Você está na página 1de 253

Chapter 1 Solutions

1.1 Problem Solutions


     
 2  −1 1
1.1.1 (a) 2  1  + 3  0  = 2
     
−1 1 1
     
     
 3   4   −3 
1   
(b)  1  −  1/2  = −1/6
  
3  
−2 −2/3 0
  
√     
√  2 2 6 6
(c) 2  √0  + 0 = 0
  3   
2 0 2
       
1 −2 −7 0
(d) 3 0 − 2  3  +  6  = 0
       
1 1 −1 0
       
       
1 0 0  x
(e) x 0 + y 1 + z 0 = y
       
0 0 1 z
       
     
1 0 −3
(f) −3 0 + π 1 =  π 
     
1 0 −3
     
       
 1   1  −1 a
(g) (a + b + c)  1  + (−a + b)  2  + (−a + 2b + c) −1 = b
       
−1 −3 2 c
       
           
−2  1  
  1  −2   1 
   
  
 
 
1.1.2 (a) Since  6  = −2 −3 we get by Theorem 1.1.2 that Span  −3 ,  6  = Span  −3 . Hence,
     

 
 
 1 
 
−2 1  1 −2 
      
 
 1 
3
the set represents the line in R with vector equation ~x = t −3, t ∈ R.
 
1
 

(b) The set only contains two vectors, so the set represents the two points (1, −3, 1), (−2, 6, −2) in R3 .

1
2 Section 1.1 Solutions

   

  1   2 
   
 
(c) Since   0  ,  1  is linearly independent (neither vector is a scalar multiple of the other), we

 −2 −1 

 

   
 1   2 
get that the set represents the plane in R3 with vector equation ~x = s  0  + t  1 , s, t ∈ R.
 
−2 −1
   
 
0
3
(d) The set only contains the zero vector, so it represents the origin in R . A vector equation is ~x = 0.
 
0
 

(e) The set is linearly independent (verify this) and so the set represents a hyperplane in R4 with vector
equation
     
1 1 3
0 0 1
~x = t1   + t2   + t3   , t1 , t2 , t3 ∈ R
1 2 0
1 1 0
 
1
1
(f) The set represents the line in R4 with vector equation ~x = t  , t ∈ R.
1
0
" # " # " # " #
1 1 1 0
1.1.3 (a) Observe that (−1) +2 − = . Hence, the set is linearly dependent. Solving for the first
2 3 4 0
" # " # " #
1 1 1
vector gives =2 − .
2 3 4
(b) Since neither vector is a scalar multiple of the other, the set is linearly independent.
(c) Since neither vector is a scalar multiple of the other, the set is linearly independent.
" # " # " #
2 −4 0
(d) Observe that 2 + = . Hence, the set is linearly dependent. Solving for the second
3 −6 0
" # " #
−4 2
vector gives = (−2)2 .
−6 3
   
1 0
(e) The only solution to c 2 = 0 is c = 0, so the set is linearly independent.
   
1 0
   

(f) Since the set contains the zero vector, it is linearly dependent by Theorem 1.1.4. Solving for the
     
0  1  4
zero vector gives 0 = 0 −3 + 0 6.
     
0 −2 1
     
       
1  1  −2 0
(g) Observe that 0 1 + 2  2  − −4 = 0. Hence, the set is linearly dependent. Solving for the
       
0 −1 2 0
       
     
 1  1 −2
second vector gives  2  = 0 1 − 21 −4.
     
−1 0 2
     
Section 1.1 Solutions 3

(h) Consider        
0  1  2  0 
0 = c1 −2 + c2 3 + c3 −1
       
0 1 4 −2
Performing operations on vectors on the right hand side, we get
   
0  c1 + 2c2 
0 = −2c1 + 3c2 − c3 
   
0 c1 + 4c2 − 2c3

Since vectors are equal if and only if there corresponding entries are equal we get the three equa-
tions in three unknowns

c1 + 2c2 = 0
−2c1 + 3c2 − c3 = 0
c1 + 4c2 − 2c3 = 0

The first equation implies that c1 = −2c2 . Substituting this into the second equation we get
7c2 − c3 = 0. Thus, c3 = 7c2 . Substituting these both into the third equation gives −12c2 = 0.
Therefore, the only solution is c1 = c2 = c3 = 0. Hence, the set is linearly independent.
(i) Observe that
         
1 2 1 2 0
1 2 0 1 0
(−2)   +   + 0   + 0   =  
 
2 4 1 3 0
1 2 0 1 0
Hence, the set is linearly dependent. Solving for the second vector gives
       
2 1 1 2
  = 2   + 0   + 0 1
2 1 0  
4 2 1 3
       
2 1 0 1

(j) Since neither vector is a scalar multiple of the other, the set is linearly independent.
" # (" #) (" #)
1 3 3
1.1.4 (a) Clearly < Span . Thus, does not span R2 and so is not a basis for R2 .
0 2 2
" #
x1
(b) Since neither vector is a scalar multiple of the other, the set is linearly independent. Let ~x = ∈
x2
R2 and consider " # " # " # " #
x1 2 1 2c1 + c2
= c1 + c2 =
x2 3 0 3c1
Comparing entries we get

x1 = 2c1 + c2
x2 = 3c1
4 Section 1.1 Solutions

2
(" # "c1#)= x2 /3 and c2 = x1 − 2x2 /3. Since there is a solution for all ~x ∈ R , we have that
Thus,
2 1
, spans R2 .
3 0
Since the set spans R2 and is linearly independent, it is a basis for R2 .
" #
x
(c) Since neither vector is a scalar multiple of the other, the set is linearly independent. Let ~x = 1 ∈
x2
2
R and consider " # " # " # " #
x1 −1 1 −c1 + c2
= c1 + c2 =
x2 1 3 c1 + 3c2
Comparing entries we get

x1 = −c1 + c2
x2 = c1 + 3c2

Solving, we get
(" c2#="(x#) x ∈ R2 ,
1 + x2 )/4 and c1 = (−3x1 + x2 )/4. Since there is a solution for all ~
−1 1
we have that , spans R2 .
1 3
Since the set spans R2 and is linearly independent, it is a basis for R2 .
(d) Observe from our work in (c) that
" # " # " #
1 −1 1 1 0
+ =
2 1 2 3 2

Thus, the set is linearly dependent and hence is not a basis.


" #
x1
(e) Since neither vector is a scalar multiple of the other, the set is linearly independent. Let ~x = ∈
x2
R2 and consider " # " # " # " #
x1 1 −3 c1 − 3c2
= c1 + c2 =
x2 0 5 5c2
Comparing entries we get

x1 = c1 − 3c2
x2 = 5c2

2
Solving,
(" # we #) c2 = x2 /5 and c1 = x1 + 3x2 /5. Since there is a solution for all ~x ∈ R , we have
" get
1 −3
that , spans R2 .
0 5
Since the set spans R2 and is linearly independent, it is a basis for R2 .
(f) The set contains the zero vector and so is linearly dependent by Theorem 1.1.4. Thus, the set
cannot be a basis.
1.1.5 (a) The set contains the zero vector and so is linearly dependent by Theorem 1.1.4. Thus, the set
cannot be a basis.
Section 1.1 Solutions 5
 
 x1 
(b) Let ~x =  x2  be any vector in R3 . Consider the equation
 
x3
 

       
 x1  −1 1  −c1 + c2 
 x2  = c1  2  + c2 1 =  2c1 + c2 
       
x3 −1 2 −c1 + 2c2

This gives the 3 equations in 2 unknowns

−c1 + c2 = x1
2c1 + c2 = x2
−c1 + 2c2 = x3

Subtracting the first equation from the third equation gives c2 = x3 − x1 . Substituting this into the
first equation we get c1 = −2x1 + x3 . Substituting both of these into the second equation gives

−5x1 + 3x3 = x2
      


 −1 1
 1
Thus, ~x is in the span of  2 , 1 if and only if −5x + 3x = x . Thus, the vector 0 is not in
     
  
      1 3 2
−1 2
 
 0
   

 −1 1
   
 
the span, so   2  , 1 does not span R3 and hence is not a basis.


−1 2
 

   
1 
 1
 
 
(c) Clearly 0 < Span  0 . Thus, the set does not span R3 and hence is not a basis.
  

   
0 1
   
 
 x1 
(d) Let ~x =  x2  be any vector in R3 . Consider the equation
 
x3
 

         
 x1  1 0 1 c1 + c3 
 x2  = c1 0 + c2 1 + c3 1 = c2 + c3  (1.1)
         
x3 1 1 0 c1 + c2

This gives the 3 equations in 3 unknowns

c1 + c3 = x1
c2 + c3 = x2
c1 + c2 = x3
6 Section 1.1 Solutions

Solving using substitution and elimination we get

1 1 1
c1 = x1 − x2 + x3
2 2 2
1 1 1
c2 = − x1 + x2 + x3
2 2 2
1 1 1
c3 = x1 + x2 − x3
2 2 2
     

 1 0 1
     
 
Thus, ~x is in the span of  0 , 1 , 1 . Hence, it spans R3 .


1 1 0
      

Moreover, if we let x1 = x2 = x3 = 0 in equation (1.1), we get that the only solution is c1 = c2 =
c3 = 0, so the set is also linearly independent. Hence, it is a basis for R3 .
(e) Observe that        
1 1 0 0
1 − 1 − 1 0 = 0
    3    
1 0 3 0
so the set is linearly dependent and hence not a basis.
 
 x1 
(f) Let ~x =  x2  be any vector in R3 . Consider the equation
 
x3
 

         
 x1   1   1  −1  c1 + c2 − c3 
 x2  = c1  1  + c2  2  + c3 −1 =  c1 + 2c2 − c3  (1.2)
         
x3 −1 −3 2 −c1 − 3c2 + 2c3

This gives the 3 equations in 3 unknowns

c1 + c2 − c3 = x1
c1 + 2c2 − c3 = x2
−c1 − 3c2 + 2c3 = x3

Solving using substitution and elimination we get

c1 = x1 + x2 + x3
c2 = −x1 + x2
c3 = −x1 + 2x2 + x3
      

  1   1  −1
     
 
Thus, ~x is in the span of   1  ,  2  , −1 . Hence, it spans R3 .



 −1 −3 

2
      
Moreover, if we let x1 = x2 = x3 = 0 in equation (1.2), we get that the only solution is c1 = c2 =
c3 = 0, so the set is also linearly independent. Hence, it is a basis for R3 .
Section 1.1 Solutions 7

   

 1 0
   
 
1.1.6 There are infinitely many correct answers. One simple choice is  0 , 1 . Since the vectors in the set

0 0

 

are standard basis vectors for R3 , we know they are linearly independent and hence the set forms a basis
for a hyperplane in R3 .
1.1.7 Assume that {~v1 , ~v2 } is linearly independent. For a contradiction, assume without loss of generality that
~v1 is a scalar multiple of ~v2 . Then ~v1 = t~v2 and hence ~v1 − t~v2 = ~0. This contradicts the fact that {~v1 , ~v2 }
is linearly independent since the coefficient of ~v1 is non-zero.
On the other hand, assume that {~v1 , ~v2 } is linearly dependent. Then there exists c1 , c2 ∈ R not both zero
such that c1~v1 + c2~v2 = ~0. Without loss of generality assume that c1 , 0. Then ~v1 = − cc12 ~v2 and hence ~v1
is a scalar multiple of ~v2 .
1.1.8 If ~v1 ∈ Span{~v2 , ~v3 }, then there exists c1 , c2 ∈ R such that ~v1 = c1~v2 + c2~v3 . Thus,

~v1 − c1~v1 − c2~v3 = ~0

where the coefficient of ~v1 is non-zero, so {~v1 , ~v2 , ~v3 } is linearly dependent.
1.1.9 Assume for a contradiction that {~v1 , ~v2 } is linearly dependent. Then there exists c1 , c2 ∈ R with c1 , c2 not
both zero such that c1~v1 + c2~v2 = ~0. Hence, we have

c1~v1 + c2~v2 + 0~v3 = ~0

with not all coefficients equal to zero which contradicts the fact that {~v1 , ~v2 , ~v3 } is linearly independent.
1.1.10 To prove this, we will prove that both sets are a subset of the other.
Let ~x ∈ Span{~v1 , ~v2 }. Then there exists c1 , c2 ∈ R such that ~x = c1~v1 + c2~v2 . Since t , 0 we get
c2
~x = c1~v1 + (t~v2 )
t

so ~x ∈ Span{~v1 , t~v2 }. Thus, Span{~v1 , ~v2 } ⊆ Span{~v1 , t~v2 }.


If ~y ∈ Span{~v1 , t~v2 }, then there exists d1 , d2 ∈ R such that

~y = d1~v1 + d2 (t~v2 ) = d1~v1 + (d2 t)~v2 ∈ Span{~v1 , ~v2 }

Hence, we also have Span{~v1 , t~v2 } ⊆ Span{~v1 , ~v2 }. Therefore, Span{~v1 , ~v2 } = Span{~v1 , t~v2 }.
1.1.11 Assume that {~v1 , ~v2 } is linearly independent and ~v3 < Span{~v1 , ~v2 }. Consider

c1~v1 + c2~v2 + c3~v3 = ~0

If c3 , 0, then we have ~v3 = − cc31 ~v1 − cc23 ~v2 which contradicts that fact that ~v3 < Span{~v1 , ~v2 }. Hence,
c3 = 0.
Thus, we have c1~v1 + c2~v2 + c3~v3 = ~0 implies c1~v1 + c2~v2 = ~0. Since {~v1 , ~v2 } is linearly independent, the
only solution to this is c1 = c2 = 0. Therefore, we have shown the only solution to c1~v1 + c2~v2 + c3~v3 = ~0
is c1 = c2 = c3 = 0, so {~v1 , ~v2 , ~v3 } is linearly independent.
8 Section 1.1 Solutions
 
 x1 
1.1.12 Let ~x =  x2  be any vector in R3 . Consider the equation
 
x3
 

       
 x1   1  0  c1 
 x2  = c1  2  + c2 1 =  2c1 + c2 
       
x3 −1 2 −c1 + 2c2

This gives the 3 equations in 2 unknowns

c1 = x1
2c1 + c2 = x2
−c1 + 2c2 = x3

Substituting c1 = x1 into the second equation we get c2 = −2x1 + x2 . Substituting both of these into the
third equation gives
−5x1 + 2x2 = x3
      

  1  0 0
   
 
Thus, ~x is in the span of  2 , 1 if and only if −5x + 2x = x . Thus, the vector 0 is not in the
  
 

 

 

 

 
 1 2 3

 −1 2  
1
   
   

  1  0
   
 
span, so   2  , 1 does not span R3 and hence is not a basis.

 −1 2 

     

      

  1  0 0
     
 
Then, by Problem 11, the set   2  , 1 , 0 is linearly independent. Consider

 −1 2 1 

       

         
 x1   1  0 0  c1 
 x2  = c1  2  + c2 1 + c3 0 =  2c1 + c2 
         
x3 −1 2 1 −c1 + 2c2 + c3

Thus, we have

c1 = x1
2c1 + c2 = x2
−c1 + 2c2 + c3 = x3
     

  1  0 0
     
 
Solving gives c1 = x1 , c2 = −2x1 + x2 , and c3 = 3x1 − 2x2 + x3 . Therefore,   2  , 1 , 0 also spans

 −1 2 1 

 

R3 and hence is a basis for R3 .
Section 1.1 Solutions 9

1.1.13 We have

~x + 2~v = ~v + (−~x)
~x + 2~v + ~x = ~v + (−~x) + ~x add ~x on the right to both sides
~x + ~x + 2~v = ~v + ~0 by V3 and V5
1~x + 1~x + (1 + 1)~v = ~v by V4, V10, and normal addition in R
(1 + 1)~x + 1~v + 1~v = ~v by V8
2~x + ~v + ~v = ~v by V10, and normal addition in R
2~x + ~v + ~v + (−~v) = ~v + (−~v) by V5, add (−~v) on the right to both sides
2~x + ~v + ~0 = ~0 by V5
2~x + ~v = ~0 by V4
2~x + ~v + (−~v) = ~0 + (−~v) by V5, add (−~v) on the right to both sides
2~x + ~0 = ~0 + (−~v) by V5
2~x = (−~v) by V4
1 1 1
(2~x) = (−~v) multiply both sides by
2 ! 2 2
1 1
2 ~x = (−~v) by V7
2 2
1
1~x = (−~v) by normal multiplication in R
2
1
~x = (−~v) by V10
2

1.1.14 (a) The statement is true. If ~v1 , ~0, then the only solution to c~v1 = ~0 is c = 0, so {~v1 } is linearly
independent. On the other hand, if ~v1 = ~0, then {~v1 } is linearly dependent by Theorem 1.1.4.
   

 1 2
   
 
(b) The statement is false. For example Span  0 , 0 is a line in R3 .


0 0
 

(c) The statement is true. Let ~x ∈ R2 . Since {~v1 , ~v2 } spans R2 there exists c1 , c2 ∈ R such that
~x = c1~v1 + c2~v2 . Then we get

~x = c1~v1 + c2~v2 + c2~v1 − c2~v1 = (c1 − c2 )~v1 + c2 (~v1 + ~v2 )

Thus, R2 ⊆ {~v1 , ~v1 + ~v2 }.


On the other hand, since ~v1 , ~v1 + ~v2 ∈ R2 by Theorem 1.1.1, we have that Span{~v1 , ~v1 + ~v2 } ⊆ R2
by Theorem 1.1.1.
Thus, Span{~v1 , ~v1 + ~v2 } = R2 .
(" # " # " #) " # (" # " #)
1 2 0 0 1 2
(d) The statement is false. The set , , is linearly dependent, but < Span , .
0 0 1 1 0 0
10 Section 1.1 Solutions
" # " #
1 0
(e) The statement is false. Let ~v1 = and ~v2 = . Then ~v1 is not a scalar multiple of ~v2 but {~v1 , ~v2 }
0 0
is linearly dependent by Theorem 1.1.4.
(f) The statement is false. If ~b = d~ = ~0, then ~x = td~ + ~b is just the origin.
(g) The statement is true. By definition of Span we have ~0 = 0~v1 + · · · + 0~vk ∈ Span{~v1 , . . . , ~vk }.
Section 1.2 Solutions 11

1.2 Problem Solutions


   
 1  2
1.2.1 (a) By Theorem 1.2.2, S1 is a subspace of R3 . Since neither  1  nor 1 is a scalar multiple of the
   
−1 4
   
    

  1  2
   
 
other, B =   1  , 1 is linearly independent. Also, Span B = S1 , so B is a basis for S1 . Thus,


−1 4
 

geometrically, S1 is a plane in R3 .
(b) By definition S2 is a subset of R2 . Observe that ~0 ∈ S2 since 0 = 0. So, S2 is non-empty.
Let ~x, ~y ∈ S2 , then x1 = x2 and y1 = y2 . Hence,
" #
x + y1
~x + ~y = 1
x2 + y2
where x1 + y1 = x2 + y2 . So ~x + ~y ∈ S2 .
For any t ∈ R we have " #
tx1
t~x = ∈ S2
tx2
since tx1 = tx2 .
Thus, S2 is a subspace of R2 by the Subspace Test.
To find a basis for S2 we need to find a linearly independent spanning set for S2 . We first find a
spanning set. Every vector ~x ∈ S2 satisfies x1 = x2 and so has the form
" # " # " #
x1 x 1
= 1 = x1 , x1 ∈ R
x2 x1 1
(" #)
1
Thus, B = spans S2 . Moreover, it contains one non-zero vector so it also linearly indepen-
1
dent. Thus, B is a basis for S2 and so S2 is a line in R2 .
 
0
3
(c) By definition S3 is a subset of R . But, observe that 0 < S3 since it does not satisfy the condition
 
0
 
on S3 . Thus, S3 is not a subspace.
 
0
(d) By definition S4 is a subset of R3 . Also, 0 ∈ S4 since 0 + 0 = 0. Thus, S4 is non-empty.
 
0
 
   
 x1  y1 
Let ~x =  x2 , ~y = y2  ∈ S4 . Then, x1 + x2 = x3 and y1 + y2 = y3 . This gives
   
x3 y3
   
 
 x1 + y1 
~x + ~y =  x2 + y2 
 
x3 + y3
 
12 Section 1.2 Solutions

and
(x1 + y1 ) + (x2 + y2 ) = x1 + x2 + y1 + y2 = x3 + y3
So, ~x + ~y satisfies the condition of S4 , so ~x + ~y ∈ S4 .
 
cx1 
Similarly, for any c ∈ R, c~x = cx2  and
 
cx3
 

cx1 + cx2 = c(x1 + x2 ) = cx3

so c~x ∈ S4 .
Thus, by the Subspace Test, S4 is a subspace of R3 .
To find a basis for S4 we need to find a linearly independent spanning set for S4 . We first find a
spanning set. Every vector ~x ∈ S4 satisfies x1 + x2 = x3 and so has the form
       
 x1   x1  1 0
 x2  =  x2  = x1 0 + x2 1 , x1 , x2 ∈ R
       
x3 x1 + x2 1 1
    

 1 0
   
 
Thus, B =  0 , 1 spans S4 . Moreover, neither vector is a scalar multiple of the other so the

1 1

     

set is linearly independent. Thus, B is a basis for S4 and so S4 is a plane in R3 .
(e) By definition S5 is non-empty subset of R4 .
   
0 0
0 0
Let ~x, ~y ∈ S5 . Then, ~x =   and ~y =  . Hence
0 0
0 0
     
0 0 0
0 0 0
~x + ~y =   +   =   ∈ S5
0 0 0
0 0 0

and for any c ∈ R


   
0 0
0 0
c~x = c   =   ∈ S5
0 0
0 0
Therefore, S5 is a subspace of R4 by the Subspace Test.
By definition, the empty set is a basis for S5 . Geometrically, S5 is the origin in R4 .
Section 1.2 Solutions 13
 
0
(f) S6 is a subset of R3 , but clearly 0 < S6 , so S6 is not a subspace.
 
0
 
 
0
0
(g) By definition S7 is a subset of R4 . Also,   ∈ S7 since 0 = 0 − 0 + 0. Thus, S7 is non-empty.
0
0
   
 x1  y1 
 x2  y 
Let ~x =  , ~y =  2  ∈ S7 . Then, x2 = x1 − x3 + x4 and y2 = y1 − y3 + y4 . This gives
 x3  y3 
x4 y4
 
 x1 + y1 
 x + y2 
~x + ~y =  2 
 x3 + y3 
x4 + y4
and
x2 + y2 = (x1 − x3 + x4 ) + (y1 − y3 + y4 ) = (x1 + y1 ) − (x3 + y3 ) + (x4 + y4 )
So, ~x + ~y satisfies the condition of S7 , so ~x + ~y ∈ S7 .
 
tx1 
tx 
Similarly, for any t ∈ R, t~x =  2  ∈ S7 since
tx3 
tx4
tx2 = t(x1 − x3 + x4 ) = (tx1 ) − (tx3 ) + (tx4 )

Thus, by the Subspace Test, S7 is a subspace of R4 .


To find a basis for S7 we need to find a linearly independent spanning set for S7 . We first find a
spanning set. Every vector ~x ∈ S7 satisfies x2 = x1 − x3 + x4 and so has the form
         
 x1   x1  1  0  0
 x   x − x + x  1 −1
  + x   + x 1 , x , x , x ∈ R
 
 2  =  1 3 4
= x
 x3  
 1 3 4 0 1 3 4
x3 0  1 


         
x4 x4 0 0 1
     


 1  0  0 


1 −1 1
 

Thus, B =  , , spans S7 .
      

 0  1  0
     

       

0

0 1

Consider         
0

1  0  0  c1 
0 1 −1 1 c − c + c 
  = c1   + c2   + c3   =  1 2 3 
0 0  1  0  c2 

        
0 0 0 1 c3
14 Section 1.2 Solutions

Comparing entries gives c1 = c2 = c3 = 0. Thus, B is also linearly independent, and so is a basis


for S7 . Therefore, S7 is a hyperplane in R4 .
 
0
0
(h) By definition S8 is a subset of R4 . Also, take a = b = c = d = 0 gives   ∈ S8 . Thus, S8 is
0
0
non-empty.
 a1 − b1   a2 − b2 
   
 b − d
, ~y =  b2 − d2  ∈ S . Then,
  
1 1
Let ~x =  8
a1 + b1 − 2d1  a2 + b2 − 2d2 

 
c1 − d1 c2 − d2

(a1 + a2 ) − (b1 + b2 )
 
 
 (b 1 + b2 ) − (d1 + d2 ) 
~x + ~y =   
(a1 + a2 ) + (b1 + b2 ) − 2(d1 + d2 )
(c1 + c2 ) − (d1 + d2 )

So, ~x + ~y satisfies the condition of S8 , so ~x + ~y ∈ S8 .


For any t ∈ R, we get

 t(a1 − b1 )   (ta1 ) − (tb1 )


   

 t(b − d )   (tb ) − (td ) 
1 1 1 1
t~x =   =  
t(a1 + b1 − 2d1 ) (ta1 ) + (tb1 ) − 2(td1 )
  
t(c1 − d1 ) (tc1 ) − (td1 )

So, t~x satisfies the condition of S8 , so t~x ∈ S8 .


Thus, by the Subspace Test, S8 is a subspace of R4 .
To find a basis for S8 we need to find a linearly independent spanning set for S8 . We first find a
spanning set. Every vector ~x ∈ S8 has the form

 a − b  −1


         
1 0  0 
 b − d 
 = a   + b   + c   + d −1 , a, b, c, d ∈ R
0  1  0  

a + b − 2d 1  1  0 −2
         
c−d 0 0 1 −1
        


 1 −1 0  0  
         
0  1  0 −1

 
Thus,  , , , spans S8 .



 1  1  0 −2



 
 
 
 
 
 
 


         
0 0 1 −1 
 

However, observe that


0 1 −1 0  0 
         
0 0  1  0 −1
  =   +   +   +  
0 1  1  0 −2
         
0 0 0 1 −1
Section 1.2 Solutions 15

So, spanning set is linearly dependent. Using Theorem 1.1.2, we get


      


 1 −1 0 
       
0  1  0

 
Span  , , = S8


 1  1  0
      

       

0

0 1

Now, consider
−1 0 c1 − c2 
         
0 1
0
  = c 0 + c  1  + c 0 =  c2 
       
0 1 1 2  1  3 0 c1 + c2 
         
0 0 0 1 c3
Comparing entries gives c1  − c2 = 0, c2 =0, c1 + c2 = 0, and c3 = 0. Thus, c1 = c2 = c3 = 0
1 −1 0
     

 

       
0  1  0

 
is the only solution so, B =  , , is also linearly independent, and so is a basis for S8 .

1 1 0
 

 

 

 

 

 

 



      
      

0

0 1

Therefore, S8 is a hyperplane in R4 .
1.2.2 By definition, a plane P in R3 has vector equation ~x = c1~v1 + c2~v2 + ~b, c1 , c2 ∈ R where {~v1 , ~v2 } is linearly
independent.
If P is a subspace of R3 , then ~0 ∈ P and hence P passes through the origin. On the other hand, if P
passes through the origin, then there exists d1 , d2 ∈ R such that
~0 = d1~v1 + d2~v2 + ~b

Thus, ~b = −d1~v1 − d2~v2 . Hence, we can write the vector equation of P as

~x = c1~v1 + c2~v2 − d1~v1 − d2~v2 = (c1 − d1 )~v1 + (c2 − d2 )~v2

Since c1 and c2 can be any real numbers, (c1 − d1 ) and (c2 − d2 ) can be any real numbers, so P =
Span{~v1 , ~v2 }. Consequently, P is a subspace of R2 by Theorem 1.2.2.
1.2.3 Since S does not contain the zero vector it cannot be a subspace.
 
 x1 
1.2.4 (a) Let ~x =  x2  be any vector in R3 . Consider the equation
 
x3
 

       
 x1  1 2 c1 + 2c2 
 x2  = c1 3 + c2 0 =  3c1 
       
x3 1 1 c1 + c2

This gives the 3 equations in 2 unknowns

c1 + 2c2 = x1
3c1 = x2
c1 + c2 = x3
16 Section 1.2 Solutions

Subtracting the third equation from the first equation gives c2 = x1 − x3 . Substituting this and the
second equation into the third equation we get

1
x2 + x1 = 2x3
3
      

 1 2 1
   
  1
Thus, ~x is in the span of  3 , 0 if and only if 3 x2 + x1 = 2x3 . Thus, the vector 0 is not in the
  
     

1 1 
0
 
span, so B does not span R3 and hence is not a basis for R3 .
(b) From our work in (a), we have that B is a basis for the subspace
    
( ) 1 2
1


   
S = ~x ∈ R3 | x2 + x1 = 2x3 = Span 
 
3 , 0

3 1 1

 

 
0
1.2.5 (a) If d , 0, then 0 < P since a(0) + b(0) + c(0) , d. Hence, P cannot be a subspace.
 
0
 

(b) If P had a basis, then there would be a spanning set for P. But, this would contradict Theorem
1.2.2, since P is not a subspace.
(c) If a = b = c = 0, then P is the empty set. If a , 0, then every vector in ~x ∈ P has the form
  d b
 x1   a − a x2 − ac x3  d/a
      
−b/a −c/a
 x2  =  x2  =  0  + x2  1  + x3  0  , x2 , x3 ∈ R
         
x3 x3 0 0 1

The remaining cases are similar.


(d) If a = b = c = 0, then P is the empty set. If a , 0, then P is a plane in R3 passing through
   

 −b/a −c/a 

 
(d/a, 0, 0) since   1  ,  0  is linearly independent. The remaining cases are similar.
  


 0 

1 
   

1.2.6 A set S is a subset of Rn if every element of S is in Rn . For S to be a subspace, it not only has to be a
subset, but it also must be closed under addition and scalar multiplication of vectors (by the Subspace
Test).
Section 1.3 Solutions 17

1.3 Problem Solutions


1.3.1 Evaluate the following:
   
 5  3
(a)  3  · 2 = 5(3) + 3(2) + (−6)(4) = −3
   
−6 4
   
   
 1   2 
−2 1/2
(b)   ·   = 1(2) + (−2)(1/2) + (−2)(1/2) + 3(−1) = −3
−2 1/2
3 −1
 √ 
 2 q √ √ √
(c)  √1  = ( 2)2 + 12 + (− 2)2 = 5
 
2
 
   
 1   3 
(d)  4  · −1 = 1(3) + 4(−1) + (−1)(−1) = 0
   
−1 −1
   
 
 1 
 2  p √
(e)   = 12 + 22 + (−1)2 + 32 = 15
−1
3
     
 2   1   2 
(f)  1  ×  1  = −1
     
−3 −1 1
     
     
 1   2  −2
(g)  1  ×  1  =  1 
     
−1 −3 −1
     
     
3 3a 0
(h) 2 × 2a = 0
     
3 3a 0
     

1.3.2 (a) Since the plane passes through (0, 0, 0) we get that a scalar equation is

x1 − 3x2 + 3x3 = 1(0) + (−3)(0) + 3(0) = 0

(b) For two planes to be parallel, their normal vectors must be scalar multiples of each other. Hence,
 
 3 
a normal vector for the required plane is  2 . Thus, a scalar equation of the plane is
 
−1
 

3x1 + 2x2 − x3 = 3(1) + 2(2) − 1(0) = 7


18 Section 1.3 Solutions

(c) A normal vector for the required plane is


     
 1   1   2 
~n =  0  ×  1  = −1
     
−2 −1 1
     

Since the plane passes through the origin, we get that a scalar equation is

2x1 − x2 + x3 = 0
     
3  2  −6
(d) We have 3 ×  1  =  9 . Any non-zero scalar multiple of this will be a normal vector for the
     
3 −1 −3
     
 
−2
plane. We pick ~n =  3 . Since the plane passes through the origin, we get that a scalar equation
 
−1
 
is
−2x1 + 3x2 − x3 = 0

(e) A normal vector for the required plane is


     
 3  2  5 
~n =  1  × 2 = −7
     
−2 1 4
     

Since the plane passes through the origin, we get that a scalar equation is

5x1 − 7x2 + 4x3 = 0

1.3.3 Determine which of the following sets are orthogonal.


   
1 −1
(a) We have 1 · −1 = −4, so the set is not orthogonal.
   
2 −1
   

(b) We have
   
 1   3 
 2  · −2 = 0
   
−1 −1
   
 1  2
 2  · 1 = 0
   
−1 4
   
 3  2
−2 · 1 = 0
   
−1 4

Hence, the set is orthogonal.


Section 1.3 Solutions 19

(c) We have
   
0 1
0 · 0 = 0
   
0 1
   
0 0
0 · 1 = 0
   
0 0
   
1 0
0 · 1 = 0
   
1 0

Hence, the set is orthogonal.


(d) We have
 √   
1/ 2  0√ 

1/ 3 · −1/ 3 = 0
 √   √ 
1/ 6 2/ 6
 √   √ 
1/ 2  1/ 2 
 √   √ 
1/ √3 · −1/ √3 = 0
1/ 6 −1/ 6
   
   √ 
 0√   1/ 2 
√ 
−1/√ 3 · −1/ √3 = 0
  
2/ 6 −1/ 6
   

Hence, the set is orthogonal.


1.3.4 We have
 √   √ 
1/ 3  1/ 2 
1/ √3 ·  0  = 0
 √   √ 
1/ 3 −1/ 2
 √   √ 
1/ 3  1/ 6 
 √   √ 
1/ √3 · −2/√ 6 = 0
   
1/ 3 1/ 6
 √   √ 
 1/ 2   1/ 6 
   √ 
 0√  · −2/√ 6 = 0
−1/ 2
   
1/ 6
20 Section 1.3 Solutions

Hence, the set is orthogonal. Also,


 √ 
1/ 3 p
 √  = 1/3 + 1/3 + 1/3 = 1
1/ √3
1/ 3
 √ 
 1/ 2  p
 0  = 1/2 + 0 + 1/2 = 1
 √ 
−1/ 2
 √ 
 1/ 6  p
 √ 
−2/√ 6 = 1/6 + 4/6 + 1/6 = 1

1/ 6

 x1 
 

1.3.5 Let ~x =  ... . Then


 
 
xn
  2
cx1 
kc~x k =  ...  = (cx1 )2 + · · · + (cxn )2 = c2 x12 + · · · + c2 xn2 = c2 (x12 + · · · + xn2 ) = c2 k~x k2
2  
 
cxn

Taking square roots of both sides gives kc~x k = c2 k~x k = |c|k~x k.
1.3.6 Using Problem 1.3.5 we get
1 ~x = 1 k~x k = 1
k~x k k~x k

1.3.7 Consider c1~v1 + c2~v2 = ~0. Then we have

0 = ~v1 · ~0 = ~v1 · (c1~v1 + c2~v2 ) = c1~v1 · ~v1 + c2~v1 · ~v2 = c1 k~v1 k2 + 0 = c1

Similarly,
0 = ~v2 · ~0 = ~v2 · (c1~v1 + c2~v2 ) = c1~v2 · ~v1 + c2~v2 · ~v2 = 0 + c2 k~v2 k2 = c2
Thus, c1~v1 + c2~v2 = ~0 implies c1 = c2 = 0, so {~v1 , ~v2 } is linearly independent.
1.3.8 We have

k~v1 + ~v2 k2 = (~v1 + ~v2 ) · (~v1 + ~v2 )


= ~v1 · ~v1 + ~v1 · ~v2 + ~v2 · ~v1 + ~v2 · ~v2
= k~v1 k2 + 0 + 0 + k~v2 k2
= k~v1 k2 + k~v2 k2

1.3.9 By definition of the cross product, we have that ~n · ~v = 0 and ~n · w


~ = 0. If ~y ∈ Span{~v, w
~ }, then there
exists c1 , c2 ∈ R such that ~y = c1~v + c2 w
~ . Hence,

~y · ~n = (c1~v + c2 w
~ ) · ~n = c1~v · ~n + c2 w
~ · ~n = 0 + 0 = 0
Section 1.3 Solutions 21
 
1
1.3.10 By definition, the set S of all vectors orthogonal to ~x = 1 is a subset of R3 . Moreover, since ~0 · ~x = 0
 
1
 
~
we have that 0 ∈ S. Thus, S is non-empty.
Let ~y,~z ∈ S. Then ~x · ~y = 0 and ~x · ~z = 0. Thus, we have
~x · (~y + ~z) = ~x · ~y + ~x · ~z = 0 + 0 = 0
and
~x · (t~y) = t(~x · ~y) = 0
for any t ∈ R. Thus, ~y + ~z ∈ S and t~y ∈ S, so S is a subspace of R3 .
1.3.11 Let ~x ∈ Rn . Then,
0  x1 
   
~0 · ~x =  ..  ·  ..  = 0x1 + · · · + 0xn = 0
   
 .   . 
0 xn
as required.
     
1 0  1 
0 1  1 
1.3.12 (a) The statement is false. If ~x =  , ~y =  , and ~z =  , then ~x · ~y = 0, ~y · ~z = 0, but ~x · ~z = 1.
0 1 −1
1 0 0
" # " #
0 1
(b) The statement if false. If ~x = and ~y = , then ~x · ~y = 0, but {~x, ~y} is linearly dependent since
0 0
it contains the zero vector.
(c) The statement is true. We have
(s~x) · (t~y) = (st)(~x · ~y) = 0
Thus, {s~x, t~y} is an orthogonal set for any s, t ∈ R.
(d) The statement is true. We have
~z · ~v = ~z · (s~x + t~y) = s(~z · ~x) + t(~z · ~y) = 0 + 0 = 0
Hence, ~z is orthogonal to ~v = s~x + t~y for any s, t ∈ R.
     
1 0 0
(e) The statement is false. Take ~x = 0, ~y = 1, and ~z = 1. Then
     
0 0 0
     

     
1 0 0
~x × (~y × ~z) = 0 × 0 = 0
     
0 0 0
     

But,      
0 1 0
(~x × ~y) × ~z = 0 × 0 = 1
     
1 0 0
     
22 Section 1.3 Solutions

(f) The statement is true. If ~x · ~n = ~b · ~n is a scalar equation of the plane, then multiplying both sides
by t , 0 gives another scalar equation for the plane ~x · (t~n) = ~b · (t~n). Thus, t~n is also a normal
vector for the plane.
Section 1.4 Solutions 23

1.4 Problem Solutions


1.4.1 (a) We have
" # " #
~u · ~v −6 2 −12/13
proj~v (~u) = ~v = =
k~vk2 13 3 −18/13
" # " # " #
−3 −12/13 −27/13
perp~v (~u) = ~u − proj~v (~u) = − =
0 −18/13 18/13

(b) We have
" # " #
~u · ~v 9 1 9/10
proj~v ~u = ~
v = =
k~vk2 10 3 27/10
" # " # " #
3 9/10 21/10
perp~v ~u = ~u − proj~v ~u = − =
2 27/10 −7/10

(c) We have
" # " #
~u · ~v 0 2 0
proj~v (~u) = ~
v = =
k~vk2 13 −3 0
" # " # " #
6 0 6
perp~v (~u) = ~u − proj~v (~u) = − =
4 0 4

(d) We have
" # " #
~u · ~v 9 3 27/13
proj~v (~u) = ~v = =
k~vk2 13 2 18/13
" # " # " #
1 27/13 −14/13
perp~v (~u) = ~u − proj~v (~u) = − =
3 18/13 21/13

(e) We have
   
1 3
~u · ~v 3    
proj~v (~u) = ~
v = 0 = 0
k~vk2 1    
0 0
     
3 3 0
perp~v (~u) = ~u − proj~v (~u) = 2 − 0 = 2
     
4 0 4
     

(f) We have
   
1 3
~u · ~v 9    
proj~v (~u) = ~
v = 1 = 3
k~vk2 3    
1 3
     
3 3  0 
perp~v (~u) = ~u − proj~v (~u) = 2 − 3 = −1
     
4 3 1
     
24 Section 1.4 Solutions

(g) We have
   
1 4
~u · ~v 8 0 0
proj~v (~u) = ~v =   =  
k~vk2 2 0 0
1 4
 2  4 −2
     
 5  0  5 
perp~v (~u) = ~u − proj~v (~u) =   −   =  
−6 0 −6
6 4 2

(h) We have
   
 1   4/17 
~u · ~v 8  4   16/17 
proj~v (~u) = ~
v =  =
k~vk2 34  4   16/17 

−1 −4/17
     
1  4/17  13/17
1  16/17   1/17 
perp~v (~u) = ~u − proj~v (~u) =   −   =  
1  16/17   1/17 
1 −4/17 21/17

(i) We have
   
1 2/3
~u · ~v 4 2 4/3
proj~v (~u) = ~v =   =  
k~vk2 6 0  0 
1 2/3
     
 3  2/3  7/3 
 1  4/3 −1/3
perp~v (~u) = ~u − proj~v (~u) =   −   =  
 1   0   1 

−1 2/3 −5/3

(j) We have

−2 2/3
   
~u · ~v −8 −4 4/3
proj~v (~u) = ~v =  = 
k~vk2 24  0   0 
−2 2/3
     
 3  2/3  7/3 
 1  4/3 −1/3
perp~v (~u) = ~u − proj~v (~u) =   −   =  
 1   0   1 
−1 2/3 −5/3
Section 1.4 Solutions 25
 
 3 
1.4.2 (a) A normal vector for the plane P is ~n = −2. Thus,
 
3
 

     
 1   3   1 
~v · ~n 0
projP (~v) = perp~n (~v) = ~v − ~
n =   22 −2 =  0 
0 −
     
k~nk2
 
−1 3 −1

This makes sense since ~v ∈ P.


 
1
(b) A normal vector for the plane P is ~n = 2. Thus,
 
2
 

     
−1 1 −14/9
~v · ~n   5  
projP (~v) = perp~n (~v) = ~v − ~n =  2  − 2 =  8/9 

k~nk2   9  
1 2 −1/9

       
 0  0 −4 1
(c) Observe that −1 × 3 =  0 . Hence, a normal vector for the plane P is ~n = 0. Thus,
       
1 1 0 0
       

     
 3  1   0 
~v · ~n   3    
projP (~v) = perp~n (~v) = ~v − ~
n = −4
  1 0 = −4

k~nk2
 
1 0 1

     
 1  −2  3 
(d) A normal vector for the plane P is ~n = −2 ×  1  =  0 . Hence,
     
1 −2 −3
     

     
1  3  3/2
~v · ~n   3    
projP (~v) = perp~n (~v) = ~v − ~
n = 1 + 0= 1 
k~nk2 18    
 
2 −3 3/2
     
1  1  −1
1.4.3 A normal vector for the plane P is ~n = 1 ×  0  =  2 .
     
1 −1 −1
     
 
1
(a) The projection of ~x = 2 onto P is
 
3
 

     
1 −1 1
~x · ~n   0    
projP (~x) = perp~n (~x) = ~x − ~n = 2 −  2  = 2
k~nk2   6   
3 −1 3
26 Section 1.4 Solutions
 
2
(b) The projection of ~x = 1 onto P is
 
0
 

     
2 −1 2
~x · ~n 0
projP (~x) = perp~n (~x) = ~x − ~
n = −
  6  2  = 1
1
     
k~nk2
 
0 −1 0

 
 1 
(c) The projection of ~x = −2 onto P is
 
1
 

     
 1  −1 0
~x · ~n   −6    
projP (~x) = perp~n (~x) = ~x − ~
n = −2 −  2  = 0
k~nk2 6    
 
1 −1 0

 
2
(d) The projection of ~x = 3 onto P is
 
3
 

     
2 −1 13/6
~x · ~n 1
projP (~x) = perp~n (~x) = ~x − ~n = 3 −  2  =  8/3 
     
k~nk2   6  
3 −1 19/6

1.4.4 (a) We have

(~x + ~y) · ~v
proj~v (~x + ~y) = ~v
k~v k2
~x · ~v + ~y · ~v
= ~v
k~v k2
~x · ~v ~y · ~v
= 2
~v + ~v
k~v k k~v k2
= proj~v (~x) + proj~v (~y)

(b) We have

(s~x) · ~v
proj~v (s~x) = ~v
k~v k2
~x · ~v
=s ~v
k~v k2
= s proj~v (~x)
Section 1.4 Solutions 27

(c) We have

proj~v (~x) · ~v

~v

proj~v proj~v (~x) =
k~v k2
 
~x·~v
k~v k2
~v · ~v
= ~v
k~v k2
~x·~v
(~v · ~v)
= ~v·~v 2 ~v
k~v k
~x · ~v
= ~v
k~v k2
= proj~v (~x)

1.4.5 (a) The statement is true. We have

~x · (−~v) (−1)(~x · ~v) ~x · ~v


proj−~v (~x) = (−~v) = [(−1)~v] = (−1)(−1) ~v = proj~v (~x)
k − ~v k2 k~v k2 k~v k2

(b) The statement is true. We have

(−~x) · ~v (−1)(~x · ~v)


proj~v (−~x) = ~v = ~v = − proj~v (~x)
k~v k2 k~v k2

(c) The statement is false. If ~x is orthogonal to ~v, then proj~v (~x) = ~0 and hence {proj~v (~x), perp~v (~x)} is
linearly dependent.
(d) The statement is true. We have
 
~x·~v
~x − k~v k2
~v · ~v
~v

proj~v perp~v (~x ) =
k~v k2
~x·~v
~x · ~v − ~v·~v
(~v · ~v)
= ~v
k~v k2
~x · ~v − ~x · ~v
= ~v = ~0
k~v k2
 
~x·~v
 ~x · ~v k~v k2
~v · ~v
perp~v proj~v (~x) = ~v − ~v
k~v k2 k~v k2
~x·~v
~x · ~v ~v·~v
(~v · ~v)
= ~v − ~v
k~v k2 k~v k2
~x · ~v ~x · ~v
= ~v − ~v
k~v k2 k~v k2
= ~0 = proj~v perp~v (~x)

28 Section 1.4 Solutions

1.4.6 Since P is a plane in R3 , we must have {~v1 , ~v2 } is linearly independent. So, by Problem 1.1.10, we only
need to prove that proj~n (~x) < P.
For a contradiction, assume that proj~n (~x) ∈ P. Then, there exists c1 , c2 ∈ R such that

~x · ~n
~n = c1~v1 + c2~v2
k~n k2

Then, !
~x · ~n ~x · ~n
~x · ~n = (~
n · ~
n ) = ~
n · ~n = (c1~v1 + c2~v2 ) · ~n = 0
k~n k2 k~n k2
since ~n is the normal vector for P and ~v1 , ~v2 ∈ P. Hence, ~x is orthogonal to ~n which implies that ~x ∈ P
which is a contradiction. Therefore, proj~n (~x) < P and the result follows from Problem 1.1.10.
Chapter 2 Solutions

2.1 Problem Solutions


2.1.1 (a) x1 = 1, x2 = 2, x3 = 3.
(b) x1 = 1, x1 = 2, x2 + x3 = 1.
(c) x1 + x2 + x3 = 1, 2x1 + 2x2 + 2x3 = 2, 3x1 + 3x2 + 3x3 = 3.
   
1 1
(d) If ~x = s 2 + 0, s ∈ R is the solution, then we have
   
1 1
   

   
 x1   s + 1
 x2  =  2s 
   
x3 s+1

Thus, x3 = x1 = s + 1 = 12 (2s) + 1 = 12 x2 + 1. Hence, we get


1 1
x1 − x3 = 0, − x2 + x3 = 1, x1 − x2 = 1
2 2
2.1.2 (a) We have
! ! !
3 10 3
6 +2 − +3 =1
7 7 7
! ! !
3 10 3
2 + − +6 =2
7 7 7
! ! !
3 10 3
4 +5 − −6 = −8
7 7 7
Since the third equation is not satisfied, ~x is not a solution of the system.
(b) We have

3(−2) + 6(−1) + 7(3) = 9


−4(−2) + 3(−1) − 3(3) = −4
(−2) − 13(−1) − 2(3) = 5

Since the third equation is not satisfied, ~x is not a solution of the system.
29
30 Section 2.1 Solutions

(c) We have

4 − 2(3) + (−1) − 3(−1) = 0


−4(4) + 5(3) + 5(−1) − 6(−1) = 0
5(4) − 3(3) + 3(−1) + 8(−1) = 0

Hence, ~x is a solution of the system.


(d) We have

(7 + 3t) + 2(−2 − 2t) + (t) = 3


−(7 + 3t) + (−2 − 2t) + 5t = −9
(7 + 3t) + (−2 − 2t) − t = 5

Hence, ~x is a solution of the system.


      
1 1 0

    
2.1.3 By Theorem 2.1.1, we have that ~x = 0 + c 0 − 1 is a solution for any c ∈ R. Thus, 3 other
  
     
0 0 1
  
     
 2   3  −1
solutions are: −1, −2,  2 .
     
−1 −2 2
     

2.1.4 (a) TRUE. Since we can write the equation as x1 + 3x2 + x3 = 0, it is linear.
(b) FALSE. Since it contains the square of a variable, it is not a linear equation.
   
1 2
(c) FALSE. The system x1 = 1, x2 = 1, and x3 = 1 has solution 1, but clearly 2 is not a solution.
   
1 2
   
" #
−1
(d) FALSE. The systems x1 + x2 = 0 has more unknowns than equations, but it has a solution .
1
2.1.5 (a) Since the right hand side of all the equations is 0, the system consists of hyperplanes which all
pass through the origin. Since, the origin lies on all of the hyperplanes, it is a solution. Therefore,
the system is consistent.
(b) The i-th equation of the system has the form

ai1 x1 + · · · + ain xn = 0

Substituting in ~x = ~0, we get


ai1 (0) + · · · + ain (0) = 0
Hence, all of the equations are satisfied, so ~x = ~0 is a solution of the system. Therefore, the system
is consistent.
Section 2.1 Solutions 31

2.1.6 (a) Observe that

2(−1) + (2) + 4(1) + 2 = 6


(−1) + 1 + 2(2) = 4
(−1)(−1) + (−4)(2) − 9(1) + 10(2) = 4

So, P(−1, 2, 1, 2) lies on all three hyperplanes.


(b) Taking t = s = 0, gives Q1 (4, −2, 0, 0) lies on all three hyperplanes.
Taking t = 1, s = 0, gives Q2 (3, −4, 1, 0) lies on all three hyperplanes.
Taking t = 0, s = 1, gives Q3 (2, 1, 0, 1) lies on all three hyperplanes.
32 Section 2.2 Solutions

2.2 Problem Solutions


2.2.1 1
" # " #
1 2 2 1 2 2 1
(a) i. ,
1 1 1 1 1 1 2
ii. Row reducing the augmented matrix we get
" # " #
1 2 2 1 1 2 2 1
∼ ∼
1 1 1 2 R2 − R1 0 −1 −1 1 (−1)R2
" # " #
1 2 2 1 R1 − 2R2 1 0 0 3

0 1 1 −1 0 1 1 −1

Hence, the rank of the coefficient matrix and the rank of the augmented matrix are 2.
iii. x3 is a free variable, so let x3 = t ∈ R. Then x1 = 3 and x2 + x3 = −1, so x2 = −1 − t.
Therefore, all solutions have the form
       
 x1   3   3   0 
~x =  x2  = −1 − t = −1 + t −1
       
x3 t 0 1
       

iv. The system is a pair of planes in R3 which intersect in a line passing through (3, −1, 0).
" # " #
2 2 −3 2 2 −3 3
(b) i. ,
1 1 1 1 1 1 9
ii.
" # " #
2 2 −3 3 1 1 1 9
R1 ↔ R2 ∼ ∼
1 1 1 9 2 2 −3 3 R2 − 2R1
" # " # " #
1 1 1 9 1 1 1 9 R1 − R2 1 1 0 6
∼ ∼
0 0 −5 −15 − 51 R2 0 0 1 3 0 0 1 3

Hence, the rank of the coefficient matrix and the rank of the augmented matrix are 2.
iii. x2 is a free variable, so let x2 = t ∈ R. Then x1 + x2 = 6, so x1 = 6 − t, and x3 = 3. Therefore,
all solutions have the form
       
 x1  6 − t 6 −1
~x =  x2  =  t  = 0 + t  1 
       
x3 3 3 0
       

iv. The system is a pair of planes in R3 which intersect in a line passing through (3, −1, 0).
   
 2 1 −1   2 1 −1 4 
(c) i.  2 −1 −3  ,  2 −1 −3 −1 
   
3 2 −1 3 2 −1 8
   
Section 2.2 Solutions 33

ii.
   
 2 1 −1 4   3 2 −1 8  R1 − R3
 2 −1 −3 −1  R1 ↔ R3 ∼  2 −1 −3 −1  ∼
   
3 2 −1 8 2 1 −1 4
  
   
 1 1 0 4   1 1 0 4 
 2 −1 −3 −1  R2 − 2R1 ∼  0 −3 −3 −9  − 31 R2 ∼
   
2 1 −1 4 R3 − 2R1 0 −1 −1 −4
  
   
 1 1 0 4  R1 − R2  1 0 −1 1  R1 + R3
0 1 1 3  ∼  0 1 1 3  R2 + 3R3 ∼
   

0 −1 −1 −4 R3 + R2 0 0 0 −1
  
   
 1 0 −1 0   1 0 −1 0 
 0 1 1 0  ∼  0 1 1 0 
   
0 0 0 −1 (−1)R3 0 0 0 1
  

Hence, the rank of the coefficient matrix is 2, and the rank of the augmented matrix is 3.
iii. Since the rank of the augmented matrix is greater than the rank of the coefficient matrix, the
system is inconsistent.
iv. The system is a set of three planes in R3 which have no common point of intersection.
   
 1 2 0   1 2 0 3 
(d) i.  1 3 −1 ,  1 3 −1 4 
   
−3 −8 2 −3 −8 2 −11
   
ii.
     
 1 2 0 3   1 2 0 3  R1 − 2R2  1 0 2 1 
 1 3 −1 4  R2 − R1 ∼  0 1 −1 1  ∼ 0 1 −1 1 
    

−3 −8 2 −11 R3 + 3R1 0 −2 2 −2 R3 + 2R2 0 0 0 0
    

Hence, the rank of the coefficient matrix and the rank of the augmented matrix are 2.
iii. x3 is a free variable, so let x3 = t ∈ R. Then x1 + 2x3 = 1 and x2 − x3 = 1, so x1 = 1 − 2t, and
x2 = 1 + t. Therefore, all solutions have the form
       
 x1  1 − 2t 1 −2
~x =  x2  =  1 + t  = 1 + t  1 
       
x3 t 0 1
       

iv. The system is three planes in R3 which intersect in a line passing through (1, 1, 0).
" # " #
1 3 −1 1 3 −1 9
(e) i. ,
2 1 −2 2 1 −2 −2
ii.
" # " #
1 3 −1 9 1 3 −1 9
∼ ∼
2 1 −2 −2 R2 − 2R1 0 −5 0 −20 − 51 R2
" # " #
1 3 −1 9 R1 − 3R2 1 0 −1 −3

0 1 0 4 0 1 0 4
Hence, the rank of the coefficient matrix and the rank of the augmented matrix are 2.
34 Section 2.2 Solutions

iii. x3 is a free variable, so let x3 = t ∈ R. Then x2 = 4, and x1 − x3 = −3, so x1 = −3 + t.


Therefore, all solutions have the form
       
 x1  −3 + t −3 1
~x =  x2  =  4  =  4  + t 0
       
x3 t 0 1
       

iv. The system is a pair of planes in R3 which intersect in a line passing through (−3, 4, 0).
" # " #
2 −2 2 1 2 −2 2 1 −4
(f) i. ,
3 −3 3 4 3 −3 3 4 9
ii.
1 −4 21 R1
" # " #
2 −2 2 1 −4 2 −2 2
∼ ∼
3 −3 3 4 9 R2 − 23 R1 0 0 0 5/2 15 52 R2
1 −1 1 1/2 −2 R1 − 21 R2
" # " #
1 −1 1 0 −5

0 0 0 1 6 0 0 0 1 6

Hence, the rank of the coefficient matrix and the rank of the augmented matrix are 2.
iii. x2 and x3 are free variables, so let x2 = s ∈ R and x3 = t ∈ R. Then x4 = 6, and x1 − x2 + x3 =
−5, so x1 = −5 + s − t. Therefore, all solutions have the form

 x1  −5 + s − t −5 −1


         
1
 x2   s   0 
 =   + s   + t  0 
1  
~x =   = 
 x3   t   0 
  
0
 
 1 
 
x4 6 6 0 0

iv. The system is a pair of hyperplanes in R4 which intersect in a plane passing through (−5, 0, 0, 6).
   
 1 1 1 6   1 1 1 6 5 
 0 2 −4 4   0 2 −4 4 −4 
(g) i.  ,  
 2 0 4 5   2 0
 
4 5 4 

−1 2 −3 4 −1 2 −3 4 9
ii.
   
 1 1 1 6 5   1 1 1 6 5 
 0 2 −4 4 −4    1
 0 2 −4 4 −4  2 R2 ∼
  ∼ 
 2 0 4 5 4  R3 − 2R1  0 −2 2 −7 −6 

 

   
−1 2 −3 4 9 R4 + R1 0 3 −2 10 14
5  R1 − R2
   
 1 1 1 6
  1 0 3 4 7 

 0 1 −2 2 −2   0 1 −2 2 −2 

 0 −2 ∼  ∼
2 −7 −6  R3 + 2R2  0 0 −2 −3 −10  (−1)R3
 
    1
0 3 −2 10 14 R4 − 3R2 0 0 4 4 20 4 R4
R1 − 3R3
   
 1 0 3 4 7   1 0 3 4 7 
 0 1 −2 2 −2   0 1 −2 2 −2  R2 + 2R3

 0 0
 ∼   ∼
2 3 10   0 0 1 1 5 

 
0 0 1 1 5 R3 ↔ R4 0 0 2 3 10 R4 − 2R3
Section 2.2 Solutions 35

1 −8  R1 − R4 0 −8 
   
 1 0 0   1 0 0

 0 1 0 4 8  R2 − 4R4  0 1 0 0 8 
 ∼ 
 0 0 1 1 5  R3 − R4  0 0 1 0 5 
 
  
0 0 0 1 0 0 0 0 1 0

Hence, the rank of the coefficient matrix and the rank of the augmented matrix are 4.
iii. The solution is x1 = −8, x2 = 8, x3 = 5, x4 = 0.
iv. The system is four hyperplanes in R4 which intersect only at the point (−8, 8, 5, 0).
 1 2 1 2   1 2 1 2 −2 
   
 2 1 2 1   2 1 2 1 2 
(h) i.  , 
 0 2 1 1   0 2 1 1 2 


1 1 2 0 1 1 2 0 6
ii.

 1 2 1 2 −2  2 −2 
   
 1 2 1
6  − 31 R2

 2 1 2 1 2  R2 − 2R1  0 −3 0 −3
 

 0 2 1 1 ∼ ∼
2   0 2 1 1 2 
  
   
1 1 2 0 6 R4 − R1 0 −1 1 −2 8
2 −2  R1 − 2R2  1 0 1 2  R1 − R3  1 0 0 1 −4 
     
 1 2 1
 
0
  
 0 1 0 1 −2   0 1 0 1 −2   0 1 0 1 −2 
 ∼ ∼
 0 2 1 1 2  R3 − 2R2  0 0 1 −1 6   0 0 1 −1 6 
    
     
0 −1 1 −2 8 R4 + R2 0 0 1 −1 6 R4 − R3 0 0 0 0 0

Hence, the rank of the coefficient matrix and the rank of the augmented matrix are 3.
iii. x4 is a free variables, so let x4 = t ∈ R. Then x1 + x4 = −4, x2 + x4 = −2, and x3 − x4 = 6.
Thus, x1 = −4 − t, x2 = −2 − t, and x3 = 6 + t. Therefore, all solutions have the form

 x1  −4 − t −4 −1


       
 x  −2 − t −2
 =   + t −1
 
~x =  2  = 
 x3   6 + t   6 
    1 
 
x4 t 0 1

iv. The system is four hyperplanes in R4 which intersect in a line passing through (−4, −2, 6, 0).
 0 1 −2   0 1 −2
   
3 
 2 2 
3   2 2
  3 1 
(i) i.  , 
 1 2 4   1 2 4 −1 

  
2 1 −1 2 1 −1 1
36 Section 2.2 Solutions

ii.

 0 1 −2 4 −1 
   
3   1 2
 
 2 2 3 1   2 2 3 1  R2 − 2R1
 R1 ↔ R3 ∼  
 ∼
 1 2 4 −1   0 1 −2 3 
 
  
2 1 −1 1 2 1 −1 1 R4 − 2R1
4 −1  4 −1  R1 − 2R2
   
 1 2
  1 2

 0 −2 −5 3   0 1 −2 3 
 ∼  
 ∼
 0 1 −2 3   0 −2 −5 3  R3 + 2R2
 
  R2 ↔ R3  
0 −3 −9 3 0 −3 −9 3 R4 + 3R2
8 −7  8 −7  R1 − 8R3
   
 1 0   1 0 
 0 1 −2 3   0 1 −2 3  R2 + 2R3

 0 0 −9 ∼  
 ∼
9   0 0 1 −1 
 
− 91 R3
  
0 0 −15 12 0 0 −15 12 R4 + 15R3
 1 0 0 −15   1 0 0 −15 
     
R1 + 15R4  1 0 0 0 

 0 1 0 1   0 1 0 1  R2 − R4  0 1 0 0 

 0 0 1 −1  ∼  ∼ 
 0 0 1 −1  R3 + R4  0 0 1 0
 

− 31 R4
 
0 0 0 −3 0 0 0 1 0 0 0 1

Hence, the rank of the coefficient matrix is 3, and the rank of the augmented matrix is 4.
iii. Since the rank of the augmented matrix is greater than the rank of the coefficient matrix, the
system is inconsistent.
iv. The system is a set of four planes in R3 which have no common point of intersection.
2.2.2 (a) We need to determine if there exists c1 , c2 ∈ R such that
       
1  1   3   c1 + 3c2 
3 = c1  2  + c2 −4 =  2c1 − 4c2 
       
1 −1 2 −c1 + 2c2

Hence, we need to solve the system of equations

c1 + 3c2 = 1
2c1 − 4c2 = 3
−c1 + 2c2 = 1

We row reduce the corresponding augmented matrix to get


     
 1 3 1   1 3 1   1 3 1 
2 −4 3 R − 2R ∼ 0 −10 1  R2 + 2R3 ∼ 0 0 5 
     
  2 1 
−1 2 1 R3 + R1 0 5 2 0 5 2

The second row corresponds to the equation 0 = 5. Thus, the system is inconsistent, so ~x < Span B.
Section 2.2 Solutions 37

(b) We need to determine if there exists c1 , c2 ∈ R such that


       
1  1   3   c1 + 3c2 
0 = c1  2  + c2 −4 =  2c1 − 4c2 
       
0 −1 2 −c1 + 2c2
Hence, we need to solve the system of equations
c1 + 3c2 = 1
2c1 − 4c2 = 0
−c1 + 2c2 = 0
We row reduce the corresponding augmented matrix to get
       
 1 3 1   1 3 1   1 3 1  R1 − 3R2  1 0 2/5 
 2 −4 0  R2 − 2R1 ∼  0 −10 −2  − 1 R2 ∼ 0 1 1/5  ∼  0 1 1/5 
 
    10  
−1 2 0 R3 + R1 0 5 1 0 5 1 R3 − 5R2 0 0 0
 

2
Hence, c1 = 5 and c2 = 51 , and so ~x ∈ Span B. We can verify this answer by checking that
     
1 1 3
  2   1  
0 =  2  + −4
5 5
0 −1 2
(c) We need to determine if there exists c1 , c2 , c3 ∈ R such that
−1  3c1 + 2c2 − c3 
         
 1  3  2 
−3 1  1   0   c1 + c2 
  = c   + c   + c   =  
−7 1 2 2  3  3 −4 2c1 + 3c2 − 4c3 
         
−9 3 −4 3 3c1 − 4c2 + 3c3
Hence, we need to solve the system of equations
3c1 + 2c2 − c3 = 1
c1 + c2 = −3
2c1 + 3c2 − 4c3 = −7
3c1 − 4c2 + 3c3 = −9
We row reduce the corresponding augmented matrix to get
2 −1 0 −3 

 3
  
1  1 1
 R1 ↔ R2  
 1 1 0 −3   3 2 −1 1  R2 − 3R1
 ∼ ∼
 2 3 −4 −7   2 3 −4 −7  R3 − 2R1
 
   
3 −4 3 −9 3 −4 3 −9 R4 − 3R1
0 −3  R1 + R2 0 −1 0 −1
     
 1 1  1 7   1 7 
 0 −1 −1 10   0 −1 −1 10 
  0 −1 −1 10 
  ∼  ∼ 
 0 1 −4 −1  R3 + R2  0 0 −5 9  0 0 −5 9
 
     
0 −7 3 0 R4 − 7R2 0 0 10 −70 R4 + 2R3 0 0 0 −52
The fourth row corresponds to the equation 0 = −52. Thus, the system is inconsistent, so ~x <
Span B.
38 Section 2.2 Solutions

(d) We need to determine if there exists c1 , c2 , c3 ∈ R such that


         
 1   5  −6  6   5c1 − 6c2 + 6c3 
−7 = c1 −3 + c2  3  + c3 −6 = −3c1 + 3c2 − 6c3 
         
−9 4 −4 8 4c1 − 4c2 + 8c3
Hence, we need to solve the system of equations
5c1 − 6c2 + 6c3 = 1
−3c1 + 3c2 − 6c3 = −7
4c1 − 4c2 + 8c3 = −9
We row reduce the corresponding augmented matrix to get
   
 5 −6 6 1   5 −6 6 1 
 −3 3 −6 −7  ∼  −3 3 −6 −7
   

4 −4 8 −9 R3 + 43 R2 0 − 55
 
0 0 3

The third row corresponds to the equation 0 = −55/3. Thus, the system is inconsistent, so ~x <
Span B.
2.2.3 (a) Consider          
0  1  1  1  2
0 = c 2 + c 3 + c −1 + c 1
         
  1   2   3   4
0 −1 1 4 1
Observe that this will give 3 equations in 4 unknowns. Hence, the rank of the matrix is at most 3,
so by Theorem 2.2.5, the system must have at least 4 − 3 = 1 free variable. Therefore, the set is
linearly dependent.
(b) Consider          
0  1   2   3   c1 + 2c2 + 3c3 
0 = c1  0  + c2  1  + c3  3  =  c2 + 3c3 
         
0 −1 −2 −1 −c1 − 2c2 − c3
This corresponds to the homogeneous system
c1 + 2c2 + 3c3 = 0
c2 + 3c3 = 0
−c1 − 2c2 − c3 = 0
To find the solution space of the system, we row reduce the corresponding coefficient matrix.
   
 1 2 3   1 2 3  R1 − 2R2
 0 1 3  ∼  0 1 3  ∼
  
 1
−1 −2 −1 R3 + R1 0 0 2 R
  
2 3
   
 1 0 −3  R1 + 3R3  1 0 0 
 0 1 3  R2 − 3R3 ∼  0 1 0 
  

0 0 1 0 0 1
  

Hence, the only solution to the system is c1 = c2 = c3 = 0. Therefore, the set is linearly indepen-
dent.
Section 2.2 Solutions 39

(c) Consider          
0 1 4 7  c1 + 4c2 + 7c3 
0 = c1 2 + c2 5 + c3 8 = 2c1 + 5c2 + 8c3 
         
0 3 6 9 3c1 + 6c2 + 9c3
This corresponds to the homogeneous system

c1 + 4c2 + 7c3 = 0
2c1 + 5c2 + 8c3 = 0
3c1 + 6c2 + 9c3 = 0

To find the solution space of the system, we row reduce the corresponding coefficient matrix.
   
 1 4 7   1 4 7 
 2 5 8  R2 − 2R1 ∼  0 −3 −6  ∼
   
3 6 9 R3 − 3R1 0 −6 −12 R3 − 2R2
 
 1 4 7 
 0 −3 −6 
 
0 0 0

Hence, we see that there will be a free-variable. Consequently, the homogeneous system has
infinitely many solutions, and so the set is linearly dependent.
(d) Consider          
0  5  −6  6   5c1 − 6c2 + 6c3 
0 = c1 −3 + c2  3  + c3 −6 = −3c1 + 3c2 − 6c3 
         
0 4 −4 8 4c1 − 4c2 + 8c3
This corresponds to the homogeneous system

5c1 − 6c2 + 6c3 = 0


−3c1 + 3c2 − 6c3 = 0
4c1 − 4c2 + 8c3 = 0

From our work in Problem 2.2.2 (d), we know the rank of the coefficient matrix is 2. Thus, there
is one free variable and so the homogeneous system has infinitely many solutions. Thus, the set is
linearly dependent.
(e) Consider          
0 3 2 1  3c1 + 2c2 + c3 
0 = c1 1 + c2 1 + c3 3 =  c1 + c2 + 3c3 
         
0 2 3 2 2c1 + 3c2 + 2c3
This corresponds to the homogeneous system

3c1 + 2c2 + c3 = 0
c1 + c2 + 3c3 = 0
2c1 + 3c2 + 2c3 = 0
40 Section 2.2 Solutions

We row reduce the corresponding coefficient matrix to get


   
 3 2 1   1 1 3 
 R1 ↔ R2 
 1 1 3  ∼  3 2 1  R2 − 3R1 ∼
 
2 3 2 2 3 2 R3 − 2R1
 
   
 1 1 3  R1 + R2  1 0 −5 
0 −1 −8 ∼ 0 −1 −8 (−1)R2 ∼
   
   
1
0 1 −4 R3 + R2 0 0 −12 − 12 R3
   
 1 0 −5  R1 + 5R3  1 0 0 
 0 1 8  R2 − 8R3 ∼  0 1 0 
  

0 0 1 0 0 1
  

Thus, we see the rank of the coefficient matrix is 3 and hence the solution is unique. Consequently,
the set is linearly independent.
(f) Consider
        
0 3

 6   0   3c1 + 6c2 
0
  = c1 1 + c2  4  + c3  2  =  c1 + 4c2 + 2c3 
       
0 3 −3 −1  3c1 − 3c2 − c3 
         
0 5 4 8 5c1 + 4c2 + 8c3
This corresponds to the homogeneous system

3c1 + 6c2 = 0
c1 + 4c2 + 2c3 = 0
3c1 − 3c2 − c3 = 0
5c1 + 4c2 + 8c3 = 0

We row reduce the corresponding coefficient matrix to get


  1  
 3 6 0 
 3 R1  1 2 0 
 1 4 2   1 4 2  R − R
2 1
 ∼   R3 − 3R1 ∼

 3 −3 −1 
   3 −3 −1 
5 4 8 5 4 8 R4 − 5R1
0  R1 − 2R2
   
 1 2 0 
  1 2

 0 2 2   0 1 1 
1
2 R2 ∼  ∼
 
 0 −9 −1   0 −9 −1  R3 + 9R2

   
0 −6 8 0 −6 8 R4 + 6R2
 1 0 −2  −2  R1 + 2R3
     
 1 0   1 0 0 
 0 1 1   0 1 1  R2 − R3  0 1 0 

 0 0 ∼  ∼ 
8   0 0 1  0 0 1
  
1
8 R3
    
0 0 14 0 0 14 R4 − 14R3 0 0 0

Thus, we see the rank of the coefficient matrix is 3 and hence the solution is unique. Consequently,
the set is linearly independent.
Section 2.2 Solutions 41

2.2.4 (a) Consider      


 x1   5  −4
 x2  = c1 −3 + c2  1 
     
x3 7 6
This corresponds to a system of 3 equations in 2 unknowns. Therefore, the rank of the system is
less than 3. Hence, by Theorem 2.2.5, the system cannot be consistent for all ~x ∈ R3 . Thus, the
set does not span R3 , and so it is not a basis for R3 .
(b) Consider          
0  1   1   6   2 
0 = c1 −1 + c2  0  + c3 −2 , +c4 −1
         
0 3 −1 5 4
Observe that this will give 3 equations in 4 unknowns. Hence, the rank of the matrix is at most 3.
So, by Theorem 2.2.5, the system must have at least 4 − 3 = 1 free variable. Therefore, the set is
linearly dependent, and so it is not a basis for R3 .
(c) Consider          
0  1   2  4 c1 + 2c2 + 4c3 
0 = c1  3  + c2 −5 + c3 1 = 3c1 − 5c2 + c3 
         
0 −2 3 0 −2c1 + 3c2
This gives the homogeneous system

c1 + 2c2 + 4c3 = 0
3c1 − 5c2 + c3 = 0
−2c1 + 3c2 = 0

Row reducing the coefficient matrix we get


     
 1 2 4   1 2 4   1 2 4  R1 − 2R2
 1
3 −5 1  R2 − 3R1 ∼  0 −11 −11  − 11 R2 ∼ 0 1 1  ∼
    

−2 3 0 R3 + 2R1 0 7 8 0 7 8 R3 − 7R2
 
   
 1 0 2  R1 − 2R3  1 0 0 
 0 1 1  R2 − R3 ∼  0 1 0 
  

0 0 1 0 0 1
  

Hence, the rank of the coefficient matrix is 3. Therefore, the only solution to the homogeneous
system is c1 = c2 = c3 = 0, so the set is linearly independent.
h i Moreover, since the rank equals the
number of rows, by Theorem 2.2.5 (3), the system A | ~b is consistent for every ~b ∈ R3 , so the set
also spans R3 . Therefore, it is a basis for R3 .
(d) Consider          
0 1  2   1  c1 + 2c2 + c3 
0 = c1 1 + c2  1  + c3  0  =  c1 + c2 
         
0 1 −1 −3 c1 − c2 − 3c3
42 Section 2.2 Solutions

This gives the homogeneous system

c1 + 2c2 + c3 = 0
c1 + c2 = 0
c1 − c2 − 3c3 = 0

Row reducing the coefficient matrix we get


     
 1 2 1   1 2 1   1 2 1  R1 − 2R2
 1 1 0 R − R ∼ 0 −1 −1  (−1)R2 ∼ 0 1 1 ∼
     
 2 1  
1 −1 −3 R3 − R1 0 −3 −4 0 −3 −4 R3 + 3R2
 
     
 1 0 −1   1 0 −1  R1 + R3  1 0 0 
 0 1 1  ∼  0 1 1  R2 − R3 ∼  0 1 0
    
 
0 0 −1 (−1)R3 0 0 1 0 0 1
   

Hence, the rank of the coefficient matrix is 3. Therefore, the only solution to the homogeneous
system is c1 = c2 = c3 = 0, so the set is linearly independent.
h i Moreover, since the rank equals the
number of rows, by Theorem 2.2.5 (3), the system A | b is consistent for every ~b ∈ R3 , so the set
~
also spans R3 . Therefore, it is a basis for R3 .
(e) Consider          
0  1   3  3  c1 + 3c2 + 3c3 
0 = c1  0  + c2  3  + c3 6 =  3c2 + 6c3


        
0 −4 −5 2 −4c1 − 5c2 + 2c3
This gives the homogeneous system

c1 + 3c2 + 3c3 = 0
3c2 + 6c3 = 0
−4c1 − 5c2 + 2c3 = 0

Row reducing the coefficient matrix we get


     
 1 3 3   1 3 3  R1 − 3R2  1 0 −3 
1
0 3 6  3 R2 ∼  0 1 2 ∼ 0 1 2 
     
 
−4 −5 2 R3 + 4R1 0 7 14 R3 − 7R2 0 0 0
   

Hence, the rank of the coefficient matrix is 2. Therefore, the homogeneous system has infinitely
many solution and so the set is linearly dependent. Consequently, it is not a basis.
(f) Consider          
0  3  4  2  3c1 + 4c2 + 2c3 
0 = c1  2  + c2 4 + c3  1  =  2c1 + 4c2 + c3 
         
0 −3 0 −2 −3c1 − 2c3
Section 2.2 Solutions 43

This gives the homogeneous system

3c1 + 4c2 + 2c3 = 0


2c1 + 4c2 + c3 = 0
−3c1 − 2c3 = 0

Row reducing the coefficient matrix we get


   
 3 4 2  R1 − R2  1 0 1 
2 4 1 ∼  2 4 1  R2 − 2R1 ∼
   
 
−3 0 −2 −3 0 −2 R3 + 3R1

   
 1 0 1  R1 − 3R2  1 0 0 
1
 0 4 −1  R2 + R3 ∼  0 4 0  4 R2 ∼
   
0 0 1 0 0 1
  
 
 1 0 0 
 0 1 0 


0 0 1

Hence, the rank of the coefficient matrix is 3. Therefore, the only solution to the homogeneous
system is c1 = c2 = c3 = 0, so the set is linearly independent.
h i Moreover, since the rank equals the
number of rows, by Theorem 2.2.5 (3), the system A | ~b is consistent for every ~b ∈ R3 , so the set
also spans R3 . Therefore, it is a basis for R3 .
h i
2.2.5 Let {~v1 , . . . , ~vn } be a set of n-vectors in Rn and let A = ~v1 · · · ~vn .

Assume that {~v1 , . . . , ~vn } is linearly independent. Then, ~


h i system c1~v1 + · · · + cn~vn = 0 has a unique
the
solution. But, this system can be represented by A | ~0 . Thus, by Theorem 2.2.5 (2), rank(A) = n.
h i
Hence, A | ~b is consistent for all ~b ∈ Rn by Theorem 2.2.5 (3). Therefore, for every ~b ∈ Rn there exists
a ~x ∈ Rn such that ~b = x1~v1 + · · · + xn~vn . Consequently, Span{~v1 , . . . , ~vn } = Rn .
h i
On the other hand, if Span{~v1 , . . . , ~vn } = Rn , then A | ~b is consistent for all ~b ∈ Rn , so rank(A) = n by
Theorem 2.2.5 (3). Thus, {~v1 , . . . , ~vn } is linearly independent by Theorem 2.2.5 (2).
(" # " #)
a b
2.2.6 Assume that B = , is a basis for R2 . Then, B must be linear independent. Hence, the only
c d
solution to " # " # " #
0 a b
= t1 + t2
0 c d
" #
a b
is t1 = t2 = 0. Thus, the coefficient matrix of the corresponding system must have rank 2. So, we
c d
cannot have a = 0 = c. Assume that a , 0. Row reducing we get
" # " # " #
a b a b a b
∼ ∼
c d R2 − ac R1 0 d − bc a aR2 0 ad − bc

Since we need rank A = 2, we must have ad − bc , 0 as required. The case of c , 0 is similar.


44 Section 2.2 Solutions

On the other hand, if ad − bc , 0, then at least one of a and c are non-zero. Hence, we can row reduce
as in the first case to find that the rank of the coefficient matrix is 2. Then, Theorem 2.2.5 implies that B
is a basis for R2 .
2.2.7 Since there is no point in common between the three planes, we know that rank A < rank[A | ~b]. But the
planes are not parallel, so rank A > 1, and there are only three rows of [A | ~b], so rank[A | ~b] ≤ 3. It
follows that rank A = 2.
2.2.8 The intersection of two planes in R3 is represented by a system of 2 linear equations in 3 unknowns.
This will correspond to a system whose coefficient matrix has 2 rows and 3 columns. Hence, the rank
of the coefficient matrix of the system is either 1 or 2. Since we are given that the system is consistent,
Theorem 2.2.5(2) tells us that the solution set has either 1 or 2 free variables. Theorem 2.2.6 tells us that
the vectors corresponding to the free variables must be linearly independent. Thus, the solution set is
either a line or a plane in R3 .
2.2.9 (a) The statement is false. The solution set of x1 + x2 = 1 is not a subspace of R2 , since it does not
contain the zero vector.
(b) The statement is false. The system x1 + x2 + x3 + x4 + x5 = 1, x1 + x2 + x3 + x4 + x5 = 2,
x1 + x2 + x3 + x4 + x5 = 3, does not have infinitely many solutions. It is not consistent.
(c) The statement is false. The system x1 + x2 + x3 = 1, 2x1 + 2x2 + 2x3 = 2, 3x1 + 3x2 + 3x3 = 3,
4x1 + 4x2 + 4x3 = 4, 5x1 + 5x2 + 5x3 = 5 has infinitely many solutions.
(d) The statement is true. We know that a homogeneous system is always consistent and the maximum
rank is the minimum of the number of equation and the number of variables. Hence, the rank of
the coefficient matrix is at most 3. Therefore, there are at least 5 − 3 = 2 free variables in the
system. Thus, there must be infinitely many solutions.
(e) The statement is false. The system x1 + x2 + x3 + x4 = 1, x1 + x2 + x3 + x4 = 2, x2 + x3 + x4 = 0,
has rank 2, but it is inconsistent, so the solution set is not a plane.
(f) The statement is true. Since the rank equals the number of equations, the system must be consistent
by Theorem 2.2.5. Then, by Theorem 2.2.6, a vector equation for the solution set must have the
form
~x = ~c + t1~v1 + t2~v2 , t1 , t2 ∈ R
where {~v1 , ~v2 } is a linearly independent set in R5 . Therefore, the solution set is a plane in R5 .
(g) The statement is true. If the solution space is a line, then it must have the form ~x = t~v1 , t ∈ R,
where ~v1 , ~0. Therefore, the system must have had 1 free variable. Hence, by Theorem 2.2.5, we
get the rank of the coefficient matrix equals the number of variables - the number of free variables.
Hence, the rank is 3 − 2 = 1.
(h) The statement is false. Consider the system x1 + x2 + x3 = 1, 2x1 + 2x2 + 2x3 = 2. The system
has m = 2 equations and n = 3 variables, but the rank of the corresponding coefficient matrix A is
1 , m.
h i
(i) If A | ~b is consistent for every ~b ∈ Rm , then rank A = m by Theorem 2.2.5(3). Since the solution
is unique, we have that rank A = n by Theorem 2.2.5(2). Thus, m = rank A = n.
Section 2.2 Solutions 45

hh i i
2.2.10 Assume that B is linearly independent. Then, the only solution to the system ~v1 · · · ~vk | ~0 is the
trivial solution. Hence, by Theorem 2.2.5, the rank of the coefficient matrix must equal the number of
columns which is k.
h i hh i i
On the other hand, if the rank of ~v1 · · · ~vk is k, then ~v1 · · · ~vk | ~0 has a unique solution by
Theorem 2.2.5. Thus, B is linearly independent.
2.2.11 (a) We have that {~v1 , . . . , ~vk } spans Rn if and only if the equation c1~v1 + · · · + ck~vk = ~b is consistent for
every ~b ∈ Rn . The equation c1~v1 + · · · + ck~vk = ~b ishconsistent
i for every ~b ∈ Rn if and only if the
corresponding system of n equation in k unknowns A | ~b is consistent for every ~b ∈ Rn . Finally,
h i
by Theorem 2.2.5(3) the corresponding system of n equation in k unknowns A | ~b is consistent
for every ~b ∈ Rn if and only if rank A = n.
(b) If k < n, then the coefficient matrix A of the system c1~v1 + · · · + ck~vk = ~b must have rank A < n
since the rank is the number of leading ones and there cannot be more leading ones than rows. But
then, we have Span{~v1 , . . . , ~vk } = Rn and rank A < n which contradicts part (a). Hence, k ≥ n.
2.2.12 (a) Since ~ui ∈ S and Span{~v1 , . . . , ~vℓ } = S, we can write each ~ui as a linear combination of ~v1 , . . . , ~vℓ .
Say,

~u1 = a11~v1 + · · · + a1ℓ~vℓ


..
.
~uk = ak1~v1 + · · · + akℓ~vℓ

Consider
c1~u1 + · · · + ck~uk = ~0
Substituting, we get
~0 = c1 (a11~v1 + · · · + a1ℓ~vℓ ) + · · · + ck (ak1~v1 + · · · + akℓ~vℓ )
0~v1 + · · · + 0~vℓ = (c1 a11 + · · · + ck ak1 )~v1 + · · · + (c1 a1ℓ + · · · + ck akℓ )~vℓ

Comparing coefficients of ~vi on both sides we get the system of equations

a11 c1 + · · · + ak1 ck = 0
..
.
a1ℓ c1 + · · · + akℓ ck = 0

This homogeneous system of ℓ equations in k unknowns (c1 , . . . , ck ) must have a unique solution as
otherwise, we would contradict the fact that {~u1 , . . . , ~uk } is linearly independent. By Theorem 2.2.5
(2), the rank of the matrix must equal the number of variables k. Therefore, ℓ ≥ k as otherwise
there would not be enough rows to contain the k leadings ones in the RREF of the coefficient
matrix.
(b) If {~v1 , . . . , ~vℓ } is a basis, then Span{~v1 , . . . , ~vℓ } = S. If {~u1 , . . . , ~uk } is a basis, then it is linearly
independent. Hence, by part (a), k ≤ ℓ. Similarly, we also have that {~u1 , . . . , ~uk } spans S and
{~v1 , . . . , ~vℓ } is linearly independent, so ℓ ≤ k. Therefore, k = ℓ.
46 Section 2.2 Solutions

h i
2.2.13 The i-th equation of the system A | ~b has the form

ai1 x1 + · · · + ain xn = bi
h i
If ~y is a solution of A | ~0 , then we have

ai1 y1 + · · · + ain yn = 0
h i
If ~z is a solution of A | ~b , then we have

ai1 z1 + · · · + ain zn = bi

So, if we take ~x = ~z + c~y we get

ai1 (z1 + cy1 ) + · · · + ain (zn + cyn ) = ai1 z1 + · · · + ain zn + c(ai1 y1 + · · · + ain yn )
= bi + c(0)
= bi

Thus, ~x = ~z + c~y is a solution for each c ∈ R.


2.2.14 (a) We have

f1 + f2 = 50
f2 + f3 + f5 = 60
f4 + f5 = 50
f1 − f3 + f4 = 40

(b) We row reduce the augmented matrix corresponding to the system in part (a).

 R1 − R2
   
 1 1 0 0 0 50   1 1 0 0 0 50
 0 1 1 0 1 60   0 1 1 0 1 60 

 0 0
 ∼   ∼
 0 1 1 50 
  0 0 0 1 1 50 

1 0 −1 1 0 40 R4 − R1 0 −1 −1 1 0 −10 R4 + R2
 1 0 −1 0 −1 −10  0 −1 0 −1 −10 
   
 1 
 0 1 1 0 1 60   0 1 1 0 1 60 

 0 0 ∼ 
0 1 1 50   0 0 0 1 1 50 
 
  
0 0 0 1 1 50 R4 − R3 0 0 0 0 0 0

Rewriting this into equation form gives

f1 − f3 − f5 = −10
f2 + f3 + f5 = 60
f4 + f5 = 50
Section 2.2 Solutions 47

We see that f3 and f5 are free variables. Thus, we get that the general solution is

 f1  −10 + f3 + f5  −10  1   1 


         
 f2   60 − f3 − f5   60  −1
 
−1
 
       
 f3  =  f3  =  0  + f3  1  + f5  0  , f3 , f5 ∈ R
 f4   50 − f5   50   0  −1
         
f5 f5 0 0 1

(c) For f1 to be non-negative, we require that f3 + f5 ≥ 10. For f2 to be non-negative, we require


f3 + f5 ≤ 60. For f4 to be non-negative we require f5 ≤ 50.
Chapter 3 Solutions

3.1 Problem Solutions


" # " # " #
2 1 3 −1 7 1
3.1.1 (a) 2 + =
−3 4 0 2 −6 10
" # " # " #
1 1 3 1 4 8 −2/3 −1
(b) − =
3 1 −6 4 1/2 3 5/24 −11/4
" # " # "√ √ √ #
1 2 4 1 6 0 2 + 2 3 √ 2 2√
(c) √ + √ =
2 0 2 3 0 6 0 2+2 3
" # " # " #
3 3 −2 −2 0 0
(d) −2 −3 =
0 −1 0 2 0 −4
" # " # " # " #
1 0 0 1 0 0 x y
(e) x +y +z =
0 0 0 0 1 0 z 0
" # " # " #
−4 3 9 −8 0 0
(f) 0 +0 =
3 5 1 5 0 0
" #" # " #
3 1 2 −2 6 −5
3.1.2 (a) =
2 1 0 1 4 −3
(b) The product doesn’t exist since the number of rows of the second matrix does not equal the number
of columns of the first matrix.
    
1 2 3 1 1
(c) 4 5 6 0 = 4
7 8 9 0 7
    
    
1 2 3 0 0 2 3
(d) 4 5 6 1 0 = 5 6
    
7 8 9 0 1 8 9
    
     
1 5  " #T 1 5  " #  8 7
 3 1   3 2
  
(e) 3 2  = 3 2  = 11 8
  
 2 1  1 1
3 −2 3 −2 7 4
   
    
1 4 −1 4 1 −1  1 0 0 
(f) 2 1 0  0 0 0  =  8 2 −2
    
3 1 1 3 1 1 15 4 −2
    

48
Section 3.1 Solutions 49

(g) The product doesn’t exist since the number of rows of the second matrix does not equal the number
of columns of the first matrix.
 
" #  5 −3 " #
−1 2 0   −1 17
(h) 2 7  =

2 1 1  11 −2
−1 −3

 T    
2  1  h i  1 
(i) 3 −1 = 2 3 5 −1 = 4
     
5 1 1
     
   
 5 −3 " # −11 7 −3
 −1 2 0  
(j)  2 7  =  12 11 7 

 2 1 1
−1 −3 −5 −5 −3
  
   T    
2  1  2 h i 2 −2 2
(k) 3 −1 = 3 1 −1 1 = 3 −3 3
      
5 1 5 5 −5 5
      
 T    
2  x1  h i  x1 
(l) 3  x2  = 2 3 5  x2  = 2x1 + 3x2 + 5x3
     
5 x3 x3
     

3.1.3 (a) We have


n
X n
X n
X n
X
tr(A + B) = (A + B)ii = [(A)ii + (B)ii ] = (A)ii + (B)ii = tr A + tr B
i=1 i=1 i=1 i=1

(b) We have
n
X X n
n X n X
X n
tr(AT B) = (AT B)ii = (AT )i j (B) ji = (A) ji (B) ji
i=1 i=1 j=1 i=1 j=1
 
" #  x1  " #
3 2 −1 4
3.1.4 We take A = , ~x =  x2 , and ~b = .
 
2 −1 5 5
x3
 

3.1.5 Simplifying the left side we get


" # " #
3c1 + 2c2 + c3 c1 + c2 + c3 0 0
=
3c1 + 2c2 + c3 c1 − c2 − 2c3 0 0

Comparing entires we get the homogeneous system of linear equations

3c1 + 2c2 + c3 = 0
c1 + c2 + c3 = 0
3c1 + 2c2 + c3 = 0
c1 − c2 − 2c3 = 0
50 Section 3.1 Solutions

Row reducing the corresponding coefficient matrix gives


   
3 2 1  1
 
0 0

1 1 1  0 1 0

3 2 ∼
1  0 0 1

   
1 −1 −2 0 0 0

Thus, the only solution is c1 = c2 = c3 = 0.


3.1.6 Simplifying the left side we get
" # " #
t1 + t2 + 2t3 2t1 + t2 + 3t3 −1 −4
=
−2t1 + t3 −t1 − t2 + t3 3 −2

Comparing entires we get the homogeneous system of linear equations

t1 + t2 + 2t3 = −1
2t1 + t2 + 3t3 = −4
−2t1 + t3 = 3
−t1 − t2 + t3 = −2

Row reducing the corresponding augmented matrix gives

2 −1   1 0 0 −2 
   
 1 1
  
 2 1 3 −4   0 1 0 3 

 −2 ∼
0 1 3   0 0 0 −1 

   
−1 −1 1 −2 0 0 0 0

Hence, t1 = −2, t2 = 3, and t3 = −1.


3.1.7 (a) The statement is false. If A~x is defined, then ~x must have the same number of entries as the number
of columns of A. Therefore, we would need ~x ∈ R2 .
(b) The statement is false. A~x gives a linear combination of the columns of A. Since A has 2 rows, the
columns of A are in R2 . Thus, A~x ∈ R2 .
(c) The statement is false. Take B = I3 . Then, by Theorem 3.1.5 we have AB = BA.
" # " #
0 1 2 0 0
(d) The statement if false. Take A = . Then A = .
0 0 0 0
" # " #
0 1 0 0
(e) The statement if false. Take A = = B. Then AB = , but neither A nor B is the zero
0 0 0 0
matrix.
3.1.8 We have (A + B)i j = (A)i j + (B)i j = (B)i j + (A)i j = (B + A)i j .
Section 3.2 Solutions 51

h i
3.1.9 Let A = ~a1 · · · ~an . We have
h i h i h i
AIn = A ~e1 · · · ~en = A~e1 ··· A~en = ~a1 ··· ~an = A

Using this result for the n × m matrix AT , we get

AT = AT Im = AT ImT = (Im A)T

Taking transposes of both sides gives A = Im A.


3.1.10 By definition of matrix-vector multiplication we get that

A~ei = 0~a1 + · · · + 0~ai−1 + 1~ai + 0~ai+1 + · · · + 0~an = ~ai

as required.
 
A1 
(a) Since A has a row of zeros, we can write A in block form as A = ~0T . Thus,
 
3.1.11
A2
 

     
A1  A1 B A1 B
AB = ~0T  B = ~0T B =  ~0T 
     
A2 A2 B A2 B
     

Hence, AB has a row of zeros.


(b) B could be the zero matrix.
3.1.12
3.1.13 (a)
(b)
(c)
3.1.14 (a)
(b)
(c)
3.1.15 (a)
(b)
52 Section 3.2 Solutions

3.2 Problem Solutions


3.2.1 (a) Since A has 2 columns for A~x to be defined we must have ~x ∈ R2 . Thus, the domain of L is R2 .
Moreover, since A has 4 rows, we get that A~x ∈ R4 . Thus, the codomain of L is R4 .
(b) We have

−2 3  " # −19


   
 3 0  2  6 

L(2, −5) =  =  
 1 5  −5 −23


4 −6 38
−2 3  " #  18 
   
 3 0  −3  −9 
L(−3, 4) =  =  
 1 5  4 17
 
 
 
4 −6 −36

(c) We have

−2 3  " # −2


   
 3 0  1
  3 
L(1, 0) =  =  
 1 5  0  1 


4 −6 4
−2 3  " #  3 
   
 3 0  0  0 
L(0, 1) =  =  
 1 5  1  5 


4 −6 −6

Thus, [L] = A.
3.2.2 (a) Since B has 4 columns for B~x to be defined we must have ~x ∈ R4 . Thus, the domain of f is R4 .
Moreover, since B has 3 rows, we get that B~x ∈ R3 . Thus, the codomain of f is R3 .
(b) We have
 
  2   
1 2 −3 0    −11
−2
f (2, −2, 3, 1) = 2 −1 0
 
3    =  9 
   
  3  
1 0 2 −1   7
 
1
 −3 
 
 
1 2 −3 0    −13
1
f (−3, 1, 4, 2) = 2 −1 0 3    =  −1 
     
  4  
1 0 2 −1   3
 
2
Section 3.2 Solutions 53

(c) We have
 
1
f (~e1 ) = B~e1 = 2
 
1
 
 
 2 
f (~e2 ) = B~e2 = −1
 
0
 
 
−3
f (~e3 ) = B~e3 =  0 
 
2
 
 
 0 
f (~e4 ) = B~e4 =  3 
 
−1
 

Thus, [ f ] = B.
3.2.3 (a) Let ~x, ~y ∈ R3 and s, t ∈ R. Then,

L(s~x + t~y) = L(sx1 + ty1 , sx2 + ty2 , sx3 + ty3 ) = ([sx1 + ty1 ] + [sx2 + ty2 ], 0)
= s(x1 + x2 , 0) + t(y1 + y2 , 0)
= sL(~x) + tL(~y)

Hence, L is linear. We have


" # " # " #
1 1 0
L(1, 0, 0) = L(0, 1, 0) = L(0, 0, 1) =
0 0 0

Hence, # "
h 1 1 0 i
[L] = L(1, 0, 0) L(0, 1, 0) L(0, 0, 1) =
0 0 0

(b) Let ~x, ~y ∈ R3 and s, t ∈ R. Then,

L(s~x + t~y) = L(sx1 + ty1 , sx2 + ty2 , sx3 + ty3 ) = ([sx1 + ty1 ] − [sx2 + ty2 ], [sx2 + ty2 ] + [sx3 + ty3 ])
= s(x1 − x2 , x2 + x3 ) + t(y1 − y2 , y2 + y3 )
= sL(~x) + tL(~y)

Hence, L is linear. We have


" # # " " #
1 −1 0
L(1, 0, 0) = L(0, 1, 0) = L(0, 0, 1) =
0 1 1

Hence,
h i "1 −1 0#
[L] = L(1, 0, 0) L(0, 1, 0) L(0, 0, 1) =
0 1 1
54 Section 3.2 Solutions

(c) Observe that L(0, 0, 0) = (1, 0, 0) and L(1, 0, 0) = (1, 1, 0), so L(0, 0, 0) + L(1, 1, 0) = (2, 1, 0). But,

L[(0, 0, 0) + (1, 0, 0)] = L(1, 0, 0) = (1, 1, 0) , L(0, 0, 0) + L(1, 0, 0)

So, L does not preserve addition, so it is not linear.


(d) Observe that L(1, 1) = (1, 1), so 2L(1, 1) = (2, 2). But, L[2(1, 1)] = L(2, 2) = (4, 4) , 2L(1, 1).
Hence, L does not preserve scalar multiplication, so L is not linear.
(e) Let ~x, ~y ∈ R3 and s, t ∈ R. Then,

L(s~x + t~y) = L(sx1 + ty1 , sx2 + ty2 , sx3 + ty3 ) = (0, 0, 0)


= s(0, 0, 0) + t(0, 0, 0) = sL(~x) + tL(~y)

Hence, L is linear. We have


     
0 0 0
L(1, 0, 0) = 0 L(0, 1, 0) = 0 L(0, 0, 1) = 0
     
0 0 0
     

Hence,  
h 0 0 0
i
[L] = L(1, 0, 0) L(0, 1, 0) L(0, 0, 1) = 0 0 0
 
0 0 0
 

(f) Let ~x, ~y ∈ R4 and s, t ∈ R. Then,

L(s~x + t~y) = L(sx1 + ty1 , sx2 + ty2 , sx3 + ty3 , sx4 + ty4 )
= ([sx4 + ty4 ] − [sx1 + ty1 ], 2[sx2 + ty2 ] + 3[sx3 + ty3 ])
= s(x4 − x1 , 2x2 + 3x3 ) + t(y4 − y1 , 2y2 + 3y3 )
= sL(~x) + tL(~y)

Hence, L is linear. We have


" # " # " # " #
−1 0 0 1
L(1, 0, 0, 0) = L(0, 1, 0, 0) = L(0, 0, 1, 0) = L(0, 0, 0, 1) =
0 2 3 0

Hence,
# "
h −1 0 0 1 i
[L] = L(1, 0, 0, 0) L(0, 1, 0, 0) L(0, 0, 1, 0) L(0, 0, 0, 1) =
0 2 3 0
" #
2
3.2.4 Let ~a = .
2
Section 3.2 Solutions 55

(a) We have
" # " #
~e1 · ~a 2 2 1/2
proj~a (~e1 ) = ~a = =
k~ak2 8 2 1/2
" # " #
~e2 · ~a 2 2 1/2
proj~a (~e2 ) = ~a = =
k~ak2 8 2 1/2

Hence,
h i "1/2 1/2#
[proj~a ] = proj~a (~e1 ) proj~a (~e2 ) =
1/2 1/2

(b) We have " #! " #" # " #


3 1/2 1/2 3 1
proj~a = =
−1 1/2 1/2 −1 1

(c) We have " #! " # " #


3 4 2 1
proj~a = =
−1 8 2 1

3.2.5 We have
     
1  3  −7/11
6   
reflP (~e1 ) = 0 −
   1  = −6/11
11   
0 −1 6/11
  
     
0  3  −6/11
2   
reflP (~e2 ) = 1 −  1  =  9/11 
  
11   
0 −1 2/11
 
     
0  3  6/11
2   
reflP (~e3 ) = 0 +  1  = 2/11
 
11   
1 −1 9/11
  

Hence,  
−7/11 −6/11 6/11
[reflP ] = −6/11 9/11 2/11
 
6/11 2/11 9/11
 

3.2.6 Let ~x, ~y ∈ Rn and s, t ∈ R. Then

perp~a (s~x + t~y) = (s~x + t~y) − proj~a (s~x + t~y)


= s~x + t~y − [s proj~a (~x) + t proj~a (~y)] since proj~a is linear
= s[~x − proj~a (~x)] + t[~y − proj~a (~y)]
= s perp~a (~x) + t perp~a (~y)

Therefore, perp~a is a linear mapping.


56 Section 3.2 Solutions

3.2.7 Let ~x, ~y ∈ Rn and s, t ∈ R. Then

DOT~v (s~x + t~y) = (s~x + t~y) · ~v


= s(~x · ~v) + t(~y · ~v)
= s DOT~v (~x) + t DOT~v (~y)

Therefore, DOT~v is a linear mapping.


3.2.8 L(~0) = L(0~x) = 0L(~x) = ~0.
3.2.9 (a) Since L has 4 columns, the domain of L is R4 .
(b) Since L has 2 rows, the codomain of L is R2 .
(c) We have
 
" #  x1  " #
1 3 2 1  x2  x1 + 3x2 + 2x3 + x4
L(~x) = [L]~x =  =
−1 3 0 5  x3  −x1 + 3x2 + 5x4
 
x4
3.2.10 Consider the rotation Rπ/3 : R2 → R2 .
" # " √ #
cos π/3 − sin π/3 1/2 − 3/2

(a) We have [Rπ/3 ] = = .
sin π/3 cos π/3 3/2 1/2
(b) We have
" √ # " √ #" √ # " √ √ #
√ √ 1/ √2 1/2 − 3/2 1/ √2 (1√− 3)/2 √2
Rπ/3 (1/ 2, 1/ 2) = [Rπ/3 ] = √ =
1/ 2 3/2 1/2 1/ 2 ( 3 + 1)/2 2

 
 3
h 1 
i
3.2.11 (a) The standard matrix of a such a linear mapping would be [L] = L(1, 0) L(0, 1) = −1 −5.
 
4 9
 
Hence, a linear mapping would be
 
 3x1 + x2 
L(~x) = [L]~x = −x1 − 5x2 
 
4x1 + 9x2
 

 2 
 3x1 + x2 
(b) We could take L(x1 , x2 ) = −x1 − 5x2 . Then, L(1, 0) = (3, −1, 4), L(0, 1) = (1, −5, 9), but L is not
 
4x1 + 9x2
 
linear since L(2, 0) = (12, −2, 8) , 2L(1, 0).
3.2.12 If L is linear, then we would get
!
1 1 1 1 1
L(1, 0) = L [(1, 1) + (1, −1)] = L(1, 1) + L(1, −1) = (2, 3) + (3, 1) = (5/2, 2)
2 2 2 2 2
Section 3.3 Solutions 57

and
!
1 1 1 1 1
L(0, 1) = L [(1, 1) − (1, −1)] = L(1, 1) − L(1, −1) = (2, 3) − (3, 1) = (−1/2, 1)
2 2 2 2 2

Hence, we can define the linear mapping by


"5
x1 − 21 x2
" # #
5/2 −1/2 2
L(~x) = [L]~x = ~x =
2 1 2x1 + x2

3.2.13 (a) Consider


c1~v1 + · · · + ck~vk = ~0
Then,

L(c1~v1 + · · · + ck~vk ) = L(~0)


c1 L(~v1 ) + · · · + ck L(~vk ) = ~0

Thus, c1 = · · · = ck = 0 since {L(~v1 ), . . . , L(~vk )} is linearly independent. Thus, {~v1 , . . . , ~vk } is


linearly independent.
(b) Take L(~x) = ~0. Then, {L(~v1 ), . . . , L(~vk )} contains the zero vector, so it is linearly dependent.
u1 
 

3.2.14 Let ~u =  ... . By definition, the i-th column of [proj~u ] is
 
 
un

~ei · ~u
proj~u (~ei ) = ~u = ui~u
k~uk2

since ~u is a unit vector. Thus,


h i
[proj~u ] = u1~u · · · un~u
 
 u1  h i
= .  u1 · · · un
..u
n

= ~u~uT

3.3 Problem Solutions


3.3.1 (a) i. If ~x ∈ ker(L), then we have
" # " #
0 x1 + x2
= L(x1 , x2 , x3 ) =
0 0
58 Section 3.3 Solutions

Hence, x2 = −x1 . Thus, we have


       
 x1   x1   1  0
~x =  x2  = −x1  = x1 −1 + x3 0 , x1 , x3 ∈ R
       
x3 x3 0 1
       

    

  1  0
   
 
Thus, B =  −1 , 0 spans ker(L) and is clearly linearly independent. Hence, B is a basis



 0 

1
    
for ker(L).
Every vector ~y ∈ Range(L) has the form
" # " #
x1 + x2 1
L(~x) = = (x1 + x2 ) , (x1 + x2 ) ∈ R
0 0
(" #)
1
Hence, C = spans Range(L) and is linearly independent, so it is a basis for Range(L).
0
ii. We have " # " # " #
1 1 0
L(1, 0, 0) = L(0, 1, 0) = L(0, 0, 1) =
0 0 0
" #
1 1 0
Thus, [L] = .
0 0 0
iii. By Theorem 3.3.4, B is a basis for Null([L]). By Theorem 3.3.5, C is a basis for Col([L]).
     

 1 0 
 1 
   

By definition, Row([L]) = Span  1 , 0 . Thus, 1 spans Row([L]) and is clearly lin-
    

 
 
 
   
 
 

 
0 0
 
 
 
 
 0

 
 
 

early independent, so it is a basis for Row([L]).
To find a basis for the left nullspace of [L] we solve the homogeneous system [L]T ~x = ~0.
Row reducing the corresponding coefficient matrix gives
   
1 0 1 0
1 0 ∼ 0 0
   
0 0 0 0

Thus, we have x1 = 0, so every ~x ∈ Null([L]T ) has the form


" # " # " #
x1 0 0
~x = = = x2
x2 x2 1
(" #)
0
Hence, spans Null([L]T ) and is clearly linearly independent, so it is a basis for Null([L]T ).
1
(b) i. If ~x ∈ ker(L), then we have
" # " #
0 x1 − x2
= L(x1 , x2 , x3 ) =
0 x2 + x3
Section 3.3 Solutions 59

Hence, x2 = x1 and x3 = −x2 = −x1 . Thus, we have


     
 x1   x1   1 
~x =  x2  =  x1  = x1  1  , x1 ∈ R
     
x3 −x1 −1
     

  

  1 
 
 
Thus, B =   1  spans ker(L) and is clearly linearly independent. Hence, B is a basis for

 −1 

   

ker(L).
Every vector ~y ∈ Range(L) has the form
" # " # " # " #
x1 − x2 1 −1 0
L(~x) = = x1 + x2 + x3 , x1 , x2 , x3 ∈ R
x2 + x3 0 1 1
(" # " #)
1 0
Hence, C = , spans Range(L) and is linearly independent, so it is a basis for
0 1
Range(L).
ii. We have " # " # " #
1 −1 0
L(1, 0, 0) = L(0, 1, 0) = L(0, 0, 1) =
0 1 1
" #
1 −1 0
Thus, [L] = .
0 1 1
iii. By Theorem 3.3.4 B is a basis for Null([L]). By Theorem 3.3.5 C is a basis for Col([L]).
         

  1  0   1  0
   
     

 
By definition, Row([L]) = Span  −1 , 1 . Thus, −1 , 1 spans Row([L]) and is
 
 
 
 
 
 
 

 
 0  1
     0  1
 

clearly linearly independent, so it is a basis for Row([L]).
To find a basis for the left nullspace of [L] we solve the homogeneous system [L]T ~x = ~0.
Row reducing the corresponding coefficient matrix gives
   
 1 0 1 0
−1 1 ∼ 0 1
   
0 1 0 0

Thus, we have x1 = x2 = 0, so Null([L]T ) = {~0}. Thus, a basis for Null([L]T ) is the empty
set.
(c) i. Observe that for any ~x ∈ R3 , we have
 
0
L(x1 , x2 , x3 ) = 0
 
0
 

So, ker(L) = R3 . Thus, a basis for ker(L) is the standard basis for R3 .
60 Section 3.3 Solutions

Every vector ~y ∈ Range(L) has the form


 
0
L(~x) = 0
 
0
 

Hence, Range(L) = {~0} and so a basis for Range(L) is the empty set.
ii. We have      
0 0 0
L(1, 0, 0) = 0 L(0, 1, 0) = 0 L(0, 0, 1) = 0
     
0 0 0
     
 
0 0 0
Thus, [L] = 0 0 0.
 
0 0 0
 

iii. By Theorem 3.3.4 the standard basis for R3 is a basis for Null([L]). By Theorem 3.3.5 the
empty set is a basis for Col([L]).
     

 0 0 0 
     
. Thus, Row([L]) = {~0} and so a basis for

By definition, Row([L]) = Span  0 , 0 , 0


0 0 0
 

Row([L]) is the empty set.
To find a basis for the left nullspace of [L] we solve the homogeneous system [L]T ~x = ~0.
Clearly, the solution space is R3 . Hence, the standard basis for R3 is a basis for Null([L]T ).
(d) i. If ~x ∈ ker(L), then we have
" # " #
0 x1
= L(x1 , x2 , x3 , x4 ) =
0 x2 + x4
Hence, x1 = 0 and x4 = −x2 . Thus, we have
       
 x1   0   0  0
 x2   x2 
 = x2   + x3 0 , x2 , x3 ∈ R
 1   
~x =   =   
 x3   x3   0 
 
1
 
x4 −x2 −1 0
    


  0  0 

 1  0

 

Thus, B =  , spans ker(L) and is clearly linearly independent. Hence, B is a basis
     
0 1
 

 

 

 

 



     

 −1 0 
 
for ker(L).
Every vector ~y ∈ Range(L) has the form
" # " # " #
x1 1 0
L(~x) = = x1 + (x2 + x4 ) , x1 , (x2 + x4 ) ∈ R
x2 + x4 0 1
(" # " #)
1 0
Hence, C = , spans Range(L) and is linearly independent, so it is a basis for
0 1
Range(L).
Section 3.3 Solutions 61

ii. We have
" # " # " # " #
1 0 0 0
L(1, 0, 0, 0) = L(0, 1, 0, 0) = L(0, 0, 1, 0) = L(0, 0, 0, 1) =
0 1 0 1
" #
1 0 0 0
Thus, [L] = .
0 1 0 1
iii. By Theorem 3.3.4 B is a basis for Null([L]). By Theorem
   3.3.5 C is a basis for Col([L]).
    

 1 0 
 
 1 0 

     
     
0 1

  0 1

 
By definition, Row([L]) = Span  , . Thus, , spans Row([L]) and is clearly
 


 0 0

 
 
 
 





 0 0

 
 
 
 


           
0 1 0 1

  
 
linearly independent. Therefore, it is a basis for Row([L]).
To find a basis for the left nullspace of [L] we solve the homogeneous system [L]T ~x = ~0.
Row reducing the corresponding coefficient matrix gives
   
1 0 1
 
0

0 1 0 1

0 ∼
0 0 0

   
0 1 0 0

Thus, we have x1 = x2 = 0, so Null([L]T ) = {~0}. Thus, a basis for Null([L]T ) is the empty
set.
(e) i. If ~x ∈ ker(L), then we have " # " #
0 x
= L(x1 , x2 ) = 1
0 x2

Hence, x1 = x2 = 0. So, ker(L) = {~0} and so a basis for ker(L) is the empty set.
Every vector ~y ∈ Range(L) has the form
" # " # " #
x1 1 0
L(~x) = = x1 + x2 , x1 , x2 ∈ R
x2 0 1
(" # " #)
1 0
Hence, C = , spans Range(L) and is linearly independent, so it is a basis for
0 1
Range(L).
ii. We have " # " #
1 0
L(1, 0) = L(0, 1) =
0 1
" #
1 0
Thus, [L] = .
0 1
iii. By Theorem 3.3.4 the empty set is a basis for Null([L]). By Theorem 3.3.5 C is a basis for
Col([L]). (" # " #) (" # " #)
1 0 1 0
By definition, Row([L]) = Span , . Thus, , is a basis for Row([L]).
0 1 0 1
62 Section 3.3 Solutions

To find a basis for the left nullspace of [L] we solve the homogeneous system [L]T ~x = ~0. We
see that the only solution is ~x = ~0, so Null([L]T ) = {~0}. Thus, a basis for Null([L]T ) is the
empty set.
(f) L(x1 , x2 , x3 , x4 ) = (x4 − x1 , 2x2 + 3x3 )
i. If ~x ∈ ker(L), then we have
" # " #
0 x4 − x1
= L(x1 , x2 , x3 , x4 ) =
0 2x2 + 3x3
Hence, x4 = x1 and x3 = − 32 x2 . Thus, we have
       
 x1   x1  1  0 
 x   x 
 = x 0 + x  1  , x , x ∈ R
   
~x =  2  =  2
1 2 1 2
 x3  −2x2 /3
 0 −2/3
   
x4 x1 1 0
   


 1  0  


0  1 
 

Thus, B =  , spans ker(L) and is clearly linearly independent. Hence, B is a
     
0 −2/3
 

 

 

 

 



    


1

0 

basis for ker(L).
Every vector ~y ∈ Range(L) has the form
" # " # " #
x4 − x1 1 0
L(~x) = = (x4 − x1 ) + (2x2 + 3x3 )
2x2 + 3x3 0 1
(" # " #)
1 0
Hence, C = , spans Range(L) and is linearly independent, so it is a basis for
0 1
Range(L).
ii. We have
" # " # " # " #
−1 0 0 1
L(1, 0, 0, 0) = L(0, 1, 0, 0) = L(0, 0, 1, 0) = L(0, 0, 0, 1) =
0 2 3 0
" #
−1 0 0 1
Thus, [L] = .
0 2 3 0
iii. By Theorem 3.3.4 B is a basis for Null([L]).
   By  Theorem  3.3.5 C is a basis for Col([L]).
  
  −1  0     −1 0


     
 

    
  

 0  2

   0  2

 
By definition, Row([L]) = Span    ,   . Thus,    ,   spans Row([L]) and is
 



 0
    3 





 0 3
   



 1

0
  1

0

clearly linearly independent. Therefore, it is a basis for Row([L]).
To find a basis for the left nullspace of [L] we solve the homogeneous system [L]T ~x = ~0.
Row reducing the corresponding coefficient matrix gives
−1 0 1 0
   
 0 2 0 1
 0 3 ∼ 0 0
   
   
1 0 0 0
Section 3.3 Solutions 63

Thus, we have x1 = x2 = 0, so Null([L]T ) = {~0}. Thus, a basis for Null([L]T ) is the empty
set.
(g) i. If ~x ∈ ker(L), then we have
0 = L(~x) = (3x1 )
Hence, x1 = 0. Therefore, ker(L) = {0}, so a basis for ker(L) is the empty set.
Every vector in the range of L has the form L(~x) = (3x1 ). Hence, {1} spans Range(L) and is
linearly independent, so it is a basis for Range(L).
ii. We have L(1) = 3. Thus, [L] = [3].
iii. By Theorem 3.3.4, the empty set is a basis for ([L]). By Theorem 3.3.5, {1} is a basis for
Col([L]). Now notice that [L]T = [L]. Hence, the empty set is also a basis for ([L]T ) and {1}
is also a basis for Row([L]).
i. If ~x ∈ ker(L), then

(0, 0, 0) = L(x1 , x2 , x3 ) = (2x1 + x3 , x1 − x3 , x1 + x3 )

Hence, 2x1 + x3 = 0, x1 − x3 = 0, and x1 + x3 = 0. This implies that x1 = x3 = 0. Therefore,


every vector in ker(L) has the form
     
 x1   0  0
~x =  x2  =  x2  = x2 1 , x2 ∈ R
     
x3 0 0
     

  

 0
 
 
Thus, B =  1 spans ker(L) and is linearly independent. Hence, it is a basis for ker(L).

0

   

Every vector ~y ∈ Range(L) has the form
     
2x1 + x3  2  1 
L(~x) =  x1 − x3  = x1 1 + x3 −1 , x1 , x3 ∈ R
     
x1 + x3 1 1
     

    

 2  1 
   
 
Thus, C =  1 , −1 spans Range(L) and is clearly linearly independent, so it is a basis for


1  1 
 

Range(L).
ii. We have      
2 0  1 
L(1, 0, 0) = 1 , L(0, 1, 0) = 0 , L(0, 0, 1) = −1
     
1 0 1
     
 
2 0 1 
Thus, [L] = 1 0 −1.
 
1 0 1
 
64 Section 3.3 Solutions

iii. By Theorem 3.3.4 B is a basis for Null([L]). By Theorem 3.3.5 C is a basis for Col([L]).
     

 2  1  1
     
 
By definition, Row([L]) = Span  0 ,  0  , 0 . However, observe that



 1 −1 1 
      

     
 1  1 2
1   3    
 0  + 0 = 0
2  2   
−1 1 1
        
  1  1   1  1

 
    

 
Hence, Row([L]) = Span  0 , 0 . Since  0  , 0 is also clearly linearly independent,
    

 
 
 
 
 

 
−1 1
     −1 1
 

it is a basis for Row([L]).
To find a basis for the left nullspace of [L], we solve [L]T ~x = ~0. Row reducing the corre-
sponding coefficient matrix gives
   
2 1 1 1 0 2/3 
0 0 0 ∼ 0 1 −1/3
   
1 −1 1 0 0 0

Hence, every vector in Null([L]T ) has the form


   2   
 x1  − 3 x3  −2/3
~x =  x2  =  13 x3  = x3  1/3  , x3 ∈ R
     
x3 x3 1
     

 

 −2/3 

 
Hence,  1/3 spans the left nullspace of [L] and is clearly linearly independent, so it is a

 
 
 1 
  

basis for the left nullspace of [L].
(h) i. If ~x ∈ ker(L), then we have
0 = L(~x) = (x1 , 2x1 , −x1 )
Hence, x1 = 0. Therefore, ker(L) = {0}, so a basis for ker(L) is the empty set.
Every vector in the range of L has the form
   
 x1   1 
L(~x) =  2x1  = x1  2  , x1 ∈ R
   
−x1 −1
   

 

  1 
 
 
Hence, C =   2  spans Range(L) and is linearly independent, so it is a basis for Range(L).


 −1 
   

   
 1   1 
ii. We have L(1) =  2 . Thus, [L] =  2 .
   
−1 −1
   
Section 3.3 Solutions 65

iii. By Theorem 3.3.4, the empty set is a basis for ([L]). By Theorem 3.3.5, C is a basis for
Col([L]).
We have Row([L]) = Span{1, 2, −1} = Span{1}. Thus, {1} is a basis for Row([L]).
To find a basis for the left nullspace of [L], we solve

0 = [L]T ~x = x1 + 2x2 − x3

Thus, every vector in the left nullspace of [L] has the form
       
 x1  −2x2 + x3  −2 1
~x =  x2  =  x2 = x 1 + x 0 , x2 , x3 ∈ R
       

 2  
  3
x3 x3 0 1
  

    

 −2 1
   
 
Hence,   1  , 0 is a basis for Null([L]T ).


 0  1
 

i. If ~x ∈ ker(L), then
(0, 0) = L(x1 , x2 ) = (x1 + x2 , 2x1 + 4x2 )
Hence, x1 + x2 = 0 and 2x1 + 4x2 = 0. This implies that x1 = x2 = 0. Therefore, ker(L) = {~0}
and so a basis for ker(L) is the empty set.
Every vector ~y ∈ Range(L) has the form
" # " # " #
x1 + x2 1 1
L(~x) = = x1 + x2 , x1 , x2 ∈ R
2x1 + 4x2 2 4
(" # " #)
1 1
Thus, C = , spans Range(L) and is clearly linearly independent, so it is a basis for
2 4
Range(L).
ii. We have " # " #
1 1
L(1, 0) = , L(0, 1) =
2 4
" #
1 1
Thus, [L] = .
2 4
iii. By Theorem 3.3.4 the empty set is a basis for Null([L]). By Theorem 3.3.5 C is a basis for
Col([L]). (" # " #) (" # " #)
1 2 1 2
By definition, Row([L]) = Span , . Hence, , spans Row([L]) and is also
1 4 1 4
clearly linearly independent, so it is a basis for Row([L]).
To find a basis for the left nullspace of [L], we solve [L]T ~x = ~0. Row reducing the corre-
sponding coefficient matrix gives
" # " #
1 2 1 0

1 4 0 1

Hence, the only vector in Null([L]T ) is the zero vector. So, a basis for the left nullspace of
[L] is the empty set.
66 Section 3.3 Solutions
(" # " #)
1 2
3.3.2 (a) By definition, , spans Col(A). Since it is also linearly independent, it is a basis for
3 4
Col(A).
(" # " #)
1 3
By definition, , spans Row(A). Since it is also linearly independent, it is a basis for
2 4
Row(A).
To find Null(A), we solve A~x = ~0. Row reducing the coefficient matrix of the corresponding
system gives " # " #
1 2 1 0

3 4 0 1
Thus, the only solutions is ~x = ~0. Hence, Null(A) = {~0} and so a basis is the empty set.
To find Null(AT ), we solve AT ~x = ~0. Row reducing the coefficient matrix of the corresponding
system gives " # " #
1 3 1 0

2 4 0 1
Thus, the only solutions is ~x = ~0. Hence, Null(AT ) = {~0} and so a basis is the empty set.
(" # " # " #) " # " # " #
1 0 1 1 1 0
(b) By definition, Col(B) = Span , , . Since =1 + (−1) , we get that
0 1 −1 −1 0 1
(" # " # " #) (" # " #)
1 0 1 1 0
Col(B) = Span , , = Span ,
0 1 −1 0 1
(" # " #)
1 0
by Theorem 1.1.2. Thus, , is a spanning set for Col(B) and is also linearly independent,
0 1
so it is a basis for Col(B).
    
1  0 

 

By definition, 
 0 ,  1  spans Row(B). Since it is also linearly independent, it is a basis for

 
1 −1   

Row(B).
To find Null(B), we solve B~x = ~0. The coefficient matrix is already in RREF, so we get x1 + x3 = 0
and x2 − x3 = 0. Let x3 = t ∈ R. Then, every vector ~x ∈ Null(B) has the form
     
 x1  −x3  −1
 x2  =  x3  = x3  1 
     
x3 x3 1
  

 −1
 
 
So,   1  spans Null(B) and is clearly linearly independent and hence is a basis for Null(B).


  1 
 

To find Null(BT ), we solve BT ~x = ~0. Row reducing the coefficient matrix of the corresponding
system gives
   
1 0  1 0
0 1  ∼ 0 1
   
1 −1 0 0

Section 3.3 Solutions 67

Thus, the only solutions is ~x = ~0. Hence, Null(BT ) = {~0} and so a basis is the empty set.
     

  1   2   0 
     
 
(c) By definition, Col(C) = Span  −1 , −2 ,  0  spans Col(C). Observe that


 

 1 3 −1 
     
 1   0   2 
2 −1 −  0  = −2
     
1 −1 3
     

    

  1   0 
   
 
Hence,  −1 ,  0  also spans Col(C). Since it is also linearly independent, it is a basis for


 

 1 −1 
   
Col(C).
By definition,
          

 1 −1  1   1  1 
     
  
   
 
Row(C) = Span  2 , −2 , 3 = Span 2 ,  3 
 
 
 
 
 
 
 
 
 


0 
 
 

0

 

−1 

   0 −1 
     

    

 1  1 
   
 
Hence,  2 ,  3  is a basis for Row(C) since it spans Row(C) and is linearly independent.


 0 −1 
     

To find Null(C), we solve C~x = ~0. We row reduce the corresponding coefficient matrix to get
   
 1 2 0  1 0 2 
−1 −2 0  ∼ 0 1 −1
   
1 3 −1 0 0 0
x1 + 2x3 = 0 and x2 − x3 = 0. Then, every vector ~x ∈ Null(C) has the form
     
 x1  −2x3  −2
 x2  =  x3  = x3  1  , x3 ∈ R
     
x3 x3 1
  

 −2
 
 
So,   1  spans Null(C) and is clearly linearly independent and hence is a basis for Null(C).


  1 
 

To find Null(C T ), we solve C T ~x = ~0. Row reducing the coefficient matrix of the corresponding
system gives
   
1 −1 1  1 −1 0
2 −2 3  ∼ 0 0 1
   
0 0 −1 0 0 0
  

Hence, x1 − x2 = 0 and x3 = 0. Then, every vector ~x ∈ Null(C T ) has the form


     
 x1   x2  1
 x2  =  x2  = x2 1 , x2 ∈ R
     
x3 0 0
68 Section 3.3 Solutions

  

 1
 
 
So,  1 spans Null(C T ) and is clearly linearly independent and hence is a basis for Null(C T ).

0

 

(d) To find a basis for Col(A), we need to determine which columns of A can be written as linear
combinations of other columns. We consider
           
0  1   2   1  1  c1 + 2c2 + c3 + c4 
0 = c1 −2 + c2 −5 + c3 −2 + c4 1 = −2c1 − 5c2 − 2c3 + c4 
           
0 −1 −3 −1 2 −c1 − 3c2 − c3 + 2c4

Row reducing the coefficient matrix of the homogeneous system gives


   
 1 2 1 1 1 0 1 7 
−2 −5 −2 1 ∼ 0 1 0 −3
   
−1 −3 −1 2 0 0 0 0

If we ignore the last column and pretend the third column is an augmented part, this shows that the
third column equals 1 times the first column (which is obvious). If we ignore the third column and
pretend the fourth column is an augmented part, we see that 7 times the first column plus -3 times
    

  1   2  
   

the second column gives the fourth column. Hence, we have that  −2 , −5 spans Col(A).


 −1 −3 
     

Since it is also linearly independent (just consider the first two columns in the system above) and
hence it is a basis for Col(A).
To find a basis for Row(A), we need to determine which rows of A can be written as linear combi-
nations of other columns. We consider

−2 −1  c1 − 2c2 − c3 


         
0 1
0 2 −5 −3 2c − 5c − 3c 
  = c   + c   + c   =  1 2 3
0 1 1 2 −2 3 −1  c1 − 2c2 − c3 
         
0 1 1 2 c1 + c2 + 2c3

Row reducing the coefficient matrix of the homogeneous system gives

1 −2 −1 1 0


   
1
2 −5 −3 0 1 
1
1 −2 −1 ∼ 0 0
  
0

   
1 1 2 0 0 0

If we ignore the last column and pretend the third column is an augmented part, this shows that
the third column equals the sum of the first two
 columns. That is, the third row of A equals the
  

  1  −2  


     
2 −5

 
sum of the first two rows. Hence, we have that  , spans Row(A). Since it is also linearly


 1 −2
    

     

1

1 

independent (just consider the first two columns in the system above) and hence it is a basis for
Row(A).
Section 3.3 Solutions 69

To find Null(A), we solve A~x = ~0. Observe that the coefficient matrix is A which we already row
reduced above. Hence, we get x1 + x3 + 7x4 = 0 and x2 − 3x4 = 0. Then, every vector ~x ∈ Null(AT )
has the form
 x1  −x3 − 7x4  −1 −7
       
 x   3x
 = x   + x  3  , x , x ∈ R
  0   
 2  =  4
 x3   3 4 3 4
   x3 
  1   0 
x4 x4 0 1
   


 −1 −7 
     
 0   3 

 
So,  , spans Null(AT ) and is clearly linearly independent and hence is a basis for

1 0
 

 

 

 





  
     

 0   1 
 
Null(A).
To find Null(AT ), we solve AT ~x = ~0. We observe that we row reduced AT above. We get x1 +x3 = 0
and x2 + x3 = 0. Then, every vector ~x ∈ Null(AT ) has the form
     
 x1  −x3  −1
x = −x = x −1 , x3 ∈ R
     
 2   3  3
x3 x3 1
  

 −1
 
 
So,  −1 spans Null(AT ) and is clearly linearly independent and hence is a basis for Null(AT ).



 1    

3.3.3 If A~x = ~b has a unique solution, then A~x = ~0 has a unique solution ~x = ~0. Thus, Null(A) = {~0}.
On the other hand, if Null(A) = {~0}, then A~x = ~0 has a unique solution. Thus, by Theorem 2.2.4,
rank(A) = n and so A~x = ~b has a unique solution.
3.3.4 By definition Range(L) is a subset of Rm and ~0 ∈ Range(L) by Lemma 3.3.1.
~ ∈ Rn such that L(~x) = ~y and L(~
Let ~y,~z ∈ Range(L). Then, there exists ~x, w w) = ~z. We now see that

~ ) = L(~x) + L(~
L(~x + w w) = ~y + ~z

and
L(t~x) = tL(~x) = t~y
So, ~y + ~z ∈ Range(L) and t~y ∈ Range(L) for all t ∈ R. Thus, Range(L) is a subspace of Rm by the
Subspace Test.
3.3.5 Consider
c1 L(~v1 ) + · · · + ck L(~vk ) = ~0
Then,
L(c1~v1 + · · · + ck~vk ) = ~0
Therefore, c1~v1 + · · · + ck~vk ∈ ker(L) and hence c1~v1 + · · · + ck~vk = ~0 since ker(L) = {~0}. Thus, c1 = · · · =
ck = 0 since {~v1 , . . . , ~vk } is linearly independent. Hence, {L(~v1 ), . . . , L(~vk )} is linearly independent.
70 Section 3.3 Solutions

3.3.6 Let [L] be the standard matrix of L. By definition of the standard matrix, we have [L] is an n × n matrix.
Thus, we get that ker(L) = {~0} if and only if

~0 = L(~x) = [L]~x

has a unique solution. By Theorem 2.2.4(2), this is true if and only if rank[L] = n. Moreover, by
Theorem 2.2.4(3), we have that ~b = [L]~x = L(~x) is consistent for all ~b ∈ Rn if and only if rank[L] = n.
Hence, ker(L) = {~0} if and only if rank[L] = n if and only if Range(L) = Rn .
(a) By definition Null(A) is a subset of Rn and we have A~0 = ~0, so ~0 ∈ Null(A). Thus, Null(A) is
non-empty.
Let ~x, ~y ∈ Null(A). Then, A~x = ~0 and A~y = ~0, so

A(~x + ~y) = A~x + A~y = ~0 + ~0 = ~0

Therefore, ~x + ~y ∈ Null(A).
For any t ∈ R, we have
A(t~x) = tA~x = t(~0) = ~0
So, t~x ∈ Null(A).
Consequently, Null(A) is a subspace of Rn by the Subspace Test.
(b) Observe that A~x = ~0 represents a homogeneous system of linear equations. Thus, the set of all ~x
which satisfy this equation is the solution space of the homogeneous system, which is a subspace
of Rn .
3.3.7 (a) By definition Null(A) is a subset of Rn . Also, we have A~0 = ~0, so ~0 ∈ Null(A).
Let ~x, ~y ∈ Null(A). Then, A~x = ~0 and A~y = ~0 and hence

A(~x + ~y) = A~x + A~y = ~0 + ~0 = ~0

Similarly, for any s ∈ R we have


A(s~x) = sA~x = s~0 = ~0
Thus, by the Subspace Test, Null(A) is a subset of Rn .
(b) Observe that every ~x ∈ Null(A) satisfies

~0 = A~x = x1~a1 + · · · + xn~an

Thus, Null(A) is the solution space of this homogeneous system which we know is a subspace by
Theorem 2.2.3.
3.3.8 We just need to prove that if B is obtained from A by applying any one of the three elementary row
operations, then Row(A) = Row(B).
 T
~a1 
Let A =  ... . Then, Row(A) = Span{~a1 , . . . , ~am }.
 
 
~aTm
Section 3.4 Solutions 71

If B is obtained from A by swapping row i and row j, then B still has the same rows as A (just in a
different order), so
Row(B) = Span{~a1 , . . . , ~am } = Row(A)

If B is obtained from A by multiplying row i by a non-zero constant c, then we have

Row(B) = Span{~a1 , . . . , ~ai−1 , c~ai , ~ai+1 , . . . , ~am }


= {t1~a1 + · · · + ti−1~ai−1 + ti (c~ai ) + ti+1~ai+1 + · · · + tm~am | t1 , . . . , tm ∈ R}
= {t1~a1 + · · · + ti−1~ai−1 + cti (~ai ) + ti+1~ai+1 + · · · + tm~am | t1 , . . . , tm ∈ R}
= Span{~a1 , . . . , ~am } = Row(A)

If B is obtained from A by applying the elementary row operation Ri + cR j , then we have

Row(B) = Span{~a1 , . . . , ~ai−1 , ~ai + c~a j , ~ai+1 , . . . , ~am }


= {t1~a1 + · · · + ti−1~ai−1 + ti (~ai + c~a j ) + ti+1~ai+1 + · · · + tm~am | t1 , . . . , tm ∈ R}
= {t1~a1 + · · · + ti−1~ai−1 + ti~ai + ti+1~ai+1 + · · · + (t j + cti )~a j + · · · + tm~am | t1 , . . . , tm ∈ R}
= Span{~a1 , . . . , ~am } = Row(A)

3.3.9 (a) If ~b = A~x for some ~x ∈ Rn , then by definition ~b ∈ Col(A). On the other hand, if ~b ∈ Col(A),
then there exists ~x ∈ Rn such that A~x = ~b, so the system is consistent. Hence, we have proven the
statement.
(b) We disprove the statement " #with a counter" example.
# Let L : R2 → R2 be defined by L(~x) = ~0.
1 1
Then, L(1, 1) = ~0 and so ∈ ker(L), but , ~0.
1 1
" #
1 1
(c) We disprove the statement with a counter example. Let A = and let L : R2 → R2 be defined
1 1
(" # " #) " #
2 3 2 3
by L(~x) = A~x. Then, observe that Range(L) = Span , , but [L] , .
2 3 2 3

3.4 Problem Solutions


3.4.1 (a) We have
"
# " # " #
2x1 − x2 3x1 + x2 5x1
(L + M)(x1 , x2 ) = L(x1 , x2 ) + M(x1 , x2 ) = + =
x1 x1 + x2 2x1 + x2

Hence, # "
5 0
[L + M] =
2 1
72 Section 3.4 Solutions

(b) We have
" # " # " #
x1 + x2 −x3 x1 + x2 − x3
(L + M)(x1 , x2 , x3 ) = L(x1 , x2 , x3 ) + M(x1 , x2 , x3 ) = + =
x1 + x3 x1 + x2 + x3 2x1 + x2 + 2x3
Hence, " #
1 1 −1
[L + M] =
2 1 2
(c) We have
" # " # " #
2x1 − x2 −2x1 + x2 0
(L + M)(x1 , x2 , x3 ) = L(x1 , x2 , x3 ) + M(x1 , x2 , x3 ) = + =
x2 + 3x3 −x2 − 3x3 0
Hence, " #
0 0 0
[L + M] =
0 0 0
3.4.2 (a) We have
" # " #
3(2x1 − x2 ) + x1 7x1 − 3x2
(M ◦ L)(x1 , x2 ) = M(L(x1 , x2 )) = M(2x1 − x2 , x1 ) = =
2x1 − x2 + x1 3x1 − x2
" # " #
2(3x1 + x2 ) − (x1 + x2 ) 5x1 + x2
(L ◦ M)(x1 , x2 ) = L(M(x1 , x2 )) = L(3x1 + x2 , x1 + x2 ) = =
3x1 + x2 3x1 + x2
" # " #
7 −3 5 1
Hence, [M ◦ L] = and [L ◦ M] = .
3 −1 3 1
(b) We have

(M ◦ L)(x1 , x2 , x3 ) = M(L(x1 , x2 , x3 )) = M(x1 − 2x2 , x1 + x2 + x3 )


   
 0   0 
= (x1 − 2x2 ) + (x1 + x2 + x3 ) = 2x1 − x2 + x3 
   
x1 − 2x2 x1 − 2x2
   
" # " #
0 − 2(x1 + x2 ) −2x1 − 2x2
(L ◦ M)(x1 , x2 ) = L(M(x1 , x2 )) = L(0, x1 + x2 , x1 ) = =
0 + (x1 + x2 ) + x1 2x1 + x2
 
0 0 0 " #
−2 −2
Hence, [M ◦ L] = 2 −1 1 and [L ◦ M] = .
 
2 1
1 −2 0
 

(c) We have

(M ◦ L)(x1 , x2 ) = M(L(x1 , x2 )) = M(3x1 + x2 , 5x1 + 2x2 )


" # " #
2(3x1 + x2 ) − (5x1 + 2x2 ) x1
= =
−5(3x1 + x2 ) − 3(5x1 + 2x2 ) −30x1 − 11x2
(L ◦ M)(x1 , x2 ) = L(M(x1 , x2 )) = L(2x1 − x2 , −5x1 − 3x2 )
" # " #
3(2x1 − x2 ) + (−5x1 − 3x2 ) x − 6x2
= = 1
5(2x1 − x2 ) + 2(−5x1 − 3x2 ) −11x2
Section 3.4 Solutions 73
" # " #
1 0 1 −6
Hence, [M ◦ L] = and [L ◦ M] = .
−30 −11 0 −11
3.4.3 Let L : Rn → Rm and M : Rm → R p be linear mappings.
(a) Let ~z ∈ R p . Since M is onto, there exists ~y ∈ Rm such that L(~y) = ~z. Since L is onto, there exists
~x ∈ Rn such that L(~x) = ~y. Hence, we have that (M ◦ L)(~x) = M(L(~x)) = M(~y) = ~z. Hence,
Range(M ◦ L) = R p .
(b) Let L : R2 → R3 be defined by L(x1 , x2 ) = (x1 , x2 , 0). Clearly Range(L) , R3 . However, if we
define M : R3 → R2 by M(x1 , x2 , x3 ) = (x1 , x2 ), then we get that

(M ◦ L)(x1 , x2 ) = M(L(x1 , x2 )) = M(x1 , x2 , 0) = (x1 , x2 )

So, Range(M ◦ L) = R2 .
(c) No, it isn’t. If M ◦ L is onto, then for any ~z ∈ R p , there exists ~x ∈ Rn such that ~z = (M ◦ L)(~x) =
M(L(~x)). Hence, ~z ∈ Range(M). So, Range(M) = R p .
3.4.4 Let L, M ∈ L and let c, d ∈ R.
V4 Let ~x, ~y ∈ Rn and s, t ∈ R. Hence,

O(s~x + t~y) = ~0 = sO(~x) + tO(~y)

so, O is linear and hence O ∈ L. For any ~x ∈ Rn we have

(L + O)(~x) = L(~x) + O(~x) = L(~x) + ~0 = L(~x)

Hence, L + O = L.
V5 Let ~x, ~y ∈ Rn and s, t ∈ R. Then,

(−L)(s~x + t~y) = −L(s~x + t~y) = −[sL(~x) + tL(~y)] = s(−L)(~x) + t(−L)(~y)

Hence, (−L) is linear and so (−L) ∈ L. For any ~x ∈ Rn we have

(L + (−L))(~x) = L(~x) + (−L)(~x) = L(~x) − L(~x) = ~0 = O(~x)

Thus, L + (−L) = O
V8 For any ~x ∈ Rn we have

[(c + d)L](~x) = (c + d)L(~x) = cL(~x) + dL(~x) = [cL + dL](~x)

Hence, (c + d)L = cL + dL.


3.4.5 We have
h i h i
(M ◦ L)(~e1 ) · · · (M ◦ L)(~en ) = M(L(~e1 )) · · · M(L(~en ))
h i
= [M](L(~e1 )) · · · [M](L(~en ))
h i
= [M] L(~e1 ) · · · L(~en )
= [M][L]
74 Section 3.4 Solutions

3.4.6 For any ~x ∈ R3 we have

O(~x) = (t1 L1 + t2 L2 )(~x) = t1 L1 (~x) + t2 L2 (~x)


" # " # " #
0 2x1 x + x2 + x3
= t1 + t2 1
0 x2 − x3 x3
" # " #
0x1 + 0x2 + 0x3 (2t1 + t2 )x1 + t2 x2 + t2 x3
=
0x1 + 0x2 + 0x3 t1 x2 + (−t1 + t2 )x3

Hence, we have the system of equations

2t1 + t2 = 0
t2 = 0
t2 = 0
t1 = 0
−t1 + t2 = 0

Therefore, the only solution is t1 = t2 = 0.


3.4.7 For any ~x ∈ R3 we have

O(~x) = (t1 L1 + t2 L2 + t3 L3 )(~x) = t1 L1 (~x) + t2 L2 (~x) + t3 L3 (~x)


" # " #
0 t1 (x1 + 3x2 ) + t2 (2x1 + 5x2 ) + t3 (x1 + x2 )
=
0 t1 (6x1 + x2 ) + t2 (8x1 + 3x2 ) + t3 (−2x1 + 3x2 )
" # " #
0x1 + 0x2 (t1 + 2t2 + t3 )x1 + (3t1 + 5t2 + t3 )x2
=
0x1 + 0x2 (6t1 + 8t2 − 2t3 )x1 + (t1 + 3t2 + 3t3 )x2

Hence, we have the system of equations

t1 + 2t2 + t3 = 0
3t1 + 5t2 + t3 = 0
6t1 + 8t2 − 2t3 = 0
t1 + 3t2 + 3t3 = 0

Row reducing the corresponding coefficient matrix gives

0 −3
   
1 2 1  1
  
3 5 1  0 1 2 
 ∼
6 8 −2 0 0 0 

   
1 3 3 0 0 0

Thus, we get
     
t1   3t1   3 
t2  = −2t3  = t3 −2 , t3 ∈ R
     
t3 t3 1
Section 3.4 Solutions 75

3.4.8 Define Id : Rn → Rn by Id(~x) = ~x. Then, for any ~x, ~y ∈ Rn and s, t ∈ R, we have

Id(s~x + t~y) = s~x + t~y = s Id(~x) + t Id(~y)

Hence, Id is linear.
For any ~x ∈ Rn we have
(L ◦ Id)(~x) = L(Id(~x)) = L(~x)
and
(Id ◦L)(~x) = Id(L(~x)) = L(~x)
as required.
Chapter 4 Solutions

4.1 Problem Solutions


4.1.1 (a) Every q(x) ∈ S1 has the form x2 (a + bx + cx2 ) = ax2 + bx3 + cx4 ∈ P4 (R). Additionally, 0 ∈ S1 ,
so S1 is a non-empty subset of P4 (R).
Let q1 (x), q2 (x) ∈ S1 . Then there exists p1 (x), p2 (x) ∈ P2 (R) such that q1 (x) = x2 p1 (x) and
q2 (x) = x2 p2 (x). Then
q1 (x) + q2 (x) = x2 p1 (x) + x2 p2 (x) = x2 [p1 (x) + p2 (x)] ∈ S1
since p1 (x) + p2 (x) ∈ P2 (R). Also, for any c ∈ R we have
cq1 (x) = c[x2 p1 (x)] = x2 [cp1 (x)] ∈ S1
since cp1 (x) ∈ P2 (R). Therefore, by the Subspace Test, S1 is a subspace of P4 (R).
" #
0 0
(b) By definition S2 is a subset of M2×2 (R) and ∈ S2 since 0 + 0 = 0. Thus, we can apply the
0 0
Subspace Test.
" # " #
a a2 b1 b2
Let 1 , ∈ S2 and t ∈ R. Then a1 + a3 = 0 and b1 + b3 = 0. Hence,
0 a3 0 b3
" # " # " #
a1 a2 b1 b2 a1 + b1 a2 + b2
+ = ∈ S2
0 a3 0 b3 0 a3 + b3
since (a1 + b1 ) + (a3 + b3 ) = a1 + a3 + b1 + b3 = 0 + 0 = 0. Similarly,
" # " #
a a ta ta2
t 1 2 = 1 ∈ S2
0 a3 0 ta3
since ta1 + ta3 = t(a1 + a3 ) = t(0) = 0. Hence, S2 is a subspace of M2×2 (R).
(c) By definition S3 is a subset of P2 (R). Also, the zero polynomial z satisfies z(x) = 0 for all x, hence
z(1) = 0 so z ∈ S3 . Therefore, we can apply the Subspace Test.
Let p, q ∈ S3 and t ∈ R. Then, p(1) = 0 and q(1) = 0. Hence,
(p + q)(1) = p(1) + q(1) = 0 + 0 = 0
Hence, p + q ∈ S2 . Similarly,
(tp)(1) = tp(1) = t(0) = 0
So, tp ∈ S3 . Consequently, S3 is a subspace of P2 (R).
76
Section 4.1 Solutions 77

(d) Clearly S4 is a non-empty subset of M2×2 (R).


" # " # " #
0 0 0 0 0 0
Let A, B ∈ S4 . Then, A = and B = . Hence, A + B = ∈ S4 . For any t ∈ R we
0 0 0 0 0 0
" #
0 0
have tA = ∈ S4 . So, S4 is a subspace of M2×2 (R) by the Subspace Test.
0 0
NOTE: S4 is called the trivial subspace of M2×2 (R).

(e) By definition S5 is a subset of P2 (R) and the zero polynomial z(x) = 0 + 0x + 0x2 ∈ S5 since 0 = 0.
Let a + bx + cx2 , d + ex + f x2 ∈ S5 . Then, a = c and d = f . Hence,

a + bx + cx2 + d + ex + f x2 = (a + d) + (b + e)x + (c + f )x2 ∈ S5

since a + d = c + f and for any t ∈ R

t(a + bx + cx2 ) = ta + tbx + tcx2 ∈ S5

because ta = tc. Consequently, by the Subspace Test, S5 is a subspace of P2 (R).


(f) By definition S6 is a subset of Mn×n (R). Also, the n×n zero matrix On,n clearly satisfies OTn,n = On,n .
Hence, we can apply the Subspace Test.
Let A, B ∈ S6 and t ∈ R. Then, AT = A and BT = B. Using properties of the transpose gives

(A + B)T = AT + BT = A + B and (tA)T = tAT = tA

so, A + B ∈ S6 and tA ∈ S6 , so S6 is a subspace of Mn×n (R).


(g) The set does not contain the zero polynomial and hence cannot be a subspace of P2 (R).
(h) By Theorem 4.1.3, this is a subspace of P3 (R).
" #
0 0
(i) By definition S9 is a subset of M2×2 (R) and ∈ S2 since 0 + 0 = 0 and 0 = 2(0). Thus, we
0 0
can apply the Subspace Test.
" # " #
a a b b2
Let 1 2 , 1 ∈ S9 and t ∈ R. Then a1 + a3 = 0, a2 = 2a4 , b1 + b3 = 0, and b2 = 2b4 .
a3 a4 b3 b4
Hence, " # " # " #
a1 a2 b b2 a + b1 a2 + b2
+ 1 = 1 ∈ S9
a3 a4 b3 b4 a3 + b3 a4 + b4
since (a1 + b1 ) + (a3 + b3 ) = a1 + a3 + b1 + b3 = 0 + 0 = 0 and a2 + b2 = 2a4 + 2b4 = 2(a4 + b4 ).
Similarly, " # " #
a1 a2 ta1 ta2
t = ∈ S9
a3 a4 ta3 ta4
since ta1 + ta3 = t(a1 + a3 ) = t(0) = 0 and ta2 = t(2a4 ) = 2(ta4 ). Hence, S9 is a subspace of
M2×2 (R).
78 Section 4.1 Solutions

4.1.2 (a) Consider

1 + x2 = c1 (1 + x + x2 ) + c2 (−1 + 2x + 2x2 ) + c3 (5 + x + x2 )
= c1 − c2 + 5c3 + (c1 + 2c2 + c3 )x + (c1 + 2c2 + c3 )x2

Comparing coefficients we get the system

c1 − c2 + 5c3 = 1
c1 + 2c2 + c3 = 0
c1 + 2c2 + c3 = 1

Row reducing the augmented matrix gives


   
 1 −1 5 1   1 0 11/3 0 
 1 2 1 0  ∼  0 1 −4/3 0
   

1 2 1 1 0 0 0 1

Hence, the system is inconsistent, so ~v < Span B.


(b) Consider
" # " # " # " # " #
2 5 1 0 0 1 3 1 c1 + 3c3 c2 + c3
= c1 + c2 + c3 =
4 4 −1 1 2 2 −3 1 −c1 + 2c2 − 3c3 c1 + 2c2 + c3

Comparing entries we get the system

c1 + 3c3 = 2
c2 + c3 = 5
−c1 + 2c2 − 3c3 = 4
c1 + 2c2 + c3 = 4

Row reducing the augmented matrix gives

  1 0 0 −4 


   
 1 0 3 2
 0 1 1 5   0 1 0 3 
  ∼ 
 −1 2 −3 4   0 0 1 2 

   
1 2 1 4 0 0 0 0

Hence, the system is consistent, so ~v ∈ Span B.


(c) Consider
         
 0   1   1   2   c1 + c2 + 2c3 
−3
  = c  2  + c  3  + c  4  = 2c1 + 3c2 + 4c3 
       
 3  1  0 
 2  1 
  3  3   c2 + 3c3
   
         
−6 −1 −1 −5 −c1 − c2 − 5c3
Section 4.1 Solutions 79

Comparing entries we get the system

c1 + c2 + 2c3 = 0
2c1 + 3c2 + 4c3 = −3
c2 + 3c3 = 3
−c1 − c2 − 5c3 = −6

Row reducing the augmented matrix gives

0 −1 
   
 1 1 2 0   1 0
  
 2 3 4 −3   0 1 0 −3 
 ∼
0 1 3 3   0 0 1 2 
 
   
−1 −1 −5 −6 0 0 0 0

Hence, the system is consistent, so ~v ∈ Span B.


(d) Consider

−3x + 3x2 − 6x3 = c1 (1 + 2x − x3 ) + c2 (1 + 3x + x2 − x3 ) + c3 (2 + 4x + 3x2 − 5x3 )


= c1 + c2 + 2c3 + (2c1 + 3c2 + 4c3 )x + (c2 + c3 )x2 + (−c1 − c2 − 5c3 )x3

Comparing coefficients we get the system

c1 + c2 + 2c3 = 0
2c1 + 3c2 + 4c3 = −3
c2 + 3c3 = 3
−c1 − c2 − 5c3 = −6

Observe this is the same system as in (c). Hence, ~v ∈ Span B.


(e) Consider
" # " # " # " # " #
0 −3 1 2 1 3 2 4 c1 + c2 + 2c3 2c1 + 3c2 + 4c3
= c1 + c2 + c3 =
3 −6 0 −1 1 −1 3 −5 c2 + 3c3 −c1 − c2 − 5c3

Comparing entries we get the system

c1 + c2 + 2c3 = 0
2c1 + 3c2 + 4c3 = −3
c2 + 3c3 = 3
−c1 − c2 − 5c3 = −6

Observe this is the same system as in (c). Hence, ~v ∈ Span B.


80 Section 4.1 Solutions

4.1.3 (a) Consider

0 + 0x + 0x2 = c1 (3 − 2x + x2 ) + c2 (2 − 5x + 2x2 ) + c3 (6 + 7x − 2x2 )


= 3c1 + 2c2 + 6c3 + (−2c1 − 5c2 + 7c3 )x + (c1 + 2c2 − 2c3 )x2

This gives the homogeneous system of equations

3c1 + 2c2 + 6c3 = 0


−2c1 − 5c2 + 7c3 = 0
c1 + 2c2 − 2c3 = 0

Row reducing the coefficient matrix gives


   
 3 2 6  1 0 4 
−2 −5 7  ∼ 0 1 −3
   
1 2 −2 0 0 0

Hence, the system has infinitely many solutions, so B is linearly dependent.


(b) By Theorem 4.1.6 the set is linearly dependent, since it contains the zero vector.
(c) Consider
" # " # " # " # " #
0 0 1 1 2 2 1 −2 c1 + 2c2 + c3 c1 + 2c2 − 2c3
= c1 + c2 + c3 =
0 0 −2 1 1 0 3 4 −2c1 + c2 + 3c3 c1 + 4c3

This gives the homogeneous system

c1 + 2c2 + c3 = 0
c1 + 2c2 − 2c3 = 0
−2c1 + c2 + 3c3 = 0
c1 + 4c3 = 0

Row reducing the coefficient matrix gives


   
 1 2 1  1 0
 
0

 1 2 −2 0 1 0

−2 ∼
1 3  0 0 1

   
1 0 4 0 0 0

Hence, the only solution is c1 = c2 = c3 = 0, so B is linearly independent.


(d) Consider

0 = c1 (1 + x − 2x2 + x3 ) + c2 (2 + 2x + x2 ) + c3 (1 − 2x + 3x2 + 4x3 )


= c1 + 2c2 + c3 + (c1 + 2c2 − 2c3 )x + (−2c1 + c2 + 3c3 )x2 + (c1 + 4c3 )x3

Hence, we have the same homogeneous system as in part (c) so this set is also linearly independent.
Section 4.1 Solutions 81

(e) Consider
" # " # " # " # " # " #
0 0 1 −1 3 1 −1 5 3 1 c + 3c2 − c3 + 3c4 −1c1 + c2 + 5c3 + c4
= c1 +c2 +c3 +c4 = 1
0 0 2 1 2 1 0 1 2 2 2c1 + 2c2 + 2c4 c1 + c2 + c3 + 2c4

This gives the homogeneous system

c1 + 3c2 − c3 + 3c4 = 0
−c1 + c2 + 5c3 + c4 = 0
2c1 + 2c2 + 2c4 = 0
c1 + c2 + c3 + 2c4 = 0

Row reducing the coefficient matrix gives

 1 3 −1
   
3 1 0 0 0
−1 1 5   
1 0 1 0 0

 2 2 0 ∼
2 0 0 1 0

   
1 1 1 2 0 0 0 1

Hence, the only solution is c1 = c2 = c3 = c4 = 0. Therefore, the set is linearly independent.


4.1.4 For any ~x ∈ R3 we have

O(~x) = (t1 L1 + t2 L2 + t3 L3 )(~x) = t1 L1 (~x) + t2 L2 (~x) + t3 L3 (~x)


" # " #
0 t1 (x1 + 3x2 ) + t2 (2x1 + 5x2 ) + t3 (x1 + x2 )
=
0 t1 (6x1 + x2 ) + t2 (8x1 + 3x2 ) + t3 (−2x1 + 3x2 )
" # " #
0x1 + 0x2 (t1 + 2t2 + t3 )x1 + (3t1 + 5t2 + t3 )x2
=
0x1 + 0x2 (6t1 + 8t2 − 2t3 )x1 + (t1 + 3t2 + 3t3 )x2

Hence, we have the system of equations

t1 + 2t2 + t3 = 0
3t1 + 5t2 + t3 = 0
6t1 + 8t2 − 2t3 = 0
t1 + 3t2 + 3t3 = 0

Row reducing the corresponding coefficient matrix gives

0 −3
   
1 2 1  1 
3 5 1  0 1 2 
6 8 −2 ∼ 0
  
0 0 

   
1 3 3 0 0 0

Thus, we get
     
t1   3t1   3 
t2  = −2t3  = t3 −2 , t3 ∈ R
     
t3 t3 1
82 Section 4.1 Solutions

4.1.5 (a) The statement is true. Assume that {~v1 , . . . , ~vℓ } is any subset of B that is linearly dependent. Then
there exists coefficients c1 , . . . , cℓ not all zero such that

c1~v1 + · · · + cℓ~vℓ = ~0

But then we have


c1~v1 + · · · + cℓ~vℓ + 0~vℓ+1 + · · · + 0~vk = ~0
with not all coefficients 0 which contradicts B is linearly independent. Hence, any subset of B is
linearly independent.
(b) The statement is false. Take ~v1 = ~0, ~v2 , ~0, and c = 0. Then, c~v1 = c~v2 , but ~v1 , ~v2 .
(c) The statement is true. We have 0~x = ~0 ∈ S by Theorem 4.1.1. If c , 0, then for any ~z ∈ V we can
~ = 1c ~z. Then, we get
let w
~z + c~0 = c~w + c~0 = c(~
w + ~0) = c~
w = ~z
Hence, c~0 = ~0 since the zero vector is unique.
4.1.6 By definition {~0V } is a non-empty subset of V. Hence, we can apply the Subspace Test. Let ~x, ~y ∈ S.
Then, ~x = ~0V = ~y. Hence,

~x + ~y = ~0V + ~0V = ~0V by V4 since V is a vector space

Hence, ~x + ~y ∈ {~0V }. We need to show that s~x = ~0 for all s ∈ R. We have 0~x = ~0 ∈ S by Theorem 4.1.1.
~ = 1s ~z. Then, we get
If s , 0, then for any ~z ∈ V we can let w

~z + s~0 = s~
w + s~0 = s(~
w + ~0) = s~
w = ~z

Hence, s~0 = ~0 ∈ {~0V }. Therefore, {~0V } is a subspace of V. Consequently, by definition, it is a vector


space under the same operations as V.
4.1.7 It is not a vector space since we know that matrix multiplication is not commutative. That is, in general
A ⊕ B = AB , AB = B ⊕ A.
4.1.8 Denote the set of all sequences by S. Let ~s = {s1 , s2 , s3 , . . .}), ~t = {t1 , t2 , t3 , . . .}, ~u = {u1 , u2 , u3 , . . .}, and
let a, b ∈ R.
V1 By definition ~s + ~t = {s1 + t1 , s2 + t2 , . . .} ∈ S.
V2 We have

~s + (~t + ~u) = ~s + {t1 + u1 , t2 + u2 , . . .} = {s1 + (t1 + u1 ), s2 + (t2 + u2 ), . . .}


= {(s1 + t1 ) + u1 , (s2 + t2 ) + u2 , . . .} = {s1 + t1 , s2 + t2 , . . .} + ~u = (~s + ~t) + ~u

V3 We have
~s + ~t = {s1 + t1 , s2 + t2 , . . .} = {t1 + s1 , t2 + s2 , . . .} = ~t + ~s

V4 Let ~0 = {0, 0, . . .}. Then, ~0 ∈ S and

~s + ~0 = {s1 + 0, s2 + 0, . . .} = {s1 , s2 , . . .} = ~s
Section 4.1 Solutions 83

V5 Let −~s = {−s1 , −s2 , . . .}. Then, −~s ∈ S and


~s + (−~s) = {s1 + (−s1 ), s2 + (−s2 ), . . .} = {0, 0, . . .} = ~0

V6 By definition a~s = {as1 , as2 , . . .} ∈ S


V7 We have
a(b~s) = a{bs1 , bs2 , . . .} = {a(bs1 ), a(bs2 ), . . .} = {(ab)s1 , (ab)s2 , . . .} = (ab){s1 , s2 , . . .} = (ab)~s

V8 We have
(a+b)~s = {(a+b)s1 , (a+b)s2 , . . .} = {as1 +bs1 , as2 +bs2 ), . . .} = {as1 , as2 , . . .}+{bs1 , bs2 , . . .} = a{s1 , s2 , . . .}+b{s1

V9 We have
a(~s+~t) = a{s1 +t1 , s2 +t2 , . . .} = {a(s1 +t1 ), a(s2 +t2 ), . . .} = {as1 +at1 , as2 +at2 , . . .} = {as1 , as2 , . . .}+{at1 , at2 , . . .} =

V10 We have
1~s = {1s1 , 1s2 , . . .} = {s1 , s2 , . . .} = ~s
Thus, S is a vector space under these operations.
4.1.9 Let V be a vector space. Prove that (−~x) = (−1)~x for all ~x ∈ V. We have
~0 = ~x + (−~x) by V5
0~x = ~x + (−~x) by Theorem 4.1.1(1)
(1 − 1)~x = ~x + (−~x) operations on reals
1~x + (−1)~x = ~x + (−~x) by V8
~x + (−1)~x = ~x + (−~x) by V10
(−~x) + ~x + (−1)~x = (−~x) + ~x + (−~x) by V5
~0 + (−1)~x = ~0 + (−~x) by V5
(−1)~x = (−~x) by V4
4.1.10 We are assuming that there exist c1 , . . . , ck−1 ∈ R such that
c1~v1 + · · · + ci−1~vi−1 + ci~vi+1 + · · · + ck−1~vk = ~vi
Let ~x ∈ Span{~v1 , . . . , ~vk }. Then, there exist d1 , . . . , dk ∈ R such that
~x = d1~v1 + · · · + di−1~vi−1 + di~vi + di+1~vi+1 + · · · + dk~vk
= d1~v1 + · · · + dk−1~vk−1 + di (c1~v1 + · · · + ci−1~vi−1 + ci~vi+1 + · · · + ck−1~vk ) + di+1~vi+1 + · · · + dk~vk
= (d1 + di c1 )~v1 + · · · + (di−1 + di ci−1 )~vi−1 + (di+1 + di ci )~vi+1 + · · · + (dk + di ck−1 )~vk
Thus, ~x ∈ Span{~v1 , . . . , ~vi−1 , ~vi+1 , . . . , ~vk }. Hence, Span{~v1 , . . . , ~vk } ⊆ Span{~v1 , . . . , ~vi−1 , ~vi+1 , . . . , ~vk }.
Clearly, we have Span{~v1 , . . . , ~vi−1 , ~vi+1 , . . . , ~vk } ⊆ Span{~v1 , . . . , ~vk } and so
Span{~v1 , . . . , ~vk } = Span{~v1 , . . . , ~vi−1 , ~vi+1 , . . . , ~vk }
as required.
84 Section 4.1 Solutions

4.1.11 If {~v1 , . . . , ~vk } is linearly dependent, then there exists c1 , . . . , ck ∈ R not all zero such that

c1~v1 + · · · + ck~vk = ~0

Assume that ci , 0. Then, we get


−c1 −ci−1 −ci+1 −ck
~vi = ~v1 + · · · + ~vi−1 + ~vi+1 + · · · + ~vk
ci ci ci ci

Thus, ~vi ∈ Span{~v1 , . . . , ~vi−1 , ~vi+1 , . . . , ~vk }.


On the other hand, if ~vi ∈ Span{~v1 , . . . , ~vi−1 , ~vi+1 , . . . , ~vk }, then there exists, c1 , . . . , ck−1 such that

c1~v1 + · · · + ci−1~vi−1 + ci~vi+1 + · · · + ck−1~vk = ~vi

Thus,
c1~v1 + · · · + ci−1~vi−1 − ~vi + ci~vi+1 + · · · + ck−1~vk = ~0
Hence, {~v1 , . . . , ~vk } is linearly dependent.
Section 4.2 Solutions 85

4.2 Problem Solutions


4.2.1 (a) Consider

0 = c1 (1 + x2 ) + c2 (1 − x + x3 ) + c3 (2x + x2 − 3x3 ) + c4 (1 + 4x + x2 ) + c5 (1 − x − x2 + x3 )
= (c1 + c2 + c4 + c5 ) + (−c2 + 2c3 + 4c4 − c5 )x + (c1 + c3 + c4 − c5 )x2 + (c2 − 3c3 + c5 )x3

This gives us a homogeneous system of 4 equations in 5 unknowns. Hence, by Theorem 2.2.4,


there must be at least one parameter in the general solution, so there are infinitely many solutions.
Thus, B is linearly dependent and hence not a basis.
(b) We need to show that for any polynomial p(x) = a + bx + cx2 we can find c1 , c2 , c3 such that

a + bx + cx2 = c1 (1 + 2x + x2 ) + c2 (2 + 9x) + c3 (3 + 3x + 4x2 )


= (c1 + 2c2 + 3c3 ) + (2c1 + 9c2 + 3c3 )x + (c1 + 4c3 )x2

ow reducing the coefficient matrix of the corresponding system gives


   
 1 2 3   1 0 0 
 2 9 3  ∼  0 1 0  .
   
1 0 4 0 0 1
 
a
Hence, there is a leading one in each row, so the system is consistent for every right hand side b
 
c
 
and B spans P2 (R) by Theorem 2.2.4. Moreover, if we let a = b = c = 0, we see that we get the
unique solution c1 = c2 = c3 = 0, and hence B is also linearly independent. Therefore, B is a
basis for P2 (R).
(c) Consider
" # " # " # " # " #
0 0 1 1 1 2 1 3 c + c2 + c3 c1 + 2c2 + 3c3
= c1 + c2 + c3 = 1
0 0 0 1 0 −1 0 2 0 c1 − c2 + 2c3

This gives us the homogeneous system of linear equations

0 = c1 + c2 + c3
0 = c1 + 2c2 + 3c3
0 = c1 − c2 + 2c3

Row reducing the coefficient matrix of the homogeneous system we get


   
1 1 1 1 0 0
1 2 3 ∼ 0 1 0 .
   
1 −1 2 0 0 1

Hence, the only solution is c1 = c2 = c3 = 0, so B is linearly independent.


86 Section 4.2 Solutions
" #
a b
To show it is also a spanning set we need to show that any matrix can be written as a linear
0 c
combination of of the elements of the set. We consider
" # " # " # " #
a b 1 1 1 2 1 3
= c1 + c2 + c3
0 c 0 1 0 −1 0 2
This gives a system of linear equations with the same coefficient matrix as above. So, using the
same row-operations we did above we see that the matrix has a leading" one
# in each row of its
a b
RREF, so by Theorem 2.2.4, the system is consistent for all matrices , so B spans U.
0 c
Thus, B is a basis for U.
(" # " # " #) " #
1 2 5 6 9 0 a b
(d) B = , , of M2×2 (R) Let ∈ M2×2 (R) and consider
3 4 7 8 1 2 c d
" # " # " # " # " #
a b 1 2 5 6 9 0 c + 5c2 + 9c3 2c1 + 6c2
= c1 + c2 + c3 = 1
c d 3 4 7 8 1 2 3c1 + 7c2 + c3 4c1 + 8c2 + 2c3
This gives a system of 4 equations in 3 unknowns. Since the rank of the coefficient
" #matrix is at
a b
most 3 we get by Theorem 2.2.4 that the equation cannot be consistent for all ∈ M2×2 (R)
c d
and hence B does not span M2×2 (R). Thus, B is not a basis.
4.2.2 (a) Observe that every vector in S1 has the form xp(x) = x(a + bx + cx2 ) = ax + bx2 + cx3 . Hence,
S1 = Span{x, x2 , x3 }. Clearly {x, x2 , x3 } is also linearly independent, and hence it is a basis for S1
and dim S1 = 3.
(b) Observe that every vector in S2 has the form
" # " # " # " #
a1 a2 a a2 1 0 0 1
= 1 = a1 + a2
0 a3 0 −a1 0 −1 0 0
(" # " #)
1 0 0 1
Thus, B = , spans S2 and is clearly linearly independent. Consequently, B is a
0 −1 0 0
basis for S2 and dim S2 = 2.
(c) Let p(x) = ax2 + bx + c be any vector in S3 . Then p(2) = 0, so a(2)2 + b(2) + c = 0. Thus,
c = −4a − 2b. Hence, every p(x) ∈ S3 has the form

ax2 + bx + (−4a − 2b) = a(x2 − 4) + b(x − 2)

Thus, S3 = Span{x2 − 4, x − 2}. Since {x2 − 4, x − 2} is also clearly linearly independent, it is a


basis for S3 . Thus, dim S3 = 2.
(d) By definition a basis is the empty set and the dimension is 0.
(e) Let p(x) = a + bx + cx2 be any vector in S5 . Then a = c, so every p(x) ∈ S5 has the form

a + bx + ax2 = a(1 + x2 ) + bx

Thus, S5 = Span{1 + x2 , x}. Since {1 + x2 , x} is also clearly linearly independent, it is a basis for
S5 . Thus, dim S5 = 2.
Section 4.2 Solutions 87
" #
a b
(f) Let A = ∈ S6 . Then A satisfies AT = A which implies that
c d
" # " #
a b a c
=
c d b d

Hence, b = c. Thus, every A ∈ S6 has the form


" # " # " # " #
a b 1 0 0 1 0 0
=a +b +d
b d 0 0 1 0 0 1
(" # " # " #)
1 0 0 1 0 0
Thus, B = , , spans S6 and is clearly linearly independent, so B is a basis
0 0 1 0 0 1
for S6 . Thus, dim S6 = 3.
(g) Every A ∈ S7 has the form
" # " # " # " #
a1 a2 a1 2a4 1 0 0 2
= = a1 + a4
a3 a4 −a1 a4 −1 0 0 1
(" # " #)
1 0 0 2
Thus, B = , spans S7 and is clearly linearly independent, so B is a basis for S7 .
−1 0 0 1
Thus, dim S7 = 2.
(h) Consider

0 = c1 (1 + x) + c2 (1 + x + x2 ) + c3 (x + 2x2 ) + c4 (x + x2 + x3 ) + c5 (1 + x2 + x3 )
= c1 + c2 + c5 + (c1 + c2 + c3 + c4 )x + (c2 + 2c3 + c4 + c5 )x2 + (c4 + c5 )x3

Row reducing the coefficient matrix of the corresponding homogeneous system gives

1 1 0 0 1 1 0 0 0 −3


   
1 1 1 1 0 0 1 0 0 4 
0 1 2 1 1 ∼ 0 0 1 0 −2
   
   
0 0 0 1 1 0 0 0 1 1

Thus, treating this an augmented matrix we get that 1+x2 +xn3 can be written as a linear combinationo
of the other vectors. Ignoring the last column shows that 1 + x, 1 + x + x2 , x + 2x2 , x + x2 + x3
is linearly independent, and hence a basis for the set it spans. Thus, dim S8 = 4.
(i) Consider
             
0  1  −1  1  0 0  c1 − c2 + c3 
0 = c1 −3 + c2  4  + c3 −1 + c4 2 + c5 1 = −3c1 + 4c2 − c3 + 2c4 + c5 
             
0 2 −1 4 2 1 2c1 − c2 + 4c3 + 2c4 + c5

Row reducing the coefficient matrix of the corresponding homogeneous system gives
   
 1 −1 1 0 0 1 0 3 2 1
−3 4 −1 2 1 ∼ 0 1 2 2 1
   
2 −1 4 2 1 0 0 0 0 0
88 Section 4.2 Solutions

Thus, we see that the last three vectors can be written as alinear   combination
 of the first two

  1  −1  
   
 
vectors. Moreover, the first two columns shows that the set  −3 , 4 is linearly independent,

 

 

 

 

 

 
 2 −1 
   
and hence a basis for the set it spans. Thus, dim S9 = 2.
(" # " # " # " # " # " #)
1 0 0 0 1 0 0 0 1 0 0 0 0 0 0 0 0 0
4.2.3 , , , , ,
0 0 0 0 0 0 0 0 0 1 0 0 0 1 0 0 0 1
4.2.4 (1) This is the contrapositive of Theorem 4.2.1.
(2) Assume {~v1 , . . . , ~vk } spans V where k < n. Then, we can use Theorem 4.1.1 to remove linearly
dependent vectors (if any) to get a basis for V. Thus, we can find a basis for V with fewer than n
vectors which contradicts Theorem 4.2.4.
(3) Assume that {~v1 , . . . , ~vn } is linearly independent, but does not span V. Then, there exists ~v ∈ V
such that ~v < Span{~v1 , . . . , ~vn }. Hence, by Theorem 4.2.2 we have that {~v1 , . . . , ~vn , ~v} is a linearly
independent set of n + 1 vectors in V. But, this contradicts (1). Therefore, {~v1 , . . . , ~vn } also spans
V.
Assume {~v1 , . . . , ~vn } spans V, but is linearly dependent. Then, there exists some vector ~vi ∈
Span{~v1 , . . . , ~vi−1 , ~vi+1 , . . . , ~vn }. Hence, by Theorem 4.2.1 we have that

V = Span{~v1 , . . . , ~vn } = Span{~v1 , . . . , ~vi−1 , ~vi+1 , . . . , ~vn }

So, V is spanned by n − 1 vectors which contradicts (2). Thus, {~v1 , . . . , ~vn } is also linearly inde-
pendent.
4.2.5 Since we know that dim P3 (R) = 4, we need to add two vectors to the set {1 + x + x3 , 1 + x2 } so that the
set is still linearly independent. There are a variety of ways of picking such vectors. We observe that
neither x2 nor x can be written as a linear combination of 1 + x + x3 and 1 + x2 , so we will try to prove
that B = {1 + x + x3 , 1 + x2 , x2 , x} is a basis for P3 . Consider

0 = c1 (1 + x + x3 ) + c2 (1 + x2 ) + c3 (x2 ) + c4 x = (c1 + c2 ) + (c1 + c4 )x + (c2 + c3 )x2 + c1 x3

Solving the corresponding homogeneous system we get c1 = c2 = c3 = c4 = 0, so B is a linearly


independent set of 4 vectors in P3 (R) and hence is a basis for P3 (R) which includes 1 + x + x3 and 1 + x2 .
4.2.6 Let ~e1 , ~e2 , ~e3 , ~e4 denote the standard basis vectors for M2×2 (R). Then, clearly {~v1 , ~v2 , ~e1 , ~e2 , ~e3 , ~e4 } spans
M2×2 (R). Consider
~0 = c1~v1 + c2~v2 + c3~e1 + c4~e2 + c5~e3 + c6~e4

Row reducing the corresponding coefficient matrix gives

0 4/5 −3/5
   
1 1 1 0 0 0 1
 
0 0

2 2 0 1 0 0 0 1 0 0 −1/5 2/5 
 ∼
2 3 0 0 1 0 0 0 1 0 −3/5 1/5 

   
1 4 0 0 0 1 0 0 0 1 −6/5 2/5

If we ignore the last two columns, we see that this implies that B = {~v1 , ~v2 , ~e1 , ~e2 } is a linearly indepen-
dent set of 4 vectors in M2×2 (R). Thus, since dim M2×2 (R) = 4 we get that B is linearly independent.
Section 4.2 Solutions 89

     
 1   0   0 
−1  2   0 
4.2.7 Three vectors in the hyperplane are ~v1 =  , ~v2 =   and ~v3 =  . Consider
 0  −1  1 
0 0 −2
   
0  c1 
0 −c + 2c 
  = c ~v + c ~v + c ~v =  1 2
0 1 1 2 2 3 3 
 −c2 + c3 
   
0 −2c3

The only solution is c1 = c2 = c3 = 0 so the vectors are linearly independent. Since a hyperplane in R4
has dimension 3 and {~v1 , ~v2 , ~v3 } is a linearly independent set of three vectors in the hyperplane, it forms
a basis for the hyperplane.
 
1
0
We need to add a vector ~v4 so that {~v1 , ~v2 , ~v3 , ~v4 } is a basis for R4 . Since   does not satisfy the equation
0
0
of the hyperplane, it is not in Span(~v1 , ~v2 , ~v3 ) and so is not a linear combination of ~v1 , ~v2 , ~v3 . Thus,
{~v1 , ~v2 , ~v3 , ~v4 } is linearly independent set with four elements in R4 and hence is a basis for R4 since R4
has dimension 4.
4.2.8 Let B = {~v1 , . . . , ~vk } be a basis for S. Then, B is a linearly independent set of k vectors in V. But,
dim V = dim S = k, so B is also a basis for V. Hence, V = S.
4.2.9 Since k < n, we have that Span{~v1 , . . . , ~vk } , V. Thus, there exists a vector ~vk+1 < Span{~v1 , . . . , ~vk }.
Thus, by Theorem 4.1.5, {~v1 , . . . , ~vk+1 } is linearly independent.
If k + 1 = n, then {~v1 , . . . , w ~ k+1 } is a basis for V. If not, we can keep repeating this procedure until we
~ k+1 , w
get {~v1 , . . . , w ~ k+2 , . . . , w
~ n }.
4.2.10 (a) This statement is true as it is the contrapositive of Theorem 4.2.3(1).
(b) This is false. Since dim P2 (R) = 3, we have that every basis for P2 (R) has 3 vectors in it.
(c) This is false. Taking a = b = 1 and c = d = 2, gives {~v1 + ~v2 , 2~v1 + 2~v2 } which is clearly linearly
dependent.
(d) This is true. If {~v1 , . . . , ~vk } is linearly independent, then it is a basis. If it is linearly dependent, then
by Theorem 4.1.5, there is some vector ~vi such that ~vi ∈ Span{~v1 , . . . , ~vi−1 , ~vi+1 , . . . , ~vk }. Then, by
Theorem 4.1.4, we have that V = Span{~v1 , . . . , ~vi−1 , ~vi+1 , . . . , ~vk }. We can continue to repeat this
process until we have a linearly indpendent spanning set.
(e) This is true. If {~v1 , . . . , ~vk } is a linearly independent set in V, then either dim V = k or, by Theorem
4.2.3, there exist w ~ k+1 , . . . , w ~ n such that {~v1 , . . . , ~vk , w
~ k+1 , . . . , w
~ n } is a basis for V. Thus, dim V =
n ≥ k.
(" # " # " # " #)
1 0 2 0 3 0 4 0
(f) This is false. , , , is clearly linearly dependent and hence not a basis
0 0 0 0 0 0 0 0
for M2×2 (R).
90 Section 4.3 Solutions

4.2.11 Let B = {~v1 , ~v2 , ~v3 } be a basis for S and C = {~ ~ 2, w


w1 , w ~ 3 } be a basis for T. Assume for a contradiction that
~
S ∪ T = {0}. Then, ~vi < Span C for i = 1, 2, 3. Therefore, {~v1 , ~v2 , ~v3 , w ~ 1, w
~ 2, w
~ 3 } is a linearly independent
set of 6 vectors in V, but this contradicts the fact that dim V = 5.

4.3 Problem Solutions


# " √ √
√ 1 3
" # " #
3 6 2 − 12 3 √2 − 24
4.3.1 (a) We have ~x = 2 + (−4) = √
−1 2 −4 3 − 2 + 16 2 2 − 12
(b) ~x = 1(1 + x) + 0(1 + x + x2 ) + 3(1 + x2 ) = 4 + x + 3x2
4.3.2 We need to find c1 , c2 , c3 and d1 , d2 , d3 such that
       
1 3 −2 −2
c1 2 + c2 7 + c3 −3 = −5
       
0 3 5 1
       
       
1 3 −2  4 
d1 2 + d2 7 + d3 −3 =  8 
       
0 3 5 −3
       

Row reducing the corresponding doubly augmented matrix gives


   
 1 3 −2 −2 4   1 0 0 11 −7/2 
 2 7 −3 −5 8  ∼  0 1 0 −3 3/2
   

0 3 5 1 −3 0 0 1 2 −3/2
   
 11  −7/2
Thus, [~u]B = −3 and [~v]B =  3/2 .
   
2 −3/2
   
Section 4.3 Solutions 91

4.3.3 (a) We need to find c1 , c2 , c3 and d1 , d2 , d3 such that


" # " # " # " #
1 1 1 0 0 1 1 −3
c1 + c2 + c3 =
0 −1 1 1 1 2 2 3
" # " # " # " #
1 1 1 0 0 1 −1 0
d1 + d2 + d3 =
0 −1 1 1 1 2 3 7

Row reducing the corresponding doubly augmented matrix gives

0 1 −1   1 0 0 −2 −2 
   
 1 1
  
 1 0 1 −3 0   0 1 0 3 1 
 ∼
 0 1 1 2 3   0 0 1 −1 2 

   
−1 1 2 3 7 0 0 0 0 0

Hence, for the first system we have c1 = −2, c2 = 3, and c3 = −1, and for the second system we
have d1 = −2, d2 = 1, and d3 = 2. Thus,
   
−2 −2
[A]B =  3  and [B]B =  1 
   
−1 2
   

(b) We need to find c1 , c2 , c3 and d1 , d2 , d3 such that


" # " # " # " #
1 1 0 1 2 0 0 1
c1 + c2 + c3 =
1 0 1 1 0 −1 1 2
" # " # " # " #
1 1 0 1 2 0 −4 1
d1 + d2 + d3 =
1 0 1 1 0 −1 1 4

Row reducing the corresponding doubly augmented matrix gives

0 −4   1 0 −2 −2 
   
 1 0 2  
0

 1 1 0 1 1   0 1 0 3 3 
 ∼
 1 1 0 1 1   0 0 1 1 −1 

   
0 1 −1 2 4 0 0 0 0 0

Hence, for the first system we have c1 = −2, c2 = 3, and c3 = 1, and for the second system we
have d1 = −2, d2 = 3, and d3 = −1. Thus,
   
−2 −2
[A]B =  3  and [B]B =  3 
   
1 −1
   

4.3.4 (a) We need to find c1 , c2 , c3 and d1 , d2 , d3 such that

c1 (1 + x + x2 ) + c2 (1 + 3x + 2x2 ) + c3 (4 + x2 ) = −2 + 8x + 5x2
d1 (1 + x + x2 ) + d2 (1 + 3x + 2x2 ) + d3 (4 + x2 ) = −4 + 8x + 4x2
92 Section 4.3 Solutions

Row reducing the corresponding doubly augmented matrix gives


   
 1 1 4 −2 −4   1 0 0 5 2 
 1 3 0 8 8  ∼  0 1 0 1 2


  
1 2 1 5 4 0 0 1 −2 −2
   
5  2 
Hence, [p(x)]B = 1 and [q(x)]B =  2 .
   
2 −2
   

(b) We need to find c1 , c2 , c3 and d1 , d2 , d3 such that

c1 (1 + x − 4x2 ) + c2 (2 + 3x − 3x2 ) + c3 (3 + 6x + 4x2 ) = −3 − 5x − 3x2


d1 (1 + x − 4x2 ) + d2 (2 + 3x − 3x2 ) + d3 (3 + 6x + 4x2 ) = x

Row reducing the corresponding doubly augmented matrix gives


   
 1 2 3 −3 0   1 0 0 −14 −17 
 1 3 6 −5 1  ∼  0 1 0 13 16 
   
−4 −3 4 −3 0 0 0 1 −5 −5
  

   
−14 −17
Hence, [p(x)]B =  13  and [q(x)]B =  16 .
   
−5 −5
   

4.3.5 To find the change of coordinates matrix from B-coordinates to C-coordinates, we need to determine the
C-coordinates of the vectors in B. That is, we need to find c1 , c2 , d1 , d2 such that
" # " # " # " # " # " #
2 5 3 2 5 5
c1 + c2 = , d1 + d2 =
1 2 1 1 2 3
We row reduce the corresponding doubly augmented matrix to get
" # " #
2 5 3 5 1 0 −1 5

1 2 1 3 0 1 1 −1
" #
−1 5
Thus, C PB = .
1 −1
To find the change of coordinates matrix from C-coordinates to B-coordinates, we need to determine the
B-coordinates of the vectors in C. That is, we need to find c1 , c2 , d1 , d2 such that
" # " # " # " # " # " #
3 5 2 3 5 5
c1 + c2 = , d1 + d2 =
1 3 1 1 3 2
We row reduce the corresponding doubly augmented matrix to get
" # " #
3 5 2 5 1 0 1/4 5/4

1 3 1 2 0 1 1/4 1/4
" #
1/4 4/5
Thus, B PC = .
1/4 1/4
Section 4.3 Solutions 93

4.3.6 To find B PC we need to find the B-coordinates of the vectors in C. Thus, we need to solve the systems

b1 (1) + b2 (−1 + x) + b3 (1 − 2x + x2 ) = 1 + x + x2
c1 (1) + c2 (−1 + x) + c3 (1 − 2x + x2 ) = 1 + 3x − x2
d1 (1) + d2 (−1 + x) + d3 (1 − 2x + x2 ) = 1 − x − x2

Row reducing the corresponding triply augmented matrix gives


   
 1 −1 1 1 1 1   1 0 0 3 3 −1 
 0 1 −2 1 3 −1  ∼  0 1 0 3 1 −3 
   
0 0 1 1 −1 −1 0 0 1 1 −1 −1
 
3 3 −1
Thus, B PC = 3 1 −3.
 
1 −1 −1
 

Similarly, to find C PB we find the C-coordinates of the vectors in B by row reducing the corresponding
triply augmented matrix to get
   
 1 1 1 1 −1 1   1 0 0 1/2 −1/2 1 
 1 3 −1 0 1 −2  ∼  0 1 0 0 1/4 −3/4 
   
1 −1 −1 0 0 1 0 0 1 1/2 −3/4 3/4

 
1/2 −1/2 1 
Thus, C PB =  0 1/4 −3/4.
 
1/2 −3/4 3/4
 

4.3.7 (a) To find S PB we need to find the S-coordinates of the vectors in B. We have
     
 1   1   1 
2 2 2
[1 − x + x ]S = −1 , [1 − 2x + 3x ]S = −2 , [1 − 2x + 4x ]S = −2


 

 

 

  
1 3 4
     

 
 1 1 1 
Hence, S PB = −1 −2 −2.
 
1 3 4
 
To find B PS we need to find the B-coordinates of the vectors in S. Thus, we need to solve the
systems

b1 (1 − x + x2 ) + b2 (1 − 2x + 3x2 ) + b3 (1 − 2x + 4x2 ) = 1
c1 (1 − x + x2 ) + c2 (1 − 2x + 3x2 ) + c3 (1 − 2x + 4x2 ) = x
d1 (1 − x + x2 ) + d2 (1 − 2x + 3x2 ) + d3 (1 − 2x + 4x2 ) = x2

Row reducing the corresponding triply augmented matrix gives


   
 1 1 1 1 0 0   1 0 0 2 1 0 
 −2 −2 −2 0 1 0  ∼  0 1 0 −2 −3 −1 
   
1 3 4 0 0 1 0 0 1 1 2 1
94 Section 4.3 Solutions
 
 2 1 0 
Hence, BPS = −2 −3 −1.
 
1 2 1
 

(b) We have     
 1 1 1   3   2 
[p(x)]S = S PB [p(x)]B = −1 −2 −2  1  = −1
    
1 3 4 −2 −2
    

Hence, p(x) = 2 − x − 2x2 .


(c) We have     
 2 1 0  1  2 
[q(x)]B = B PS [q(x)]S = −2 −3 −1 0 = −3
    
1 2 1 1 2
    

(d) We have     
 1 1 1   2  1
[r(x)]S = S PB [r(x)]B = −1 −2 −2 −3 = 0
    
1 3 4 2 1
    

Hence, r(x) = 1 + x2 , as expected.


4.3.8 To find the change of coordinates matrix from B to S we need to find the coordinates of the vectors in
B with respect to S. We have
" # " # " # " #
3 0 1 0 0 1 0 0
=3 +0 +3
0 3 0 0 0 0 0 1
" # " # " # " #
−1 3 1 0 0 1 0 0
= −1 +3 +0
0 0 0 0 0 0 0 1
" # " # " # " #
1 −3 1 0 0 1 0 0
=1 + (−3) +1
0 1 0 0 0 0 0 1

Hence,  
3 −1 1 
S PB =  0 3 −3
 
3 0 1
To find the change of coordinates matrix from S to B we need to find the coordinates of the vectors in S
with respect to B. We need to find a1 , a2 , a3 , b1 , b2 , b3 , c1 , c2 , c3 such that
" # " # " # " #
1 0 3 0 −1 3 1 −3
= a1 + a2 + a3
0 0 0 3 0 0 0 1
" # " # " # " #
0 1 3 0 −1 3 1 −3
= b1 + b2 + b3
0 0 0 3 0 0 0 1
" # " # " # " #
0 0 3 0 −1 3 1 −3
= c1 + c2 + c3
0 1 0 3 0 0 0 1
Section 4.3 Solutions 95

Row reducing the augmented matrix to the corresponding system of equations gives
   
 3 −1 1 1 0 0   1 0 0 1/3 1/9 0 
 0 3 −3 0 1 0  ∼  0 1 0 −1 0 1 
   
3 0 1 0 0 1 0 0 1 −1 −1/3 1

Hence,  
1/3 1/9 0
B PS = 
 −1 0 1


−1 −1/3 1

It is easy to check that B PS S PB = I.


 
1 2 2
4.3.9 Let A = 2 4 1. Find a matrix B such that AB = I. We can think of A as being the change of
 
3 8 0
 
      

 1 2 2
     
 
coordinates matrix S PB from the basis B =  2 , 4 , 1 to the standard basis S for R3 . Hence,


3 8 0
       

Theorem 4.3.3 tells us that we can take B = B PS . So, we need find the B-coordinates of the vectors in
S. That is, we need to solve
       
1 2 2 1
b1 2 + b2 4 + b3 1 = 0
       
3 8 0 0
       
       
1 2 2 0
c1 2 + c2 4 + c3 1 = 1
       
3 8 0 0
       
       
1 2 2 0
d1 2 + d2 4 + d3 1 = 0
       
3 8 0 1
       

Row reducing the corresponding triply augmented matrix gives


   
 1 2 2 1 0 0   1 0 0 −4/3 8/3 −1 
 2 4 1 0 1 0  ∼  0 1 0 1/2 −1 1/2 
   
3 8 0 0 0 1 0 0 1 2/3 −1/3 0

Thus, we take  
−4/3 8/3 −1 
B = B PS =  1/2 −1 1/2
 
2/3 −1/3 0
 

It is easy to verify that AB = I.


96 Section 4.3 Solutions

4.3.10 If B = {~v1 , . . . , ~vn } is a basis for V, show that {[~v1 ]B , . . . , [~vn ]B } is a basis for Rn . Consider

~0 = c1 [~v1 ]B + · · · + cn [~vn ]B

Then, by Theorem 4.3.2, we get


~0 = [c1~v1 + · · · + cn~vn ]B

Hence, by definition of B-coordinates we get that

c1~v1 + · · · + cn~vn = 0~v1 + · · · + 0~vn = ~0

Thus, since B is linearly independent, this implies that c1 = · · · = cn = 0. Consequently, {[~v1 ]B , . . . , [~vn ]B }
is a linearly independent set of n vectors in Rn . Hence, by Theorem 4.2.3, it is a basis for Rn .
4.3.11 Observe that [~vi ]B = ~ei . Thus, we have that [~vi ]C = ~ei and so

~vi = 0~
w1 + · · · + 0~
wi−1 + 1~ wi+1 + · · · + 0~
wi + 0~ ~i
wn = w

Thus, ~vi = w
~ i for all i.
Chapter 5 Solutions

5.1 Problem Solutions


5.1.1 (a) Using the result of Example 5.1.4 we get
" #−1 " # " #
2 1 1 4 −1 1 4 −1
= =
−3 4 2(4) − 1(−3) −(−3) 2 11 3 2

(b) Using the result of Example 5.1.4 we get


" #−1 " # " #
1 3 1 −6 −3 1 −6 −3
= =−
1 −6 1(−6) − 3(1) −1 1 9 −1 1
" #
2 4
(c) Using the result of Example 5.1.4 we get is not invertible since 2(6) − 4(3) = 0.
3 6
(d) Row reducing the multiple augmented system [A | I] we get
   
 1 0 0 1 0 0   1 0 0 1 0 0 
0 2 0 0 1 0 ∼ 0 1 0 0 1/2 0
   
   
0 0 3 0 0 1 0 0 1 0 0 1/3
 −1  
1 0 0 1 0 0 
Thus, 0 2 0 = 0 1/2 0 .
   
0 0 3 0 0 1/3
   

(e) Row reducing the multiple augmented system [A | I] we get


   
 1 2 3 1 0 0   1 0 −1 0 −8/3 5/3 
 4 5 6 0 1 0  ∼  0 1 2 0 7/3 −4/3 
   
7 8 9 0 0 1 0 0 0 1 −2 1

Therefore, A is not invertible since the RREF of A is not I.

97
98 Section 5.1 Solutions

(f) Row reducing the multiple augmented system [A | I] we get


   
 1 2 1 1 0 0   1 0 0 −4 −2/3 5/3 
 0 −2 4 0 1 0  ∼  0 1 0 2 1/6 −2/3 

  
3 4 4 0 0 1 0 0 1 1 1/3 −1/3

 −1  
1 2 1 −4 −2/3 5/3 
Thus, 0 −2 4 =  2 1/6 −2/3.
   
3 4 4 1 1/2 −1/3
   

(g) Row reducing the multiple augmented system [A | I] we get


   
 3 1 −2 1 0 0   1 0 0 1/10 −3/10 1/2 
 1 2 1 0 1 0  ∼  0 1 0 1/10 7/10 −1/2 
   
2 1 1 0 0 1 0 0 1 −3/10 −1/10 1/2
  

 −1  
3 1 −2  1/10 −3/10 1/2 
Thus, 1 2 1  =  1/10 7/10 −1/2.
   
2 1 1 −3/10 −1/10 1/2
   

(h) Row reducing the multiple augmented system [A | I] we get


   
 1 0 1 1 0 0   1 0 0 1 0 −1/3 
 0 2 −3 0 1 0  ∼  0 1 0 0 1/2 1/2 
   
0 0 3 0 0 1 0 0 1 0 0 1/3

 −1  
1 0 1  1 0 −1/3
Thus, 0 2 −3 = 0 1/2 1/2 .
   
0 0 3 0 0 1/3
   

(i) Row reducing the multiple augmented system [A | I] we get

 −1 0 −1 2 1 0 0 0   −1 0 −1
   
2 1 0 0 0 
 1 −1 −2 4 0 1 0 0   0 −1 −3 6 1 1 0 0 
  ∼  
 −3 −1 5 2 0 0 1 0   0 0 11 −10 −4 −1 1 0 

  
−1 −2 4 4 0 0 0 1 0 0 0 0 1 −1 −1 1

Therefore, A is not invertible since the RREF of A is not I.


(j) Row reducing the multiple augmented system [A | I] we get
   
 1 2 3 2 1 0 0 0   1 0 0
 
0 4 3 6 7 
 1 1 1 1 0 1 0 0   0 1 0 0 −5 −2 −7 −8 

 −1 −3 ∼ 
1 4 0 0 1 0   0 0 1 0 5 1 6 7 

  
0 1 −3 −5 0 0 0 1 0 0 0 1 −4 −1 −5 −6
 −1  
 1 2 3 2 
  4 3 6 7 
 1 1 1 1  −5 −2 −7 −8
Thus,  
  =  .
−1 −3 1 4 
 


5 1 6 7 

0 1 −3 −5 −4 −1 −5 −6
Section 5.1 Solutions 99

5.1.2 Row reducing the multiple augmented system [A | I] we get


   
 1 −2 2 1 0 0   1 0 0 −3 −10 −2 
 −1 1 0 0 1 0  ∼  0 1 0 −3 −9 −2 
   
3 −2 −3 0 0 1 0 0 1 −1 −4 −1
 
−3 −10 −2
Thus, A−1 = −3 −9 −2.
 
−1 −4 −1
 

(a) The solution to A~x = d~ is  


11
~x = A−1 d~ =  9 
 
4
 

(b) The solution to A~x = d~ is  


−3
~x = A−1 d~ = −3
 
−1
 

(c) The solution to A~x = d~ is  11 √ 


 6 − 2 2

~x = A−1 d~ =  32 − 2 2 
 
 5 √ 
6 − 2

5.1.3 Using the result of Example 5.1.4 we get


"√ #−1 " 3# " #
2 3/2 1 0 −
√2 = − 4 0 2

= √   1
1/2 0 3 1
2(0) − 2 2 − 2 2 3 2/3 −4 2/3

(a) The solution to A~x = d~ is # "


−1 ~0
~x = A d =
2/3

(b) The solution to A~x = d~ is " #


2
~x = A−1 d~ = √
−2 2/3

(c) The solution to A~x = d~ is " #


−1 ~ √ 2π √
~x = A d = 2 4
3 3 − 3 2π
h i
5.1.4 We need to find all 3 × 2 matrices B = ~b1 ~b2 such that
h i h i
~e1 ~e2 = AB = A~b1 A~b2
100 Section 5.1 Solutions

This gives us two systems A~b1 = ~e1 and A~b2 = ~e2 . We solve both by row reducing a double augmented
matrix. We get " # " #
3 1 0 1 0 1 0 0 2/5 −1/5

1 2 0 0 1 0 1 0 −1/5 3/5
   
 2/5  0
Thus, the general solution of the first system is −1/5 + t 0 and the general solution of the second
   
0 1
   
   
−1/5 0
system is s  3/5  + s 0. Therefore, every right inverse of A has the form
   
0 1
   

 
 2/5 −1/5
B = −1/5 3/5 
 
t s
 

h i
5.1.5 We need to find all 3 × 2 matrices B = ~b1 ~b2 such that
h i h i
~e1 ~e2 = AB = A~b1 A~b2

This gives us two systems A~b1 = ~e1 and A~b2 = ~e2 . We solve both by row reducing a double augmented
matrix. We get " # " #
1 −2 1 1 0 1 0 1 −1 2

1 −1 1 0 1 0 1 0 −1 1
   
−1 −1
Thus, the general solution of the first system is −1 + t  0  and the general solution of the second
   
0 1
   
   
−1
  −1
 
system is s  1  + s  0 . Therefore, every right inverse of A has the form
   
0 1
   

 
−1 − t 2 − s
B =  −1 1 
 
t s
 

" #T
1 −2 1
5.1.6 Observe that B = . Also, observe that if AB = I2 , then (AB)T = I2 and so BT AT = I2 . Thus,
1 −1 1
" #
−1 − t −1 t
from our work in Problem 5, we have that all left inverses of B are .
2−s 1 s
5.1.7 To find all
" left inverses
# of B, we use the same trick we did in Problem 6. We will find all right inverses
1 0 3
of BT = . We get
2 −1 3
" # " #
1 0 3 1 0 1 0 3 1 0

2 −1 3 0 1 0 1 3 2 −1
Section 5.1 Solutions 101
   
1 −3
Thus, the general solution of the first system is 2 + t −3 and the general solution of the second system
   
0 1
   
   
 0  −3
is s −1 + s −3. Therefore, every left inverse of B has the form
 
0 1
   
" #
1 − 3t 2 − 3t t
B=
−3s −1 − 3s s
5.1.8 (a) Using our work in Example 4, we get that
" # " #
−1 1 1 −1 −1 1
A = =
2(1) − 1(3) −3 2 3 −2
" # " #
1 5 −2 −5 2
B−1 = =
1(5) − 2(3) −3 1 3 −1
#"
5 9
(b) We have AB = and hence
6 11
" # " #
−1 1 11 −9 11 −9
(AB) = = = B−1 A−1
5(11) − 9(6) −6 5 −6 5
# "
4 2
(c) We have 2A = , so
6 2
" # " #
−1 1 2 −2 −1/2 1/2 1
(2A) = = = A−1
4(2) − 2(6) −6 4 3/2 −1 2
" # " # " #
2 3 1 1 −3 −1 3
(d) We have AT = , so (AT )−1 = 2(1)−3(1) = and
1 1 −1 2 1 −2
" #" # " #
T T −1 2 3 −1 3 1 0
A (A ) = =
1 1 1 −2 0 1
5.1.9 We have
AT (A−1 )T = [A−1 A]T = I T = I
Hence, (AT )−1 = (A−1 )T by Theorem 5.1.4.
5.1.10 Consider
c1 A~v1 + · · · + ck A~vk = ~0
Then, we have
A(c1~v1 + · · · + ck~vk ) = ~0
Since A is invertible, the only solution to this equation is
c1~v1 + · · · + ck~vk = A−1~0 = ~0
Hence, we get c1 = · · · = ck = 0, because {~v1 , . . . , ~vk } is linearly independent.
102 Section 5.1 Solutions

5.1.11 (a) Consider B~x = ~0. Multiplying on the left by A gives

AB~x = A~0 = ~0

Since AB is invertible, this has the unique solution ~x = (AB)−1~0 = ~0. Thus, B is invertible.
Since AB and B−1 are invertible, we have that A = (AB)B−1 is invertible by Theorem 5.1.5(3).
" #
1 0 0
(b) Let C = and D = C T . Then C and D are both not invertible since they are not square,
0 1 0
" #
1 0
but CD = is invertible.
0 1
h i
5.1.12 Assume that A has a right inverse B = ~b1 · · · ~bm . Then, we have
h i h i h i
~e1 · · · ~em = Im = AB = A ~b1 ··· ~bm = A~b1 · · · A~bm

Thus, A~bi = ~ei . Then, for any ~y = y1~e1 + · · · + ym~em ∈ Rm , we have

~y = y1~e1 + · · · + ym~em = y1 A~b1 + · · · + ym A~bm = A(y1~b1 + · · · + ym~bm )

Hence, A~x = ~y is consistent for all ~y ∈ Rm . Therefore, rank A = m by Theorem 2.2.4(3). But then we
must have n ≥ m which is a contradiction.
5.1.13 (a) The statement if false. The 3 × 2 zero matrix cannot have a left inverse.
(b) The statement is true. By the Invertible Matrix Theorem, we get that if Null(AT ) = {~0}, then AT is
invertible. Hence, A is invertible by Theorem 5.1.5.
" # " # " #
1 0 −1 0 0 0
(c) The statement is false. A = and B = are both invertible, but A + B = is
0 1 0 −1 0 0
not invertible.
" #
1 0
(d) The statement is false. A = satisfies AA = A, but A is not invertible.
0 0
(e) Let ~b ∈ Rn . Consider the system of equations A~x = ~b. Then, we have

AA~x = A~b
~x = (AA)−1 A~b

Thus, A~x = ~b is consistent for all ~b ∈ Rn and hence A is invertible by the Invertible Matrix
Theorem.
" # " #
0 0 1 0
(f) The statement is false. If A = and B = , then AB = O2,2 and B is invertible.
0 0 0 1
(" # " # " # " #)
1 0 1 0 0 1 0 1
(g) The statement is true. One such basis is , , , .
0 1 0 2 1 0 2 0
(h) If A has a column of zeroes, then AT has a row of zeroes and hence rank(AT ) < n. Thus, AT is not
invertible and so A is also not invertible by the Invertible Matrix Theorem.
Section 5.1 Solutions 103

(i) If A~x = ~0 has a unique solution, then rank(A) = n by Theorem 2.2.5 and so A is invertible and
hence the columns of A form a basis for Rn by the Invertible Matrix Theorem. Thus, Col(A) = Rn ,
so the statement is true.
(j) If rank(AT ) = n, then AT is invertible by the Invertible Matrix theorem, and hence A is also
invertible by the Invertible Matrix Theorem. Thus, rank(A) = n by Theorem 5.1.6.
(k) If A is invertible, then AT is also invertible. Thus, the columns of AT form a basis for Rn and hence
must be linearly independent.
(l) We have that I = −A2 + 2A = A(−A + 2I). Thus, A is invertible by Theorem 5.1.4. In particular
A−1 = −A + 2I.
" # " # " # " # " #
1 1 2 1 2 −1 2 −1 5 3
5.1.14 (a) Let A = and B = . Then, A−1 = and B−1 = . Then, AB =
1 2 3 2 −1 1 −3 2 8 5
" # " #
5 −3 5 −4
and (AB)−1 = , but A−1 B−1 = .
−8 5 −5 3
(b) If (AB)−1 = A−1 B−1 = (BA)−1 , then we must have AB = BA.
104 Section 5.2 Solutions

5.2 Problem Solutions


5.2.1 (a) The matrix is elementary. The corresponding row operation is R1 + 3R2 .
(b) The matrix is not elementary.
(c) The matrix is elementary. The corresponding row operation is R2 − 4R1 .
(d) The matrix is elementary. The corresponding row operation is R2 ↔ R3 .
(e) The matrix is not elementary.
(f) The matrix is elementary. The corresponding row operation is (−4)R3 .
(g) The matrix is not elementary.
(h) The matrix is elementary. The corresponding row operation is R1 ↔ R4 .
(i) The matrix is not elementary.
(j) The matrix is not elementary.
 
1 0 −2
5.2.2 (a) 0 1 0 
 
0 0 1
 
 
0 1 0
(b) 1 0 0
 
0 0 1
 
 
1 √0 0
(c) 0 2 0
 
 
0 0 1
 
1 0 0
(d) 0 1 0
 
0 2 1
 
 
1 0 0
(e) 0 0 1
 
0 1 0
 
 
3 0 0
(f) 0 1 0
 
0 0 1
 
 
1 0 0
(g) 2 1 0
 
0 0 1
 
 
1 0 0 
(h) 0 1 0 
 
0 0 1/2
 
Section 5.2 Solutions 105

5.2.3 (a) We row reduce A to I keeping track of our elementary row-operations.


       
 1 2   1 2   1 2  R1 − 2R2  1 0 
 −1 1  R2 + R1 ∼  0 3  13 R2 ∼ 0 1  ∼  0 1  = R
    
  
2 1 R3 − 2R1 0 −3 0 −3 R3 + 3R2 0 0
    

Hence,
         
1 0 0  1 0 0 1 0 0 1 −2 0 1 0 0
E1 = 1 1 0 E2 =  0 1 0 E3 = 0 1/3 0 E4 = 0 1 0 E5 = 0 1 0
         
0 0 1 −2 0 1 0 0 1 0 0 1 0 3 1
         

and E5 E4 E3 E2 E1 A = R. Then

A = E1−1 E2−1 E3−1 E4−1 E5−1 R


      
 1 0 0 1 0 0 1 0 0 1 2 0 1 0 0  1 0 
= −1 1 0 0 1 0 0 3 0 0 1 0 0 1 0  0 1
      

0 0 1 2 0 1 0 0 1 0 0 1 0 −3 1 0 0
     

(b) We row reduce A to I keeping track of our elementary row-operations.


" # " # " # " #
1 −1 2 1 −1 2 1 −1 2 R1 + R2 1 0 1
∼ ∼ ∼ =R
2 1 1 R2 − 2R1 0 3 −3 31 R2 0 1 −1 0 1 −1
" # " # " #
1 0 1 0 1 1
Hence, E1 = E2 = E3 = and E3 E2 E1 A = R. Then
−2 1 0 1/3 0 1
" #" #" #" #
−1 −1 −1 1 0 1 0 1 −1 1 0 1
A = E1 E2 E3 R =
2 1 0 3 0 1 0 1 −1

(c) We row reduce A to I keeping track of our elementary row-operations.


    1
 0 1 3   2 2 4  2 R1
 0 −1 −2  R1 ↔ R3 ∼  0 −1 −2  (−1)R2 ∼
   
2 2 4 0 1 3
     
 1 1 2  R1 − R2  1 0 0   1 0 0 
 0 1 2  ∼  0 1 2  R2 − 2R3 ∼ 0 1 0  = R
     
0 1 3 R3 − R2 0 0 1 0 0 1
  

Hence,
     
0 0 1 1/2 0 0 1 0 0
E1 = 0 1 0 E2 =  0 1 0 E3 = 0 −1 0
     
1 0 0 0 0 1 0 0 1
     
     
1 −1 0 1 0 0 1 0 0 
E4 = 0 1 0 E5 = 0 1 0 E6 = 0 1 −2
     
0 0 1 0 −1 1 0 0 1
     
106 Section 5.2 Solutions

and E6 E5 E4 E3 E2 E1 A = R. Then
A = E1−1 E2−1 E3−1 E4−1 E5−1 E6−1 R
       
0 0 1 2 0 0 1 0 0 1 1 0 1 0 0 1 0 0  1 0 0 
= 0 1 0 0 1 0 0 −1 0 0 1 0 0 1 0 0 1 2  0 1 0 
       
1 0 0 0 0 1 0 0 1 0 0 1 0 1 1 0 0 1 0 0 1
       

(d) We row reduce A to I keeping track of our elementary row-operations.


   
 2 4 1   1 2 2 
 1 2 2  1 R ↔ R2
∼  2 4 1  R2 − 2R1 ∼
 
 
1 2 2 1 2 2 R3 − R1
 
     
 1 2 2   1 2 2  R1 − 2R3  1 2 0 
 1
 0 0 −3  − 3 R2 ∼  0 0 1  ∼ 0 0 1  = R
    
0 0 0 0 0 0 0 0 0
   

Hence,
         
0 1 0  1 0 0  1 0 0 1 0 0 1 0 −2
E1 = 1 0 0 E2 = −2 1 0 E3 =  0 1 0 E4 = 0 −1/3 0 E5 = 0 1 0 
         
0 0 1 0 0 1 −1 0 1 0 0 1 0 0 1
         

and E5 E4 E3 E2 E1 A = R. Then
A = E1−1 E2−1 E3−1 E4−1 E5−1 R
      
0 1 0 1 0 0 1 0 0 1 0 0 1 0 2  1 2 0 
= 1 0 0 2 1 0 0 1 0 0 −3 0 0 1 0  0 0 1 
      
0 0 1 0 0 1 1 0 1 0 0 1 0 0 1 0 0 0
      

(e) We row reduce A to I keeping track of our elementary row-operations.


     
 1 0 2   1 0 2   1 0 2 
 0 1 3   0 1 3   0 1 3 

 −1 1 1

 R3 + R1 ∼  
 0 1 3  R3 − R2 ∼ 
 0
 ∼
 0 0 

    1
−2 2 5 R4 + 2R1 0 2 9 R4 − 2R2 0 0 3 3 R4
 1 0 2  R1 − 2R3  1 0
     
 1 0 2  0 
 0 1 3   0 1 3  R − 3R  0 1 0 
  ∼   2 3
∼   = R
 0 0 0   0 0 1   0 0 1 
     
0 0 1 R3 ↔ R4 0 0 0 0 0 0
Hence,
       
1 0 0 0
 1 0 0 0
 1 0 0 0 1 0 0 0

0 1 0 0 0 1 0 0 0 1 0 0
 E = 0 1 0 0

E1 =  E2 =  E3 =  4
1 0 1 0 0 0 1 0 0 −1 1 0 0 0 1 0
  
   
0 0 0 1 2 0 0 1 0 0 0 1 0 −2 0 1
1 0 −2 0
       
1 0 0 0 
 1 0 0 0
 1 0 0 0

0 1 0 0  0 1 0 0  

 0 1 0 0

 0 1 −3 0
E5 =  E6 =   E7 =   E8 = 
0 0 1 0  0 0 0 1 0 0 1 0 0 0 1 0
 
  
0 0 0 1/3 0 0 1 0 0 0 0 1 0 0 0 1
Section 5.2 Solutions 107

and E8 E7 E6 E5 E4 E3 E2 E1 A = R. Then

A =E1−1 E2−1 E3−1 E4−1 E5−1 E6−1 E7−1 E8−1 R


     
 1 0 0 0  1 0 0 0 1 0 0 0 1 0 0

0 1

0 0 0

 0 1 0 0  0 1 0 0 0 1 0 0 0 1 0 0 0 1 0 0
=     
−1 0 1 0  0 0 1 0 0 1 1 0 0 0 1 0 0 0 1 0
  
  
0 0 0 1 −2 0 0 1 0 0 0 1 0 2 0 1 0 0 0 3
    
1 0 0 0 1 0 2 0 1 0 0 0  1 0 0 
  
0 1 0 0 0 1 0 0 0 1 3 0  0 1 0 
  
0 0 0 1 0 0 1 0 0 0 1 0  0 0 1 
 
    
0 0 1 0 0 0 0 1 0 0 0 1 0 0 0

(f) We row reduce A to I keeping track of our elementary row-operations.


     
 1 2 1 2   1 2 1 2   1 2 1 2  R1 − 2R2
0 0 1 2  ∼  0 0 1 2 ∼ 0 1 4 8  ∼
     
 
−1 −1 3 6 R3 + R1 0 1 4 8 R2 ↔ R3 0 0 1 2
   
   
 1 0 −7 −14  R1 + 7R3  1 0 0 0 
0 1 4 8  R2 − 4R3 ∼  0 1 0 0  = R
   

0 0 1 2 0 0 1 2
 

Hence,
         
1 0 0 1 0 0 1 −2 0 1 0 7 1 0 0 
E1 = 0 1 0 E2 = 0 0 1 E3 = 0 1 0 E4 = 0 1 0 E5 = 0 1 −4
         
1 0 1 0 1 0 0 0 1 0 0 1 0 0 1
         

and E5 E4 E3 E2 E1 A = R. Then

A =E1−1 E2−1 E3−1 E4−1 E5−1 R


      
 1 0 0 1 0 0 1 2 0 1 0 −7 1 0 0  1 0 0 0 
=  0 1 0 0 0 1 0 1 0 0 1 0  0 1 4  0 1 0 0 
      
−1 0 1 0 1 0 0 0 1 0 0 1 0 0 1 0 0 1 2
      

5.2.4 (a) We row reduce A to I keeping track of our elementary row-operations.


" # " # " # " #
2 1 2 1 R1 − R2 1 0 1 0
∼ ∼ ∼
3 3 13 R2 1 1 1 1 R2 − R1 0 1
# " #" #"
1 0 1 −1 1 0
Hence, E1 = E2 = E3 = and E3 E2 E1 A = I. Then, A−1 = E3 E2 E1
0 1/3 0 1 −1 1
and
" #" #" #
1 0 1 1 1 0
A = E1−1 E2−1 E3−1 =
0 3 0 1 1 1
108 Section 5.2 Solutions

(b) We row reduce A to I keeping track of our elementary row-operations.


       
 1 1 3   1 1 3  R1 − R2  1 0 4  R1 − 4R3  1 0 0 
 1 2 2  R2 − R1 ∼  0 1 −1  ∼ 0 1 −1  R2 + R3 ∼  0 1 0 
       
1 1 1 R3 − R1 0 0 −2 − 12 R3 0 0 1 0 0 1
   

Hence,
       
 1 0 0  1 0 0 1 −1 0 1 0 0 
E1 = −1 1 0 E2 =  0 1 0 E3 = 0 1 0 E4 = 0 1 0 
       
0 0 1 −1 0 1 0 0 1 0 0 −1/2
       
   
1 0 −4 1 0 0
E5 = 0 1 0  E6 = 0 1 1
   
0 0 1 0 0 1
   

and E6 E5 E4 E3 E2 E1 A = I. Hence,

A−1 = E6 E5 E4 E3 E2 E1

and

A =E1−1 E2−1 E3−1 E4−1 E5−1 E6−1


      
1 0 0 1 0 0 1 1 0 1 0 0  1 0 4 1 0 0 
= 1 1 0 0 1 0 0 1 0 0 1 0  0 1 0 0 1 −1
      
0 0 1 1 0 1 0 0 1 0 0 −2 0 0 1 0 0 1
      

(c) We row reduce A to I keeping track of our elementary row-operations.


" # " # " # " #
0 3 1 −2 1 −2 1 0
R1 ↔ R2 ∼ 1 ∼ R1 + 2R2 ∼
1 −2 0 3 3 R2 0 1 0 1

Hence,
" # "# " #
0 1 1 0 1 2
E1 = E2 = E3 =
1 0 0 1/3 0 1

and E3 E2 E1 A = I. Hence,
A−1 = E3 E2 E1
and
#"
" #" #
0 1 1 0 1 −2
A =E1−1 E2−1 E3−1 =
1 0 0 3 0 1
Section 5.2 Solutions 109

(d) We row reduce A to I keeping track of our elementary row-operations.


     
0 2 6   1 4 4  1 4 4  R1 − 4R2
 R ↔ R2 
 
1 4 4  1 ∼  0 2 6 1
2 R2 ∼ 0 1 3 ∼
   
  
−1 2 8 −1 2 8 R3 + R1 0 6 12 R3 − 6R2
  
     
 1 0 −8   1 0 −8  R1 + 8R3  1 0 0 
0 1 3  ∼  0 1 3  R2 − 3R3 ∼  0 1 0 
     
  1
0 0 −6 − 6 R3 0 0 1 0 0 1
  

Hence,
       
0 1 0 1 0 0 1 0 0 1 −4 0
E1 = 1 0 0 E2 = 0 1/2 0 E3 = 0 1 0 E4 = 0 1 0
       
0 0 1 0 0 1 1 0 1 0 0 1
       
       
1 0 0 1 0 0  1 0 8 1 0 0 
E5 = 0 1 0 E6 = 0 1 0  E7 = 0 1 0 E8 = 0 1 −3
       
0 −6 1 0 0 −1/6 0 0 1 0 0 1
       

and E8 E7 E6 E5 E4 E3 E2 E1 A = I. Hence,

A−1 = E8 E7 E6 E5 E4 E3 E2 E1

and

A =E1−1 E2−1 E3−1 E4−1 E5−1 E6−1 E7−1 E8−1


        
0 1 0 1 0 0  1 0 0 1 4 0 1 0 0 1 0 0  1 0 −8 1 0 0
= 1 0 0 0 2 0  0 1 0 0 1 0 0 1 0 0 1 0  0 1 0  0 1 3
        
0 0 1 0 0 1 −1 0 1 0 0 1 0 6 1 0 0 −6 0 0 1 0 0 1
        

(e) We row reduce A to I keeping track of our elementary row-operations.

2 0 21 R1
" # " #
4 2
R1 − R2 ∼ ∼
2 2 2 2 12 R2
" # " #
1 0 1 0

1 1 R2 − R1 0 1

Hence, " # # " " # "


#
1 −1 1/2 0 1 0 1 0
E1 = E2 = E3 = E4 =
0 1 0 1 0 1/2 −1 1
and E4 E3 E2 E1 A = I. Hence,
A−1 = E4 E3 E2 E1
and
" #" #" #" #
1 1 2 0 1 0 1 0
A =E1−1 E2−1 E3−1 E4−1 =
0 1 0 1 0 2 1 1
110 Section 5.2 Solutions

(f) We row reduce A to I keeping track of our elementary row-operations.


        
 1 0 −1   1 0 −1   1 0 −1  1 0 −1 
 R1 + R3 
 
 −2 0 −2  R2 + 2R1 ∼  0 0 −4  R ↔ R ∼ 0 1 0 ∼  0 1 0  ∼
      

2 3
−4 1 4 R3 + 4R1 0 1 0 0 0 −4 − 41 R3 0 0 1
  

Hence,
     
1 0 0 1 0 0 1 0 0
E1 = 2 E2 = 0 1 0 E3 = 0 0 1
1 0
     
0 0 1 4 0 1 0 1 0
     
   
1 0 0  1 0 1
E4 = 0 1 0  E5 = 0 1 0
   
0 0 −1/4 0 0 1
   

and E5 E4 E3 E2 E1 A = I. Hence,
A−1 = E5 E4 E3 E2 E1
and

A =E1−1 E2−1 E3−1 E4−1 E5−1


     
 1 0 0  1 0 0 1 0 0 1 0 0  1 0 −1
= −2 1 0  0 1 0 0 0 1 0 1 0  0 1 0 
         
0 0 1 −4 0 1 0 1 0 0 0 −4 0 0 1
     

(g) We row reduce A to I keeping track of our elementary row-operations.


" # " #
1 3 1 3
∼ 1 ∼
−3 1 R2 + 3R1 0 10 10 R2
" # " #
1 3 1 0
R1 − 3R2 ∼
0 1 0 1

Hence, # " " # " #


1 0 1 0 1 −3
E1 = E2 = E3 =
3 1 0 1/10 0 1
and E3 E2 E1 A = I. Hence,
A−1 = E3 E2 E1
and
" #" #" #
1 0 1 0 1 3
A =E1−1 E2−1 E3−1 =
−3 1 0 10 0 1
Section 5.2 Solutions 111

(h) We row reduce A to I keeping track of our elementary row-operations.

 1 −2  1 −2 4 1 


      
4 1  1 0 4 1  1 0 0
R + 2R R − 2R
 
 −1 3 −4 −1  R + R  0 1 0 0  1 2  0 1 0 0  1 3  0 1 0
  2 1
∼   
 ∼   ∼ 
 0 1 2 0  0 1 2 0   0 0 2 0   0 0 2
  
  R3 − R2
  
−2 4 −8 −1 R4 + 2R1 0 0 0 1 0 0 0 1 0 0 0

Hence,
     
1 0 0 0
 1 0 0 0
 1 2 0 0

1 1 0 0 0 1 0 0 0 1 0 0
E1 =  E2 =  E3 = 
0 0 1 0 0 0 1 0 0 0 1 0
  
  
0 0 0 1 2 0 0 1 0 0 0 1
0 −2 0 0 −1
       
1 0 0 0
 1  1 0
 1 0 0 0

0 1 0 0 0 1 0 0 0 1 0 0  0 1 0 0
E4 =  E5 =   E6 =  E7 = 
0 −1 1 0 0 0 1 0 0 0 1 0  0 0 1/2 0
  
   
0 0 0 1 0 0 0 1 0 0 0 1 0 0 0 1

and E7 E6 E5 E4 E3 E2 E1 A = I. Hence,

A−1 = E7 E6 E5 E4 E3 E2 E1

and

A =E1−1 E2−1 E3−1 E4−1 E5−1 E6−1 E7−1


0 1 −2 0 0 1
       
1 0 0 0  1 0 0   
0 0 0 1

0 2 0 1

0 0 1 1

0 0 0

1 −1 0 0  0 1 0 0 0 1 0 0 0 1 0 0 0 1 0 0 0 1 0 0 0 1 0 0
=   
0 0 1 0  0 0 1 0 0 0 1 0 0 1 1 0 0 0 1 0 0 0 1 0 0 0 2 0
     
     
0 0 0 1 −2 0 0 1 0 0 0 1 0 0 0 1 0 0 0 1 0 0 0 1 0 0 0 1

5.2.5 Since multiplying on the left is the same as applying the corresponding elementary row operations we
get
" #" #" #" #" #
1 2 1/2 0 0 1 1 0 1 1
A=
0 1 0 1 1 0 −3 1 0 3
" #" #" #" #
1 2 1/2 0 0 1 1 1
=
0 1 0 1 1 0 −3 0
" #" #" #
1 2 1/2 0 −3 0
=
0 1 0 1 1 1
" #" #
1 2 −3/2 0
=
0 1 1 1
" #
1/2 2
=
1 1
112 Section 5.3 Solutions

5.2.6 (a) We row reduce A to I keeping track of our elementary row-operations.


" # " #
1 2 1 2
∼ R1 − R2 ∼
2 6 R2 − 2R1 0 2
" # " #
1 0 1 0
1 ∼
0 2 2 R2 0 1

Hence, " # " # "#


1 0 1 −1 1 0
E1 = E2 = E3 =
−2 1 0 1 0 1/2
and E3 E2 E1 A = I.
(b) Since multiplying on the left is the same as applying the corresponding elementary row operations
we get
" #" #" #
1 0 1 −1 3
~x =
0 1/2 0 1 −1
" #" #
1 0 4
=
0 1/2 −1
" #
4
=
−1/2

(c) We have
" # " #
1 2 3 1 2 3
∼ R1 − R2 ∼
2 6 5 R2 − 2R1 0 2 −1
" # " #
1 0 4 1 0 4
1 ∼
0 2 −1 2 R2 0 1 −1/2

5.2.7 If A is invertible, then by Corollary 5.2.7, there exists a sequence of elementary matrices E1 , . . . , Ek such
that A = E1−1 · · · Ek−1 . If B is row equivalent to A, then there exists a sequence of elementary matrices
F1 , . . . , Fk such that Fk · · · F1 A = B. Thus,

B = Fk · · · F1 E1−1 · · · Ek−1

Therefore, B is a product of invertible matrices and hence is invertible by Theorem 5.1.5(3).


5.2.8 Let B = AE. Taking transposes of both sides gives BT = E T AT . We observe that the transpose of an
elementary matrix is still an elementary matrix. Thus, BT is the matrix obtained from A by performing
the elementary row operation associated with E T on AT . But, the rows of AT are just the columns of B.
Thus, multiplying on the right by an elementary performs an elementary column operation on A.
Section 5.3 Solutions 113

5.3 Problem Solutions


5.3.1 The cofactor matrix is
" #
8 −5
(a)
1 4
 
 6 −3 3 
(b)  3 6 −1
 
−6 −1 2
 
 
 11 6 −8
(c) −3 8 7 
 
10 −9 12
 
 
−29 −13 −4
(d) −19 7 16 
 
−11 23 14
 

2 1
5.3.2 (a) = 2(4) − 1(−3) = 11
−3 4

1 3
(b) = 1(−6) − 3(1) = −9
1 −6

4 2
(c) = 4(3) − 2(6) = 0
6 3

1 2 −1
(d) 0 2 7 = 1(2)(3) = 6
0 0 3

3 0 −4
3+2 3 −4
(e) −2 0 5 = (−1)(−1)
= 3(5) − (−4)(−2) = 7
3 −1 8 −2 5

1 2 1
1+3 5 2
(f) 5 2 0 = 1(−1) = 5(0) − 2(3) = −6
3 0 0 3 0

(g) Using row operations to simplify the determinant we get



1 2 3 1 2 3
−1 3 −8 = 0 5 −5

2 −5 2 0 −9 −4

Expanding along the first column we get



1+1 5 −5
= 1(−1) = 5(−4) − (−5)(−9) = −65
−9 −4
114 Section 5.3 Solutions

(h) Using a column operation we get



2 3 1 5 3 1
−2 2 0 = 0 2 0

1 3 4 4 4 4

Expanding along the second column gives



2+2 5 1
= 2(−1) = 2[5(4) − 1(4)] = 32
4 4

(i) Using row operations we get



6 8 −8 6 8 −8 6 8 −8
7 5 8 = 13 13 0 = 13(4) 1 1 0 = 0

−2 −4 8 4 4 0 1 1 0

(j) Using row operations we get



1 10 7 −9 1 10 7 −9 0 11 6 −10
7 −7 7 7 1 −1 1 1 1 −1 1 1
= (7) = 7
2 −2 6 2 2 −2 6 2 0 0 4 0

−3 −3 4 1 −3 −3 4 1 0 −6 7 4

Expanding along the third row we get



0 11 −10
det B = (7)(4) 1 −1 1
0 −6 4

Now expanding along the first column gives



11 −10
det B = 28(−1) = (−28)(44 − 60) = 448
−6 4

(k) Using row operations we get

a2 a3

1 a
0 b − a b2 − a2 b3 − a3
det A = 2 2 3 3
0 c − a c − a c − a
0 d − a d2 − a2 d3 − a3

b − a b2 − a2 b3 − a3
= c − a c2 − a2 c3 − a3
d − a d2 − a2 d3 − a3

1 b + a b2 + ba + a2
= (b − a)(c − a)(d − a) 1 c + a c2 + ca + a2
1 d + a d2 + da + a2
Section 5.3 Solutions 115

1 b + a b2 + ba + a2
= (b − a)(c − a)(d − a) 0 c − b c2 + ca − b2 − ba
0 d − b d2 + da − b2 − ba

c − b c2 + ca − b2 − ba
= (b − a)(c − a)(d − a)
d − b d2 + da − b2 − ba

1 a + b + c
= (b − a)(c − a)(d − a)(c − b)(d − b)
1 a + b + d
= (b − a)(c − a)(d − a)(c − b)(d − b)(d − c)

(l) Using row operations we get



−1 2 6 4 −1 2 6 4


3 5 6

0 3 5 6 0 3 5 6
= = (−1)(−1)1+1 1 10 2
1 −1 4 −2 0 1 10 2
4 7 6
1 2 1 2 0 4 7 6

0 −25 0
1+2 1 2
= (−1) 1 10
2 = (−1)(−25)(−1)
= −25(−2) = 50
0 −33 −2 0 −2

5.3.3 If A is invertible, then AA−1 = I. By Theorem 5.3.9 we get

1 = det I = det(AA−1 ) = det A det A−1

Since A is invertible, det A , 0 and so we get

1
det A−1 =
det A
5.3.4 By Theorem 5.3.9 and Corollary 5.3.10 we get

1
det B = det(P−1 AP) = det P−1 det A det P = det A det P = det A
det P
5.3.5 (a) We have AAT = AA−1 = I. Hence, by Theorem 5.3.9 and Corollary 5.3.10 we get

1 = det I = det(AAT ) = det A det AT = det A det A = (det A)2

Hence, det A = ±1.


" #
1 0
(b) Take A = . Then A = AT and det A = −1.
0 −1
5.3.6 (a) The statement is true. If the columns of A are linearly independent, then A is invertible and hence,
det A , 0 by the Invertible Matrix Theorem.
" # " # " #
1 0 −1 0 0 0
(b) The statement is false. If A = and det B = , then det(A + B) = det = 0, but
0 1 0 −1 0 0
det A + det B = 1 + 1 = 1.
116 Section 5.3 Solutions
" # " #
0 0 0
(c) The statement is false. Take A = and ~b = . Then, A~x = ~b is consistent, but det A = 0.
0 0 0
(d) The statement is true. By Theorem 5.3.9 we get 0 = det(AB) = det A det B. Hence, det A = 0 or
det B = 0.
5.3.7 We will use induction on "n. # " #
a b c d
Base Case: n = 2. If A = , then B = . Hence,
c d a b

det A = ad − bc = −(bc − ad) = − det B

Inductive Hypothesis: Assume that the result holds for any (n − 1) × (n − 1) matrix.
Inductive Step: Suppose that B is an n × n matrix obtained from A by swapping two rows. If the i-th
row of A was not swapped, then the cofactors of the i-th row of B are (n − 1) × (n − 1) matrices which
can be obtained from the cofactors of the i-th row of A by swapping the same two rows. Hence, by the
inductive hypothesis, the cofactors Ci∗j of B and Ci j of A satisfy Ci∗j = −Ci j . Hence,

∗ ∗
det B = ai1Ci1 + · · · + ainCin = ai1 (−Ci1 ) + · · · + ain (−Cin )

= − ai1Ci1 + · · · + ainCin = − det A
Section 5.4 Solutions 117

5.4 Problem Solutions


" #
5 −3
5.4.1 (a) We have adj A = . Hence,
2 1
" #
11 0
A(adj A) = = (det A)I
0 11
" #
3 −5
(b) We have adj A = . Hence,
1 4
" #
17 0
A(adj A) = = (det A)I
0 17
" #
10 −3
(c) We have adj A = . Hence,
−4 1
" #
−2 0
A(adj A) = = (det A)I
0 −2

 
6 −6 16 
(d) We have adj A = 0 3 −7. Hence,
 
0 0 2
 

 
6 0 0
A(adj A) = 0 6 0 = (det A)I
 
0 0 6
 

 
−1 4 −2
(e) We have adj A =  2 −9 5 . Hence,
 
3 −11 6
 

 
1 0 0
A(adj A) = 0 1 0 = (det A)I
 
0 0 1
 

 
−1 −8 −4
(f) We have adj A = −2 −12 −4. Hence,
 
−2 −8 −4
 

 
4 0 0
A(adj A) = 0 4 0 = (det A)I
 
0 0 4
 
118 Section 5.4 Solutions
" #
d −b
5.4.2 (a) We have det A = ad − bc and adj A = , thus
−c a
" #
−1 1 d −b
A =
ad − bc −c a
 
−1 t + 3 −3 
(b) We have det A = −17 − 2t and adj A =  5 2 − 3t −2t − 2. Hence,
 
−2 −11 −6
 

 
−1 t + 3 −3 
1
A−1 =  5 2 − 3t −2t − 2
 
−17 − 2t 
−2 −11 −6

 
4t + 1 −1 −3t − 1
(c) We have det A = 14t + 1 and adj A =  −10 14 11 . Hence,
 
3 + 2t −4 2t − 3
 

 
4t + 1 −1 −3t − 1
−1 1 
A =  −10 14 11 

14t + 1 
3 + 2t −4 2t − 3

 
 cd 0 −cb
(d) We have det A = acd − b2 c and adj A =  0 ad − b2 0 . Hence,
 
−cb 0 ac
 

 
 cd 0 −cb
1
A−1 = 0 ad − b2 0 
 
acd − b2 c 

−cb 0 ac

5.4.3 Solve the following systems of linear equations using Cramer’s Rule.
" #
3 −1
(a) The coefficient matrix is A = , so det A = 21 + 4 = 25. Hence,
4 7

1 2 −1 19
x1 = =
25 5 7 25


1 3 2 7
x2 = =
25 4 5 25

" #
19/25
Thus, the solution is ~x = .
7/25
Section 5.4 Solutions 119
" #
2 1
(b) The coefficient matrix is A = , so det A = 14 − 3 = 11. Hence,
3 7

1 1 1 9
x1 = =
11 −2 7 11


1 2 1 7
x2 = =−
11 3
−2 11
" #
9/11
Thus, the solution is ~x = .
−7/11
" #
2 1
(c) The coefficient matrix is A = , so det A = 14 − 3 = 11. Hence,
3 7

1 3 1 16
x1 = =
11 5 7 11


1 2 3 1
x2 = =
11 3 5 11

" #
16/11
Thus, the solution is ~x = .
1/11
 
 2 3 1 
(d) The coefficient matrix is A =  1 1 −1, so det A = 6. Hence,
 
−2 0 2
 


1 3 1
1 4
x1 = −1 1 −1 =
6 6
1 0 2

2 1 1
1 −3
x2 = 1 −1 −1 =
6 6
−2 1 2

2 3 1
1 7
x3 = 1 1 −1 =
6 6
−2 0 1
 
 2/3 
Hence, the solution is ~x = −1/2.
 
7/6
 
120 Section 5.5 Solutions
 
5 1 −1
(e) The coefficient matrix is A = 9 1 −1, so det A = −16. Hence,
 
1 −1 5
 


4 1 −1
1 12
x1 = 1 1 −1 =
−16 −16
2 −1 5

5 4 −1
1 −166
x2 = 9 1 −1 =
−16 −16
1 2 5

5 1 4
1 −42
x3 = 9 1 1 =
−16 −16
1 −1 2

 
−3/4
Thus, the solution is ~x =  83/7 .
 
21/8
 

5.5 Problem Solutions


" #
2 3
5.5.1 (a) A = det = |2(2) − 3(−3)| = 13
−3 2
" #
4 −1
(b) A = det = |4(1) − (−1)(1)| = 5
1 1
" #
4 −3
(c) A = det = |4(3) − (−3)(1)| = 15
1 3
" #
2 −2
(d) A = det = |2(2) − (−2)(3)| = 10
3 2
   
 5 1 1  0 6 31 

5.5.2 (a) V = det −1 4 2  = det 0 3 −4 = 117
   
 1 −1 −6 1 −1 6 
   
1 1 −2 1 1 −2
   
(b) V = det 1 3 1  = det 0 2 3  = 6
4 4 −5 0 0 3
   
   
2 2 1  0 0 1
  
(c) V = det 3 −1 5 = det −7 −11 5 = 63

4 −5 2 0 −9 2
   
     
1 1 1 1 1 1
 
1 1  1 1 1 1 
  
1 1 2 0 0 0 1 −1 0 0 1 −1
5.5.3 V = det   = det 
 = det 
 = 13
2 1 3 5 0 −1 1 3  0 −1 1 3 
    
1 3 0 7 0 2 −1 6 0 0 1 12
Section 5.5 Solutions 121

5.5.4 Since adding a multiple of one row to another does not change the determinant, we get
h i h i
V = | det ~v1 · · · ~vn | = | det ~v1 · · · ~vn + t~v1 |

5.5.5 (a) We have


h i h i
area(L(~u), L(~v)) = det L(~u) L(~v) = det [L]~u [L]~v
h i h i
= det([L] ~u ~v = det[L] det ~u ~v
= | det[L]| area(~u, ~v)

(b) The area scaling factor is " #


2 8
| det[L]| = det = 18
−1 5

(c) The area scaling factor is


| det[M ◦ L]| = | det[M][L]|
Chapter 6 Solutions

6.1 Problem Solutions


6.1.1 (a) We have
" # " # " #
−1 1 2
L(1, −2) = =1 −1
−3 −2 1
" # " # " #
−1 1 2
L(2, 1) = = (−4) +6
−3 −2 1

Hence,
h i " 1 −4#
[L]B = [L(1, −2)]B [L(2, 1)]B =
−1 6

(b) We have
" # " # " #
6 1 1
L(1, 1) = = 10 −4
2 1 2
" # " # " #
10 1 1
L(1, 2) = = 19 −9
1 1 2

Hence,
h i " 10 19 #
[L]B = [L(1, 1)]B [L(1, 2)]B =
−4 −9

(c) We have
" # " # " #
6 −3 1
L(−3, 5) = = −2 +0
−10 5 −2
" # " # " #
−1 −3 1
L(1, −2) = =0 −1
2 5 −2

Hence,
h i "−2 0 #
[L]B = [L(−3, 5)]B [L(1, −2)]B =
0 −1

122
Section 6.1 Solutions 123

(d) We have
       
4 1  1   0 
L(1, 1, 1) = 0 = (−1) 1 + 5 −1 − 6 −1
       
5 1 0 −1
       
       
 1  1  1   0 
L(1, −1, 0) = −1 = 1 1 + 0 −1 + 2 −1
       
−1 1 0 −1
       
       
−3 1  1   0 
L(0, −1, −1) =  0  = 0 1 − 3 −1 + 3 −1
       
−3 1 0 −1
       

Hence,  
h i −1 1 0 
[L]B = [L(1, 1, 1)]B [L(1, −1, 0)]B [L(0, −1, −1)]B =  5 0 −3
 
−6 2 3
 

(e) We have
       
 16   2  1  0 
L(2, −1, −1) = −2 = (−1) −1 + 5 1 − 6  0 
       
10 −1 1 −1
       
       
 1   2  1  0 
L(1, 1, 1) = −1 = (−1) −1 + 5 1 − 6  0 
       
−1 −1 1 −1
       
       
−3  2  1  0 
0 −1
L(0, 0, −1) =   = (−1)   + 5   − 6  0  1
       
−3 −1 1 −1
       

Hence,  
h i −1 1 0 
[L]B = [L(2, −1, −1)]B [L(1, 1, 1)]B [L(0, 0, −1)]B =  5 0 −3
 
−6 2 3
 

(f) We have
       
0 0 2 1
L(0, 1, 0) = 2 = 2 1 + 0 0 + 0 0
       
0 0 1 1
       
       
4 0 2 1
L(2, 0, 1) = 0 = 0 1 + 2 0 + 0 0
       
2 0 1 1
       
       
−1 0 2 1
L(1, 0, 1) =  0  = 0 1 + 0 0 − 1 0
       
−1 0 1 1
       
124 Section 6.1 Solutions

Hence,  
h i 2 0 0 
[L]B = [L(0, 1, 0)]B [L(2, 0, 1)]B [L(1, 0, 1)]B = 0 2 0 
 
0 0 −1
 

(g) We have
       
0 0 −1 1
L(0, 1, 0) = 2 = 2 1 + 0  0  + 0 1
       
0 0 1 1
       
       
−1 0 −1 1
L(−1, 0, 1) =  0  = 0 1 + 1  0  + 0 1
       
1 0 1 1
       
       
3 0 −1 1
L(1, 1, 1) = 4 = 2 1 − 1  0  + 2 1
       
1 0 1 1
       

Hence,  
h i 2 0 2 
[L]B = [L(0, 1, 0)]B [L(−1, 0, 1)]B [L(1, 1, 1)]B = 0 1 −1
 
0 0 2
 

6.1.2 (a) We have  


 4 
[L(~x)]B = [L]B [~x]B = 19
 
15
 

Thus,        
2 −1 1  4 
L(~x) = 4 1 + 19  0  + 15 1 = 19
       
0 1 0 19
       

(b) We have  
 0 
[L(~y)]B = [L]B [~y]B = −2
 
3
 

Thus,        
2 −1 1 5
L(~y) = 0 1 − 2  0  + 3 1 = 3
       
0 1 0 1
       
Section 6.1 Solutions 125

(c) From our work on similar matrices, we have that [L]B = P−1 [L]P where P is the change of
coordinates matrix from B-coordinates to standard coordinates. Thus,
  
2 −1 1 0 1 0
[L] = P[L]B P−1 = 1 0 1 0 4 2] = [] [
   
0 1 0 3 3 0
  

6.1.3 (a) We have  


 6 
[L(~x)]B = [L]B [~x]B = 13
 
2
 

Thus,        
1 0 1  8 
L(~x) = 6 1 + 13 1 + 2 0 = 19
       
0 1 1 15
       

(b) We have  
−3
[L(~y)]B = [L]B [~y]B =  6 
 
2
 

Thus,        
1 0 1 5
L(~y) = 3 1 + 6 1 + 2 0 = 9
       
0 1 1 8
       

(c) From our work on similar matrices, we have that [L]B = P−1 [L]P where P is the change of
coordinates matrix from B-coordinates to standard coordinates. Thus,
  
1 0 1 0 0 3 
[L] = P[L]B P−1 = 1 1 0 5 0 −1] =
[] [
  
0 1 1 0 2 2
  

6.1.4 (a) ("


Our geometrically
# " #) natural basis should include ~a and a vector orthogonal to ~a. We pick B =
3 1
, . Then we have
−1 3
# " # "
3 1
proj~a (~a) = ~a = 1 +0
−1 3
" #! " # " # " #
1 0 3 1
proj~a = =0 +0
3 0 −1 3

Therefore, " #
1 0
[proj~a ]B =
0 0
126 Section 6.1 Solutions

(b) ("
For #our
" geometrically
#) natural basis, we include ~b and a vector orthogonal to ~b. We pick B =
1 −1
, . Then we have
1 1
" # " #
~ ~ 1 −1
proj~b (b) = b = 1 +0
1 1
" #! " # " #
−1 1 −1
proj~b = ~0 = 0 +0
1 1 1

Hence, # "
1 0
[perp~b ]B =
0 0

(c) ("
For #our
" geometrically
#) natural basis, we include ~b and a vector orthogonal to ~b. We pick B =
1 −1
, . Then we have
1 1
" # " #
~ ~ 1 −1
perp~b (b) = 0 = 0 +0
1 1
" #! " # " # " #
−1 −1 1 −1
perp~b = =0 +1
1 1 1 1

Hence, " #
0 0
[perp~b ]B =
0 1

(d) For our geometrically natural basis, we include the normal vector ~n for the plane of reflection and a
basis for the plane. To pick a basis for the plane, we need to pick a set of two linearly independent
vectors which are orthogonal to ~n. Thus, we pick
      

  1  0 1
     
 
B=  0  , 1 , 0

−1 0 1

 

Then we get
     
 1  0 1
refl~n (~n) = −~n = −  0  + 0 1 + 0 0
     
−1 0 1
     
         
0 0  1  0 1
refl~n 1 = 1 = 0  0  + 1 + 0 0
         
0 0 −1 0 1
         
         
1 1  1  0 1
refl~n 0 = 0 = 0  0  + 0 1 + 1 0
         
1 1 −1 0 1
         
Section 6.1 Solutions 127

Hence,  
−1 0 0
[refl~n ]B =  0 1 0
 
0 0 1
 

6.1.5 Let A, B, and C be n × n matrices.


(a) Taking P = I we get I −1 AI = A, so by definition A is similar to A.
(b) If A is similar to B, then there exists an invertible matrix P such that P−1 AP = B. If B is similar to
C, then there exists an invertible matrix Q such that Q−1 BQ = C. Hence,

Q−1 P−1 APQ = Q−1 BQ = C

Thus,
(PQ)−1 A(PQ) = C
So, A is similar to C.
6.1.6 If AT A is similar to I, then there exists an invertible matrix P such that

P−1 AT AP = I

Hence, AT A = PP−1 = I, so AT = A−1 .


If AT = A−1 , then AT A = I. Taking P = I we get I −1 AT AI = I, so AT A is similar to I.
6.1.7 We prove the result by induction.
Base Case: n = 1: If A is similar to B, then there exists an invertible matrix P such that P−1 AP = B, so
A1 is similar to B1 .
Inductive Hypothesis: Assume that for some positive integer k we have P−1 Ak P = Bk .
Inductive Step: We have

Bk+1 = Bn B = (P−1 Ak P)(P−1 AP) = P−1 Ak PP−1 AP = P−1 Ak AP = P−1 Ak+1 P

Thus, by induction, An is similar to B n for all positive integers n.


6.1.8 If A and B are similar, then there exists an invertible matrix P such that P−1 AP = B which we can
rewrite as P−1 A = BP−1 . Since P is invertbile, there exists elementary matrices E1 , . . . , Ek such that
P−1 = Ek · · · E1 . Thus, we have

rank(Ek · · · E1 A) = rank(BEk · · · E1 )
rank(A) = rank(BEk · · · E1 ) by Corollary 5.2.5
T T
rank(A ) = rank([BEk · · · E1 ] )
rank(AT ) = rank(E1T · · · EkT BT )
rank(AT ) = rank(BT ) by Corollary 5.2.5
rank(A) = rank(B)

as required.
128 Section 6.2 Solutions
# " " #
1 0 1 1
6.1.9 (a) The statement is false. Let A = and B = , then rank A = rank B, but tr A , tr B, so A
0 0 2 2
and B cannot be similar by Theorem 6.1.1.
(b) The statement is true. If A and B are similar, then there exists an invertible matrix P such that
P−1 AP = B. Thus, B is a product of invertible matrices, so B is invertible by Theorem 5.1.5.
" # " #
2 0 1 0
(c) The statement is false. Let A = . Its RREF is R = . We have det A , det R, so A and
0 0 0 0
R cannot be similar by Theorem 6.1.1.
(d) The statement is true. If A is similar to the diagonal matrix D = diag(d11 , d22 , . . . , dnn ), then
there exists an invertible matrix P such that P−1 AP = D and so A = PDP−1 . By Theorem 5.3.2,
Theorem 5.3.9, and Corollary 5.3.10 we get

1
det A = det(PDP−1 ) = det P det D det P−1 = det D det P = det D = d11 d22 · · · dnn
det P

6.2 Problem Solutions


6.2.1 (a) We have     
−1 1 −4 1 −4
A~v1 =  5 −5 −4 1 = −4
    
−5 11 10 1 16
    

is not a scalar multiple of ~v1 . Thus, ~v1 is not an eigenvector.


(b) We have       
−1 1 −4  2   12   2 
A~v2 =  5 −5 −4  2  =  12  = 6  2 
       
−5 11 10 −3 −18 −3
      

Thus, ~v2 is an eigenvector.


(c) By definition, the zero vector cannot be an eigenvector.
6.2.2 (a) We have       
 1 3 3  −1 −1 −1
A~v1 =  6 7 12  −1 = −1 = 1 −1
       
−3 −3 −5 1 1 1
      

Thus, ~v1 is an eigenvector.


(b) We have       
 1 3 3   3  −6  3 
A~v2 =  6 7 12  −6 =  12  = −2 −6
       
−3 −3 −5 3 −6 3
      

Thus, ~v2 is an eigenvector.


Section 6.2 Solutions 129

(c) We have     
 1 3 3   2  −1
A~v3 =  6 7 12  −2 =  10 
    
−3 −3 −5 1 −5
    

is not a scalar multiple of ~v3 . Thus, ~v3 is not an eigenvector.


6.2.3 (a) We have
3 − λ 2
C(λ) = det(A − λI) = = λ2 − 8λ + 7 = (λ − 7)(λ − 1)
4 5 − λ
Hence, the eigenvalues are λ1 = 7 and λ2 = 1 with aλ1 = 1 = aλ2 . Therefore, by Theorem 6.2.4,
we have that gλ1 = 1 = gλ2 .
(b) We have
−2 − λ 0
C(λ) = det(A − λI) = = λ2 + 4λ + 4 = (λ + 2)2
5 −2 − λ
Thus, λ1 = −2 is the only eigenvalue and aλ1 = 2. We have
" # " #
0 0 1 0
A − λ1 I = ∼
5 0 0 0
(" #)
0
Hence, a basis for Eλ1 is . So, gλ1 = 1.
1
(c) We have
4 − λ −3 √ √
C(λ) = det(A − λI) = = λ2 − 7 = (λ − 7)(λ + 7)
3 −4 − λ

√ √
Hence, the eigenvalues are λ1 = 7 and λ2 = − 7. We have aλ1 = 1 = aλ2 . Therefore, by
Theorem 6.2.4, gλ1 = 1 = gλ2 .
(d) We have

−3 − λ −2 −2
−3 − λ −2
C(λ) = det(A − λI) = 2
2−λ 1 = (1 − λ)

2 2 − λ
0 0 1 − λ
= −(λ − 1)(λ2 + λ − 2) = −(λ − 1)2 (λ + 2)

Hence, the eigenvalues are λ1 = 1 with aλ1 = 2, and λ2 = −2 with aλ2 = 1. Since λ2 has algebraic
multiplicity 1 it also has geometric multiplicity 1 by Theorem 6.2.4. For λ1 we get
   
−4 −2 −2 1 1/2 1/2
A − λ1 I =  2 1 1  ∼ 0 0 0 
   
0 0 0 0 0 0
   

    

 −1 −1
   
 
Thus, a basis for Eλ1 is   2  ,  0  . Thus, λ1 has geometric multiplicity 2.


  0   2 
 

130 Section 6.2 Solutions

(e) We have

−3 − λ 6 −2
C(λ) = det(A − λI) = −1 2 − λ −1
1 −3 −λ

−3 − λ 6 −2 −3 − λ 8 −2
= 0 −1 − λ −1 − λ = 0 0 −1 − λ
1 −3 −λ 1
−3 + λ −λ
= −(λ + 1)(λ2 − 1) = −(λ + 1)2 (λ − 1)

Hence, the eigenvalues are λ1 = −1 with algebraic multiplicity 2 and λ2 = 1 with algebraic mul-
tiplicity 1. Since λ2 has algebraic multiplicity 1 it also has geometric multiplicity 1 by Theorem
6.2.4. For λ1 we get
   
−2 6 −2 1 −3 1
A − λ1 I = −1 3 −1 ∼ 0 0 0
   
1 −3 1 0 0 0
   

Thus, rank(A − λ1 I) = 1, so gλ1 = 3 − 1 = 2 by Theorem 2.2.6.


(f) We have

4 − λ 2 2
C(λ) = det(A − λI) = 2 4−λ 2
2 2 4 − λ

4 − λ 0 2 4 − λ 0 2
= 2 2−λ 2 = 2 2−λ 2
2 −2 + λ 4 − λ 4 0 6 − λ
= −(λ − 1)(λ2 − 10λ + 16) = −(λ − 2)2 (λ − 8)

Hence, the eigenvalues are λ1 = 2 with aλ1 = 2, and λ2 = 8 with aλ2 = 1. Since λ2 has algebraic
multiplicity 1 it also has geometric multiplicity 1 by Theorem 6.2.4. For λ1 we get
   
2 2 2 1 1 1
A − λ1 I = 2 2 2 ∼ 0 0 0
   
2 2 2 0 0 0
   

Thus, rank(A − λ1I) = 1, so gλ1 = 3 − 1 = 2 by Theorem 2.2.6.


(g) We have

1 − λ 3 1
1 − λ 2
C(λ) = det(A − λI) = 0
1−λ 2 = (1 − λ)

2 1 − λ
0 2 1 − λ
= −(λ − 1)(λ2 − 2λ − 3) = −(λ − 1)(λ − 3)(λ + 1)

Hence, the eigenvalues are λ1 = 1, λ2 = 3, and λ3 = −1 all with algebraic multiplicity 1. Thus,
they all have geometric multiplicity 1 by Theorem 6.2.4.
Section 6.2 Solutions 131

(h) We have

2 − λ 2 −1
C(λ) = det(A − λI) = −2 1 − λ 2
2 2 −1 − λ

1 − λ 2 −1 1 − λ 2 −1
= 0 1−λ 2 = 0 1 − λ 2
1 − λ 2 −1 − λ 0
0 −λ
= −λ(λ − 1)2

Hence, the eigenvalues are λ1 = 0 with algebraic multiplicity 1 and λ2 = 1 with algebraic mul-
tiplicity 2. Since λ1 has algebraic multiplicity 1 it also has geometric multiplicity 1 by Theorem
6.2.4. For λ2 we get
   
 1 2 −1 1 0 −1
A − λ2 I = −2 0 2  ∼ 0 1 0 
   
2 2 −2 0 0 0
   

Thus, rank(A − λ2 I) = 2, and so gλ2 = 3 − 2 = 1 by Theorem 2.2.6.


(i) We have

1 − λ 6 −4
C(λ) = det(A − λI) = 5 −3 − λ 1
−5 6 2 − λ

1 − λ 6 −4 1 − λ 10 −4
= 5 −3 − λ 1 = 5 −4 − λ 1
0 3 − λ 3 − λ 0 0 3 − λ
= (3 − λ)(λ2 + 3λ − 54) = −(λ − 3)(λ + 9)(λ − 6)

Hence, the eigenvalues are λ1 = 3, λ2 = −9, and λ3 = 6 all with algebraic multiplicity 1. Thus,
they all have geometric multiplicity 1 by Theorem 6.2.4.
(j) We have

1 − λ 0 1
C(λ) = det(A − λI) = 0 2−λ 1
0 0 2 − λ
= (1 − λ)(2 − λ)2

Hence, the eigenvalues are λ1 = 1 and λ2 = 2. The algebraic multiplicity of λ1 is 1, so the


algebraic multiplicity is also 1 by Theorem 6.2.4. For λ2 = 2, we have
   
−1 0 1 1 0 0
A − λ2 I =  0 0 1 ∼ 0 0 1
   
0 0 0 0 0 0
   

Thus, rank(A − λ2 I) = 2, and so gλ2 = 3 − 2 = 1 by Theorem 2.2.6.


132 Section 6.2 Solutions

6.2.4 Geometrically, we are looking for vectors that are mapped to a scalar multiple of themselves. For a
rotation by π/3, the only vector that will be mapped to a scalar multiple of itself is the zero vector. Thus,
Rπ/3 has no real eigenvalues and eigenvectors. Since the projection onto ~v, is always a scalar multiple of
~v, the only eigenvectors can be vectors that a parallel to ~v or orthogonal to ~v. Indeed, by definition, we
have proj~v ~x = k~~xv·~kv2 ~v. Thus, if ~x = t~v, for t , 0, then we get

(t~v) · ~v
proj~v (t~v) = ~v = t~v = 1(t~v)
k~vk2
Thus, λ1 = 1 is an eigenvalue with Eλ1 = Span{~v}.
If ~y is any non-zero vector orthogonal to ~v, then we get

proj~v ~y = ~0 = 0~v

Hence, λ2 = 0 is an eigenvalue with Eλ2 = Span{~y}.


6.2.5 If A is invertible and ~v is an eigenvector of A, then there exists λ ∈ R such that A~v = λ~v. Multiplying
both sides on the left by A−1 gives
~v = A−1 (λ~v) = λA−1~v
Since ~v , ~0, this implies that λ , 0. Thus, A−1~v = λ1 ~v. Hence, ~v is also an eigenvector of A−1 with
eigenvalue λ1 .
" #
a b
6.2.6 Let A = . We have that
c d
" # " # " # " #
1 1 1 1
A =2 and A =3
2 2 3 3
This gives " # " # " # " #
a + 2b 2 a + 3b 3
= and =
c + 2d 4 c + 3d 9
Solving a + 2b = 2 and "a + 3b = # 3 gives b = 1 and a = 0. Solving c + 2d = 4 and c + 3d = 9 gives d = 5
0 1
and c = −6. Thus, A = .
−6 5
 
0 0 0 1
0 0 0 0
6.2.7 Let A =  . Then, observe that C(λ) = λ4 , so λ = 0 is an eigenvalue with aλ = 4. However,
0 0 0 0
0 0 0 0
since rank(A) = 1, we have by Theorem 6.2.4, that gλ = 3. Thus, gλ < aλ as required.
 
1 0 0 0
0 2 0 0
6.2.8 Let A =  . Then, observe that
0 0 3 0
0 0 0 4
C(λ) = (1 − λ)(2 − λ)(3 − λ)(4 − λ)

Thus, A has 4 distinct eigenvalues 1, 2, 3, 4.


Section 6.3 Solutions 133

6.2.9 (a) We have

(AB)(5~u + 3~v) = A(5B~u + 3B~v) = A(25~u + 9~v) = 25A~u + 9A~v = 150~u + 90~v = 30(5~u + 3~v)

So, 5~u + 3~v is an eigenvector of C with eigenvalue 30.


(b) Since ~u and ~v correspond to different eigenvalues, {~u, ~v} is linearly independent and hence a basis
for R2 . Thus, there exists coefficients c1 , c2 such that w
~ = c1~u + c2~v. Therefore

" #
6
w = AB(c1~u + c2~v) = c1 (AB)~u + c2 (AB)~v = c1 (30~u) + c2 (30~v) = 30(c1~u + c2~v) = 30~
AB~ w=
42
# " " #
1 0 1 0
6.2.10 It does not imply that λµ is an eigenvalue of AB. Let A = and B = . Then λ = 2 is an
0 2 0 1/2
" #
1 0
eigenvalue of A and µ = 1 is an eigenvalue of B, but λµ = 2 is not an eigenvalue of AB = .
0 1
6.2.11 By definition of matrix-vector multiplication, we have that

c
 

A~v =  ...  = c~v


 
 
c

Thus, ~v is an eigenvector with eigenvalue c.


6.2.12 Observe that if A is upper or lower triangular, then A − λI is also upper or lower triangular. Thus, by
Theorem 5.3.2, we have that

det(A − λI) = (a11 − λ)(a22 − λ) · · · (ann − λ)

Therefore, the eigenvalues of A are a11 , a22 , . . . , ann as required.


6.2.13 If A and B are similar, then there exists an invertible matrix P such that P−1 AP = B. We get that the
characteristic polynomial for B is

C B (λ) = det(B − λI) = det(P−1 AP − λP−1 P)


= det(P−1 (A − λI)P)
= det P−1 det(A − λI) det P
= det(A − λI)

which is the characteristic polynomial for A. Thus, the result follows.


6.2.14 If A is not invertible, then det A = 0 and hence 0 = det(A − 0I) and so 0 is an eigenvalue of A.
On the other hand, if 0 is an eigenvalue of A, then 0 = det(A − 0I) = det A and so A is not invertible.
134 Section 6.3 Solutions

6.3 Problem Solutions


6.3.1 (a) We have
3 − λ −1
C(λ) = = λ2 − 6λ + 8 = (λ − 2)(λ − 4)
−1 3 − λ
Thus, the eigenvalues of A are λ1 = 2 and λ2 = 4. Since all the eigenvalues have algebraic
multiplicity 1, we know that A is diagonalizable.
" # " #
1 −1 1 −1
For λ1 = 2 we have A − λ1 I = ∼ .
−1 1 0 0
(" #)
1
Hence, a basis for Eλ1 is .
1
" # " #
−1 −1 1 1
For λ2 = 4 we have A − λ2 I = ∼ .
−1 −1 0 0
(" #)
−1
Hence, a basis for Eλ2 is .
1
" #
1 −1
It follows that A is diagonalized by P = . The resulting diagonal matrix D has the eigen-
1 1
" #
2 0
values of A as its diagonal entries, so D = .
0 4
(b) We have
1 − λ 2
C(λ) = = λ2 − 2λ + 1 = (λ − 1)2
0 1 − λ
Thus, the only eigenvalue of A is λ1 = 1 which has algebraic multiplicity 2.
" # " #
0 2 0 1
For λ1 = 1 we have A − λ1 I = ∼ .
0 0 0 0
(" #)
1
A basis for Eλ1 is . Hence, the geometric multiplicity of λ1 is 1. Since the geometric
0
multiplicity is less than the algebraic multiplicity the matrix is not diagonalizable.
(c) We have
4 − λ −2
C(λ) = = λ2 − 11λ + 24 = (λ − 3)(λ − 8)
−2 7 − λ
Thus, the eigenvalues of A are λ1 = 3 and λ2 = 8. Since all the eigenvalues have algebraic
multiplicity 1, we know that A is diagonalizable.
" # " #
1 −2 1 −2
For λ1 = 3 we have A − λ1 I = ∼ .
−2 4 0 0
(" #)
2
Hence, a basis for Eλ1 is .
1
Section 6.3 Solutions 135
" # " #
4 −2 1 1/2
For λ2 = 8 we have A − λ2 I = ∼ .
−2 −1 0 0
(" #)
−1
Hence, a basis for Eλ2 is .
2
" #
2 −1
It follows that A is diagonalized by P = . The resulting diagonal matrix D has the eigen-
1 2
" #
3 0
values of A as its diagonal entries, so D = .
0 8
(d) We have

5 − λ −1 1 5 − λ −1 1 5 − λ −1 2
C(λ) = −1 5 − λ 1 = 0 6 − λ 6 − λ = 0 6−λ 0
1 1 5 − λ 1
1 5 − λ 1
1 4 − λ
= (6 − λ)(λ2 − 9λ + 18) = −(λ − 6)2 (λ − 3)

Thus, the eigenvalues of A are λ1 = 6 and λ2 = 3.


   
−1 −1 1  1 1 −1
For λ1 = 6 we have A − λ1 I = −1 −1 1  ∼ 0 0 0 .
   
1 1 −1 0 0 0
   

    

 −1 1
   
 
Hence, a basis for Eλ1 is   1  , 0 .



 0 

1
   

   
 2 −1 1 1 0 1
For λ2 = 3 we have A − λ2 I = −1 2 1 ∼ 0 1 1.
   
1 1 2 0 0 0
   

  

 −1
 
 
Hence, a basis for Eλ2 is  −1 .

 1 

   

 
−1 1 −1
It follows that A is diagonalized by P =  1 0 −1. The resulting diagonal matrix D has the
 
0 1 1
 
 
6 0 0
eigenvalues of A as its diagonal entries, so D = 0 6 0.
0 0 3
 

(e) We have

1 − λ 0 −1
C(λ) = 2 2−λ 1 = (2 − λ)(λ2 − 6λ + 9) = −(λ − 2)(λ − 3)2
4 0 5 − λ
136 Section 6.3 Solutions

Thus, the eigenvalues of A are λ1 = 2 and λ2 = 3.


   
−2 0 −1 1 0 1/2
For λ2 = 3 we have A − λ2 I =  2 −1 1  ∼ 0 1 0 .
   
4 0 2 0 0 0
   
  

 −1
 
 
Hence, a basis for Eλ2 is   0  . Consequently, the matrix is not diagonalizable since gλ2 < aλ2 .

  2 

 

(f) We have

1 − λ 1 2 2 − λ 1 2 2 − λ 1 2
C(λ) = 2 −λ 1 = 2 − λ −λ 1 = 0 −1 − λ −1
1 −1 −1 − λ 0
−1 −1 − λ 0
−1 −1 − λ
= (2 − λ)(λ2 + 2λ) = −λ(λ − 2)(λ + 2)
Thus, the eigenvalues of A are λ1 = 0, λ2 = 2, and λ3 = −2.
   
1 1 2  1 0 1/2
For λ1 = 0 we have A − λ1 I = 2 0 1  ∼ 0 1 3/2.
   
1 −1 −1 0 0 0
   
  

 −1
 
 
Hence, a basis for Eλ1 is  −3 .


  2 
 

   
−1 1 2  1 −1 0
For λ2 = 2 we have A − λ2 I =  2 −2 1  ∼ 0 0 1.
   
1 −1 −3 0 0 0
   
  

 1
 
 
Hence, a basis for Eλ2 is  1 .

0

   

   
3 1 2 1 0 3/4 
For λ3 = −2 we have A − λ3 I = 2 2 1 ∼ 0 1 −1/4.
   
1 −1 1 0 0 0
   
  

 −3
 
 
Hence, a basis for Eλ3 is   1  .


  4 
 

 
−1 1 −3
It follows that A is diagonalized by P = −3 1 1 . The resulting diagonal matrix D has the
 
2 0 4
 
 
0 0 0 
eigenvalues of A as its diagonal entries, so D = 0 2 0 .
 
0 0 −2
 
Section 6.3 Solutions 137

(g) We have

1 − λ −2 3 2 − λ 0 2 − λ 2 − λ 0 0
C(λ) = 2 6−λ −6 = 2 6−λ −6 = 2 6−λ −8
1 2 −1 − λ 1
2 −1 − λ 1
2 −2 − λ
= (2 − λ)(λ2 − 4λ + 4) = −(λ − 2)3

Thus, the only eigenvalues of A is λ1 = 2 with algebraic multiplicity 3.


   
−1 −2 3  1 2 −3
We have A − λ1 I =  2 4 −6 ∼ 0 0 0 .
   
1 2 −3 0 0 0
   
    

 −2 3
   
 
Hence, a basis for Eλ1 is   1  , 0 . Thus, gλ1 = 2 < aλ1 , so A is not diagonalizable.



 0 

1
   

(h) We have

3 − λ 1 1 3 − λ 1 0 3 − λ 1 0
C(λ) = −4 −2 − λ −5 = −4 −2 − λ −3 + λ = −2 −λ 0
2 2 5 − λ 2
2 3−λ 2
2 3 − λ
= (3 − λ)(λ2 − 3λ + 2) = −(λ − 3)(λ − 2)(λ − 1)

Thus, the eigenvalues of A are λ1 = 3, λ2 = 2, and λ3 = 1.


   
 0 1 1  1 0 0
For λ1 = 3 we have A − λ1 I = −4 −5 −5 ∼ 0 1 1.
   
2 2 2 0 0 0
   
  

  0 
 
 
Hence, a basis for Eλ1 is  −1 .


 1 
 

   
 1 1 1  1 1 0
For λ2 = 2 we have A − λ2 I = −4 −4 −5 ∼ 0 0 1.
   
2 2 3 0 0 0
   
  

 −1
 
 
Hence, a basis for Eλ2 is   1  .

 0 

   

   
 2 1 1  1 0 −1
For λ3 = 1 we have A − λ3 I = −4 −3 −5 ∼ 0 1 3 .
   
2 2 4 0 0 0
   
  

  1 
 
 
Hence, a basis for Eλ3 is  −3 .


  1 
 

138 Section 6.3 Solutions
 
 0 −1 1 
It follows that A is diagonalized by P = −1 1 −3. The resulting diagonal matrix D has the
 
1 0 1
 
 
3 0 0
eigenvalues of A as its diagonal entries, so D = 0 2 0.
0 0 1
 

(i) We have

−3 − λ −3 5 2 − λ −3 5 2 − λ −3 5
C(λ) = 13 10 − λ −13 = 0 10 − λ −13 = 0 10 − λ −13
3 2 −1 − λ 2 − λ 2 −1 − λ 0 5 −6 − λ
= (2 − λ)(λ2 − 4λ + 5)

The roots of λ2 − 4λ + 5 are not real, so A is not diagonalizable over R.


(j) We have

8 − λ 6 −10 8 − λ 6 −10 8 − λ −4 −10
C(λ) = 5 1−λ −5 = 0 −2 − λ 2 + λ = 0 0 2 + λ
5 3 −7 − λ 5
3 −7 − λ 5
−4 − λ −7 − λ
= −(2 + λ)(λ2 − 4λ − 12) = −(λ + 2)2 (λ − 6)

Thus, the eigenvalues of A are λ1 = −2 and λ2 = 6.


   
10 6 −10 1 3/5 −1
For λ1 = −2 we have A − λ1 I =  5 3 −5  ∼ 0 0 0 .
   
5 3 −5 0 0 0
   

  


 h i 1 

Hence, a basis for Eλ1 is  −3/5 1 0 , 0 .


  
  

 1
   
2 6 −10 1 0 −2
For λ2 = 6 we have A − λ2 I = 5 −5 −5  ∼ 0 1 −1.
   
5 3 −13 0 0 0
   

  

 2
 
 
Hence, a basis for Eλ2 is  1 .


1
 

 
−3/5 1 2
It follows that A is diagonalized by P =  1 0 1. The resulting diagonal matrix D has the
 
0 1 1
 
 
−2 0 0
eigenvalues of A as its diagonal entries, so D =  0 −2 0.
 
0 0 6
 
Section 6.3 Solutions 139

(k) We have

2 − λ 1 0 2 − λ 1 0 2 − λ 1 0
C(λ) = 1 3−λ 1 = 0 3−λ 1 = 0 3−λ 1
3 1 4 − λ −1 + λ 1 4 − λ −3 + 2λ 0 4 − λ
= (2 − λ)(3 − λ)(4 − λ) − 3 + 2λ = λ3 − 9λ2 + 24λ − 21

We find that C(λ) has non-real eigenvalues and so is not diagonalizable over R.
(l) We have

1 − λ 1 1 1 1 − λ 0

0 1

1 1−λ 1 1 1 −λ 0 1
C(λ) = =
1−λ

1 1 1 1 0 −λ 1
1 − λ 1 λ λ 1 − λ
1
1 1

1 − λ 0 0 1


1 − λ 0

1
1 −λ 0 1
= = (−λ) 1 −λ 1
1 0 −λ 1
2
λ 2 − λ
λ 2 − λ
2
0

1 − λ 0 1
= (−λ) 1 −λ 1 = λ2 (λ2 − 4λ) = λ3 (λ − 4)
3 0 3 − λ

Thus, the eigenvalues of A are λ1 = 0 and λ2 = 4.


   
1 1 1 1 1 1 1 1

1 1 1 1 0 0 0 0
For λ1 = 0 we have A − λ1 I =   ∼  .
1 1 1 1 0 0 0 0

1 1 1 1 0 0 0 0
      


 −1 −1 −1 
       
 1   0   0 

 
Hence, a basis for Eλ1 is  , , .



  0   1   0 

 
 
 
 
 
 



 0       
0 1 
 

−3 1 0 −1
   
1 1  1 0
 1 −3 1   
1  0 1 0 −1
For λ2 = 4 we have A − λ2 I =  
 ∼ .
 1 1 −3 1  0 0 0 −1
  
1 1 1 −3 0 0 0 0
  


 1 

1

 

Hence, a basis for Eλ2 is  .
   
1
 

 

 



  

1
 
140 Section 6.3 Solutions

−1 −1 −1 1
 
 1 0 0 1
It follows that A is diagonalized by P =  . The resulting diagonal matrix D has
 0 1 0 1

0 0 1 1
 
0 0 0 0
0 0 0 0
the eigenvalues of A as its diagonal entries, so D =  .
0 0 0 0
0 0 0 4
" #
3 −1
(a) We have [L] = . Thus, from our work in problem 6.3.1(a), we get that if we take
−1 3
(" # " #) " #
1 −1 2 0
B= , , then [L]B = .
1 1 0 4
" #
1 3
(b) We have [L] = .
4 2
We have
1 − λ 3
C(λ) = = λ2 − 3λ = 10 = (λ − 5)(λ + 2)
4 2 − λ
Thus, the eigenvalues of A are λ1 = 5 and λ2 = −2. Since all the eigenvalues have algebraic
multiplicity 1, we know that [L] is diagonalizable.
" # " #
−4 3 4 −3
For λ1 = 5 we have A − λ1 I = ∼ .
4 −3 0 0
(" #)
3
Hence, a basis for Eλ1 is .
4
" # " #
3 3 1 1
For λ2 = −2 we have A − λ2 I = ∼ .
4 4 0 0
(" #)
−1
Hence, a basis for Eλ2 is .
1
(" # " #) " #
3 −1 5 0
Therefore, we take B = , and our we get [L]B = .
4 1 0 −2
 
−2 2 2
(c) We have [L] = −3 3 2.
 
−2 2 2
 
We have

−2 − λ 2 2 −2 − λ 2 2
C(λ) = −3 3−λ 2 = −3 3 − λ 2
−2 2 2 − λ λ
0 −λ

−2 − λ 2 −λ
= −3 3 − λ −1 = λ(−2 + 3λ − λ2 )
λ 0 0
Section 6.3 Solutions 141

= −λ(λ − 2)(λ − 1)

Thus, the eigenvalues of A are λ1 = 0, λ2 = 2, and λ3 = 1. Since all the eigenvalues have algebraic
multiplicity 1, we know that [L] is diagonalizable.
   
−2 2 2 1 −1 0
For λ1 = 0 we have A − λ1 I = −3 3 2 ∼ 0 0 1.
   
−2 2 2 0 0 0
   

  

 1
 
 
Hence, a basis for Eλ1 is  1 .

0

 

   
−4 2 2 1 0 −1
For λ2 = 2 we have A − λ2 I = −3 1 2 ∼ 0 1 −1.
   
−2 2 0 0 0 0
   

  

 1
 
 
Hence, a basis for Eλ2 is  1 .

1

   

   
−3 2 2 1 0 −1 
For λ3 = 1 we have A − λ3 I = −3 2 2 ∼ 0 1 −1/2.
   
−2 2 1 0 0 0
   

  

 2
 
 
Hence, a basis for Eλ3 is  1 .


2
   

        


 1 1 2

 0 0 0
Therefore, we take B =  1 , 1 , 1 and our we get [L]B = 0 2 0.
     

0 1 2
 
0 0 1
  

6.3.2 (a) By definition A is similar to D = diag(λ1 , . . . , λn ). Hence, by Theorem 6.1.1, we have that

tr A = tr diag(λ1 , . . . , λn ) = λ1 + · · · + λn

(b) By definition A is similar to D = diag(λ1 , . . . , λn ). Hence, by Theorem 6.1.1, we have that

det A = det diag(λ1 , . . . , λn ) = λ1 · · · λn


" #
1 1
6.3.3 A = is invertible, but not diagonalizable.
0 1
" #
1 0
B= is invertible and diagonalizable.
0 1
" #
0 1
C= is not invertible and not diagonalizable.
0 0
142 Section 6.4 Solutions
" #
0 0
E= is not invertible, but diagonalizable.
0 0
This shows that there is no connection between a matrix being invertible and being diagonalizable.
6.3.4 If A is diagonalizable with eigenvalues λ1 , . . . , λn , then there exists an invertible matrix P such that

P−1 AP = diag(λ1 , . . . , λn )

Then, observe that

P−1 (A − λ1 I)P = P−1 AP − λ1 P−1 P = diag(λ1 , . . . , λn ) − λ1 I = diag(0, λ2 − λ1 , . . . , λn − λ1 )

as required.
6.3.5 If A is diagonalizable, then there exists an invertible matrix P and diagonal matrix D such that P−1 AP =
D. Thus,
D = DT = (P−1 AP)T = PT AT (P−1 )T
Let Q = (P−1 )T , then Q−1 = PT and so we have that

Q−1 AT Q = D

and so AT is also diagonalizable.


(a) Observe that
C(λ) = det(A − λI) = λ2 − (a + d)λ + ad − bc
Thus, the eigenvalues of A are
p
−(a + d) ± (a + d)2 − 4(ad − bc)
λ=
2
This will only have real roots when

0 ≤ (a + d)2 − 4(ad − bc) = a2 + 2ad + d2 − 4ad + 4bc = a2 − 2ad + d2 − 4bc = (a − d)2 + 4bc

as required.
" #
0 1
(b) If A = , then (a − d)2 + 4bc = (0 − 0)2 + 4(1)(0) = 0, but A is not diagonalizble since a0 = 2,
0 0
but g0 = 1 < a0 .
If A is the zero matrix, then (a − d)2 + 4bc = 0, but A is diagonalizable since it is already diagonal.
(c) If (a−d)2 +4bc = (2x)2 where x ∈ Z, then observe that a−d must be even since (a−d)2 = (2x)2 −4bc
is even. Let a − d = 2y where y ∈ Z, then we have that the eigenvalues of A are
p
−2y ± (2x)2 −2y ± 2|x|
λ= = = −y ± |x|
2 2
which are both integers.
Section 6.4 Solutions 143

6.4 Problem Solutions


6.4.1 We have
4 − λ −1
C(λ) = = λ2 − 9λ + 18 = (λ − 3)(λ − 6)
−2 5 − λ
Thus, the eigenvalues of A are λ1 = 3 and λ2 = 6.
For λ1 = 3 we get " # " #
1 −1 1 −1
A − λ1 I = ∼
−2 2 0 0
(" #)
1
Hence, a basis for Eλ1 is .
1
For λ2 = 6 we get " # " #
−2 −1 1 1/2
A − λ2 I = ∼
−2 −1 0 0
(" #)
−1
Hence, a basis for Eλ2 is .
2
" # " #
1 −1 3 0
It follows that A is diagonalized by P = to D = . Thus, we have A = PDP−1 and so
1 2 0 6
#"" #" #
3 2/3 1/3 27 0
3 −1 2/3 1/3
A = PD P =
−1/3 1/3 0 216 −1/3 1/3
" #
90 −63
=
−126 153

It is easy to verify that this does equal A3 .


6.4.2 We have
−6 − λ −10
C(λ) = = λ2 − λ − 2 = (λ − 2)(λ + 1)
4 7 − λ
Thus, the eigenvalues of A are λ1 = 2 and λ2 = −1.
For λ1 = 2 we get " # " #
−8 −10 1 5/4
A − λ1 I = ∼
4 5 0 0
(" #)
−5
Hence, a basis for Eλ1 is .
4
For λ2 = −1 we get "# " #
−5 −10 1 2
A − λ2 I = ∼
4 8 0 0
(" #)
−2
Hence, a basis for Eλ2 is .
1
144 Section 6.4 Solutions
" # " #
−5 −2 2 0
It follows that A is diagonalized by P = to D = . Thus, we have A = PDP−1 and so
4 1 0 −1

−5 −2 2100
" #" # " #
100 100 −1 0 1 1 2
A = PD P =
4 1 0 (−1)100 3 4 −5
−5 · 2100 + 8 −5 · 2101 + 10
 
1 
=  
3  102 103

2 −4 2 −5

6.4.3 We have
2 − λ 2
C(λ) = = λ2 + 3λ − 4 = (λ − 1)(λ + 4)
−3 −5 − λ
Thus, the eigenvalues of A are λ1 = 1 and λ2 = −4.
For λ1 = 1 we get "
# " #
1 2 1 2
A − λ1 I = ∼
−3 −6 0 0
(" #)
−2
Hence, a basis for Eλ1 is .
1
For λ2 = −4 we get " # " #
6 2 1 1/3
A − λ2 I = ∼
−3 −1 0 0
(" #)
−1
Hence, a basis for Eλ2 is .
3
" # " #
−2 −1 1 0
It follows that A is diagonalized by P = to D = . Thus, we have A = PDP−1 and so
1 3 0 −4
" #" # " #
100 100 −1 −2 −1 1 0 1 3 1
A = PD P =
1 3 0 4100 −5 −1 −2
1 −6 + 4100 −2 + 2 · 4100
" #
=−
5 3 − 3 · 4100 1 − 6 · 4100

6.4.4 We have
−2 − λ 2
C(λ) = = λ2 − 3λ − 4 = (λ + 1)(λ − 4)
−3 5 − λ
Thus, the eigenvalues are λ1 = −1 and λ2 = 4.
For λ1 = −1 we have " # " #
−1 2 1 −2
A − λ1 I = ∼
−3 6 0 0
(" #)
2
Hence, a basis for Eλ1 is .
1
Section 6.4 Solutions 145

For λ2 = 4 we have " # " #


−6 2 1 −1/3
A − λ2 I = ∼
−3 1 0 0
(" #)
1
Hence, a basis for Eλ2 is .
1
# " " #
2 1 −1 0
It follows that A is diagonalized by P = to D = . Thus, we have A = PDP−1 and so
1 3 0 4
" #" # " #
200 200 −1 2 1 1 0 1 3 −1
A = PD P =
1 3 0 4200 5 −1 2
1 6 − 4200 −2 + 2 · 4200
" #
=
5 3 − 3 · 420 −1 + 6 · 4200
6.4.5 We have

−2 − λ 1 1 −1 − λ 1 1
C(λ) = −1 −λ 1 = −1 − λ −λ 1
−2 2 1 − λ 0
2 1 − λ

−1 − λ 1 1
= 0 −1 − λ 0 = −(λ + 1)2 (λ − 1)
0 2 1 − λ
Thus, the eigenvalues are λ1 = −1 and λ2 = 1.
For λ1 = −1 we have    
−1 1 1 1 −1 −1
A − λ1 I = −1 1 1 ∼ 0 0 0 
   
−2 2 2 0 0 0
   
   

 1 1
   
 
Hence, a basis for Eλ1 is  1 , 0 .


0 1
 

For λ2 = 1 we have    
−3 1 1 1 0 −1/2
A − λ2 I = −1 −1 1 ∼ 0 1 −1/2
   
−2 2 0 0 0 0
   
 

 1
 
 
Hence, a basis for Eλ2 is  1 .


2
 

   
1 1 1 −1 0 0 
It follows that A is diagonalized by P = 1 0 1 to D =  0 −1 0. Thus, we have A = PDP−1
   
0 1 2 0 0 1
   
and so

A100 = PD100 P−1 = PIP−1 = I


146 Section 6.4 Solutions

6.4.6 If λ is an eigenvalue of A, then there exists a non-zero vector ~v such that A~v = λ~v. We will prove by
induction that An~v = λn~v.
Base Case: n = 1. We have A1~v = λ1~v.
Inductive Hypothesis: Assume that Ak~v = λk~v.
Inductive Step: We have

Ak+1~v = Ak (A~v) = Ak (λ~v) = λ(Ak~v) = λ(λk~v) = λk+1~v

as required.
6.4.7
6.4.8
6.4.9
6.4.10
6.4.11
Chapter 7 Solutions

7.1 Problem Solutions


7.1.1 (a) Row reducing we get " # " #
0 1 3 1 0 −3

1 2 3 0 1 3
    
  1  0 (" # " #)
   

  1 0
Hence, a basis for the row space is   0  , 1 , a basis for the column space is , . To find



 −3 3  
 0 1
a basis for the nullspace we solve

x1 − 3x3 = 0
x2 + 3x3 = 0

x3 is a free variable, so we let x3 = t ∈ R. Then we have any vector ~x in the nullspace satisfies
     
 x1   3t   3 
 x2  = −3t = t −3
     
x3 t 1
  

  3 
 
 
Thus, a basis for the nullspace is  −3 .


  1 
 

To find a basis for the left nullspace, we row reduce the transpose of the matrix. We get
   
0 1 1 0
1 2 ∼ 0 1
   
3 3 0 0

Thus, the left nullspace is {~0}. Therefore, a basis for the left nullspace is the empty set.
(b) Row reducing we get " # " #
1 2 −1 1 2 0

2 4 3 0 0 1

147
148 Section 7.1 Solutions

    
 1 0 (" # " #)
   

  1 −1
Hence, a basis for the row space is  2 , 0, a basis for the column space is , . To find



0 1 
 2 3
a basis for the nullspace we solve

x1 + 2x2 = 0
x3 = 0

x2 is a free variable, so we let x2 = t ∈ R. Then we have any vector ~x in the nullspace satisfies
     
 x1  −2t −2
x = t = t  1 
     
 2   
x3 0 0
  

 −2
 
 
Thus, a basis for the nullspace is   1  .


  0 
 

To find a basis for the left nullspace, we row reduce the transpose of the matrix. We get
   
 1 2 1 0
 2 4 ∼ 0 1
   
−1 3 0 0

Thus, the left nullspace is {~0}. Therefore, a basis for the left nullspace is the empty set.
(c) Row reducing we get
   
2 −1 5 1 0 2 
3 4 2 ∼ 0 1 −1
   
1 1 1 0 0 0
       

 1  0   
 2 −1
     
 
Hence, a basis for the row space is  0 , 1 , a basis for the column space is 3 ,  4 . To
    
 
 
 
 
 
 

  
2 −1    1  1 
 

find a basis for the nullspace we solve

x1 + 2x3 = 0
x2 − x3 = 0

x3 is a free variable, so we let x3 = t ∈ R. Then we have any vector ~x in the nullspace satisfies
     
 x1  −2t −2
x t
 2  =   = t  1 
     
x3 t 1
  

 −2
 
 
Thus, a basis for the nullspace is   1  .


  1 
 

Section 7.1 Solutions 149

To find a basis for the left nullspace, we row reduce the transpose of the matrix. We get
   
 2 3 1 1 0 1/11
−1 4 1 ∼ 0 1 3/11
   
5 2 1 0 0 0

Thus we need to solve the system

1
x1 + x3 = 0
11
3
x2 + x3 = 0
11
Let x3 = t ∈ R. Then every vector ~x in the left nullspace satisfies
     
 x1   −t/11  −1/11
 x2  = −3t/11 = t −3/11
     
x3 t 1
 

 −1/11 

 
Hence, a basis for the left nullspace is  −3/11 .

 
  
 1 
 
 
 

(d) Row reducing we get


   
 1 −2 3 5  1 −2 0 2
−2 4 0 −4 ∼ 0 0 1 1
   
3 −6 −5 1 0 0 0 0
  
    
  1  0     
 1   3 


 

−2 0
 
 
   

 
Hence, a basis for the row space is    ,   , a basis for the column space is  −2 ,  0  . To
      

 0 1 
 
 

 3 −5

         
 2

1

find a basis for the nullspace we solve

x1 − 2x2 + 2x4 = 0
x3 + x4 = 0

x2 and x4 are free variables, so we let x2 = s ∈ R and x4 = t ∈ R. Then we have any vector ~x in
the nullspace satisfies
 x1  2s − 2t −2
       
2
 x   s 
 = s   + t  0 
1  
 2  = 
 x3   −t  0
 
−1
 
   
x4 t 0 1
    


 2 −2 
     
1  0 

 
Thus, a basis for the nullspace is  , .

0 −1
 

 

 

 

 



     

0

1 

150 Section 7.1 Solutions

To find a basis for the left nullspace, we row reduce the transpose of the matrix. We get

 1 −2 3  1 0 −5/3


   
−2 4 
6  0 1 −7/3
 ∼
 3 0 −5 0 0 0 

   
5 −4 1 0 0 0

Thus we need to solve the system

5
x1 − x3 = 0
3
7
x2 − x3 = 0
3

Let x3 = t ∈ R. Then every vector ~x in the left nullspace satisfies


     
 x1  5t/3 5/3
x = 7t/3 = t 7/3
     
 2   
x3 t 1
  

 5
 
 
Hence, a basis for the left nullspace is  7.

3

 

(e) Row reducing we get


   
 3 2 5 3 1 0 3 0
−1 0 −3 1 0 1 −2 0
 1 1 1 1 ∼ 0 0 0 1
   
   
1 4 −5 1 0 0 0 0
            

 1  0  0  
  3  2 3 
 
 
       
−1 0 1


 0  1  0
  
 
Hence, a basis for the row space is    ,   ,   , a basis for the column space is    ,   ,   .
      


  3  −2
       0  
 

  1  1
       1  

0  0  1

 
  1  4 1

 

To find a basis for the nullspace we solve

x1 + 3x3 = 0
x2 − 2x3 = 0
x4 = 0

x3 is a free variable, so we let x3 = t ∈ R. Then we have any vector ~x in the nullspace satisfies

 x1  −3t −3


     
 x2   2t 
  =   = t  2 
 
 x3   t   1 
     
x4 0 0
Section 7.1 Solutions 151
  


 −3 
   
 2 

 
Thus, a basis for the nullspace is  .

1
 
 
 


  
  


 0 
 
To find a basis for the left nullspace, we row reduce the transpose of the matrix. We get

3 −1 1 1  1 0 0 −3


   
2 0 1 4  0 1 0 0 
5 −3 1 −5 ∼ 0 0 1 10 
   
   
3 1 1 1 0 0 0 0
Thus we need to solve the system

x1 − 3x4 = 0
x2 = 0
x3 + 10x4 = 0

Let x4 = t ∈ R. Then every vector ~x in the left nullspace satisfies


     
 x1   3t   3 
 x2   0 
 = t  0 
 
  = 
 x3  −10t −10
     
x4 t 1
 


  3  


 0 
 

Hence, a basis for the left nullspace is  .
   
−10
 

 

 



  


 1 
 

7.1.2 We have that dim Null(A) = 2. Thus, by the Dimension Theorem

dim Col(A) = rank(A) = 3 − dim Null(A) = 3 − 2 = 1

We are also given that


   
0 h i 1
0 = ~a1 ~a2 ~a3 0 = ~a1
 
0 0
   
0 h i 0
0
  = 1 2 ~a ~a ~a 1 = ~a2
 
3   
0 0

Also, ~a3 , ~0, since ~e3 is not in the nullspace of A. Thus, a basis for Col(A) is {~a3 }.
7.1.3 Pick ~z ∈ {RE~x | ~x ∈ Rn }. Then ~z = RE~x for some ~x ∈ Rn . By definition of matrix-vector multiplication
we have that E~x = ~y ∈ Rn for some ~y ∈ Rn . Thus, ~z = R~y ∈ {R~y | ~y ∈ Rn }.
On the other hand, pick w ~ ∈ {R~y | ~y ∈ Rn }. Since E is an invertible matrix, for any ~y ∈ Rn we have that
there exists a unique vector ~x ∈ Rn such that E~x = ~y. Thus, ~z = RE~x ∈ {RE~x | ~x ∈ Rn }.
Therefore, the sets are subsets of each other and hence equal.
152 Section 7.1 Solutions

7.1.4 The result follows immediately from the Invertible Matrix Theorem.
7.1.5 (a) If ~x ∈ Null(BT ), then by definition ~0 = BT ~x. Taking transposes of both sides gives
~0T = (BT ~x)T = ~xT (BT )T = ~xT B

~b1 
 T

~bn . Then, B =  ... . If ~x ∈ Null(BT ), then we have


h i  
(b) Let B = ~b1 · · · T
 
~bTn

~b1 · ~x
 

~0 = BT ~x =  .. 
 
 . 
~bn · ~x

Thus, ~bi · ~x = 0 for 1 ≤ i ≤ n. Hence, if ~y ∈ Col(B), then

~x · ~y = ~x · (c1~b1 + · · · + cn~bn ) = c1 (~x · ~b1 ) + · · · + cn (~x · ~bn ) = 0

7.1.6 We need to find a matrix A such that A~x = ~0 if and only if ~x is a linear combination of the columns of A.
Observe that we need dim Null(A) = dim Col(A) = rank(A). So, by the Dimension Theorem, we have
2 = rank(A)
" + dim# Null(A) = 2 rank(A). Thus, we need " rank(A)
# = 1. Hence, we can pick A to be of the
x x 0 1
form A = 1 2 . It is easy to see that taking A = gives
0 0 0 0
(" #)
1
Col(A) = Span = Null(A)
0

7.1.7 First observe by the Dimension Theorem that dim Null(AT ) = m − r.


 T
~e1 
Let E =  ...  and observe that the bottom r rows of R are all zeros since rank(A) = r.
 
 
~eTm
Thus, EA = R implies that ~eTi A = ~0 for r + 1 ≤ i ≤ m. Hence, ~er+1 , . . . , ~em ∈ Null(AT ). Moreover,
since E is invertible, the columns of E are linearly independent by the Invertible Matrix Theorem. Thus,
{~er+1 , . . . , ~em } is a linearly independent set of m−r vectors in Null(AT ) as hence forms a basis for Null(AT )
as required.
7.1.8 If A and B are similar, then there exists an invertible matrix P such that P−1 AP = B. Multiply on the left
by P to get AP = PB. Then, since P is invertible, it can be written as a product of elementary matrices,
say P = E1 · · · Ek . Hence, by Corollary 5.2.4, we get

rank(PB) = rank(E1 · · · Ek B) = rank(B)

Similarly, since the rank of a matrix equals the rank of the transpose of a the matrix, we get

rank(AP) = rank((AP)T ) = rank(EkT · · · E1T A) = rank(AT ) = rank A

Thus, rank(A) = rank(AP) = rank(PB) = rank(B) as required.


Section 7.1 Solutions 153

7.1.9 (a) If A~x = ~0, then AT A~x = AT ~0 = ~0. Hence, the nullspace of A is a subset of the nullspace of AT A.
On the other hand, consider AT A~x = ~0. Then,

kA~x k2 = (A~x) · (A~x) = (A~x)T (A~x) = ~xT AT A~x = ~xT ~0 = 0

Thus, A~x = ~0. Hence, the nullspace of AT A is also a subset of the nullspace of A, and the result
follows.
(b) Using part (a), we get that dim(Null(AT A)) = dim(Null(A)). Thus, the Dimension Theorem gives

rank(AT A) = n − dim(Null(AT A)) = n − dim(Null(A)) = rank(A)

as required.
7.1.10 Let ~x ∈ Col(B). Then, there exists ~y such that B~y = ~x. Now, observe that

A~x = A(B~y) = (AB)~y = Om,n~y = ~0

Thus, ~x ∈ Null(A). Hence, Col(B) is a subset of Null(A) and thus, since Col(B) is a vector space, Col(B)
is a subspace of Null(A).
 
1 1
7.1.11 (a) A = 1 0
 
1 2
 
" #
1 1 1
(b) B =
1 0 2
(c) We first observe that C must have 3 columns. So, lets make C a 1 × 3 matrix. We require that

c11 + c12 + c13 = 0


c11 + 2c13 = 0

Row reducing the corresponding coefficient matrix gives


" # " #
1 1 1 1 0 2

1 0 2 0 1 −1
 
−2
Thus, we get that ~c = t  1 . Hence, we can take
 
1
 

h i
C = −2 1 1
 
−2
(d) Using our work in part (c), we see that we can take D =  1 .
 
1
 

(e) No, there cannot be such a matrix F since we would have dim(Col(F)) = 2, dim(Null(F)) = 2,
and F would have to have 3 columns, but this would contradict the Dimension Theorem.
Chapter 8 Solutions

8.1 Problem Solutions


" # " #
a1 a2
8.1.1 (a) Let , ∈ R2 and s, t ∈ R. Then we have
b1 b2
" # " #! i " sa1 + ta2 #
a1 a2 h 0
L s +t = L sa1 + ta2 , sb1 + tb2 =
b1 b2 0 sb1 + tb2
" # " # " #! " #!
a1 0 a2 0 a1 a
=s +t = sL + tL 2
0 b1 0 b2 b1 b2
Hence, L is linear.
(b) We have L(x + x2 ) = −1 + x2 and L(2x + 2x2 ) = −2 + 4x2 , so L(x + x2 ) + L(2x2 ) = −3 + 5x2 . But
L[(x + x2 ) + (2x + 2x2 )] = L(3x + 3x2 ) = −3 + 9x2
Hence, L is not linear.
" #
1 0
(c) Since L(0) = , ~0 M2×2 (R) we have that L is not linear by Theorem 8.2.1.
0 0
" # " #
a b1 a2 b2
(d) Let 1 , ∈ M2×2 (R) and s, t ∈ R. Then we have
c1 d1 c2 d2
 
" # " #! " #!  sa1 + ta2 + sb1 + tb2 
a b a b sa1 + ta2 sb1 + tb2
T s 1 1 +t 2 2 =T
 
=  sb1 + tb2 + sc1 + tc2 

c1 d1 c2 d2 sc1 + tc2 sd1 + td2
sc1 + tc2 − (sa1 + ta2 )
 
   
a1 + b1  a2 + b2  " #! " #!
a1 b1 a2 b2
= s b1 + c1  + t b2 + c2  = sT + tT
   
c1 d1 c2 d2
c1 − a1 c2 − a2
   

Thus, T is linear.
(e) Let a1 + b1 x + c1 x2 + d1 x3 , a2 + b2 x + c2 x3 + d2 x3 ∈ P3 (R) and s, t ∈ R. Then,
D[s(a1 + b1 x + c1 x2 + d1 x3 ) + t(a2 + b2 x + c2 x3 + d2 x3 )] = D(sa1 + ta2 + (sb1 + tb2 )x + (sc1 + tc2 )x2 + (sd1 +
= (sb1 + tb2 ) + 2(sc1 + tc2 )x + 3(sd1 + td2 )x2
= s(b1 + 2c1 x + 3d1 x2 ) + t(b2 + 2c2 x + 3d2 x2 )
= sD(a1 + b1 x + c1 x2 + d1 x3 ) + tD(a2 + b2 x + c2 x3 +
154
Section 8.1 Solutions 155

Thus, D is linear.
(f) We have " #! " #!
1 0 −1 0
L +L =1+1=2
0 1 0 −1
but "
# " #! " #!
1 0 −1 0 0 0
L + =L =0
0 1 0 −1 0 0
So, L is not linear.
8.1.2 (a) If L is linear, then we have
L(a1 , a2 , a3 ) = a1 L(1, 0, 0) + a2 L(0, 1, 0) + a3 L(0, 0, 1)
= a1 (1 + x) + a2 (1 − x2 ) + a3 (1 + x + x2 )
= (a1 + a2 + a3 )1 + (a1 + a3 )x + (−a2 + a3 )x2

(b) If L is linear, then we have


L(a + bx + cx2 ) = aL(1) + bL(x) + cL(x2 )
Observe that
# " # "" # " #
2 2 2 2 1 2 1 0 0 0
L(x ) = L(1 + x + x ) − L(1) − L(x) = − = =
2 3 1 2 0 1 1 0
Thus,
# " " # " # " #
2 1 2 1 0 0 0 a+b 2a
L(a + bx + cx ) = a +b +c =
1 2 0 1 1 0 a + c 2a + b
8.1.3 Let ~x, ~y ∈ V, s, t ∈ R and let B = {~v1 , . . . , ~vn }. Since ~x, ~y ∈ V there exists b1 , . . . , bn , c1 , . . . , cn ∈ R such
that
~x = b1~v1 + · · · + bn~vn and ~y = c1~v1 + · · · + cn~vn
Hence,
L(s~x + t~y) = L s(b1~v1 + · · · + bn~vn ) + t(c1~v1 + · · · + cn~vn )
 

= L[(sb1 + tc1 )~v1 + · · · + (sbn + tcn )~vn ]


 sb1 + tc1 
 

= 
 .. 
 . 

sbn + tcn
b1  c1 
   

= s  .  + t  ... 


 ..   
   
bn cn
= sL(b1~v1 + · · · + bn~vn ) + tL(c1~v1 + · · · + cn~vn )
= sL(~x) + tL(~y)
hence, L is linear.
156 Section 8.1 Solutions

8.1.4 ~0 = 0L(~x) = L(0~x) = L(~0).


8.1.5 By definition the domain of M ◦ L is V. Also, for any ~v ∈ V, we have

(M ◦ L)(~v) = M(L(~v))

Observe that L(~v) ∈ W which is in the domain of M. Hence, we get that M(L(~v)) ∈ Range(M) which is
in W. Thus, the codomain of M ◦ L is W.
Let ~x, ~y ∈ V and s, t ∈ R. Then,

(M ◦ L)(s~x + t~y) = M(L(s~x + t~y)) = M(sL(~x) + tL(~y)) = sM(L(~x)) + tM(L(~y))


= s(M ◦ L)(~x) + t(M ◦ L)(~y)

Hence, M ◦ L is linear.
8.1.6 (a) Consider c1~v1 + · · · + ck~vk = ~0. Then, we have

L(c1~v1 + · · · + ck~vk ) = L(~0)


c1 L(~v1 ) + · · · + ck L(~vk ) = ~0 by Problem 4

Thus, c1 = · · · = ck = 0 since {L(~v1 ), . . . , L(~vk )} is linearly independent. Thus, {~v1 , . . . , ~vk } must
also be linearly independent.
(b) If L : R2 → R2 is the linear mapping defined by L(~x) = ~0 and {~v1 , ~v2 } = {~e1 , ~e2 } is the standard
basis for R2 , then {~v1 , ~v2 } is linearly independent, but {L(~v1 ), L(~v2 )} is linearly dependent since it
contains the zero vector.
8.1.7 Let L, M ∈ L and t ∈ R.
(a) By definition tL is a mapping with domain V. Also, since W is closed under scalar multiplication,
we have that (tL)(~v) = tL(~v) ∈ W. Thus, the codomain of tL is W. For any ~x, ~y ∈ V and c, d ∈ R
we have

(tL)(c~x + d~y) = tL(c~x + d~y) = t[cL(~x) + dL(~y)]


= ctL(~x) + dtL(~y) = c(tL)(~x) + d(tL)(~y)

Hence, tL ∈ L.
(b) For any ~v ∈ V we have

[t(L + M)](~v) = t[(L + M)(~v)] = t[L(~v) + M(~v)] = tL(~v) + tM(~v) = [tL + tM](~v)

Thus, t(L + M) = tL + tM.


8.1.8 Assume that L is invertible. Then, there exists a linear mapping L−1 : Rn → Rn such that (L◦L−1 )(~x) = ~x.
Hence, by definition of the standard matrix, we get

~x = L(L−1 )(~x) = [L][L−1 ]~x

Thus, [L][L−1 ] = I by Theorem 3.1.4. So, [L] is invertible.


Section 8.1 Solutions 157

If [L] is invertible, then there exists a matrix A such that A[L] = I = [L]A. Define M : Rn → Rn by
M(~x) = A~x. Then, for any ~x ∈ Rn we have

(M ◦ L)(~x) = M(L(~x)) = A[L]~x = I~x = ~x

and
(L ◦ M)(~x) = L(M(~x)) = [L]A~x = I~x = ~x
Thus, M = L−1 .
" #
−1 2 1
8.1.9 Using our work from problem 7, we know that the standard matrix of L is the inverse of [L] = .
3 −4
" #
−1 4/11 1/11
We have that [L] = . Consequently,
3/11 −2/11
!
4 1 3 2
L−1 (x1 , x2 ) = x1 + x2 , x1 − x2
11 11 11 11

8.1.10 Observe that

(L ◦ R)(~x) = L(R(x1 , x2 , x3 , . . .))


= L(0, x1 , x2 , x3 , . . .)
= (x1 , x2 , x3 )(R ◦ L)(~x) = R(L(x1 , x2 , x3 , . . .))
= R(x2 , x3 , . . .)
= (0, x2 , x3 , . . .)

Thus, (L ◦ R)(~x) = ~x, but (R ◦ L)(~x) , ~x.


158 Section 8.2 Solutions

8.2 Problem Solutions


8.2.1 (a) Every vector in Range(L) has the form
" # " # " #
a 0 1 0 0 0
L(a + bx + cx2 ) = =a +b
0 b 0 0 0 1
(" # " #)
1 0 0 0
Thus, a basis for Range(L) is , . Thus, rank(L) = 2 and so the Rank-Nullity
0 0 0 1
Theorem gives
nullity(L) = dim P2 (R) − rank(L) = 3 − 2 = 1

(b) If a + bx + cx2 ∈ ker(L), then

0 + 0x + 0x2 = L(a + bx + cx2 ) = (a − b) + (b + c)x2

Thus, a − b = 0 and b + c = 0. So, a = b = −c. So, every vector a + bx + cx2 ∈ ker(L) has the
form −c − cx + cx2 = c(−1 − x + x2 ). Thus, a basis for ker(L) is {−1 − x + x2 }. Consequently,
nullity(L) = 1 and the Rank-Nullity Theorem gives

rank(L) = dim P2 (R) − nullity(L) = 3 − 1 = 2

(c) Every vector in Range(T ) has the form


"
# " # " #
2 0 0 0 0 0 0
T (a + bx + cx ) = = (a + c) + (b + c)
a+c b+c 1 0 0 1
(" # " #)
0 0 0 0
Thus, , spans Range(T ) and is clearly linearly independent. Thus, it is a basis for
1 0 0 1
Range(T ). Hence, rank(T ) = dim Range(T ) = 2. By the Rank-Nullity Theorem we get

nullity(L) = dim P2 (R) − rank(T ) = 3 − 2 = 1


 
a
(d) If b ∈ ker(L), then
 
c
 
 
a
0 = L b = (a + b) + (a + b + c)x
 
c
 

Hence, a + b = 0 and a + b + c = 0. Thus, c = 0 and b = −a. So, every vector in ker(L) has the
form      
a  a   1 
b = −a = a −1
     
   
c 0 0
Section 8.2 Solutions 159

  

  1 
 
 
Thus, a basis for ker(L) is  −1 . Thus, nullity(L) = 1 and

  0 

 

rank(L) = dim R3 − nullity(L) = 3 − 1 = 2

by the Rank-Nullity Theorem.


" #
a b
(e) If A = ∈ ker(T ), then
c d
" # " #
0 0 a c
= T (A) =
0 0 b d
Thus, a = b = c = d = 0 and so A = O2,2 . Hence, ker(T ) = {O2,2 } and so nullity(T ) = 0.
Therefore, by the Rank-Nullity Theorem we have that rank(T ) = dim M2×2 (R) − 0 = 4.
(f) If a + bx + cx2 ∈ ker(M), then we have
" # " #
0 0 2 −a − 2c 2b − c
= M(a + bx + cx ) =
0 0 −2a + 2c −2b − c
Thus, we have −a − 2c = 0, −2a + 2c = 0, 2b − c = 0, and −2b − c = 0. These imply that
a = b = c = 0. Thus, nullity(M) = 0 and so rank(M) = dim P2 (R) − 0 = 3 by the Rank-Nullity
Theorem.
8.2.2 If {L(~v1 ), . . . , L(~vk )} spans W, then Range(L) = W. Thus, rank(L) = dim Range(L) = dim W. So, by the
Rank-Nullity Theorem, we have

dim V = rank(L) + nullity(L) = dim W + nullity(L)

Since nullity(L) ≥ 0, we have that dim V ≥ dim W.


" #
0
8.2.3 One possible example is to take L : P3 (R) → R2 defined by L(a + bx + cx2 + dx3 ) = . Then, we have
0
(" #)
0
that dim P3 (R) ≥ dim R2 , and Range(L) = , R2 as required.
0
" #
x
8.2.4 Using our work in Problem 7.1.6, we see that we can take L : R → R defined by L(x1 , x2 ) = 2 .
2 2
0
Then, we have (" #)
1
Range(L) = Span = ker(L)
0
8.2.5 If Range(L) = W, then rank(L) = dim Range(L) = dim W = n. Hence, by the Rank-Nullity Theorem we
get
nullity(L) = dim V − rank(L) = n − n = 0
Thus, ker(L) = {~0}.
Similarly, if ker(L) = {~0}, then nullity(L) = 0, so the Rank-Nullity Theorem gives

dim(Range(L)) = rank(L) = dim V − nullity(L) = n − 0 = n

Thus, Range(L) = W.
160 Section 8.3 Solutions

8.2.6 (a) Since the rank of a linear mapping is equal to the dimension of the range, we consider the range
of both mappings. Let ~x ∈ Range(M ◦ L). Then there exists ~v ∈ V such that ~x = (M ◦ L)(~v) =
M(L(~v)) ∈ Range(M).
Hence, Range(M ◦ L) is a subset, and hence a subspace, of Range(M). Therefore, dim Range(M ◦
L) ≤ dim Range(M) which implies rank M ◦ L ≤ rank M.
(b) The kernel of L is a subspace of the kernel of M ◦ L, because if L(~x) = ~0, then
(M ◦ L)(~x) = M(L(~x)) = M(~0) = ~0
Therefore
nullity(L) ≤ nullity(M ◦ L)
so
n − nullity(L) ≥ n − nullity(M ◦ L)
Hence rank(L) ≥ rank(M ◦ L), by the Rank-Nullity Theorem.

8.3 Problem Solutions


8.3.1 (a) We have
D(1) = 0 = 0(1) + 0(x)
D(x) = 1 = 1(1) + 0(x)
D(x2 ) = 2x = 0(1) + 2(x)
" #
0 1 0
Thus C [D]B = .
0 0 2
(b) We have
L(1, −1) = x2 = 0(1 + x2 ) + 1(1 + x) + 1(−1 − x + x2 )
L(1, 2) = 3 + x2 = 3(1 + x2 ) − 2(1 + x) − 2(−1 − x + x2 )
 
0 3 
Thus C [L]B = 1 −2.
 
1 −2
 

(c) We have
" #! " # " # " #
1 1 1 1 −1
T = =1 +0
0 0 1 1 1
" #! " # " # " #
0 1 1 3 1 1 −1
T = = +
1 0 2 2 1 2 1
" #! " # " # " #
1 0 1 1 1 1 −1
T = = −
0 1 0 2 1 2 1
" #! " # " # " #
0 0 1 1 −1
T = =1 +0
1 1 1 1 1
Section 8.3 Solutions 161
" #
1 3/2 1/2 1
Hence, C [T ]B = .
0 1/2 −1/2 0
(d) We have
" # " # " #
2 1 1 1
L(1 + x ) = =0 +1
2 1 2
" # " # " #
0 1 1
L(x − x2 ) = =0 +0
0 1 2
" # " # " #
2 0 1 1
L(x ) = = (−1) +1
1 1 2
" # " # " #
0 1 1
L(x3 ) = = (−1) +1
1 1 2
" #
0 0 −1 −1
Hence, C [L]B = .
1 0 1 1
(e) We have

L(1) = 1 = 1(1) + 0(x + 1) + 0(x + 1)2


L(x − 1) = x − 1 = (−2)(1) + 1(x + 1) + 0(x + 1)2
L(x2 − 2x + 1) = x2 − 2x + 1 = 4(1) − 4(x + 1) + 1(x + 1)2
 
1 −2 4 
Hence, C [L]B = 0 1 −4.
 
0 0 1
 

(f) We have

L(1, −1, 0) = 1 − x = 1(1 − x) + 0(x)


L(1, 0, −1) = 0 + x = 0(1 − x) + 1(x)
L(0, 1, 1) = 1 + 2x = 1(1 − x) + 3(x)
" #
1 0 1
Hence, C [L]B = .
0 1 3
8.3.2 (a) We have

L(1) = 0 = 0 + 0x + 0x2
L(x) = 1 = 1 + 0x + 0x2
L(x2 ) = 2x = 0 + 2x + 0x2
 
0 1 0
Hence, [L]B = 0 0 2.
 
0 0 0
 
162 Section 8.3 Solutions

(b) For ease, name the vectors in B as ~v1 , ~v2 , ~v3 , ~v4 .
" #! " #
1 1 1 1
L = = 1~v1 + 0~v2 + 0~v3 + 0~v4
0 0 0 0
" #! " #
1 0 1 0
L = = 0~v1 + 1~v2 + 0~v3 + 0~v4
0 1 0 1
" #! " #
1 1 1 1
L = = 0~v1 + 0~v2 + 1~v3 + 0~v4
0 1 0 1
" #! " #
0 0 0 1
L = = 0~v1 + −1~v2 + 1~v3 + 0~v4
1 0 0 0
 
1 0 0 0 

0 1 0 −1
Thus, [L]B =  .
0 0 1 1 

0 0 0 0
(c) We have
"#! " # " # " #
1 0 2 0 1 0 2 0
T = =0 +1
0 1 0 3 0 1 0 3
" #! " # " # " #
2 0 5 0 1 0 2 0
T = =1 +2
0 3 0 7 0 1 0 3
" #
0 1
Thus, [L]B = .
1 2
(d) We have

L(1 + x2 ) = 1 + x2 = 1(1 + x2 ) + 0(−1 + x) + 0(1 − x + x2 )


L(−1 + x) = −1 + x2 = (−1)(1 + x2 ) + 2(−1 + x) + 2(1 − x + x2 )
L(1 − x + x2 ) = 1 = (1)(1 + x2 ) + (−1)(−1 + x) + (−1)(1 − x + x2 )
 
1 −1 1 
Thus, [L]B = 0 2 −1.
 
0 2 −1
 

8.3.3 Let B = {~v1 , . . . , ~vn }. Then for 1 ≤ i ≤ n we have

L(~vi ) = ~vi = 0~v1 + · · · + 0~vi−1 + ~vi + 0~vi+1 + · · · + 0~vn

Thus, h i
[L]B = [L(~v1 )]B ··· [L(~vn )]B = I
Section 8.4 Solutions 163
" # (" #)
1 1 1
8.3.4 Let A = and define L : R2 → R2 by L(~x) = A~x. Clearly we have Range(L) = Span . Let
1 1 1
(" # " #)
1 1
B= , . Then, we get
1 −1
" #! " # " # " #
1 2 1 1
L = =2 +0
1 2 1 −1
" #! " # " # " #
1 0 1 1
L = =0 +0
−1 0 1 −1
" #
2 0
Thus, [L]B = . Clearly, Col([L]B ) , Range(L).
0 0

8.4 Problem Solutions


" #
a
8.4.1 (a) We define L : P1 (R) → R by L(a0 + a1 x) = 0 .
2
a1

Linear: Let any two elements of P1 (R) be ~a = a0 + a1 x and ~b = b0 + b1 x and let s, t ∈ R then
L(s~a + t~b) = L s(a0 + a1 x) + t(b0 + b1 x)


= L (sa0 + tb0 ) + (sa1 + tb1 )x
" # " # " #
sa0 + tb0 a0 b
= =s + t 0 = sL(~a) + tL(~b)
sa1 + tb1 a1 b1
Therefore, L is linear.
One-to-one: If a0 + a1 x ∈ ker(L), then
" # " #
0 a
= L(a0 + a1 x) = 0
0 a1

Hence, a0 = a1 = 0 and so ker(L) = {~0}. Consequently, L is one-to-one.


" # " #
a0 a
Onto: For any ∈ R we have L(a0 + a1 x) = 0 . Thus, L is onto.
2
a1 a1
Thus, L is an isomorphism from P1 (R) to R2 .
" #
a a1
(b) We define L : P3 (R) → M2×2 (R) by L(a0 + a1 x + a2 x + a3 x ) = 0
2 3
.
a2 a3

Linear: Let any two elements of P3 (R) be ~a = a0 + a1 x + a2 x2 + a3 x3 and ~b = b0 + b1 x + b2 x2 + b3 x3


and let s, t ∈ R then
L(s~a + t~b) = L s(a0 + a1 x + a2 x2 + a3 x3 ) + t(b0 + b1 x + b2 x2 + b3 x3 )


= L (sa0 + tb0 ) + (sa1 + tb1 )x + (sa2 + tb2 )x2 + (sa3 + tb3 )x3

" # " # " #
sa0 + tb0 sa1 + tb1 a a b b
= = s 0 1 + t 0 1 = sL(~a) + tL(~b)
sa2 + tb2 sa3 + tb3 a2 a3 b2 b3
164 Section 8.4 Solutions

Therefore, L is linear.
One-to-one: If a0 + a1 x + a2 x2 + a3 x3 ∈ ker(L), then
" # " #
0 0 a a1
= L(a0 + a1 x + a2 x2 + a3 x3 ) = 0
0 0 a2 a3

Hence, a0 = a1 = a2 = a3 = 0 and so ker(L) = {~0}. Therefore, L is one-to-one.


" # " #
a0 a1 a a1
Onto: For any ∈ M2×2 (R) we have L(a0 + a1 x + a2 x + a3 x ) = 0
2 3
. Hence, L is
a2 a3 a2 a3
onto.
Thus, L is an isomorphism from P3 to M2×2 (R).
(c) We know that a general vector in P" has the#form (x−1)(a2 x2 +a1 x+a0 ). Thus, we define L : P → U
a a
by L (x − 1)(a2 x2 + a1 x + a0 ) = 2 1 .

0 a0

Linear: Let any two elements of P be ~a = (x − 1)(a2 x2 + a1 x + a0 ) and ~b = (x − 1)(b2 x2 + b1 x + b0 )


and let s, t ∈ R then

L(s~a + t~b) = L (x − 1)(sa2 + tb2 )x2 + (sa1 + tb1 )x + (sa0 + tb0 )



" #
sa2 + tb2 sa1 + tb1
=
0 sa0 + tb0
" # " #
a2 a1 b2 b1
=s +t = sL(~a) + t(~b)
0 a0 0 b0

Therefore L is linear.
One-to-one: Assume ~a ∈ ker(L). Then
" # " #
0 0 2  a2 a1
= L (x − 1)(a2 x + a1 x + a0 ) =
0 0 0 a0

This gives a2 = a1 a0 = 0. Hence ker(L) = {~0} so L is one-to-one.


Onto: Since dim P = dim U and L is one-to-one, by Theorem 8.4.5 L is also onto.
Thus, L is an isomorphism from P to U.
−1
   
1
1  0 
(d) Observe that every vector in S has the form x2   + x3   and that a every vector in U has the
0  0 
0 1
    


 1 −1 


 1  0 


2
form a(1 + x) + cx . In particular,  , is a basis for S and {1 + x, x2 } is a basis for U. Thus,
 
 
 
 
 
0 0
 

 

 

 

 



    
 

0

1 

Section 8.4 Solutions 165

we define L : S → U by
−1
    
 1
 1  0 
L a   + b   = a(1 + x) + bx2
  
 0  0 
0 1

−1 −1
       
1 1
1  0  1  0 
Linear: Let any two elements of S be ~a = a1   + b1   and ~b = a2   + b2   and let s, t ∈ R
0  0  0  0 
0 1 0 1
then

−1
    
 1
 1  0 
L(s~a + t~b) = L (sa1 + ta2 )   + (sb1 + tb2 )  
 0  0 
0 1
= (sa1 + ta2 )(1 + x) + (sb1 + tb2 )x2
= sa1 (1 + x) + ta2 (1 + x) + sb1 x2 + tb2 x2
= s[a1 (1 + x) + b1 x2 ] + t[a2 (1 + x) + b2 x2 ] = sL(~a) + tL(~b)

Therefore L is linear.
One-to-one: Assume L(~a) = L(~b). Then a1 (1 + x) + b1 x2 = a2 (1 + x) + b2 x2 . This gives a1 = b1
and a2 = b2 hence ~a = ~b so L is one-to-one.
−1
   
1
1  0 
Onto: For any a(1 + x) + bx2 ∈ U we can pick ~a = a   + b   ∈ S so that we have L(~a) =
0  0 
0 1
a(1 + x) + bx2 hence L is onto.

Thus, L is an isomorphism from S to U.


(e) Let {~v1 , . . . , ~vn } be a basis for V. Define L : V → Rn by

L(b1~v1 + · · · + bn~vn ) = b1~e1 + · · · + bn~en

Linear: Let b1~v1 + · · · + bn~vn , c1~v1 + · · · + cn~vn ∈ V and let s, t ∈ R then

L[s(b1~v1 + · · · + bn~vn ) + t(c1~v1 + · · · + cn~vn )] = L (sb1 + tc1 )~v1 + · · · + (sbn + tcn )~vn
 

= (sb1 + tc1 )~e1 + · · · + (sbn + tcn )~en


= s(b1~e1 + · · · + bn~en ) + t(c1~e1 + · · · + cn~en )
= sL(b1~v1 + · · · + bn~vn ) + tL(c1~v1 + · · · + cn~vn )

Therefore, L is linear.
166 Section 8.4 Solutions

One-to-one: If L(b1~v1 + · · · + bn~vn ) = L(c1~v1 + · · · + cn~vn ), then

b1~e1 + · · · + bn~en = c1~e1 + · · · + cn~en

Hence, bi = ci for 1 ≤ i ≤ n and so L is one-to-one.


Onto: For any ~x = x1~e1 + + · · · + xn~en ∈ Rn we have L(x1~v1 + · · · + xn~vn ) = x1~e1 + + · · · + xn~en = ~x.
So, L is onto.
Thus, L is an isomorphism from V to Rn .
(f) Let {~v1 , . . . , ~vn } be a basis for V. Define L : V → Pn−1 (R) by

L(b1~v1 + · · · + bn~vn ) = b1 + b2 x + b3 x2 + · · · + bn xn−1

Linear: Let b1~v1 + · · · + bn~vn , c1~v1 + · · · + cn~vn ∈ V and let s, t ∈ R then

L[s(b1~v1 + · · · + bn~vn ) + t(c1~v1 + · · · + cn~vn )] = L (sb1 + tc1 )~v1 + · · · + (sbn + tcn )~vn
 

= (sb1 + tc1 ) + (sb2 + tc2 )x + · · · + (sbn + tcn )xn−1


= s(b1 + b2 x + · · · + bn xn−1 ) + t(c1 + c2 x + · · · + cn xn−1 )
= sL(b1~v1 + · · · + bn~vn ) + tL(c1~v1 + · · · + cn~vn )

Therefore, L is linear.
One-to-one: If L(b1~v1 + · · · + bn~vn ) = L(c1~v1 + · · · + cn~vn ), then

b1 + b2 x + b3 x2 + · · · + bn xn−1 = c1 + c2 x + c3 x2 + · · · + cn xn−1

Hence, bi = ci for 1 ≤ i ≤ n and so L is one-to-one.


Onto: For any b1 + b2 x + b3 x2 + · · · + bn xn−1 ∈ Pn−1 (R) we have L(b1~v1 + · · · + bn~vn ) = b1 + b2 x +
b3 x2 + · · · + bn xn−1 . So, L is onto.
Thus, L is an isomorphism from V to Pn−1 (R).
8.4.2 If L is one-to-one, then ker(L) = {~0} by Lemma 8.4.1. Thus, nullity(L) = 0 and so by the Rank-Nullity
Theorem we get
dim(Range(L)) = rank(L) = dim V − nullity(L) = n − 0 = n
Since dim(Range(L)) = dim W, we have Range(L) = W, so L is onto.
Similarly, if L is onto, then rank(L) = dim(Range(L)) = dim W = n. Hence, by the Rank-Nullity
Theorem,
nullity(L) = dim V − rank(L) = n − n = 0
Thus, L is one-to-one by Lemma 8.4.1.
Section 8.4 Solutions 167

8.4.3 Consider c1 L(~v1 ) + · · · + cn L(~vn ) = ~0. Then

L(c1~v1 + · · · + cn~vn ) = ~0

Hence, c1~v1 + · · · + cn~vn ∈ ker(L). Since L is an isomorphism, it is one-to-one and thus ker(L) = {~0} by
Lemma 8.4.1. Thus, we get c1~v1 + · · · + cn~vn = ~0. This implies that c1 = · · · = cn = 0 since {~v1 , . . . , ~vn }
is linearly independent. Consequently, {L(~v1 ), . . . , L(~vn )} is a linearly independent set of n vectors in an
n-dimensional vector space, and hence is a basis.
8.4.4 ~ ∈ W. Since M is onto, there exist a ~u ∈ U such that M(~u) = w
(a) Let w ~ . Then, since L is onto, there
exists a ~v ∈ V such that L(~v) = ~u. Hence,

~
(M ◦ L)(~v) = M(L(~v)) = M(~u) = w

as required.
(b) Let L : R → R2 be defined by L(x1 ) = (x1 , 0). Clearly, L is not onto. Now, define M : R2 → R by
M(x1 , x2 ) = x1 . Then, we get

(M ◦ L)(x1 ) = M(L(x1 )) = M(x1 , 0) = x1

Hence, Range(M ◦ L) = R so M ◦ L is onto.


~ ∈ W, there exists a ~v ∈ V such that
(c) It is not possible. Observe that if M ◦ L is onto, then for any w

~ = (M ◦ L)(~v) = M(L(~v))
w

~ ∈ Range(M), and so M is onto.


Hence, w
8.4.5 (a) By definition [L(~x)]C = A[~x]B . Hence [L(~x)]C ∈ Col(A). Thus, we define F : Range(L) → Col(A)
by F(~
w) = [~w]C .
We know that taking coordinates is a linear operation, so F is linear.
w) = ~0 gives [~
F is one-to-one since F(~ w]C = ~0, so w
~ = ~0.
 x1 
 

F is onto: Let B = {~v1 , . . . , ~vn } and let ~y ∈ Col(A). Then, ~y = A~x for some ~x =  ...  ∈ Rn . Let
 
 
xn
~v = x1~v1 + · · · xn~vn so that [~v]B = ~x. Then,

F(L(~v)) = [L(~v)]C = A[~v]B = A~x = ~y

(b) Since Range(L) is isomorphic to Col(A), they have the same dimension. Hence,

rank(L) = dim(Range(L)) = dim(Col(A)) = rank(A)


168 Section 8.4 Solutions

8.4.6 Since L and M are one-to-one, we have nullity(L) = 0 = nullity(M) by Lemma 8.4.1. Since the
range of a linear mapping is a subspace of its codomain, we have that dim V ≥ dim Range(L) and
dim U ≥ dim Range(M). Thus, by the Rank-Nullity Theorem we get

dim V ≥ Range(L) = rank(L) = dim U

and
dim U ≥ Range(M) = rank(M) = dim V
Hence, dim U = dim V and so U and V are isomorphic by Theorem 8.4.2.
8.4.7 (a) We disprove the statement with a counter example. Let U = R2 and V = R2 and L : U → V be the
linear mapping defined by L(~x) = ~0. Then, clearly dim U = dim V, but L is not an isomorphism
since it isn’t one-to-one nor onto.
(b) By definition, we have that dim(Range(L)) = rank(L). Thus, the Rank-Nullity Theorem gives us
that
dim(Range(L)) = dim V − nullity(L)
If dim V < dim W, then dim V − nullity(L) < dim W and hence the range of L cannot be equal to
W. Thus, we have proven the statement is true.
(c) We disprove the statement with a counter example. Let V = R3 and W = R2 and L : V → W be
the linear mapping defined by L(~x) = ~0. Then, clearly dim V > dim W, but L is not onto.
(d) The Rank-Nullity Theorem tells us that

nullity(L) = dim V − rank(L) = dim V − dim W > 0

Thus, nullity(L) , {~0} and hence L is not one-to-one by Lemma 8.4.1. Therefore, we have proven
the statement is true.
Chapter 9 Solutions

9.1 Problem Solutions


*" # " #+
1 2 1 2
9.1.1 (a) , = 1(1) + 2(2) + 0(0) + (−1)(−1) = 6
0 −1 0 −1
*" # " #+
3 −4 4 1
(b) , = 3(4) + (−4)(1) + 2(2) + 5(−1) = 7
2 5 2 −1
*" # " #+
2 3 −5 3
(c) , = 2(−5) + 3(3) + (−4)(1) + 1(5) = 0
−4 1 1 5
*" # " #+
0 0 3 7
(d) , = 0(3) + 0(7) + 0(−4) + 0(3) = 0
0 0 −4 3
9.1.2 (a) hx, 1 + x + x2 i = 0(1) + 1(3) + 2(7) = 17
(b) h1 + x2 , 1 + x2 i = 1(1) + 2(2) + 5(5) = 30
(c) hx + x2 , 1 + x + x2 i = 0(1) + 2(3) + 6(7) = 48
(d) hx, xi = 0(0) + 1(1) + 2(2) = 5
9.1.3 (a) For any ~x, ~y,~z ∈ R3 and s, t ∈ R, we have

h~x, ~xi = 2x12 + 3x22 + 4x32 ≥ 0

and 0 = h~x, ~xi = 2x12 + 3x22 + 4x32 if and only if x1 = x2 = x3 = 0. Thus, h~x, ~xi = 0 if and only if
~x = ~0. So, it is positive definite.

h~x, ~yi = 2x1 y1 + 3x2 y2 + 4x3 y3 = 2y1 x1 + 3y2 x2 + 4y3 x3 = h~y, ~xi
Thus, it is symmetric.

hs~x + t~y,~zi = 2(sx1 + ty1 )z1 + 3(sx2 + ty2 )z2 + 4(sx3 + ty3 )z3
= s(2x1 z1 + 3x2 z2 + 4x3 z3 ) + t(2y1 z1 + 3y2 z2 + 4y3 z3 ) = sh~x, ~xi + th~y,~zi

Thus, it is also bilinear. Hence, it is an inner product.

169
170 Section 9.1 Solutions

(b) For any ~x, ~y,~z ∈ R3 and s, t ∈ R, we have

h~x, ~xi = 2x12 − x1 x2 − x2 x1 + 2x22 + x32 = 2x12 − 2x1 x2 + 2x22 + x32


= x12 + x12 − 2x1 x2 + x22 + x22 + x32
= x12 + (x1 − x2 )2 + x22 + x32

Thus, h~x, ~xi ≥ 0 and h~x, ~xi = 0 if and only if ~x = ~0. So, it is positive definite.

h~x, ~yi = 2x1 y1 − x1 y2 − x2 y1 + 2x2 y2 + x3 y3 = 2y1 x1 − y1 x2 − y2 x1 + 2y2 x2 + y3 x3 = h~y, ~xi

Thus, it is symmetric.

hs~x + t~y,~zi = 2(sx1 + ty1 )z1 − (sx1 + ty1 )z2 − (sx2 + ty2 )z1 + 2(sx2 + ty2 )z2 + (sx3 + ty3 )z3
= s(2x1 z1 − x1 z2 − x2 z1 + 2x2 z2 + x3 z3 ) + t(2y1 z1 − y1 z2 − y2 z1 + 2y2 z2 + y3 z3 )
= sh~x, ~xi + th~y,~zi

Thus, it is also bilinear. Hence, it is an inner product.


9.1.4 (a) Observe that if h1 − x2 , 1 − x2 i = 0(0) + 0(0) = 0, but 1 − x2 is not the zero vector in P2 (R), so the
function is not positive definite and hence not an inner product.
(b) Let p, q, r ∈ P2 (R) and s, t ∈ R. We have

hp, pi =2[p(−2)]2 + [p(1)]2 + [p(2)]2 ≥ 0

and hp, pi = 0 if and only if p(−2) = p(1) = p(2) = 0. Since p is a polynomial of degree at most
2 with three roots we get that p = 0. Hence, hp, pi = 0 if and only if p = 0.

hp, qi =p(−2)q(−2) + p(1)q(1) + p(2)q(2)


=q(−2)p(−2) + q(1)p(1) + q(2)p(2) = hq, p, i
hsp + tq, ri =(sp + tq)(−2)[r(−2)] + (sp + tq)(1)[r(1)] + (sp + tq)(2)[r(2)]
= [sp(−2) + tq(−2)]r(−2) + [sp(1) + tq(1)]r(1) + [sp(2) + tq(2)]r(2)
= s[p(−2)r(−2) + p(1)r(1) + p(2)r(2)] + t[q(−2)r(−2) + q(1)r(1) + q(2)r(2)] = shp, ri + thq, ri

Thus, it is an inner product.


(c) Let p, q, r ∈ P2 (R) and a, b ∈ R. We have

hp, pi =2[p(−1)]2 + 2[p(0)]2 + 2[p(1)]2 − 2p(−1)p(1)


=[p(−1)]2 + 2[p(0)]2 + [p(1)]2 + [p(−1)]2 − 2p(−1)p(1) + [p(1)]2
=[p(−1)]2 + 2[p(0)]2 + [p(1)]2 + [p(−1) − p(1)]2 ≥ 0
Section 9.1 Solutions 171

and hp, pi = 0 if and only if p(−1) = p(0) = p(1) = 0. Thus, hp, pi = 0 if and only if p = 0.

hp, qi =2p(−1)q(−1) + 2p(0)q(0) + 2p(1)q(1) − p(−1)q(1) − p(1)q(−1)


=2q(−1)p(−1) + 2q(0)p(0) + 2q(1)p(1) − q(−1)p(1) − q(1)p(−1) = hq, p, i
hap + bq, ri =2[ap(−1) + bq(−1)]r(−1) + 2[ap(0) + bq(0)]r(0) + 2[ap(1) + bq(1)]r(1)
− [ap(−1) + bq(−1)]r(1) − [ap(1) + bq(1)]r(−1)
 
=a 2p(−1)r(−1) + 2p(0)r(0) + 2p(1)r(1) − p(−1)r(1) − p(1)r(−1)
 
+ b 2q(−1)r(−1) + 2q(0)r(0) + 2q(1)r(1) − q(−1)r(1) − q(1)r(−1)
= ahp, ri + bhq, ri

Thus, it is an inner product.


9.1.5 (a) Observe that *" # " #+
0 0 0 0
, = 2(0)2 + 02 + 2(0)2 = 0
0 1 0 1
" #
0 0
but, , ~0. Hence, the function is not positive definite and hence not an inner product.
0 1
(b) Observe that *" # " #+
0 0 0 0
, = 02 + 02 − 02 − 12 = −1
0 1 0 1
Hence, the function is not positive definite and hence not an inner product.
9.1.6 We have
h~v, ~0i = h~v, 0~vi = 0h~v, ~vi = 0
as required.
9.1.7 We have
hA~x, ~yi = (A~x) · ~y = (A~x)T ~y = ~xT AT ~y = ~x · (AT ~y) = h~x, AT ~yi
9.1.8 We have

h~x, ~yi = hx1~e1 + x2~e2 , y1~e1 + y2~e2 i


= hx1~e1 , y1~e1 + y2~e2 i + hx2~e2 , y1~e1 + y2~e2 i
= hx1~e1 , y1~e1 i + hx1~e1 , y2~e2 i + hx2~e2 , y1~e1 i + hx2~e2 , y2~e2 i
= x1 y1 h~e1 , ~e1 i + x1 y2 h~e1 , ~e2 i + x2 y1 h~e2 , ~e1 i + x2 y2 h~e2 , ~e2 i

as required.
9.1.9 Let f, g, h ∈ C[−π, π] and a, b ∈ R. We have
Z π
h f, f i = ( f (x))2 dx ≥ 0
−π
172 Section 9.2 Solutions

since ( f (x))2 ≥ 0. Moreover, we have h f, f i = 0 if and only if f (x) = 0 for all x ∈ [−π, π].
Z π Z π
h f, gi = f (x)g(x) dx = g(x) f (x) dx = hg, f, i
Z−ππ −π

ha f + bg, bi = (2 f (x) + bg(x))h(x) dx


−π
Z π Z π
=a f (x)h(x) dx + b g(x)h(x) dx = ah f, hi + bhg, hi
−π −π

Thus, it is an inner product.


" # " #
1 −1
9.1.10 (a) The statement is false. Consider the standard inner product on R2 and take ~x = and ~y = .
0 0
Then, we get ~x · ~y = −1.
(b) The statement is false. We have

ha~x + b~y, c~x + d~yi = ha~x + b~y, c~xi + ha~x + b~y, d~yi
= ha~x, c~xi + hb~y, c~xi + ha~x, d~yi + hb~y, d~yi
= abh~x, ~xi + bch~y, ~xi + adh~x, ~yi + bdh~y, ~yi
= abh~x, ~xi + (bc + ad)h~x, ~yi + bdh~y, ~yi

9.2 Problem Solutions


9.2.1 (a) We have

h1 + x, −2 + 3xi = 0(−5) + 1(−2) + 2(1) = 0


h1 + x, 2 − 3x2 i = 0(−1) + 1(2) + 2(−1) = 0
h−2 + 3x, 2 − 3x2 i = (−5)(−1) + (−2)(2) + 1(−1) = 0

Therefore, it is an orthogonal set. Since it does not contain the zero vector, we get that it is a
linearly independent by Theorem 9.2.3. Thus, it is a linearly independent set of 3 vectors in P2 (R)
and hence it is an orthogonal basis for P2 (R).
(b) We have
h1 + x, 1i 3
=
k1 + xk2 5
h−2 + 3x, 1i −6 1
= =−
k − 2 + 3xk2 30 5
2
h2 − 3x , 1i 0
= =0
k2 − 3x2 k2 6
 
 3/5 
Hence, [1]B = −1/5.
 
0
 
Section 9.2 Solutions 173

(c) We have

h1 + x, xi 2
=
k1 + xk2 5
h−2 + 3x, xi 6 1
= =
k − 2 + 3xk2 30 5
h2 − 3x2 , xi 0
= =0
k2 − 3x2 k2 6
 
2/5
Hence, [x]B = 1/5.
 
0
 

(d) We have

h1 + x, x2 i 2
=
k1 + xk2 5
h−2 + 3x, x2 i −4 2
2
= =−
k − 2 + 3xk 30 15
h2 − 3x2 , x2 i −2 1
2 2
= =−
k2 − 3x k 6 3
 
 2/5 
Hence, [x2 ]B = −2/15.
 
−1/3
 

(e) i. Consider

3 − 7x2x = c1 (1 + x) + c2 (−2 + 3x) + c3 (2 − 3x2 ) = (c1 − 2c2 + 2c3 ) + (c1 − 3c2 )x + (−3c3 )x2

Row reducing the corresponding augmemanted matrix gives


   
 1 −2 2 3   1 0 0 −3/5 
 1 −3 0 −7  ∼  0 1 0 −32/15 
   
0 0 −3 1 0 0 1 −1/3

ii. We have

[3 − 7x + x2 ]B = 3[1]B − 7[x]B + 1[x2 ]B


     
 3/5  2/5  2/5 
= 3 −1/5 − 7 1/5 + −2/15
     
0 0 −1/3
     
 
 −3/5 
= −32/15
 
−1/3
 
174 Section 9.2 Solutions

iii. We have
h1 + x, 3 − 7x + x2 i −3
=
k1 + xk2 5
2
h−2 + 3x, 3 − 7x + x i −64 32
= =−
k − 2 + 3xk2 30 15
h2 − 3x2 , 3 − 7x + x2 i −2 1
= =−
k2 − 3x2 k2 6 3
 
 −3/5 
Hence, [3 − 7x + x2 ]B = −32/15.
 
−1/3
 

9.2.2 (a) We have


   
* 1  2+
   
−2 , 2 = 1(2) + (−2)(2) + 2(1) = 0
2 1
   
   
* 1  −2+
   
−2 ,  1  = 1(−2) + (−2)(1) + 2(2) = 0
2 2
   
   
*2 −2+
   
2 ,  1  = 2(−2) + 2(1) + 1(2) = 0
1 2

Therefore, B is an orthogonal set in R3 . Since it does not contain the zero vector, we get that it is
a linearly independent by Theorem 9.2.3. Thus, it is a linearly independent set of 3 vectors in R3
and hence it is an orthogonal basis for R3 .
(b) We have
   
* 1   1 +
−2 , −2 = 12 + (−2)2 + 22 = 9
   
2 2
   
   
*2 2+
2 , 2 = 22 + 22 + 12 = 9
   
1 1
   
   
*−2 −2+
 1  ,  1  = (−2)2 + 12 + 22 = 9
   
2 2
   

Thus, an orthonormal basis for S is


     

  1/3  2/3 −2/3 


 , 2/3 ,  1/3 

C= −2/3
    

 
      


 2/3
  
1/3
 
2/3 

Section 9.2 Solutions 175

(c) We have
   
* 1  4+
   
−2 , 3 = 1(4) + (−2)(3) + 2(5) = 8
2 5
   
   
*2 4+
   
2 , 3 = 2(4) + 2(3) + 1(5) = 19
1 5
   
*−2 4+
   
 1  , 3 = (−2)(4) + 1(3) + 2(5) = 5
2 5

Thus, using Theorem 9.2.4 we get


 
 8/9 
[~x]B = 19/9
 
5/9
 

(d) We have
   
* 1/3  4+ !
    1 2 2
−2/3 , 3 = (4) + − (3) + (5) = 8/3
3 3 3
2/3 5
   
   
*2/3 4+
    2 2 1
2/3 , 3 = (4) + (3) + (5) = 19/3
3 3 3
1/3 5
   
*−2/3 4+
    2 1 2
 1/3  , 3 = − (4) + (3) + (5) = 5/3
3 3 3
2/3 5
   

Thus, using Corollary 9.2.5 we get


 
 8/3 
[~x]C = 19/3
 
5/3
 

9.2.3 (a) We have


   
* 1   3 +
   
 2  , −1 = 1(3) + 2(2)(−1) + (−1)(−1) = 0
−1 −1
   
   
* 1  3+
   
 2  , 1 = 1(3) + 2(2)(1) + (−1)(7) = 0
−1 7
   
* 3  3+
   
−1 , 1 = 3(3) + 2(−1)(1) + (−1)(7) = 0
−1 7
176 Section 9.2 Solutions

Therefore, B is an orthogonal set in R3 under the given inner product. Since it does not contain
the zero vector, we get that it is a linearly independent by Theorem 9.2.3. Thus, it is a linearly
independent set of 3 vectors in R3 and hence it is an orthogonal basis for R3 .
(b) We have
   
* 1   1 +
 2  ,  2  = 12 + 2(2)2 + (−1)2 = 10
   
−1 −1
   
   
* 3   3 +
2 2 2
   
−1 , −1 = 3 + 2(−1) + (−1) = 12
−1 −1
   
*3 3+
1 , 1 = 32 + 2(1)2 + 72 = 60
   
7 7
   

Hence, an orthonormal basis C for R3 under this inner product is


      
1 3 3
 1   1   1  


 
C= √ 2 , √ −1 , √ 1


 
 
 

 10 −1

 
 
 12 −1

 
 60 7 
  

(c) We have
   
* 1  1+
   
 2  , 1 = 1(1) + 2(2)(1) + (−1)(1) = 4
−1 1
   
   
* 3  1+
   
−1 , 1 3(1) + 2(−1)(1) + (−1)(1) = 0
−1 1
   
*3 1+
   
1 , 1 = 3(1) + 2(1)(1) + 7(1) = 12
7 1
   

   
 4/10  2/5
Therefore, by Theorem 9.2.4 we have [~x]B =  0/12  =  0 .
   
12/60 1/5
   

(d) We have
   
*  1  1+
1     4
√  2  , 1 = √
10 −1 1
    10
    
 1  3  1
* +
 √ −1 , 1 = 0
 12    
−1 1
Section 9.2 Solutions 177
   
* 3 1+
1     12
√ 1 , 1 = √
60 7 1 60
 √ 
 4/ 10 
 
Therefore, by Corollary 9.2.5 we have [~x]C =  0√ .
12/ 60
 

9.2.4 (a) We have


*" # " #+
1 1 1 −1
, = 1(1) + 1(−1) + (−1)(1) + (−1)(−1) = 0
−1 −1 1 −1
*" # " #+
1 1 −1 1
, = 1(−1) + 1(1) + (−1)(1) + (−1)(−1) = 0
−1 −1 1 −1
*" # " #+
1 −1 −1 1
, = 1(−1) + (−1)(1) + 1(1) + (−1)(−1) = 0
1 −1 1 −1

Therefore, it is an orthogonal set. Since it does not contain the zero vector, we get that it is a
linearly independent by Theorem 9.2.3. By definition, it is also a spanning set, and hence it is an
orthogonal basis for S.
(b) We have
*" # " #+
1 1 1 1
, = 12 + 12 + (−1)2 + (−1)2 = 4
−1 −1 −1 −1
*" # " #+
1 −1 1 −1
, = 12 + (−1)2 + 12 + (−1)2 = 4
1 −1 1 −1
*" # " #+
−1 1 −1 1
, = (−1)2 + 12 + 12 + (−1)2 = 4
1 −1 1 −1

Hence, an orthonormal basis C for S is


(" # " # " #)
1/2 1/2 1/2 −1/2 −1/2 1/2
C= , ,
−1/2 −1/2 1/2 −1/2 1/2 −1/2

(c) We have
*" # " #+
1/2 1/2 −3 6
, =3
−1/2 −1/2 −2 −1
*" # " #+
1/2 −1/2 −3 6
, = −5
1/2 −1/2 −2 −1
*" # " #+
−1/2 1/2 −3 6
, =4
1/2 −1/2 −2 −1
 
 3 
Therefore, [~x]C = −5.
 
4
 
178 Section 9.2 Solutions

9.2.5 (a) The matrix satisfies AAT = I, so it is orthogonal.


(b) The matrix satisfies AAT = I, so it is orthogonal.
(c) The first column is not a unit vector, so the matrix is not orthogonal.
(d) The matrix satisfies AAT = I, so it is orthogonal.
(e) The first column is not a unit vector, so the matrix is not orthogonal.
(f) The columns are standard basis vectors and so form an orthonormal basis for R3 . Hence, the
matrix is orthogonal.
(g) The first column is not a unit vector, so the matrix is not orthogonal.
9.2.6 We first look for a polynomial a + bx + cx2 ∈ P2 (R) that is orthogonal to both 1 − x2 and x − x2 .

ha + bx + cx2 , 1 − x2 i = (a − b + c)(0) + (a)(1) + (a + b + c)(0) = a


ha + bx + cx2 , x − x2 i = (a − b + c)(−2) + (a)(0) + (a + b + c)(0) = −2a + 2b − 2c

This implies that a = 0 and −2a + 2b − 2c = 0, so any polynomial of the form bx + cx2 is orthogonal to
each polynomial in B. We use x + x2 , and normalize it to 21 (x + x2 ). This polynomial extends B to an
orthonormal basis of P2 (R).
9.2.7 We have p p √ p
kt~vk = ht~v, t~vi = t2 h~v, ~vi = t2 h~v, ~vi = |t|k~vk
9.2.8 (a) Since PPT = I we get

1 = det I = det PPT = det P det PT = (det P)2

Thus, det P = ±1.


(b) Let λ be a real eigenvalue of P with corresponding eigenvector ~x. Then, using (a) we get

k~xk = kP~xk = kλ~xk = |λ|k~xk

Hence, we get |λ| = 1 and so λ = ±1 since λ is real.


" #
0 1
(c) Take P = . Then P is clearly orthogonal and the eigenvalues of P are ±i.
−1 0
(d) If PT = P−1 and QT = Q−1 , then

(PQ)T = QT PT = Q−1 P−1 = (PQ)−1

Thus, PQis orthogonal.


9.2.9 The matrix whose columns are the vectors in B is orthogonal. Thus, the rows also form an orthonormal
basis for R3 by Theorem 9.2.6. Thus, such an orthonormal basis is
 √   √   √ 
  1/ 6   −2/ 6   1/ 6 


 √ √ √ 


, ,
    
 1/ √3   1/ 3   1/ √3 
   


 

0 −1/ 2 
     
1/ 2

 
Section 9.3 Solutions 179

9.2.10 We have

~v1 · ~v1 ~v1 · ~v2 · · · ~v1 · ~vn


 
 T
~v1 

i ~v2 · ~v1 ~v2 · ~v2 · · · ~v2 · ~vn 
Q Q =  ...  ~v1
T   h
··· ~vm =  .

.. .. ..
 ..

  . . . 
~vTm  
~vn · ~v1 ~vn · ~v2 · · · ~vn · A~vn

But, {~v1 , . . . , ~vm } is orthonormal, so ~vi · ~vi = 1 and ~vi · ~v j = 0 for all i , j. Hence we have QT Q = I as
required.
9.2.11 (a) We have

< ~x, ~y >= < c1~v1 + · · · + cn~vn , d1~v1 + · · · + dn~vn >


=c1 < ~v1 , d1~v1 + · · · + dn~vn > + · · · + cn < ~vn , d1~v1 + · · · + dn~vn >
=c1 d1 < ~v1 , ~v1 > + · · · + c1 dn < ~v1 , ~vn > +c2 d1 < ~v2 , ~v1 > +c2 d2 < ~v2 , ~v2 > +
c2 d2 < ~v2 , ~v2 > + · · · + c2 dn < ~v2 , ~vn > + · · · + cn d1 < ~vn , ~v1 > + · · · + cn dn < ~vn , ~vn >

But, since {~v1 , . . . , ~vn } is orthonormal we have < ~vi , ~vi >= 1 and < ~vi , ~v j >= 0 for i , j, hence we
get
< ~x, ~y >= c1 d1 + · · · + cn dn

(b) Taking ~y = ~x, the result of part (a) gives k~xk =< ~x, ~x >= c21 + · · · + c2n .
9.2.12 (a) We have d(~x, ~y) = k~x − ~yk ≥ 0 by Theorem 9.2.1(1).
(b) We have 0 = d(~x, ~y) = k~x − ~yk But, by Theorem 9.2.1(1), we have that k~x − ~yk = 0 if and only if
~x − ~y = ~0 as required.
(c) We have
q q q
d(~x, ~y) = k~x−~yk = h~x − ~y, ~x − ~yi = h(−1)(~y − ~x), (−1)(~y − ~x)i = (−1)(−1)h~y − ~x, ~y − ~xi = k~y−~xk = d(~y, ~x)

(d) Using Theorem 9.2.1(4) we get

d(~x,~z) = k~x − ~zk = k~x − ~y + ~y − ~zk ≤ |~x − ~y| + k~y − ~zk = d(~x, ~y) + d(~y,~z)

(e) We have
d(~x, ~y) = k~x − ~yk = k~x + ~z − ~z − ~yk = k~x + ~z − (~y + ~z)k = d(~x + ~z, ~y + ~z)

(f) Using Theorem 9.2.1(2) we get

d(c~x, c~y) = kc~x − c~yk = kc(~x − ~y)k = |c| k~x − ~yk = |c| d(~x, ~y)
180 Section 9.3 Solutions

9.3 Problem Solutions


9.3.1 Let the vectors in B be denoted w ~ 1, w
~ 2 , and w ~ 3 respectively. We first take ~v1 = w ~ 1 . Next, we get
     
2  1  5/2
w2 , ~v1 i
h~ 2 − −1  0  =  2 
~2 −
w ~
v 1 =
k~v1 k2   2    
3 −1 5/2
 
5
So, we take ~v2 = 4. Finally, we get
 
5
 

w3 , ~v1 i
h~ h~ w3 , ~v2 i
~v3 = w ~3 − ~v1 − ~v2
k~v1 k2 k~v2 k2
       
1  1  5 −2/3
0 22
= 3 −  0  − 4 = 5/3 
       
  2   66   
1 −1 5 −2/3

Consequently, {~v1 , ~v2 , ~v3 } is an orthogonal basis for R3 .


9.3.2 Let the vectors in B be denoted w ~ 1, w
~ 2 , and w ~ 3 respectively. We first take ~v1 = w ~ 1 . Next, we get
     
 2  −3  −8/11 
h~w2 , ~v1 i   −10   
~2 −
w ~v1 = −2 −  1  = −12/11

k~v1 k2 11   
2 −1 12/11
 

 
 2 
So, we take ~v2 =  3 . Finally, we get
 
−3
 

w3 , ~v1 i
h~ w3 , ~v2 i
h~
~v3 = w ~3 − 2
~v1 − ~v2
k~v1 k k~v2 k2
       
 2  −3  2  0
  −2   16    
=  3  − 1 − 3 = 1
  11   22    
 
−1 −1 −3 1

Consequently, {~v1 , ~v2 , ~v3 } is an orthogonal basis for R3 .


9.3.3 Take p1 (x) = 1. Then
hx, 1i 0
p2 (x) = x − 1= x− 1= x
k1k2 3
hx2 , 1i hx2 , xi
p3 (x) = x2 − 1 − x
k1k2 kxk2
2 0 2
= x2 − 1 − x = x2 −
3 2 3
Instead, we take p3 (x) = −2 + 3x2 . Then, {1, x, −2 + 3x2 } is an orthogonal basis for P2 (R). To make it
an orthonormal basis, we divide each vector by its length to get { √13 , √12 x, √16 (−2 + 3x2 )}.
Section 9.3 Solutions 181

9.3.4 Take p1 (x) = 1. Then


hx, 1i 3
p2 (x) = x − 2
1= x− 1= x−1
k1k 3
2 2
hx , 1i hx , x − 1i
p3 (x) = x2 − 2
1− (x − 1)
k1k kx − 1k2
5 4 1
= x2 − 1 − (x − 1) = x2 − 2x +
3 2 3
Instead, we take p3 (x) = 1 − 6x + 3x2 . Then, {1, x − 1, 1 − 6x + 3x2 } is an orthogonal basis for P2 (R). To
make it an orthonormal basis, we divide each vector by its length to get { √13 , √12 (x−1), √118 (1−6x+3x2 )}.
9.3.5 To find an orthogonal basis for S, we apply the Gram-Schmidt Procedure. Let the vectors in the spanning
set be denoted w~ 1, w
~ 2 , and w
~ 3 respectively. Then, we first take ~v1 = w ~ 1 . Next, we get

−1 −2  1/2 


     
w2 , ~v1 i
h~  3  12  2   3/2 
~2 −
w ~
v 1 =   −
 1  16  2  = −1/2
   
k~v1 k2      
1 2 −1/2
 
 1 
 3 
So, we take ~v2 =  . Finally, we get
−1
−1

w3 , ~v1 i
h~ w3 , ~v2 i
h~
~v3 = w ~3 − 2
~v1 − ~v2
k~v1 k k~v2 k2
−2
       
 1  1 0
 2  −2  2  9  3   0 
=   −   −   =  
−2 16  2  12 −1 −1
0 2 −1 1

Consequently, {~v1 , ~v2 , ~v3 } is an orthogonal basis for S.


9.3.6 Let the vectors in B be denoted w ~ 1, w ~ 2, w
~ 3 , and w
~ 4 respectively. Then, we first take ~v1 = w ~ 1 . Next, we
get " # " # " #
w2 , ~v1 i
h~ 1 0 5 −1 1 1/2 1/2
~2 −
w ~v1 = + =
k~v1 k2 −1 −1 10 2 2 0 0
" #
1 1
So, we take ~v2 = . Therefore, we see that w ~ 3 ∈ Span{~ ~ 2 } = Span{~v1 , ~v2 }, so we can ignore it.
w1 , w
0 0
Finally, we get
w4 , ~v1 i
h~ w4 , ~v2 i
h~
~v3 = w
~4 − ~v1 − ~v2
k~v k2 k~v2 k2
" # 1 " # " # " #
1 0 1 −1 1 1 1 1 3/5 −3/5
= − − =
0 1 10 2 2 2 0 0 −1/5 4/5

Consequently, {~v1 , ~v2 , ~v3 } is an orthogonal basis for S.


182 Section 9.4 Solutions

9.3.7 We let w1 = x. Then


h2x2 + x, xi 2hx2 , xi + hx, xi 2(−2) + 4
w2 = 2x2 + x − x = 2x2 + x − x = 2x2 + x − x = 2x2 + x
hx, xi 4 4
h2, xi h2, 2x2 + xi
w3 = 2 − x− (2x2 + x)
hx, xi h2x2 + x, 2x2 + xi
2·2 2 · 2h1, x2 i + 2h1, xi
=2− x− (2x2 + x)
4 4hx , x i + 2hx2 , xi + 2hx, x2 i + hx, xi
2 2

4(−2) + 2 · 2 1 1
=2−x− (2x2 + x) = 2 − x + (2x2 + x) = 2 + x2 − x
4 · 3 + 2(−2) + 2(−2) + 4 2 2
So an orthogonal basis is B2 = {x, 2x2 + x, 2 + x2 − 21 x}.
9.3.8 (a) Consider
~0 = c1~v1 + c2~v2 + c3~e1 + c4~e2 + c5~e3 + c6~e4
Row reducing the corresponding coefficient matrix gives
   
 1 3 1 0 0 0 1 0 1/4 0 0 3/4 
−1 1 0 1 0 0 0 1 1/4 0

0 −1/4
−1 1 0 0 1 0 ∼ 0 0 0
  
1 0 1 

   
1 −1 0 0 0 1 0 0 0 0 1 1

Thus, {~v1 , ~v2 , ~e2 , ~e3 } is a basis for R4 .


(b) Observe that ~v1 and ~v2 are already orthogonal. Hence, the next step in the Gram-Schmidt proce-
dure gives
h~e2 , ~v1 i h~e2 , ~v2 i
~e2 − ~v1 − ~v2
k~v1 k2 k~v2 k2
 1 
 
     
0 −1  3   0 
1 −1   1  1   2/3 
=   −  1  −   =  
0 4   12  1  −1/3
−1
0 −1 1/3
1
 
 0 
 2 
Hence, we take ~v3 =  . Next, we get
−1
1
h~e3 , ~v1 i h~e3 , ~v2 i h~e3 , ~v3 i
~v4 = ~e3 − 2
~v1 − 2
~v2 − ~v2
k~v1 k k~v2 k k~v3 k2
         
0 1 3 0  0 
0 −1 −1 1  1  −1  2   0 
=   −   −   −   =  
1 4 −1 12  1  6 −1 1/2
0 1 −1 1 1/2

So, we have extended {~v1 , ~v2 } to an orthogonal basis {~v1 , ~v2 , ~v3 , ~v4 } for R4 .
Section 9.4 Solutions 183

9.4 Problem Solutions


 
 x1 
9.4.1 (a) Let ~x =  x2  ∈ S⊥1 . Then, we have
 
x3
 
   
3  x1 
0 = 2 ·  x2  = 3x1 + 2x2 + x3
   
1 x3
   
   
 1   x1 
0 = −2 ·  x2  = x1 − 2x2 + 3x3
   
3 x3
   

Row reducing the corresponding coefficient matrix gives


" # " #
3 2 1 1 0 1

1 −2 3 0 1 −1
  

 −1
 
 
Thus, we get B =   1  spans S⊥1 and B is clearly linearly independent, so it is a basis for S⊥1 .

 1 

   

 
 x1 
 x 
(b) Let ~x =  2  ∈ S⊥2 . Then, we have
 x3 
x4
   
 1   x1 
−2  x 
0 =   ·  2  = x1 − 2x2 + x4
 0   x3 
1 x4
The general solution to this linear equation is
−2 −1
     
0
 1  0  0 
~x = a   + b   + c   , a, b, c ∈ R
 0  1  0 
0 0 1
Therefore, we have       


 −2 0 −1 

  1  0  0 


 
S2 = Span  , ,
 
 
 
 
 
 
 
0 1 0
 
 
 
 
 
 
 


      
      


 0

0 1 

     


 −2 0 −1 


 1  0  0 
 

Since B =  , , is linearly independent by Theorem 2.2.6., we have that B is a basis
      

  0  1  0 
     

       

 0

0 1 

for S⊥2 .
184 Section 9.4 Solutions

   

 1  x1 
 
  ⊥
(c) We first observe that a basis for S3 is  0 . Let ~
x =  x2  ∈ S3 . Then
  
 
 

1
 
 x 3
   
1  x1 
0 = 0 ·  x2  = x1 + x3
   
1 x3
   

The general solution to this linear equaiton is


   
−1 0
~x = a  0  + b 1 , a, b ∈ R
   
1 0
   

    

 −1 0
   
 
Consequently, we have that B =   0  , 1 spans S⊥3 and is clearly linearly independent, so B is



 1 

0
    
a basis for S⊥3 .
" #
a1 a2
(d) Let A = ∈ S⊥4 . Then
a3 a4
" # " #
a a2 1 0
0=h 1 , i = a1 + a3
a3 a4 1 0
" # " #
a a2 2 −1
0=h 1 , i = 2a1 − a2 + a3 + 3a4
a3 a4 1 3
Row reducing the coefficient matrix of the homogeneous system gives
" # " #
1 0 1 0 1 0 1 0

2 −1 1 3 0 1 1 −3
Hence, we get the general solution is
−1
     
a1  0
a  −1 3
 2  = s   + t   ,
  s, t ∈ R
a3   1  0
 
a4 0 1
(" # " #) (" # " #)
−1 −1 0 3 −1 −1 0 3
Therefore, the orthogonal complement of S4 is S⊥4 = Span , . Since ,
1 0 0 1 1 0 0 1
is also clearly linearly independent, it is a basis for S⊥4 .
" #
a a2
(e) Let A = 1 ∈ S⊥5 . Then
a3 a4
" # " #
a1 a2 2 1
0=h , i = 2a1 + a2 + a3 + a4
a3 a4 1 1
" # " #
a a2 −1 1
0=h 1 , i = −a1 + a2 + 3a3 + a4
a3 a4 3 1
Section 9.4 Solutions 185

Row reducing the coefficient matrix of the homogeneous system gives


" # " #
2 1 1 1 1 0 −2/3 0

−1 1 3 1 0 1 7/3 1

Hence, we get the general solution is


     
a1   2/3   0 
−7/3
a2   + t −1 ,
 
  =
a3  s  s, t ∈ R
 1 
  0 
   
a4 0 1
(" # " #)
2/3 −7/3 0 −1
Therefore, the orthogonal complement of S5 is S⊥5 = Span , . Since
1 0 0 1
(" # " #)
2/3 −7/3 0 −1
, is also clearly linearly independent, it is a basis for S⊥5 .
1 0 0 1
(f) We first need to find a basis for S6 . Every vector in S6 has the form
" # " # " # " #
−b − d b −1 1 0 0 0 0
=b +c +d
c d 0 0 1 0 0 1
(" # " # " #)
−1 1 0 0 0 0
Thus, B = , , spans S6 and is clearly linearly independent, so it is a basis
0 0 1 0 0 1
for S6 .
" #
a a2
Let A = 1 ∈ S⊥6 . Then
a3 a4
"# " #
a1 a2 −1 1
0=h , i = −a1 + a2
a3 a4 0 0
" # " #
a a 0 0
0=h 1 2 , i = a3
a3 a4 1 0
" # " #
a1 a2 0 0
0=h , i = a4
a3 a4 0 1
   
a1  1
a  1
We get that the general solution of this system is  2  = s  , s ∈ R.
a3  0
a4 0
     


 1 
 

 1 
 1     

  1

 
Therefore, the orthogonal complement of S6 is S⊥6 = Span  . Since  is also clearly
  
0 0
     

   
 
  
  
 
  
0 0
 
 
 

linearly independent, it is a basis for S⊥6 .
186 Section 9.4 Solutions

9.4.2 (a) We need to find the general form of any vector orthogonal to x2 + 1. If a + bx + cx2 ∈ S⊥ , then we
have
0 = ha + bx + cx2 , x2 + 1i = (a − b + c)(2) + a(1) + (a + b + c)(2) = 5a + 4c
Hence, c = − 45 a. Thus, we have
!
2 5 2 5 2
a + bx + cx = a + bx − ax = a 1 − x + bx
4 4

Hence, S⊥ is spanned by B = {4 − 5x2 , x}. Observe that

h4 − 5x2 , xi = (−1)(−1) + 4(0) + (−1)(1) = 0

So, B = {4 − 5x2 , x} is an orthogonal basis for S⊥ .


(b) We need to find the general form of any vector orthogonal to x2 . If a + bx + cx2 ∈ S⊥ , then we
have
0 = ha + bx + cx2 , x2 i = (a − b + c)(1) + a(0) + (a + b + c)(1) = 2a + 2c
Hence, c = −a. Thus, we have
 
a + bx + cx2 = a + bx − ax2 = a 1 − x2 + bx

Hence, S⊥ is spanned by B = {1 − x2 , x}. Observe that

h1 − x2 , xi = (0)(−1) + 1(0) + (0)(1) = 0

So, B = {1 − x2 , x} is an orthogonal basis for S⊥ .


 
1
2
9.4.3 (a) We apply the Gram-Schdmit Procedure. Let ~v1 = w
~ 1 =  .
0
1
     
2 1  1 
w2 , ~v1 i
h~ 1 6 2 −1
~v2 = w
~2 − ~v1 =   −   =  
k~v1 k2 1 6 0  1 
2 1 1
−1
       
2 1  1 
w3 , ~v1 i
h~ w3 , ~v2 i
h~ 1 6 2 6 −1 1  1 
~v3 = w
~3 − ~v1 − ~v2 =   −   −   =  
k~v1 k2 k~v2 k2 3 6 0 4  1  2  3 
2 1 1 −1

Then, normalizing the vectors we get the orthonormal basis


( " # " # " #)
1 1 2 1 1 −1 1 −1 1
√ , , √
6 0 1 2 1 1 2 3 3 −1
Section 9.4 Solutions 187

(b) Denote the vectors in the orthonormal basis from part (a) by ~c1 , ~c2 , and ~c3 respectively. Then we
get

projS (~y) = h~y, ~c1 i~c1 + h~y, ~c2 i~c2 + h~y, ~c3 i~c3
−1 6
       
1  1 
12 1 2 12 1 −1 −12  1  0
= √ √   +   +  = 
6 6 0 2 2  1  12  3  0
1 1 −1 6

(c) We have

perpS (~z) = ~z − projS (~z) = ~z − h~z, ~c1 i~c1 + h~y, ~c2 i~c2 + h~y, ~c3 i~c3
−1 1 3/2
           
1 1  1 
0 3 1 2 3 1 −1 −3  1  0  0 
=   − √ √   +   +   =   −  
0 6 6 0 2 2  1  12  3  0  0 
2 1 1 −1 2 3/2
−1/2
 
 0 
=  
 0 
1/2
"# " # " #
1 0 1 1 2 0
9.4.4 (a) Denote the given basis by ~z1 = , ~z2 = , ~z3 = ~ 1 = ~z1 . Then, we get
. Let w
−1 1 1 1 1 1
" # " # " #
~z2 · w
~1 1 1 1 1 0 2/3 1
~ 2 = ~z2 − projw~ 1 (~z2 ) = ~z2 −
w ~1 =
w − = To simplify calculations
k~
w1 k 2 1 1 3 −1 1 4/3 2/3
" #
2 3
we use w ~2 = instead. Then, we get
4 2
" # " # " # " #
~z3 · w
~1 ~ 2)
(~z3 · w 2 0 2 1 0 10 2 3 8/11 −10/11
~ 3 = z3 −
w ~
w 1 − ~
w 2 = − − =
w1 k2
k~ w2 k2
k~ 1 1 3 −1 1 33 4 2 5/11 −3/11
"#
8 −10
~3 =
We pick w . Then the set {~ ~ 2, w
w1 , w ~ 3 } is an orthogonal basis for S.
5 −3
(b) From our work in (a)
" #
~x · w
~1 ~x · w
~2 ~x · w
~3 11 19 −23 35/9 13/9
projS (A) = ~
w 1 + ~
w 2 + ~
w 3 = ~
w 1 + ~
w 2 + ~
w 3 =
k~w1 k2 k~w2 k2 k~w3 k2 3 33 198 −35/18 93/18

(c) From our work in (a)


" #
~x · w
~1 ~x · w
~2 ~x · w
~3 0 12 −18 0 2
projS (B) = ~1 +
w ~2 +
w ~3 = w
w ~1 + w ~2 + ~3 =
w
k~w1 k2 k~w2 k2 k~w3 k2 3 33 198 1 1

(d) Since projS (B) = B we have that B ∈ S. Since projS (A) , A, we have A < S.
188 Section 9.4 Solutions

9.4.5 (a) We let ~v1 = 1. Then

hx − x2 , 1i −2 2
~v2 = x − x2 − 2
1 = x − x2 − 1 = + x − x2
k1k 3 3

Thus, an orthogonal basis for S is {1, 2 + 3x − 3x2 }.


(b) We have

h1 + x + x2 , 1i h1 + x + x2 , 2 + 3x − 3x2 i
projS (1 + x + x2 ) = 1 +
k1k2 k2 + 3x − 3x2 k
5 4
= 1 + (2 + 3x − 3x2 )
3 24
1 1
= 2 + x − x2
2 2
1 3
perpS (1 + x + x ) = 1 + x + x − projS (1 + x + x2 ) = −1 + x + x2
2 2
2 2

(c) Since projS and perpS are linear mappings, from our work in (b) we get

projS (2 + 2x + 2x2 ) = 2 projS (1 + x + x2 ) = 4 + x − x2

and
perpS (2 + 2x + 2x2 ) = 2 perpS (1 + x + x2 ) = −2 + x + 3x2
9.4.6 We first extend the set to a basis for R3 . We take
     

  1  1 0
     
 
B = {~ ~ 2, w
w1 , w ~ 3} =   0  , 0 , 1

 
−1 0 0
 

 
 1 
We then apply the Gram-Schmidt procedure. Take ~v1 =  0 . Then
 
−1
 

     
1  1  3/5
w2 , ~v1 i
h~   2    
~2 −
w ~
v 1 = 0 − 0= 0 
k~v1 k2 5    
 
0 −1 2/5
 
3
Hence, we take ~v2 = 0.
 
2
 

       
0  1  3 0
w3 , ~v1 i
h~ w3 , ~v2 i
h~
~v3 = w
~3 − ~
v − ~
v = 1 − 0 0 − 0 0 = 1
       
2 1 2 2
k~v1 k k~v2 k

 
 
 

0 −1 2 0
   

Hence, an orthogonal basis is {~v1 , ~v2 , ~v3 }.


Section 9.6 Solutions 189

9.4.7 Let {~v1 , . . . , ~vk } be an orthonormal basis for W. Then, for any ~u, ~v ∈ V and s, t ∈ R, we have
projW (s~u + t~v) =hs~u + t~v, ~v1 i~v1 + · · · + hs~u + t~v, ~vk i~vk
=s h~u, ~v1 i~v1 + · · · + h~u, ~vk i~vk + t h~v, ~v1 i~v1 + · · · + h~v, ~vk i~vk
 

= s projW ~u + t projW ~v
Hence, projW is linear.
~ ∈ W⊥ . Then h~
Let w ~ = h~
w, ~vi i = 0 for 1 ≤ i ≤ k. Hence projW w w, ~vk i~vk = ~0, so
w, ~v1 i~v1 + · · · + h~
~ ∈ ker(projW ). Therefore, W⊥ ⊆ ker(projW ).
w
Let ~x ∈ ker(projW ), then ~0 = projW ~x = h~x, ~v1 i~v1 + · · · + h~x, ~vk i~vk
Since {~v1 , . . . , ~vk } is linearly independent, we get that h~x, ~vi i = 0 for 1 ≤ i ≤ k. Therefore, ~x ∈ W⊥ by
theorem. Hence, ker(projW ) ⊆ W⊥ . Consequently, W⊥ = ker(projW ) as required.
9.4.8 A basis for S⊥ is {p4 (x)} = {1}. Then
h1, −1 + x + x2 − x3 i −4
projS⊥ (−1 + x + x2 − x3 ) = 2
1= = −1
||1|| 4
Therefore,
projS (−1 + x + x2 − x3 ) = (−1 + x + x2 − x3 ) − projS⊥ (−1 + x + x2 − x3 ) = x + x2 − x3

9.5 Problem Solutions


9.5.1 By the Fundamental Theorem of Linear Algebra the orthogonal complement of the rowspace is the
nullspace of A. We have
   
 1 1 3 1  1
 
0 2 3 

 2 2 6 2  0 1 1 −2

−1 0 ∼
−2 −3 0 0 0 0 

   
3 1 7 7 0 0 0 0
    


 −2 −3 
    
−1  2 

 


So, a basis for Row(A) is  , .

1 0
 

 

 

 

 



    
    

 0

1 

9.5.2 We will prove that {~v1 , . . . , ~vk , w


~ 1, . . . , w
~ ℓ } is a linearly independent spanning set for U ⊕ W. Consider
c1~v1 + · · · + ck~vk + ck+1 w ~ ℓ = ~0
~ 1 + · · · + ck+ℓ w
Then, we have
c1~v1 + · · · + ck~vk = −ck+1 w
~ 1 − · · · − ck+ℓ w
~ℓ
The vector on the left is in U and the vector on the right is in W. But, the only vector that is both in U and
W is the zero vector. Therefore, each ci = 0 and hence {~v1 , . . . , ~vk , w
~ 1, . . . , w
~ ℓ } is linearly independent.
For any ~v ∈ U ⊕ W we have that ~v = ~u + w ~ by definition of U ⊕ W. We can write ~u = c1~v1 + · · · + ck~vk
~ = d1 w
and w ~ 1 + · · · + dℓ w
~ ℓ and hence
~v = c1~v1 + · · · + ck~vk + d1 w
~ 1 + · · · + dℓ w
~ℓ
and so {~v1 , . . . , ~vk , w
~ 1, . . . , w
~ ℓ } also spans U ⊕ W.
190 Section 9.6 Solutions

9.6 Problem Solutions


   
1 2  1
9.6.1 (a) We have A = 1 1  and ~b = 2. Hence, the vector ~x that minimizes kA~x − ~bk is
   
1 −1 5
   

~x = (AT A)−1 AT ~b
 
" #−1 " # 1 
3 2 1 1 1  
= 2 
2 6 2 1 −1  
5
" #" #
1 6 −2 8
=
4 −2 3 −1
" #
25/7
=
−19/14

   
 1 5  1
(b) We have A = −2 −7 and ~b = 1. Hence, the vector ~x that minimizes kA~x − ~bk is
   
1 2 0
   

~x = (AT A)−1 AT ~b
 
" #−1 " # 1
6 21 1 −2 1  
= 1
21 78 5 −7 2  
0
" #" #
1 78 −21 −1
=
27 −21 6 −2
" #
−4/3
=
−1/3

   
2 1   4 
(c) We have A = 2 −1 and ~b = −1. Hence, the vector ~x that minimizes kA~x − ~bk is
   
3 2 8
   

~x = (AT A)−1 AT ~b
 
" #−1 " # 4 
17 6 2 2 3  
= −1
6 6 1 −1 2  
8
" #" #
1 6 −6 30
=
66 −6 17 21
" #
9/11
=
59/22
Section 9.6 Solutions 191

   
1 2
 2
1 2 3
(d) We have A =   and ~b =  . Hence, the vector ~x that minimizes kA~x − ~bk is
1 3 
 2
1 3 3

~x = (AT A)−1 AT ~b
 
" # 2
#−1 "
4 10 1 1 1 2 3
=
10 26 2 2 3 3 2
 
 
3
" #" #
1 26 −10 10
=
4 −10 4 25
" #
5/2
=
0

1 −1 0
   
 2 
2 −1 1
 and ~b = −2. Hence, the vector ~x that minimizes kA~x − ~bk is
 
(e) We have A = 
2 2 1  3 
 
1 −1 0 −2

~x = (AT A)−1 AT ~b
 
 −1    2 
10 0 4  1 2 2 1   
 −2
=  0 7 1 −1 −1 2 −1  
  
 3 
4 1 2 0 1 1 0  
  
−2
 
 5/3 
=  5/3 
 
−11/3
 

 2 −2
   
1
−1 1 
 and ~b = 2. Hence, the vector ~x that minimizes kA~x − ~bk is
 
(f) We have A =   
 3 1 

1
 
2 −1 2

~x = (AT A)−1 AT ~b
 
" #−1 " # 1
18 −4 2 −1 3 2 2
=
−4 7 −2 1 1 −1 1
 
 
2
" #
9/22
=
1/11
192 Section 9.6 Solutions

9.6.2 (a) By the Approximation Theorem, the vector in P that is closet to ~x is


 
 0 
~x · ~n
projP (~x) = perp~n (~x) = ~x − ~
n = 3/2
k~nk2  
3/2

(b) By the Approximation Theorem, the vector in P that is closet to ~y is


 
 0 
~y · ~n
projP (~y) = perp~n (~y) = ~y − ~n = 1/2
 
k~nk2
1/2
 

(c) By the Approximation Theorem, the vector in P that is closet to ~x is


 
 1 
~z · ~n
projP (~z) = perp~n (~z) = ~z − ~
n = −1
k~nk2  
1

(d) In our calculations in (a), (b), and (c), we see that ~y · ~n > ~x · ~n and ~z ∈ P, so ~y is the furthest from
P.
   
1 −1 −3
9.6.3 (a) Let X = 1 0  and ~y =  2 . Then we get
   
1 1 2
   
" #
T −1 T 1/3
~a = (X X) X ~y =
5/2

Hence, the best fitting line is y = 13 + 52 x.


1 −1 −3
   

 and ~y = −1. Then we get


1 0   
(b) Let X = 
1 1   0 
1 2 1
" #
T −7/5
−1 T
~a = (X X) X ~y =
13/10

Hence, the best fitting line is y = − 57 + 13 10 x.

1 −2 4
   
 0 
1 −1 1
 and ~y = −2. Then we get
 
9.6.4 (a) Let X = 
1 1 1  0 
 
1 2 4 1
 
−3/2
~a = (X T X)−1 X T ~y =  2/5 
 
1/2
 

Hence, the best fitting quadratic is y = − 32 + 25 x + 12 x2 .


Section 9.6 Solutions 193

1 −2 4  0 


   
1 −1 1  1 
 
 
(b) Let X = 1 0 0 and ~y = −1. Then we get
1 1 1  3 
   
1 2 4 1
 
4/5
~a = (X T X)−1 X T ~y = 2/5
 
0
 

Hence, the best fitting quadratic is y = 54 + 25 x + 0x2 .


−1 (−1)2 
   
4
9.6.5 Let ~y = 1. Since the desired polynomial has the form bx + cx2 , the design matrix is X =  0 02  =
   
2 
1 1 1
  
 
−1 1
 0 0.
 
1 1
We have
 
" #−1 " # 4 " #
2 0 −1 0 1   −3/2
~a = (X T X)−1 X T ~y = 1 =
0 2 1 0 1   5/2
1

So, y = − 23 x + 52 x2 .
Chapter 10 Solutions

10.1 Problem Solutions


10.1.1 (a) We have C(λ) = det(A − λI) = λ2 + 4λ + 4 = (λ + 2)2 . We pick λ1 = −2. We have
" # " #
4 4 1 1
A + 2I = ∼
−4 −4 0 0
# "
−1
Hence, a unit eigenvector for λ1 = −2 is ~v1 = √1 . We extend {~v1 } to the orthonormal basis
12
" # " # " #
1 −1 1 −2 −8
{~v1 , ~v2 } for R2 with ~v2 = √12 . Taking Q = √12 gives QT AQ = = T.
1 1 1 0 −2
(b) We have C(λ) = det(A − λI) = λ2 − 5λ − 24 = (λ − 8)(λ + 3). We pick λ1 = 8. We have
" # " #
−7 4 1 −4/7
A − 8I = ∼
7 −4 0 0
" #
4
Hence, a unit eigenvector for λ1 = 8 is ~v1 = √1 . We extend {~v1 } to the orthonormal basis
65 7
" # " # " #
2 1 −7 1 4 −7 T 8 −3
{~v1 , ~v2 } for R with ~v2 = √65 . Taking Q = √65 gives Q AQ = = T.
4 7 4 0 −3
 
1
(c) Observe that λ = −3 is an eigenvalue of A with corresponding eigenvector ~v1 = 0. We extend
 
0
 
     

  1  0  0  
     
 
{~v1 } to the basis  0 , 1 , 0 for R3 . Thus, we pick P1 = I. Thus, we have


0 0 1
 

 
−3 4 5 
 −3 ~bT 

PT1 AP1 =  0 1 4 = 

~0 A1 
0 7 4
 

#" " #
1 4 ~ 4
where A1 = and b = .
7 4 5

194
Section 10.1 Solutions 195
" # " #
4 −7 8 −3
Our work in part (b) tells us that we can take Q = √1 and get T 1 = .
65 7 4 0 −3
 
 1 0 0√ 
1 ~0T   √

 = 0 4/ 65 −7/ 65. Let P = P1 P2 = P2 , then

We now take P2 = ~ √ √
0 Q  
0 7/ 65 4/ 65

√ √
 −3 51/ 65 −8/ 65
~
 T 
−3 b Q
PT AP =  ~
  
 =  0 8 −3 
  
0 T1 
0 0 −3

 
1
(d) Observe that λ = 7 is an eigenvalue of A with corresponding eigenvector ~v1 = 0. We extend {~v1 }
 
0
 
      

 1 0 0 
     

to the basis  0 , 1 , 0 for R3 . Thus, we pick P1 = I. Thus, we have



0 0 1
      

 
7 −6 5  ~ T 
 7 b 
PT1 AP1 = 0 1 4 = 

~0 A1 
0 6 3
 

"
# " #
1 4 ~ −6
where A1 = and b = .
6 3 5

We have C(λ) = det(A − λI) = λ2 − 4λ − 21 = (λ + 3)(λ − 7). So, λ1 = −3 and


" # " #
4 4 1 1
A + 3I = ∼
6 6 0 0
" #
−1
Hence, a unit eigenvector for λ1 = −3 is ~v1 = √12 . We extend {~v1 } to the orthonormal basis
1
" # " # " #
2 1 1 1 −1 1 T −3 2
{~v1 , ~v2 } for R with ~v2 = √2 . Taking Q = √2 gives Q A1 Q = = T1.
1 1 1 0 7
 
 1 0√ 0√ 
1 ~0T  

 = 0 −1/ 2 1/ 2. Let P = P1 P2 = P2 , then

We now take P2 = ~ √ √ 
0 Q 
0 1/ 2 1/ 2
√ √
 7 11/ 2 −1/ 2
~
 T
7 b Q 
PT AP = ~

 = 0 −3 2 
0 T1  
0 0 7
196 Section 10.1 Solutions
 
0
(e) Observe that λ = 2 is an eigenvalue of A with corresponding eigenvector ~v1 = 1. We extend {~v1 }
 
0
 
        

 0 1 0 0 1 0
     
  3
to the basis  1 , 0 , 0 for R . Thus, we pick P = 1 0 0. Thus, we have
  
 
 
 
 
 
 
  1
0 0 1
 
 0 0 1
 
2 2 1   ~ T 
 2 b 
PT1 AP1 = 0 1 −1 = ~

0 A1

0 4 5
 

" # " #
1 −1 ~ 2
where A1 = and b = .
4 5 1
We have C(λ) = det(A − λI) = λ2 − 6λ + 9 = (λ − 3)2 . So, λ1 = 3 and
" # " #
−2 −1 1 1/2
A − 3I = ∼
4 2 0 0
" #
−1
Hence, a unit eigenvector for λ1 = 3 is ~v1 = √15 . We extend {~v1 } to the orthonormal basis
2
" # " # " #
2 1 2 1 −1 2 T 3 5
{~v1 , ~v2 } for R with ~v2 = √5 . Taking Q = √5 gives Q A1 Q = = T1.
1 2 4 0 3
 
 1 0√ 0√ 
1 ~0T  

 = 0 −1/ 5 2/ 5. Let P = P1 P2 , then

We now take P2 = ~ √ √ 
0 Q 
0 2/ 5 1/ 5
√ 
 2 0 5
~
 T 
2 b Q 
PT AP = ~

 = 0 3 5 

0 T1 
0 0 3

(f) We first need to find a real eigenvalue of A. We have



−λ 1 1 −λ 1 1 −λ 1 −1
C(λ) = 2 3 − λ 6 = 0 1 − λ 2 − 2λ = 0 1 − λ 0
−1 −1 −2 − λ −1 −1 −2 − λ −1 −1 −λ

Thus, λ1 = 1 is an eigenvalue. Next, we need to pick a unit eigenvector corresponding to λ1 . We


have    
−1 1 1  1 0 1
A − I =  2 2 6  ∼ 0 1 2
   
−1 −1 −3 0 0 0
   
 √ 
−1/ 6
√ 
Thus, we take ~v1 = −2/ 6.

 √ 
1/ 6
Section 10.1 Solutions 197
 √ 
1/ 2
Next, we need to extend this to an orthonormal basis for R3 . We take w
~ 2 =  0√  and w
~3 =
 
1/ 2
 
 √ 
−1/ 3
 √  h i
 1/ √3 . Then we define P1 = ~v1 w
~2 w
~3 .
1/ 3
We find that  √ √ 
1 −10/ 3 −3 2
PT1 AP1 = 0 −1 0 
 
 √ 
0 4/ (6) 1
Notice that this is almost upper triangular. Weh can make iti upper triangular, by swapping the order
~ 3 . In particular, if we take P2 = ~v1 w
~ 2 and w
of w ~3 w~ 2 , then
 √ √ 
1 −10/ 3 −3 2 

PT2 AP2 = 0 1 4/ (6) = T
 
 
0 0 −1

is upper triangular.
(g) Observe that λ = 2 is an eigenvalue of A with corresponding h i ~v1 = ~e3 . We extend {~v1 }
eigenvector
4
to the basis ~e3 , ~e1 , ~e2 , ~e4 for R . Thus, we pick P1 = ~e3 ~e1 ~e2 ~e4 . Thus, we have


2 −1 1 0 
 
0 2 0 1 
PT1 AP1 =  
0 0 2 −1
0 0 0 2

is upper triangular.
10.1.2 If A is orthogonally similar to B, then there exists an orthogonal matrix P such that PT AP = B. If B is
orthogonally similar to C, then there exists an orthogonal matrix Q such that QT BQ = C. Hence,

C = QT BQ = QT PT APQ = (PQ)T A(PQ) = RT AR

where R = PQ is orthogonal since a product of orthogonal matrices is orthogonal. Thus, A is orthogonal


similar to C.
10.1.3 (a) If A and B are orthogonally similar, then there exists an orthogonal matrix P such that PT AP = B.
Since P is orthogonal, we get that P is invertible, and hence

B−1 = (PT AP)−1 = P−1 A−1 (PT )−1

Let Q = (PT )−1 . Then, QT = ((PT )−1 )T = P−1 . So, we have

B−1 = QT A−1 Q

and thus A−1 and B−1 are orthogonally similar.


198 Section 10.2 Solutions

(b) If A and B are orthogonally similar, then there exists an orthogonal matrix P such that PT AP = B.
Thus, we have
B2 = (PT AP)(PT AP) = PT A(PPT )AP = PT A2 P
Thus, A2 and B2 are also orthogonally similar.
(c) If A and B are orthogonally similar, then there exists an orthogonal matrix P such that PT AP = B.
Thus,
BT = (PT AP)T = PT AT (PT )T = PT AP = B
(d) If A and B are orthogonally similar, then there exists an orthogonal matrix P such that PT AP = B.
Assume that there exists an orthogonal matrix Q such that QT BQ = T is upper triangular. Then,

T = QT (PT AP)Q = (PQ)T A(PQ)

where PQ is orthogonal since both P and Q are orthogonal. Thus, A is also orthogonally similar
to T .

10.2 Problem Solutions



2 − λ 2
10.2.1 (a) We have C(λ) = det(A − λI) = = λ2 − 4λ = λ(λ − 4)
2 2 − λ
Hence, the eigenvalues are λ1 = 0 and λ2 = 4.
" # " # (" #)
2 2 1 1 −1
For λ1 = 0 we get A − λ1 I = ∼ . Thus, a basis for Eλ1 is .
2 2 0 0 1
" # " # (" #)
−2 2 1 −1 1
For λ2 = 4 we get A − λ2 I = ∼ . Thus, a basis for Eλ2 is .
2 −2 0 0 1
2
After" normalizing,
√ √the# basis vectors for the eigenspaces
" #form an orthonormal basis for R . Hence,
−1/√ 2 1/ √2 0 0
P= is orthogonal and PT AP = .
1/ 2 1/ 2 0 4
(b) We have

−λ 2 −2 −λ 0 −2 −λ 0 −2
C(λ) = 2 1 − λ 0 = 2 1 − λ 0 = 2 1 − λ 0
−2 0 1 − λ −2 1 − λ 1 − λ −4
0 1 − λ
= −(λ − 1)(λ2 − λ − 8)

Hence, λ1 √= 1 is an eigenvalue, √ and by the quadratic formula, we get the other eigenvalues are
λ2 = (1 + 33)/2 and λ3 = (1 − 33)/2.
   
−1 2 −2 1 0 0 
For λ1 = 1 we get A − λ1 I =  2 0 0  ∼ 0 1 −1
   
−2 0 0 0 0 0
   
  

 0
 
 
A basis for the eigenspace of λ1 is  1


   
 1
Section 10.2 Solutions 199
 1+ √33   √ 
− 2√ −2  1 0 −1+ 33 
√  2   4 
1+ 33 1− 33
For λ2 = 2 we get A − λ2 I =  2 2 0√  ∼ 0 1
  1


1− 33  0 0 0

−2 0 2
 √ 
1 − 33


 


A basis for the eigenspace of λ2 is   −4  .


 
 
 



 4 

 1− √33   √ 
− 2√ −2  1 0 −1− 33 
√  2   4 
1− 33 1+ 33
For λ3 = 2 we get A − λ3 I =  2 2 0√  ∼ 0 1 1


 
1+ 33  0 0 0

−2 0 2
 √ 



 1 + 33 


A basis for the eigenspace of λ3 is  −4 .
 
 

 
 

   

 4 

After normalizing, the √ basis vectors √ for the eigenspaces
√ form√an orthonormal basis for R3 . Hence,
 
 0
√ (1 − 33)/(66 − √ 2 33) (1 + 33)/(66 + √ 2 33)
 
P = 1/ 2 −1/(66 − 2√ 33) −1/(66 + 2√ 33)  orthogonally diagonalizes A to
 √
1/(66 − 2 33)

1/ 2 1/(66 + 2 33)
 
1 √0 0 
T  
P AP = 0 (1 + 33)/2

 0
√ .
0 0 (1 − 33)/2
 

4 − λ −2
(c) We have C(λ) = det(A − λI) = = λ2 − 11λ − 24 = (λ − 3)(λ − 8)
−2 7 − λ
Hence, the eigenvalues are λ1 = 3 and λ2 = 8.
" # " # (" #)
1 −2 1 −2 2
For λ1 = 3 we get A − λ1 I = ∼ . Thus, a basis for Eλ1 is .
−2 4 0 0 1
" # " # (" #)
−4 −2 1 1/2 −1
For λ2 = 8 we get A − λ2 I = ∼ . Thus, a basis for Eλ2 is .
−2 −1 0 0 2
2
After" normalizing,
√ √the# basis vectors for the eigenspaces
" #form an orthonormal basis for R . Hence,
2/ √5 −1/√ 5 3 0
P= is orthogonal and PT AP = .
1/ 5 2/ 5 0 8
(d) We have

2 − λ −2 −5 7 − λ 0 −7 + λ 7 − λ 0 0
C(λ) = −2 −5 − λ −2 = −2 −5 − λ −2 = −2 −5 − λ −4
−5 −2 2 − λ −5 −2 2 − λ −5 −2 −3 − λ
= −(λ − 7)(λ2 + 8λ + 7) = −(λ − 7)(λ + 1)(λ + 7)

Hence, the eigenvalues are λ1 = 7, λ2 = −1, and λ3 = −7.


   
−5 −2 −5 1 0 1
For λ1 = 7 we get A − λ1 I = −2 −12 −2 ∼ 0 1 0
   
−5 −2 −5 0 0 0
   
200 Section 10.2 Solutions

  

 −1
 
 
A basis for the eigenspace of λ1 is   0 

  1 

 

   
 3 −2 −5 1 0 −1
For λ2 = −1 we get A − λ2 I = −2 −4 −2 ∼ 0 1 1 
   
−5 −2 3 0 0 0
   
   

  1  
 
 
A basis for the eigenspace of λ2 is  −1 .


 1 
   

   
 9 −2 −5 1 0 −1
For λ3 = −7 we get A − λ3 I = −2 2 −2 ∼ 0 1 −2
   
−5 −2 9 0 0 0
   
  

 1
 
 
A basis for the eigenspace of λ3 is  2.


1
 

After normalizing,
√ the basis√vectors for the eigenspaces form an orthonormal basis for R3 . Hence,

 
−1/ 2 1/ 3 1/ 6
 
√ √  7 0 0 
−1/√ 3 2/ √6 orthogonally diagonalizes A to PT AP = 0 −1 0 .

P =  0
 

0 0 −7
   
1/ 2 1/ 3 1/ 6

1 − λ −2
(e) We have C(λ) = det(A − λI) = = λ2 + λ − 6 = (λ + 3)(λ − 2)
−2 −2 − λ
Hence, the eigenvalues are λ1 = −3 and λ2 = 2.
" # " # (" #)
4 −2 1 −1/2 1
For λ1 = −3 we get A − λ1 I = ∼ . Thus, a basis for Eλ1 is .
−2 1 0 0 2
" # " # (" #)
−1 −2 1 2 −2
For λ2 = 2 we get A − λ2 I = ∼ . Thus, a basis for Eλ2 is .
−2 −4 0 0 1
2
After" normalizing,
√ √the# basis vectors for the eigenspaces
" form
# an orthonormal basis for R . Hence,
1/ √5 −2/√ 5 −3 0
P= is orthogonal and PT AP = .
2/ 5 1/ 5 0 2
(f) We have

1 − λ 0 2
C(λ) = 0 1−λ 4 = (1 − λ)(λ2 − 3λ − 14) + 2(−1)(1 − λ)(2) = −(λ − 1)(λ2 − 3λ − 18) = −(λ − 1
2 4 2 − λ

The eigenvalues are λ1 = 2, λ2 = 6, and λ3 = −3.


   
0 0 2 1 2 0
For λ1 = 2 we get A − 2I = A = 0 0 4 ∼ 0 0 1
   
2 4 1 0 0 0
   
  

 −2
 
 
A basis for the eigenspace of λ1 is   1  .


   
 0 
Section 10.2 Solutions 201
   
−5 0 2  1 0 −2/5
For λ2 = 6 we get A − 6I =  0 −5 4  ∼ 0 1 −4/5
   
2 4 −4 0 0 0
   
  
2

 

A basis for the eigenspace of λ2 is  4 .


   

5 
   
4 0 2 1 0 1/2
For λ3 = −3 we get A + 3I = 0 4 4 ∼ 0 1 1 
   
2 4 5 0 0 0
   
  

  −1  
 
 
A basis for the eigenspace of λ3 is  −2 .


  2 
 

After normalizing,
√ √the basis vectors for the eigenspaces form an orthonormal basis for R3 . Hence,
 
−2/ 5 2/ 45 −1/3
 
1 0 0 
 √ √
P =  1/ 5 4/ 45 −2/3 orthogonally diagonalizes A to PT AP = 0 6 0 .
  

0 0 −3
   
0 5/ 45 2/3
(g) We have

1 − λ 1 1 1 − λ 1 1 1 − λ 2 1
C(λ) = 1 1−λ 1 = 1 1 − λ 1 = 1 2 − λ 1
1 1 1 − λ 0
λ −λ 0
0 −λ
= −λ(λ2 − 3λ) = −λ2 (λ − 3)

The eigenvalues are λ1 = 0 and λ2 = 3.


   
1 1 1 1 1 1
For λ1 = 0 we get A − 0I = A = 1 1 1 ∼ 0 0 0
   
1 1 1 0 0 0
   
    

 −1 −1
   
 
A basis for the eigenspace of λ1 is   1  ,  0  = {~ ~ 2 }.
w1 , w

    
 
 0 1 
However, we need an orthogonal basis for each eigenspace, so we apply the Gram-Schmidt pro-
cedure to this  basis.      
−1 −1 −1 −1/2
~ 2 ·~v1
Pick ~v1 =  1 . Then ~v2 = w ~ 2 − wk~ ~v =  0  − 12  1  = −1/2.
       
v1 k2 1
0 1 0 1
       
    
−1 −1

 

 1  , −1
Thus,   is an orthogonal basis for the eigenspace of λ1 .


     

 0   
2 

   
−2 1 1  1 0 −1
For λ2 = 3 we get A − 3I =  1 −2 1  ∼ 0 1 −1
   
1 1 −2 0 0 0
   
  

 1
 
 
A basis for the eigenspace of λ2 is  1.


1
 

202 Section 10.2 Solutions

After normalizing,
√ the
√ basis√vectors for the eigenspaces form an orthonormal basis for R3 . Hence,
 
−1/ 2 −1/ 6 1/ 3
 
0 0 0
 √ √ √ 
P =  1/ 2 −1/ 6 1/ 3 orthogonally diagonalizes A to PT AP = 0 0 0.
 
√ √ 
0 0 3
  
0 2/ 6 1/ 3
(h) We have

2 − λ 2 4 2 − λ 2 2 + λ 2 − λ 2 2 + λ
C(λ) = 2 4−λ 2 = 2 4−λ 0 = 2 4−λ 0 = −(λ + 2)(λ2 − 10λ + 16) =
4 2 2 − λ 4
2 −2 − λ 6 − λ
4 0

The eigenvalues are λ1 = −2, λ2 = 8, and λ3 = 2.


   
4 2 4 1 0 1
For λ1 = −2 we get A + 2I = A = 2 6 2 ∼ 0 1 0
   
4 2 4 0 0 0
   
 
−1
 

   
 
 
A basis for the eigenspace of λ1 is   0  .



 1    
   
−6 2 4  1 0 −1
For λ2 = 8 we get A − 8I =  2 −4 2  ∼ 0 1 −1
   
4 2 −6 0 0 0
   
  

 1
 
 
A basis for the eigenspace of λ2 is  1 .


1
 

   
0 2 4 1 0 −1
For λ3 = 2 we get A − 2I = 2 2 2 ∼ 0 1 2 
   
4 2 0 0 0 0
   
  

  1 
 
 
A basis for the eigenspace of λ3 is  −2 .

 1 

   

After normalizing,
√ √the basis√vectors for the eigenspaces form an orthonormal basis for R3 . Hence,
 
−1/ 2 1/ 3 1/ 6 
 
√ √ −2 0 0
1/ √3 −2/√ 6 orthogonally diagonalizes A to PT AP =  0 8 0.
 
P =  0
 
 √
0 0 2
  
1/ 2 1/ 3 1/ 6
(i) We have

2 − λ −4 −4 2 − λ −4 −4 2 − λ −8 −4
C(λ) = −4 2 − λ −4 = −4 2−λ −4 = −4 −2 − λ −4
−4 −4 2 − λ 0 −6 + λ 6 − λ 0 0 6 − λ
= −(λ − 6)(λ2 − 36) = −(λ − 6)2 (λ + 6)

The eigenvalues are λ1 = 6 and λ2 = −6.


Section 10.2 Solutions 203
   
−4 −4 −4 1 1 1
For λ1 = 6 we get A − 6I = −4 −4 −4 ∼ 0 0 0
   
−4 −4 −4 0 0 0
   
    
−1 −1

 

A basis for the eigenspace of λ1 is   1  ,  0  = {~ ~ 2 }.
w1 , w

 
  0   1 
     

However, we need an orthogonal basis for each eigenspace, so we apply the Gram-Schmidt pro-
cedure to this basis.
       
−1 −1 −1 −1/2
~ 2 ·~v1
Pick ~v1 =  1 . Then ~v2 = w ~ 2 − wk~ ~v =  0  − 12  1  = −1/2.
       
v1 k2 1
0 1 0 1
       
    
−1 −1

 

 1  , −1
Thus,   is an orthogonal basis for the eigenspace of λ1 .


     

 0   
2 

   
 8 −4 −4 1 0 −1
For λ2 = −6 we get A + 6I = −4 8 −4 ∼ 0 1 −1
   
−4 −4 8 0 0 0
   
  

 1
 
 
A basis for the eigenspace of λ2 is  1 .


1
 

After normalizing,
√ the
√ basis√vectors for the eigenspaces form an orthonormal basis for R3 . Hence,
 
−1/ 2 −1/ 6 1/ 3
 
√ √ √ 6 0 0 
P =  1/ 2 −1/ 6 1/ 3 orthogonally diagonalizes A to PT AP = 0 6 0 .
   
√ √ 
0 0 −6
  
0 2/ 6 1/ 3
(j) We have

5 − λ −1 1 5 − λ −1 1 5 − λ −2 1
C(λ) = −1 5 − λ 1 = −1 5 − λ 1 = −1 4 − λ 1
1 1 5 − λ 0
6 − λ 6 − λ 0
0 6 − λ
= −(λ − 6)(λ2 − 9λ + 18) = −(λ − 6)2 (λ − 3)

The eigenvalues are λ1 = 6 and λ2 = 3.


   
−1 −1 1  1 1 −1
For λ1 = 6 we get A − 6I = −1 −1 1  ∼ 0 0 0 
   
1 1 −1 0 0 0
   
    

 −1 1
   
 
A basis for the eigenspace of λ1 is   1  , 0 = {~ w1 , w~ 2 }.


     
 0 1
However, we need an orthogonal basis for each eigenspace, so we apply the Gram-Schmidt pro-
cedure to this basis.
       
−1 1 −1 1/2
~ 2 ·~v1   1    
~ 2 − wk~
Pick ~v1 =  1 . Then ~v2 = w ~
v = 0
  2  1  = 1/2.
+
  
v1 k2 1  
0 1 0 1
 
204 Section 10.2 Solutions

    

 −1 1
   
 
Thus,   1  , 1 is an orthogonal basis for the eigenspace of λ1 .

 0  2

 

   
 2 −1 1 1 0 1
For λ2 = 3 we get A − 3I = −1 2 1 ∼ 0 1 1
   
1 1 2 0 0 0
   
  

 −1
 
 
A basis for the eigenspace of λ2 is  −1 .


 1 
   

After normalizing,
√ √the basis√vectors for the eigenspaces form an orthonormal basis for R3 . Hence,
 
−1/ 2 1/ 6 −1/ 3
 
√ √ √ 6 0 0
P =  1/ 2 1/ 6 −1/ 3 orthogonally diagonalizes A to PT AP = 0 6 0.
   
√ √ 
0 0 3
  
0 2/ 6 1/ 3
(k) We have

−λ 1 −1 −λ 1 −1
C(λ) = 1 −λ 1 = 1 −λ 1
−1 1 −λ 0 1 − λ 1 − λ

−λ 2 −1
= 1 −1 − λ 1 = (1 − λ)(λ2 + λ − 2) = −(λ − 1)2 (λ + 2)
0 0 1 − λ

Thus, the eigenvalues are λ1 = 1 and λ2 = −2.


We have    
−1 1 −1 1 −1 1
A − I =  1 −1 1  ∼ 0 0 0
   
−1 1 −1 0 0 0
   
   

 1 −1
   
 
Thus, a basis for Eλ1 is  1 ,  0  . Since, this isn’t an orthogonal basis, we need to apply the

0  1 

 

 
1
Gram-Schimdt procedure. We take w ~ 1 = 1 and get
 
0
 

     
−1 1 −1/2
 0  − −1 1 =  1/2 
  2    
1 0 1
 √   √ 
1/ 2 −1/ 6

Remembering that we need an orthonormal basis, we take ~v1 =  0√  and ~v2 =  1/ 6 .
   
 √ 
1/ 2
 
2/ 6
Section 10.2 Solutions 205

Next, we have    
 2 1 −1 1 0 −1
A + 2I =  1 2 1  ∼ 0 1 1 
   
−1 1 2 0 0 0
   
 √ 
 1/ 3 
√ 
Hence, ~v3 = −1/ 3.

 √ 
1/ 3
h i
Thus, {~v1 , ~v2 , ~v3 } is an orthonormal basis for R3 and hence, taking P = ~v1 ~v2 ~v3 gives
PT AP = diag(1, 1, −2)
(l) We have

1 − λ 1 1 1 1 − λ 0

0 1 1 − λ 0

0 1

1 1−λ 1 1 1 −λ 0 1 1 −λ 0 1
C(λ) = = =
1−λ

1 1 1 1 0 −λ 1 1 0 −λ 1
1 − λ 1 λ λ 1 − λ 1 λ λ 1 − λ
1
1 1

1 − λ 0 0 1

1 −λ 0 1
= = (−λ)2 (λ2 − 4λ) = −λ3 (λ − 4)
1 0 −λ 1
0 3 − λ
3
0
The eigenvalues are λ1 = 0 and λ2 = 4.
   
1 1 1 1 1 1 1 1
  
1 1 1 1 0 0 0 0
For λ1 = 0 we get A − 0I =  ∼
1 1 1 1 0 0 0 0

  
1 1 1 1 0 0 0 0
      

 −1 −1 −1 

       
 1   0   0 

 
A basis for the eigenspace of λ1 is  , , = {~ ~ 2, w
w1 , w ~ 3 }.

  0   1   0 
      


       

 0

0 1 

However, we need an orthogonal basis for each eigenspace, so we apply the Gram-Schmidt pro-
cedure to this  basis.
−1 −1 −1 −1/2
     

  − 1  1  = −1/2.


 1   0     
~ 2 ·~v1
Pick ~v1 =  . Then w ~ 2 − wk~ v1 k 2 ~
v 1 =  1  2  0   1 
 0       
0 0 0 0
−1
 
−1
So, we pick ~v2 =  . Next,
 2 
0
−1 −1 −1 −1/3
       
w~ 3 · ~v1 ~ 3 · ~v2
w  0  1  1  1 −1 −1/3
~3 −
w ~v 1 − ~
v 2 =   −   −   = 
 0  2  0  6  2  −1/3

k~v1 k2 k~v2 k2        
1 0 0 1
206 Section 10.2 Solutions

−1
 
−1
Thus, we take ~v3 =   and we get that ~v1 , ~v2 , ~v3 is an orthogonal basis for the eigenspace of λ1 .

−1
 
3
−3 1 1  1 0 0 −1
   
1
 1 −3 1   
1  0 1 0 −1
For λ2 = 4 we get A − 4I =  ∼
 1 1 −3 1  0 0 1 −1
  
  
1 1 1 −3 0 0 0 0
  


 1 

1

 

A basis for the eigenspace of λ2 is  .
   
1
 

 

 



  

1
 

After normalizing,
√ the
√ basis vectors
√ for the eigenspaces form an orthonormal basis for R3 . Hence,
−1/ 2 −1/ 6 −1/ 12 1/2 
0 0 0 0

 √ √ √ 
 1/ 2 −1/ 6 −1/ 12 1/2 0 0 0 0
P =  √ √  orthogonally diagonalizes A to PT AP =  .
 0 2/ 6 −1/√ 12 1/2 0 0 0 0

0 0 3/ 12 1/2
 0 0 0 4
10.2.2 Observe that {~v1 , ~v2 , ~v3 } is an orthogonal
√ set,
√ but we need an orthogonal matrix. So, we normalize the
 1 
 − 3 −2/ 5 2/ 45
 √ √ 
vectors and take P = −2/3 1/ 5 4/ 45.
 √ 
2/3 0 5/ 45
We want to find A such that PT AP = diag(−3, 1, 6). Thus,
 
1 0 2
A = P diag(−3, 1, 6)PT = 0 1 4
 
2 4 2
 

10.2.3 If A is orthogonally diagonalizable, then there exists an orthogonal matrix P and diagonal matrix D
such that D = PT AP = P−1 AP. Since A and P are invertible, we get that D is invertible and hence,

D−1 = (P−1 AP)−1 = P−1 A−1 (P−1 )−1 = PT A−1 P

Hence, A−1 is orthogonally diagonalizable


10.2.4 (a) We have (AT A)T = AT (AT )T = AT A, so AT A is symmetric.
(b) We have (AAT )T = (AT )T AT = AAT , so AAT is symmetric.
(c) We have (AB)T = BT AT = BA, so it seems that AB does not have to
" be symmetric.
# "To demonstrate
#
0 1 1 1
it might not be symmetric, we give a counter example. Take A = and B = . Then, A
1 1 1 1
" #
1 1
and B are symmetric, but AB = is not symmetric.
2 2
(d) We have (AA)T = AT AT = AA, so A2 is symmetric.
Section 10.3 Solutions 207

10.2.5 Assume that there exists a symmetric matrix B such that A = B2 . Since B is symmetric, there exists an
orthogonal matrix P such that PT BP = diag(λ1 , . . . , λn ). Hence, we have B = PDPT and so

A = B2 = (PDPT )(PDPT ) = PD2 PT

Thus, PT AP = D2 = diag(λ21 , . . . , λ2n ), so the eigenvalues of A are all non-negative.



On the other hand, assume that all eigenvalues µ1 , . . . , µn of A are non-negative. Define λi = µi which
exists since µi is non-negative. Then, since A is symmetric, there exists an orthogonal matrix Q such that

QT AQ = diag(µ1 , . . . , µn ) = diag(λ21 , . . . , λ2n )

So,
A = Q diag(λ21 , . . . , λ2n )QT = [Q diag(λ1 , . . . , λn )QT ][Q diag(λ1 , . . . , λn )QT ]
Thus, we define B = Q diag(λ1 , . . . , λn )QT . We have that QT BQ = diag(λ1 , . . . , λn ) and so B is symmet-
ric since it is orthogonally diagonalizable.
" √ √ #
1/ √2 1/ √2
10.2.6 (a) The statement is false. The matrix P = is orthogonal, but it is not symmetric,
−1/ 2 1/ 2
so it is not orthogonally diagonalizable.
(b) The statement is false by the result of 4(c).
(c) The statement is true. If A is orthogonally similar to B, then there exists an orthogonal matrix
Q such that QT AQ = B. If B is symmetric, then there exists an orthogonal matrix P such that
PT BP = D. Hence,
D = PT (QT AQ)P = PT QT AQP = (QP)T A(QP)
where QP is orthogonal since a product of orthogonal matrices is orthogonal. Hence, A is orthog-
onally diagonalizable.
(d) If A is symmetric, then it is orthogonally diagonalizable by the Principal Axis Theorem. Thus, A
is diagonalizable and hence gλ = aλ by Corollary 6.3.4.

10.3 Problem Solutions


" #
1 3/2
10.3.1 (a) i. A =
3/2 1
ii. We have
1 − λ 3/2 5 1
C(λ) = = λ2 − 2λ − 5/4 = (λ − )(λ + )
3/2 1 − λ 2 2
5
Hence, the eigenvalues are λ1 = 2 and λ2 = − 12 , so the quadratic form and symmetric matrix
are indefinite.
208 Section 10.3 Solutions

iii. The corresponding diagonal form is Q = 52 y21 − 12 y22


" # " # (" #)
−3/2 3/2 1 −1 1
For λ1 = −5/2 we get A − λ1 I = ∼ . Thus, a basis for Eλ1 is .
3/2 −3/2 0 0 1
" # " # (" #)
3/2 3/2 1 1 −1
For λ2 = −1/2 we get A − λ2 I = ∼ . Thus, a basis for Eλ2 is .
3/2 3/2 0 0 1
" #
1 −1
Hence, P = √12 diagonalizes A. So, computing ~x = P~y we get that the required
1 1
change of variables is x1 = √y12 − √y22 , x2 = √y12 + √y22 .
" #
8 2
(b) i. A =
2 11
ii. We have
8 − λ 2
C(λ) = = (λ − 12)(λ − 7)
2 11 − λ
Hence, the eigenvalues are λ1 = 12 and λ2 = 7, so the quadratic form and symmetric matrix
are positive definite.
iii. The corresponding diagonal form is Q = 12y21 + 7y22
" # " # (" #)
−4 2 1 −1/2 1
For λ1 = 12 we get A − λ1 I = ∼ . Thus, a basis for Eλ1 is .
2 −1 0 0 2
" # " # (" #)
1 2 1 2 −2
For λ2 = 7 we get A − λ2 I = ∼ . Thus, a basis for Eλ2 is .
2 4 0 0 1
" #
1 1 −2
Hence, P = √5 diagonalizes A. So, computing ~x = P~y we get that the required
2 1
change of variables is x1 = √15 y1 − √25 y2 , x2 = √25 y1 + √15 y2 .
" #
1 4
(c) i. A =
4 −5
ii. We have
1 − λ 4
C(λ) = = (λ − 3)(λ + 7)
4 −5 − λ
Hence, the eigenvalues are λ1 = 3 and λ2 = −7, so the quadratic form and symmetric matrix
are indefinite.
iii. The corresponding diagonal form is Q = 3y21 − 7y22
" # " # (" #)
−2 4 1 −2 2
For λ1 = 3 we get A − λ1 I = ∼ . Thus, a basis for Eλ1 is .
4 −8 0 0 1
" # " # (" #)
8 4 1 1/2 −1
For λ2 = −7 we get A − λ2 I = ∼ . Thus, a basis for Eλ2 is .
4 2 0 0 2
" #
1 2 −1
Hence, P = √5 diagonalizes A. So, computing ~x = P~y we get that the required
1 2
change of variables is x1 = √25 y1 − √15 y2 , x2 = √15 y1 + √25 y2 .
Section 10.3 Solutions 209
" #
−1 2
(d) i. A =
2 2
ii. We have
−1 − λ 2
C(λ) = = (λ − 3)(λ + 2)
2 2 − λ
Hence, the eigenvalues are λ1 = 3 and λ2 = −2, so the quadratic form and symmetric matrix
are indefinite.
iii. The corresponding diagonal form is Q = 3y21 − 2y22
" # " # (" #)
−4 2 1 −1/2 1
For λ1 = 3 we get A − λ1 I = ∼ . Thus, a basis for Eλ1 is .
2 −1 0 0 2
" # " # (" #)
1 2 1 2 −2
For λ2 = −2 we get A − λ2 I = ∼ . Thus, a basis for Eλ2 is .
2 4 0 0 1
" #
1 −2
Hence, P = √15 diagonalizes A. So, computing ~x = P~y we get that the required
2 1
change of variables is x1 = √15 y1 − √25 y2 , x2 = √25 y1 + √15 y2 .
 
4 2 2
(e) i. A = 2 4 2
 
2 2 4
 
ii. We have
C(λ) = −(λ − 2)2 (λ − 8)
Hence, the eigenvalues are λ1 = 2 and λ2 = 8, so the quadratic form and symmetric matrix
are positive definite.
iii. The corresponding diagonal form is Q = 2y21 + 2y22 + 8y23
 √ √ √ 
−1/√ 2 −1/ √6 1/ √3
We find that a matrix which orthogonally diagonalizes A is P =  1/ 2 −1/ 6 1/ 3
 √ √ 
0 2/ 6 1/ 3
diagonalizes A. So, computing ~x = P~y we get that the required change of variables is
1 1 1
x1 = − √ y1 − √ y2 + √ y3
2 6 3
1 1 1
x2 = √ y1 − √ y2 + √ y3
2 6 3
2 1
x3 = √ y2 + √ y3
6 3
 
 0 2 −2
(f) i. A =  2 1 0 
 
−2 0 −1
 
ii. We have
C(λ) = −λ(λ − 3)(λ + 3)
Hence, the eigenvalues are λ1 = 0, λ2 = 3, and λ3 = −3 so the quadratic form and symmetric
matrix are indefinite.
210 Section 10.3 Solutions

iii. The corresponding diagonal form is Q = 3y22 − 3y23


 
−1/3 −2/3 2/3 
We find that a matrix which orthogonally diagonalizes A is P =  2/3 −2/3 −1/3 diag-
 
2/3 1/3 2/3
 
onalizes A. So, computing ~x = P~y we get that the required change of variables is

1 2 2
x1 = − y1 − y2 + y3
3 3 3
2 2 1
x2 = y1 − y2 − y3
3 3 3
2 1 2
x3 = y1 + y2 + y3
3 3 3
 
 4 −1 1
(g) i. A = −1 4 1
 
1 1 4
 

ii. We have
C(λ) = −(λ − 5)2 (λ − 2)
Hence, the eigenvalues are λ1 = 5 and λ2 = 2 so the quadratic form and symmetric matrix
are positive definite.
iii. The corresponding diagonal form is Q = 5y21 + 5y22 + 2y23
 √ √ √ 
−1/ 2 1/ 6 −1/ 3
 √ √ √ 
We find that a matrix which orthogonally diagonalizes A is P =  1/ 2 1/ 6 −1/ 3
 √ √ 
0 2/ 6 1/ 3
diagonalizes A. So, computing ~x = P~y we get that the required change of variables is

1 1 1
x1 = − √ y1 + √ y2 − √ y3
2 6 3
1 1 1
x2 = √ y1 + √ y2 − √ y3
2 6 3
2 1
x3 = √ y2 + √ y3
6 3
 
−4 1 1 
(h) i. A =  1 −4 −1
 
1 −1 −4
 

ii. We have
C(λ) = −(λ + 3)2 (λ + 6)
Hence, the eigenvalues are λ1 = −3 and λ2 = −6 so the quadratic form and symmetric matrix
are negative definite.
iii. The corresponding diagonal form is Q = −3y21 − 3y22 − 6y23
Section 10.3 Solutions 211

 √ √ √ 
1/ 2 1/ 6 −1/ 3
 √ √ √ 
We find that a matrix which orthogonally diagonalizes A is P = 1/ 2 −1/ 6 1/ 3 
 √ √ 
0 2/ 6 1/ 3
diagonalizes A. So, computing ~x = P~y we get that the required change of variables is
1 1 1
x1 = √ y1 + √ y2 − √ y3
2 6 3
1 1 1
x2 = √ y1 − √ y2 + √ y3
2 6 3
2 1
x3 = √ y2 + √ y3
6 3
10.3.2 (a) Let λ1 , λ2 be the eigenvalues of A. If det A > 0 then ac − b2 > 0 so a and c must both have the
same sign. Thus, c > 0. We know that det A = λ1 λ2 and λ1 + λ2 = a + c and so λ1 and λ2 must
have the same sign since det A > 0 and we have λ1 + λ2 = a + c > 0 so we must have λ1 and λ2
both positive so Q is positive definite.
(b) Let λ1 , λ2 be the eigenvalues of A. If det A > 0 then ac − b2 > 0 so a and c must both have the
same sign. Thus, c < 0. We know that det A = λ1 λ2 and λ1 + λ2 = a + c and so λ1 and λ2 must
have the same sign since det A > 0 and we have λ1 + λ2 = a + c < 0 so we must have λ1 and λ2
both negative so Q is negative definite.
(c) Let λ1 , λ2 be the eigenvalues of A. If det A < 0 then det A = λ1 λ2 < 0 so λ1 and λ2 must have
different signs thus Q is indefinite.
10.3.3 If h , i is an inner product on Rn , then it is symmetric and positive definite. Observe that ~yT A~x is a 1 × 1
matrix, and hence it is symmetric. Thus,

~xT A~y = h~x, ~yi = h~y, ~xi = ~yT A~x = (~yT A~x)T = ~xT AT ~y

Hence, ~xT A~y = ~xT AT ~y for all ~x, ~y ∈ Rn . Therefore, A = AT by Theorem 3.1.4. Observe that

~xT A~x = h~x, ~xi > 0

whenever ~x , ~0, so A is positive definite.


~ ∈ Rn and s, t ∈ R. Then,
Assume that A is a positive definite, symmetric matrix. Let ~x, ~y, w

h~x, ~xi = ~xT A~x ≥ 0

for all ~x , ~0 and h~x, ~xi = 0 if and only if ~x = ~0. Thus, h , i is positive definite.
We have
h~x, ~yi = ~xT A~y = (~xT A~y)T = ~yT AT ~x = ~yT A~x = h~y, ~xi
So, h , i is symmetric.
We have

~ i = (s~x + t~y)T A~
hs~x + t~y, w w = (s~xT + t~yT )A~
w = s~xT A~
w + t~yT A~
w
~ i + th~y, w
= sh~x, w ~i

So, h , i is also bilinear. Therefore, it is an inner product.


212 Section 10.3 Solutions

10.3.4 (a) If A is a positive definite symmetric matrix, then ~xT A~x > 0 for all ~x , ~0. Hence, for 1 ≤ i ≤ n,
we have
aii = ~eTi A~ei > 0
as required.
(b) If A is a positive definite symmetric matrix, then all the eigenvalues of A are positive. Thus, since
the determinant of A is the product of the eigenvalues, we have that det A > 0 and hence A is
invertible.
10.3.5 We first must observe that A + B is symmetric. Since the eigenvalues of A and B are all positive, the
quadratic forms ~xT A~x and ~xT B~x are positive definite. Let ~x , ~0. Then ~xT A~x > 0 and ~xT B~x > 0 , so
~xT (A + B)~x = ~xT A~x + ~xT B~x > 0 , and the quadratic form ~xT (A + B)~x is positive definite. Thus the
eigenvalues of A + B must be positive.
Section 10.4 Solutions 213

10.4 Problem Solutions


" #
1 4
10.4.1 (a) The corresponding symmetric matrix is . We find that the characteristic polynomial is
4 1
C(λ) = (λ − 5)(λ + 3). So, we have eigenvalues λ1 = 5 and λ2 = −3.
" # " # " #
−4 4 1 −1 1
For λ1 = 5 we get A − λ1 I = ∼ . So, a corresponding eigenvector is ~v1 = .
4 −4 0 0 1
" # " # " #
4 4 1 1 −1
For λ2 we get A − λ2 I = ∼ . So, a corresponding eigenvector is ~v2 = .
4 4 0 0 1

Thus, we have the hyperbola 5y21 − 3y22 = 6 with principal


axis ~v1 for x1 and ~v2 for y1 . The asymptotes
q of the hyper-
5
bola are when 0 = 5y21 − 3y22 ⇒ y2 = ± 3 y1 . Plotting
this we get the graph in the y1 y2 -plane is

" #
1
To sketch x12 + 8x1 x2 + x22 = 6 in the x1 x2 -plane we first draw the y1 -axis in the direction of ~v1 =
1
" #
−1
and the y2 -axis in the direction of ~v2 = . Next, to convert the asymptotes into the x1 x2 -plane,
1
" # " √ √ #" # " #
y1 T 1/ √2 1/ √2 x1 1 x1 + x2
we use the change of variables = P ~x = = √ . So, the
y2 −1/ 2 1/ 2 x2 2 −x1 + x2
asymptotes in the x1 x2 -plane are
r
5
y2 = ± y1
3
r !
1 5 1
√ (−x1 + x2 ) = ± √ 1(x + x2 )
2 3 2
√ √
3(−x1 + x2 ) = ± 5(x1 + x2 )
√ √ √ √
( 3 ∓ 5)x2 = ( 3 ± 5)x1
√ √
3± 5
x2 = √ √ x1
3∓ 5

Plotting gives the graph to the right.


214 Section 10.4 Solutions
" #
3 −1
(b) The corresponding symmetric matrix is . We find that the characteristic polynomial is
−1 3
C(λ) = (λ − 4)(λ − 2). So, we have eigenvalues λ1 = 4 and λ2 = 2.
" # " # " #
−1 −1 1 1 −1
For λ1 we get A − λ1 I = ∼ . Thus, a corresponding eigenvector is ~v1 = .
−1 −1 0 0 1
" # " # " #
1 −1 1 −1 1
For λ2 we get A − λ2 I = ∼ . So, a corresponding eigenvector is ~v2 = .
−1 1 0 0 1
Thus, we have the ellipse 4y21 + y22 = 12 with principal axis ~v1 for x1 and ~v2 for y1 .
Plotting this we get the graph in the y1 y2 -plane is

To sketch 3x12 − 2x1 x2 + 3x22 = 12 in the x1 x2 -plane we first draw the y1 -axis in the direction of
" # " #
−1 1
~v1 = and the y2 -axis in the direction of ~v2 = . Plotting gives
1 1
Section 10.4 Solutions 215
" #
−4 2
(c) The corresponding symmetric matrix is . We find that the characteristic polynomial is
2 −7
C(λ) = (λ + 8)(λ + 3). So, we have eigenvalues λ1 = −8 and λ2 = −3.
" # " # " #
4 2 1 1/2 −1
For λ1 we get A − λ1 I = ∼ . So, a corresponding eigenvector is ~v1 = .
2 1 0 0 2
" # " # " #
−1 2 1 −2 2
For λ2 we get A − λ2 I = ∼ . So, a corresponding eigenvector is ~v2 = .
2 −4 0 0 1
Thus, we have the ellipse −8y21 − 3y22 = −8 with principal axis ~v1 for x1 and ~v2 for y1 .
Plotting this we get the graph in the y1 y2 -plane is

To sketch −4x12 + 4x1 x2 − 7x22 = −8 in the x1 x2 -plane we first draw the y1 -axis in the direction of
" # " #
−1 2
~v1 = and the y2 -axis in the direction of ~v2 = . Plotting gives
2 1
216 Section 10.4 Solutions
" #
−3 −2
(d) The corresponding symmetric matrix is . We find that the characteristic polynomial is
−2 0
C(λ) = (λ + 4)(λ − 1). So, we have eigenvalues λ1 = 1 and λ2 = −4.
" # " # " #
−4 −2 1 1/2 −1
For λ1 we get A − λ1 I = ∼ . So, a corresponding eigenvector is ~v1 = .
−2 −1 0 0 2
" # " # " #
1 −2 1 −2 2
For λ2 we get A − λ2 I = ∼ . So, a corresponding eigenvector is ~v2 = .
−2 4 0 0 1

Thus, we have the hyperbola y21 − 4y22 = 4 with principal


axis ~v1 for x1 and ~v2 for y1 . The asymptotes of the hyper-
bola are when 0 = y21 − 4y22 ⇒ y2 = ± 12 y1 . Plotting this
we get the graph in the y1 y2 -plane is

" #
−1
To sketch −3x12 −4x1 x2 = 4 in the x1 x2 -plane we first draw the y1 -axis in the direction of ~v1 =
2
" #
2
and the y2 -axis in the direction of ~v2 = . Next, to convert the asymptotes into the x1 x2 -plane,
1
" # " √ √ #" # " #
y1 T −1/√ 5 2/ √5 x1 1 −x1 + 2x2
we use the change of variables = P ~x = = √ . So, the
y2 2/ 5 1/ 5 x2 5 2x1 + x2
asymptotes in the x1 x2 -plane are

1
y2 = ± y1
2 !
1 1 1
√ (2x1 + x2 ) = ± √ (−x1 + 2x2 )
5 2 5
2(2x1 + x2 ) = ±(−x1 + 2x2 )
(2 ∓ 2)x2 = (−4 ∓ 1)x1

Thus, x1 = 0 or x2 = − 34 x1 . Plotting gives the graph


to the right.
Section 10.4 Solutions 217
" #
−1 2
(e) The corresponding symmetric matrix is . We find that the characteristic polynomial is
2 2
C(λ) = (λ − 3)(λ + 2). So, we have eigenvalues λ1 = 3 and λ2 = −2.
" # " # " #
−4 2 1 −1/2 1
For λ1 we get A − λ1 I = ∼ . So, a corresponding eigenvector is ~v1 = .
2 −1 0 0 2
" # " # " #
1 2 1 2 −2
For λ2 we get A − λ2 I = ∼ . So, a corresponding eigenvector is ~v2 = .
2 4 0 0 1

Thus, we have the hyperbola 3y21 − 2y22 = 6 with principal


axis ~v1 for x1 and ~v2 for y1 . The asymptotes
q of the hyper-
3
bola are when 0 = 3y21 − 2y22 ⇒ y2 = ± 2 y1 . Plotting
this we get the graph in the y1 y2 -plane is

To sketch −x12 + 4x1 x2 + 2x22 = 6 in the x1 x2 -plane we first draw the y1 -axis in the direction of
" # " #
1 −2
~v1 = and the y2 -axis in the direction of ~v2 = . Next, to convert the asymptotes into the x1 x2 -
2 1
" # " √ √ #" # " #
y1 T 1/ √5 2/ √5 x1 1 x1 + 2x2
plane, we use the change of variables = P ~x = = √ . So,
y2 −2/ 5 1/ 5 x2 5 −2x1 + x2
the asymptotes in the x1 x2 -plane are
r
3
y2 = ± y1
2
r !
1 3 1
√ (−2x1 + x2 ) = ± √ (x1 + 2x2 )
5 2 5
√ √
2(−2x1 + x2 ) = ± 3(x1 + 2x2 )
√ √ √ √
( 2 ∓ 2 3)x2 = (2 2 ± 3)x1
√ √
2 2± 3
x2 = √ √ x1
2∓2 3

Plotting gives the graph to the right.


218 Section 10.4 Solutions
" #
4 2
(f) The corresponding symmetric matrix is . We find that the characteristic polynomial is
2 4
C(λ) = (λ − 2)(λ − 6). So, we have eigenvalues λ1 = 2 and λ2 = 6.
" # " # " #
2 2 1 1 −1
For λ1 we get A − λ1 I = ∼ . So, a corresponding eigenvector is ~v1 = .
2 2 0 0 1
" # " # " #
−2 2 1 −1 1
For λ2 we get A − λ2 I = ∼ . So, a corresponding eigenvector is ~v2 = .
2 −2 0 0 1
Thus, we have the ellipse 2y21 + 6y22 = 12 with principal axis ~v1 for x1 and ~v2 for y1 .
Plotting this we get the graph in the y1 y2 -plane is

To sketch 4x12 + 4x1 x2 + 4x22 = 12 in the x1 x2 -plane we first draw the y1 -axis in the direction of
" # " #
−1 1
~v1 = and the y2 -axis in the direction of ~v2 = . Plotting gives
1 1

"#
2 −2
(g) The corresponding symmetric matrix is A = . The eigenvalues are λ1 = 3 and λ2 =
−2 −1
" # " #
−2 1
−2 with corresponding eigenvectors ~v1 = , ~v2 = . Thus, A is diagonalized by P =
1 2
Section 10.4 Solutions 219
" √ √ # " #
−2/√ 5 1/ √5 3 0
to D = . Hence, we have the hyperbola 3y21 − 2y22 = 6 with princi-
1/ 5 2/ 5 0 2
pal axis ~v1 , and ~v2 . Since this is a hyperbola we need to graph the asymptotes. They are when
0 = 3y21 − 2y22 .
q
Hence, the equations of the asymptotes are y2 = ± 32 y1 . Graphing give the diagram below to the
left. We have " # " #" # " #
y1 T 1 −2 1 x1 1 −2x1 + x2
= P ~x = √ = √
y2 5 1 2 x2 5 x1 + 2x2
So, the asymptotes in the x1 x2 -plane are given by
r !
1 3 1
√ (x1 + 2x2 ) = ± √ (−2x1 + x2 )
5 2 5
Solving for x2 gives √
−1 ± 2 3/2
x2 = √ x1
2 ± 3/2
which is x2 ≈ 0.449x1 and x2 ≈ −4.449x1 .
Alternately, we can find that the direction vectors of the asymptotes are
" # " #" # " √ √ #
x1 1 −2 1 1 (−2 + 3/2)/
= P~x = √ √ = √ √5
x2 5 1 2 3/2 (1 + 2 3/2)/ 5

and " # " #" # " √ √ #


1 −2 (−2 − 3/2)/ √ 5
x1
= P~x = √
1 √1 = √
x2 5 1 2 − 3/2 (1 − 2 3/2)/ 5
These are the direction vectors of the lines x2 ≈ −4.449x1 and x2 ≈ 0.449x1 .
Also, the principal axes are ~v1 and ~v2 , so graphing this gives the diagram below to the right.
220 Section 10.4 Solutions
" #
3 2
(h) The corresponding symmetric matrix is A = . The eigenvalues are λ1 = 1 and λ2 = 5
2 3
" # " #
−1 1
with corresponding eigenvectors ~v1 = , ~v2 = . Thus, A is orthogonally diagonalized by
1 1
" √ √ # " #
−1/√ 2 1/ √2 1 0
P= to D = . Thus, we have the ellipse
1/ 2 1/ 2 0 5

y21 + 5y22 = 5

Graphing this in the y1 y2 -plane gives the diagram below to the left.
The principal axes are ~v1 and ~v2 . So, graphing it in the x1 x2 -plane gives the diagram below to the
right.
Section 10.5 Solutions 221

10.5 Problem Solutions


" #
4 −1
10.5.1 (a) The corresponding symmetric matrix is A = . Hence, we have
−1 4

C(λ) = (λ − 5)(λ − 3)

Hence, the eigenvalues of A are λ1 = 5 and λ2 = 3. Therefore, the maximum is 5 and the minimum
is 3.
" #
3 5
(b) The corresponding symmetric matrix is A = . Hence, we have
5 −3
√ √
C(λ) = (λ − 34)(λ + 34)
√ √ √
√ of A are λ1 =
Hence, the eigenvalues 34 and λ2 = − 34. Therefore, the maximum is 34 and
the minimum is − 34.
 
−1 0 0 
(c) The corresponding symmetric matrix is A =  0 8 2 . Hence, we have
 
0 2 11
 

C(λ) = (λ − 7)(λ + 1)(λ − 12)

Hence, the eigenvalues of A are λ1 = 7, λ2 = −1, and λ3 = 12. Therefore, the maximum is 12 and
the minimum is −1.
 
 3 −2 4
(d) The corresponding symmetric matrix is A = −2 6 2. Hence, we have
4 2 3
 

C(λ) = (λ − 7)2 (λ + 2)

Hence, the eigenvalues of A are λ1 = 7 and λ2 = −2. Therefore, the maximum is 7 and the
minimum is −2.
 
2 1 1
(e) The corresponding symmetric matrix is A = 1 2 1. Hence, we have
1 1 2
 

C(λ) = (λ − 1)2 (λ − 4)

Hence, the eigenvalues of A are λ1 = 1 and λ2 = 4. Therefore, the maximum is 4 and the minimum
is 1.
222 Section 10.6 Solutions

10.6 Problem Solutions


" #
T 5 2
10.6.1 (a) We have A A = . The eigenvalue of AT A are (ordered from greatest to least) λ1 = 9 and
2 8
" √ # " √ #
1/ √5 −2/√ 5
λ2 = 4. Corresponding normalized eigenvectors are ~v1 = for λ1 and ~v2 = for λ2 .
2/ 5 1/ 5
T
Hence,
" A√A is orthogonally
√ # diagonalized by
1/ √5 −2/√ 5
V = . The singular values of A are σ1 = 3 and σ2 = 2. Thus the matrix Σ is
2/ 5 1/ 5
" #
3 0
Σ= . Next compute
0 2
" #" √ # " √ #
1 1 2 2 1/ √5 2/ √5
~u1 = A~v1 = =
σ1 3 −1 2 2/ 5 1/ 5
" #" √ # " √ #
1 1 2 2 −2/√ 5 −1/√ 5
~u2 = A~v2 = =
σ2 2 −1 2 1/ 5 2/ 5
" √ √ #
2/ √5 −1/√ 5
Thus we have U = . Then A = UΣV T as required.
1/ 5 2/ 5
" #
T 73 36
(b) We have A A = . The eigenvalue of AT A are (ordered from greatest to least) λ1 = 100
36 52
" # " #
4/5 −3/5
and λ2 = 25. Corresponding normalized eigenvectors are ~v1 = for λ1 and ~v2 = for
3/5 4/5
λ2 . Hence,
" AT A is#orthogonally diagonalized by
4/5 −3/5
V = . The singular values of A are σ1 = 10 and σ2 = 5. Thus the matrix Σ is
3/5 4/5
" #
10 0
Σ= . Next compute
0 5
" #" # " #
1 1 3 −4 4/5 0
~u1 = A~v1 = =
σ1 10 8 6 3/5 1
" #" # " #
1 1 3 −4 −3/5 1
~u2 = A~v2 = =
σ2 5 8 6 4/5 0
" #
0 1
Thus we have U = . Then A = UΣV T as required.
1 0
" #
T 4 6
(c) We have A A = . The eigenvalue of AT A are (ordered from greatest to least) λ1 = 16 and
6 13
" √ # " √ #
1/ √5 −2/√ 5
λ2 = 1. Corresponding normalized eigenvectors are ~v1 = for λ1 and ~v2 = for λ2 .
2/ 5 1/ 5
Hence, AT A is orthogonally diagonalized by
Section 10.6 Solutions 223
" √ √ #
1/ √5 −2/√ 5
V = . The singular values of A are σ1 = 4 and σ2 = 1. Thus the matrix Σ is
2/ 5 1/ 5
" #
4 0
Σ= . Next compute
0 1
" #" √ # " √ #
1 1 2 3 1/ 5
~u1 = A~v1 = √ = 2/ √5
σ1 4 0 2 2/ 5 1/ 5
" #" √ # " √ #
1 1 2 3 −2/ 5
~u2 = A~v2 = √ = −1/√ 5
σ2 1 0 2 1/ 5 2/ 5
"√ √ #
2/ √5 −1/√ 5
Thus we have U = . Then A = UΣV T as required.
1/ 5 2/ 5
" #
T 6 6
(d) We have A A = . The eigenvalues of AT A are (ordered from greatest to least) λ1 = 15 and
6 11
λ2 = 2.
" √ # " √ #
2/ √13 −3/√ 13
Normalized eigenvectors are ~v1 = for λ1 and ~v2 = for λ2 .
3/ 13 2/ 13
" √ √ #
T 2/ √13 −3/√ 13
Hence, A A is orthogonally diagonalized by V = .
3/ 13 2/ 13
√ 
 15 0 
√ √ √ 
The singular values of A are σ1 = 15 and σ2 = 2. Thus the matrix Σ is Σ =  0

2.
0 0
 

Next compute
√ 
1 3 " √ # 11/√ 195
  
1 1  2/ 13
~u1 = √
 
A~v1 = √ 2 1 =  7/ 195 

σ1 15 1 1 3/ 13  √ 
5/ 195
 √ 
 
1 3 " √ #  3/ 26 
1 1   −3/√ 13  √ 
~u2 = A~v2 = √ 2 1 = −4/ 26
σ2 2 1 1 2/ 13 √ 
−1/ 26

We then need to extend {~u1 , ~u2 } to an orthonormal basis for R3 . Since we are in R3 , we can use the
cross product. We have
     
11  3   13 
 7  × −4 =  26 
     
5 −1 −65
 √   √ √ √ 
11/ 95 3/ 26 1/ √30 
 1/ 30 
 √   √ √

So, we can take ~u3 =  2/ 30 . Thus, we have U = 7/ 195 −4/ 26 2/ 30  . Then,
√   √ √ √ 
−5/ 30 5/ 195 −1/ 26 −5/ 30

A = UΣV T as required.
224 Section 10.6 Solutions
" #
12 −6
(e) We have AT A = . The eigenvalues of AT A are (ordered from greatest to least) λ1 = 15
−6 3
and λ2 = 0.
√ # " " √ #
−2/√ 5 1/ √5
Corresponding normalized eigenvectors are ~v1 = for λ1 and ~v2 = for λ2 .
1/ 5 2/ 5
" √ √ #
T −2/√ 5 1/ √5
Hence, A A is orthogonally diagonalized by V = .
1/ 5 2/ 5
√ 
√  15 0
The singular values of A are σ1 = 15 and σ2 = 0. Thus, the matrix Σ is Σ =  0 0.
 
 
0 0
Next compute √ 
√ # −1/ 3
  
2 −1 " √
1 1   −2/√ 5  
~u1 = A~v1 = √ 2 −1

= −1/ 3
σ1 15 2 −1 1/ 5 √ 
−1/ 3

We then need to extend {~u1 } to an orthonormal basis for R3 . One way of doing this is to find an
    

 −1 −1
T T    
 
orthonormal basis for Null(A ). A basis for Null(A ) is   1  ,  0  . To make this an orthogonal



 0 

1 
   
 
−1
basis, we apply the Gram-Schmidt procedure. We take ~v1 =  1 , and then we get
 
0
 
     
−1 −1 −1/2
 0  − 1  1  = −1/2
  2    
1 0 1
 √   √ 
−1/ 2 −1/ 6
 √  √ 
Thus, we pick ~u2 =  1/ 2  and ~u3 = −1/ 6. Consequently, we have


0
   
2 6
 √ √ √ 
−1/ 3 −1/ 2 −1/ 6
√ √ √ 
U = −1/ 3 1/ 2 −1/ 6 and A = UΣV T as required.

√ √
−1/ 3
 
0 2/ 6
 
3 0 0
(f) We have AT A = 0 3 0. The only eigenvalue of AT A is λ1 = 3 with multiplicity 3.
 
0 0 3
 

Since AT A is diagonal, we can take V = I.


√ √ √
The singular values of A are σ1 = 3, σ2 = 3, and σ3 = 3. So, we get that Σ =
√ 
 3 √0 0 
 
 0 3 √0 
.
 0 0 3
 
0 0 0
Section 10.6 Solutions 225

Next compute
 
 1 
1 1  1 
~u1 = A~v1 = √  
σ1 3 −1
0
 
0
1 1 1
~u2 = A~v2 = √  
σ2 3 1
1
 
 1 
1 1 −1
~u3 = A~v3 = √  
σ3 3  0 
1

We then need to extend {~u1 , ~u2 , ~u3 } to an orthonormal basis for R4 . We find that a basis for Null(AT )
−1
 


 
   
 0 

 
is  . After normalizing this vector, we take



 −1

 
 


   
1 

 
 √ √ √ 
−1/√ 3 0√ 1/ √3 −1/ 3
 1/ √3 1/ √3 −1/ 3 0√ 
 
T
U =   and A = UΣV as required.
−1/ 3 1/ 3 0 −1/ 3
 √ √ √  
0 1/ 3 1/ 3 1/ 3
" #
T 4 8
(g) We have A A = . The eigenvalue of AT A are (ordered from greatest to least) λ1 = 20 and
8 16
" √ # " √ #
1/ √5 −2/√ 5
λ2 = 0. Corresponding normalized eigenvectors are ~v1 = for λ1 and ~v2 = for λ2 .
2/ 5 1/ 5
T
Hence, " A√ A is orthogonally √ # diagonalized by
1/ √5 −2/√ 5 √
V= . The singular values of A are σ1 = 20 and σ2 = 1. Thus the matrix Σ is
2/ 5 1/ 5
√ 
 20 0
0 0
 
Σ =  . Next compute
 0 0

0 0
   
1 2 " √ # 1/2
  
1 1 1 2 1/ √5 1/2
~u1 = A~v1 = √  =  
σ1 20 1 2 2/ 5 1/2


1 2 1/2
226 Section 10.6 Solutions

We then need to extend {~u1 } to an orthonormal basis for R4 . We pick

−1/2
     
 1/2   1/2 
−1/2 −1/2 −1/2
~u2 =   , ~u3 =   , ~u4 =  
 1/2   1/2  −1/2
1/2 −1/2 1/2

1/2 −1/2 1/2


 
1/2 
1/2 −1/2 −1/2 −1/2
Thus, we have U =   . Then A = UΣV T as required.
1/2 1/2 1/2 −1/2

1/2 1/2 −1/2 1/2
(h) Observe that the matrix is the transpose of the matrix in part (d). Hence, we have
√ T
1/2 −1/2 1/2
 
1/2   20 0 " √
 1/ 5 2/ √5#

1/2 −1/2 −1/2 −1/2 
A =    0 0 √ √ 
1/2 1/2 1/2 −1/2  0 0 −2/ 5 1/ 5 

  
1/2 1/2 −1/2 1/2 0 0
 
" √ √ #"√ #  1/2 1/2 1/2 1/2 

1/ √5 −2/√ 5 20 0 0 0 −1/2 −1/2 1/2 1/2 
=
0 0 0 0  1/2 −1/2 1/2 −1/2
 
2/ 5 1/ 5  
1/2 −1/2 −1/2 1/2

Thus, we take

1/2 −1/2 1/2


 
"√ √ # "√ # 1/2 
1/ √5 −2/√ 5 1/2 −1/2 −1/2 −1/2
20 0 0 0
U= , Σ= , V =  
2/ 5 1/ 5 0 0 0 0 1/2 1/2 1/2 −1/2

1/2 1/2 −1/2 1/2

and get a singular value decomposition A = UΣV T .


NOTE: Make sure that you have the correct matrix V.
(i) It would be difficult to find the SVD of A directly. Therefore, as we did in part (e), we will first
find the SVD of AT . " #
T T T T 36 12
We have (A ) A = AA = . The eigenvalue of AT A are (ordered from greatest to
12 29
" #
4/5
least) λ1 = 45 and λ2 = 20. Corresponding normalized eigenvectors are ~v1 = for λ1 and
3/5
" #
−3/5
~v2 = for λ2 . Hence, AT A is orthogonally diagonalized by
4/5
√ √
" #
4/5 −3/5
V = . The singular values of A are σ1 = 45 and σ2 = 20. Thus the matrix Σ is
3/5 4/5
Section 10.6 Solutions 227
√ 
 45
√0 


Σ =  0 20. Next compute
0 0
 

   √ 
4 3 " #  5/√ 45 
1 1   4/5
~u1 =
 
A~v1 = √  4 2 = 22/ 1125
σ1 45 −2  3/5 √
4
 
4/ 45
 
0√
 
4 3 " #  
1 1   −3/5 
~u2 = = −2/ √ 125
 
A~v2 = √  4 2
σ2 20 −2  4/5
4 11/ 125
 

We then need to extend {~u1 , ~u2 } to an orthonormal basis for R3 . We pick


 
 2/3 
~u3 = ~u1 × ~u2 = −11/15
 
2/15
 

h i
Thus, we have U = ~u1 ~u2 ~u3 . Then, A = (AT )T = (UΣV T )T = VΣT U T .
" #
T 40 0
10.6.2 (a) We have A A = . Thus, the eigenvalues of AT A are λ1 = 40 and λ2 = 10. Hence, the
0 10

maximum of kA~xk subject to k~xk = 1 is 40.
" #
T 6 −3
(b) We have A A = . Thus,
−3 14

C(λ) = λ2 − 20λ + 75 = (λ − 15)(λ − 5)

Hence, the√eigenvalues of AT A are λ1 = 15 and λ2 = 5. So, the maximum of kA~xk subject to


k~xk = 1 is 15.
(c) "We have#seen that AT A and AAT have the same non-zero eigenvalues. Hence, we find that AAT =
14 5
, and see by inspection that the eigenvalues are λ1 = 9 and λ2 = 19. Hence, the maximum
5 14

of kA~xk subject to k~xk = 1 is 19.
10.6.3 By definition, the singular values of PA are the eigenvalues of (PA)T (PA) = AT PT PA = AT A. But, the
eigenvalues of AT A are the singular values of A as required.
10.6.4 If ~u is a left singular vector of A, then A~u = ~0, or AT ~u = σ~v and A~v = σ~u.
If AT ~u = ~0, then AAT ~u = A~0 = ~0 = 0~u, so ~u is an eigenvector of AAT .
If AT ~u = σ~v, then
AAT ~u = A(σ~v) = σ(A~v) = σ(σ~u) = σ2~u
Hence, ~u is an eigenvector of AAT .
Replacing A by AT in our result above gives that ~v is a right singular vector if and only if ~v is an
eigenvector of AT A.
228 Section 10.6 Solutions

h i h i
10.6.5 Let U = ~u1 · · · ~um and V = ~v1 · · · ~vn .
By Lemma 10.6.6, {~u1 , . . . , ~ur } forms an orthonormal basis for Col(A).
By the Fundamental Theorem of Linear Algebra, the nullspace of AT is the orthogonal complement of
Col(A). We are given that {~u1 , . . . , ~um } forms an orthonormal basis for Rn and hence a basis for the
orthogonal compliment of Span{~u1 , . . . , ~ur } is B = {~ur+1 , . . . , ~um }. Thus B is a basis for Null(AT ).
By Theorem 10.6.1, we have that kA~vi k = λi . But, since UΣV T is a singular value decomposition of A,
we have that λr+1 , . . . , λn are the zero eigenvalues of A. Hence, {~vr+1 , . . . , ~vn } is an orthonormal set of
n − r vectors in the nullspace of A. But, since A has rank r, we know by the Rank-Nullity Theorem, that
dim Null(A) = n − r. So {~vr+1 , . . . , ~vn } is a basis for Null(A).
Hence, by the Fundamental Theorem of Linear Algebra we have that the orthogonal complement of
Null(A) is Row(A). Hence {~v1 , . . . , ~vr } forms a basis for Row(A).
 
4 2 2
10.6.6 (a) We get AT A = 2 5 1 which has eigenvalues λ1 = 8, λ2 = 4, and λ3 = 2 and corresponding
 
2 1 5
 
 √     √ 
1/ 3  0√  −2/ 6
√ √
unit eigenvectors ~v1 = 1/ 3, ~v2 = −1/√ 2, and ~v3 =  1/ 6 . Thus, B = {~v1 , ~v2 , ~v3 }.
     
 √   √ 
1/ 2
 
1/ 3 1/ 6
(b) We have
  √   √ 
2 1 1 1/ √3 4/ √24
1 1 
~u1 =
   
A~v1 = √ 0 2 0 1/ 3 = 2/ 24
σ1 8 0 0 2 1/ 3
   √   √ 
2/ 24
   
2 1 1  0√   0√ 

1 1
~u2 =
   
A~v2 = 0 2 0 −1/√ 2 = −1/√ 2
σ2 2
0 0 2 1/ 2

1/ 2
  
 √   √ 
2 1 1 −2/√ 6 −2/√ 12

1 1 
~u3 =
   
A~v3 = √ 0 2 0  1/ 6  =  2/ 12 
σ3 √
2 0 0 2  1/ 6   2/ 12 
  √

Hence, C = {~u1 , ~u2 , ~u3 }.


(c) We have
 √   √     √ 
4/ 3 √ 4/ 24  0√  −2/ 12
 √   √   √ 
L(~v1 ) = A~v1 = 2/ 3 = 8 2/ 24 + 0 −1/√ 2 + 0  2/ 12 
 
√ √  √ 
1/ 2
     
2/ 3 2/ 24 2/ 12
   √     √ 
 0√  4/ 24  0√  −2/ 12
 √   √ 
L(~v2 ) = A~v2 = −2/√ 2 = 0 2/ 24 + 2 −1/√ 2 + 0  2/ 12 
   
√  √ 
2/ 2 1/ 2
     
2/ 24 2/ 12
 √   √     √ 
−2/ 6 4/ 24  0√  √ −2/ 12
√ √ √
L(~v3 ) = A~v3 =  2/ 6  = 0 2/ 24 + 0 −1/√ 2 + 2  2/ 12 
       
 √   √   √ 
1/ 2
 
2/ 6 2/ 24 2/ 12
Section 10.6 Solutions 229

Hence, √ 
 8 0 0 
 
C [L]B =   0 2 √0 
0 0 2

NOTE: Observe that A is not even diagonalizable.


10.6.7 Using block multiplication, we get

A = UΣV T
σ1
 
0 ··· 0
 .. 
 0 . 
  T 
.. ..  ~v1 

h i  σr . .   . 
= ~u1 · · · ~um  .
 . ..   .. 
 . . 0   T 
 ~vn
 ..
.


 0
···

0 0 0
 T
~v 
h i  .1 
= σ1~u1 · · · σr~ur ~0 · · · ~0  .. 
 
~vTn
= σ1~u1~vT1 + · · · + σr~ur~vTr
Chapter 11 Solutions

11.1 Problem Solutions


11.1.1 (a) (3 + 4i) − (−2 + 6i) = 5 − 2i
(b) (2 − 3i) + (1 + 3i) = 3
(c) (−1 + 2i)(3 + 2i) = −7 + 4i
(d) −3i(−2 + 3i) = 9 + 6i
(e) 3 − 7i = 3 + 7i
(f) (1 + i)(1 − i) = 2 = 2
√ √ √
(g) |(1 + i)(1 − 2i)(3 + 4i)| = |1 + i| |1 − 2i| |3 + 4i| = 2( 5)(5) = 5 10

(h) |1 + 6i| = 37
√ √
(i) | 32 − 2i| = 13 22
2 2 1+i 2(1+i)
(j) 1−i = 1−i 1+i = 2 =1+i
4−3i 4−3i 3+4i 24 7i
(k) 3−4i = 3−4i 3+4i = 25 + 25
2+5i 2+5i −3+6i 4 1
(l) −3−6i = −3−6i −3+6i = − 5 − 15 i
11.1.2 (a) We have

2iz = 4
z = −2i

(b) We have

1 1 + 5i
1−z= −z = −1 +
1 − 5i 26
25 5
z= − i
26 26

230
Section 11.1 Solutions 231

(c) We have
(1 + i)z + (2 + i)z = 2
(3 + 2i)z = 2
2
z=
3 + 2i
6 4
z= − i
13 13
11.1.3 (a) Row reducing gives
   
 1 2+i i 0 1+i   1 2 + i i 0 1 + i  R1 − iR2
 i −1 + 2i 0 2i −i  R2 − iR1 ∼  0 0 1 2i 1 − 2i ∼
   

1 2+i 1+i 2i 2 − i R3 − R1 0 0 1 2i 1 − 2i R3 − R2
 
 1 2 + i 0 2 −1 
 0 0 1 2i 1 − 2i 
 
0 0 0 0 0

Hence, the system is consistent with two parameters. Let z2 = s ∈ C and z4 = t ∈ C. Then, the
general solution is
 −1  −2 − i  −2 
     
 0   1 
 + t  0 
 
~z =   + s 
1 − 2i  0 
 
−2i
 
0 0 1
(b) Row reducing gives
   
 i 2 −3 − i 1   i 2 −3 − i 1  R1 − iR2
 1 + i 2 − 2i −4 i  R2 − R1 ∼  1 −2i −1 + i −1 + i  ∼
   
1
i 2 −3 − 3i 1 + 2i R3 − R1 0 0 −2i 2i 2 iR3
   
 0 0 −2 2+i  0 0 0 i
 R1 + 2R3 ∼  1 −2i −1 + i −1 + i 
 
1 −2i −1 + i −1 +i

   
0 0 1 −1 0 0 1 −1
Hence, the system is inconsistent.
(c) Row reducing gives
   
 i 1+i 1 2i  −iR3  1 1−i −i 2 
 1 − i 1 − 2i −2 + i −2 + i

 ∼

1 − i 1 − 2i −2 + i −2 +i  R2 + (1 − i)R1 ∼

 
2i 2i 2 4 + 2i − 21 iR3 1 1 −i 1 − 2i R3 − R1
   
 1 1 − i −i 2  R1 − (1 − i)R2  1 0 −1 − 4i 3 − 7i 
 0 1 −1 + 2i −4 + 3i ∼  0 1 −1 + 2i −4 + 3i  ∼
   

1
0 i 0 −1 − 2i R3 − iR2 0 0 2+i 2 + 2i 2+i R3
 

 R1 + (1 + 4i)R3  1 0 0 13 9 


  
 1 0 −1 − 4i 3 − 7i 5 − 5i 

 0 1 −1 + 2i −4 + 3i  R2 + (1 − 2i)R2 ∼  0 1 0 −2 + i 
  
6 2
0 0 1 5 + 5i 0 0 1 65 + 25 i
13
Hence, the solution is z1 = 5 − 95 i, z2 = −2 + i, and z3 = 6
5 + 25 i.
232 Section 11.1 Solutions

(d) Row reducing gives


   
 1 −1 i 2i   1 −1 i 2i  R1 − R2
 1 + i −i i −2 + i  R2 − (1 + i)R1 ∼  0 1 1 −i  ∼
   
1 − i −1 + 2i 1 + 2i 3 + 2i R3 − (1 − i)R2 0 i i 1 R3 − iR2
 
 1 0 1 + i i 
 0 1 1 −i 
 
0 0 0 0

Hence, the system is consistent with one parameter. Let z3 = t ∈ C. Then, the general solution is
   
 i  −1 − i
~z = −i + t  −1 
   
0 1
   

11.1.4 (a) Let z1 = a1 + b1 i, z2 = a2 + b2 i, and z3 = a3 + b3 i. We have

z1 (z2 + z3 ) = (a1 + b1 i)(a2 + b2 i + a3 + b3 i)


= (a1 + b1 i)[(a2 + a3 ) + (b2 + b3 )i]
= a1 (a2 + a3 ) − b1 (b2 + b3 ) + b1 (a2 + a3 )i + a1 (b2 + b3 )i
= a1 a2 − b1 b2 + b1 a2 i + a1 b2 i + a1 a3 − b1 b3 + b1 a3 i + a1 b3 i
= (a1 + b1 i)(a2 + b2 i) + (a1 + b1 i)(a3 + b3 i)
= z1 z2 + z1 z3

(b) Let z = a + bi. Define 1


z = a
a2 +b2
+ a2−b
+b2
i. Then 1
z ∈ C since a
a2 +b2
∈ R and −b
a2 +b2
∈ R. Also, we have
!
a −b
z(1/z) = (a + bi) 2 + i
a + b2 a2 + b2
! " #
 a  −b −b a
=a 2 2
−b 2 2
+ a 2 2
+b 2 i
a +b a +b a +b a + b2
a2 + b2
= 2 + 0i
a + b2
=1

(c) If z1 = a + bi ∈ R, then b = 0. Hence, z1 = a + 0i = a − 0i = a + 0i = z1


On the other hand, assume that z1 = z1 . Then, we have a − bi = a + bi which implies that b = 0.
Thus, z1 ∈ R.
(d) Let z1 = a + bi and z2 = c + di. Then,

z1 z2 = ac − bd + (ad + bc)i = ac − bd − (ad + bc)i = ac − bd + (−ad − bc)i = (a − bi)(c − di) = z1 z2


Section 11.2 Solutions 233

(e) We have

|z1 + z2 |2 = (z1 + z2 )(z1 + z2 )


= (z1 + z2 )(z1 + z2 )
= z1 z1 + z1 z2 + z2 z1 + z2 z2
= |z1 |2 + z1 z2 + z1 z2 + |z2 |2
= |z1 |2 + 2 Re(z1 z2 ) + |z2 |2
≤ |z1 |2 + 2|z1 z2 | + |z2 |2
= |z1 |2 + 2|z1 ||z2 | + |z2 |2
= (|z1 | + |z2 |)2

11.1.5 Let z1 = a + bi and z2 = c + di. If z1 + z2 is a negative real number, then b = −d and a + c < 0. Also, we
have z1 z2 = (ac − bd) + (ad + bc)i is a negative real number, so ad + bc = 0 and ac − bd < 0. Combining
these we get
0 = ad + bc = ad − dc = d(a − c)
so either d = 0 or a = c. If a = c, then ac − bd < 0 implies that a2 + b2 < 0 which is impossible. Hence,
we must have d = 0. But then b = −d = 0 so z1 and z2 are real numbers.
11.1.6 Let z = a + bi. Then a2 + b2 = 1. Hence
1 1 1 1 − a + bi 1 − a + bi
= = =
1 − z 1 − a − bi 1 − a − bi 1 − a + bi (1 − a)2 + b2
1 − a + bi 1 − a + bi
= 2 2
=
1 − 2a + a + b 1 − 2a + 1
1−a b
= + i
2 − 2a 2 − 2a
!
1 1
Hence, Re = .
1−z 2
11.1.7 Let z = a + bi. We will prove this by induction on n. If n = 1, then z = z. Assume that zk = (z)k for
some integer k. Then, by Theorem 5.1.3 part 5,

zk+1 = zk z = zk z = (z)k z = (z)k+1

as required.

11.2 Problem Solutions


   
z1  y1 
11.2.1 (a) By definition S1 is a subset of C3 and ~0 ∈ S1 since i(0) = 0. Let ~z = z2  , ~y = y2  ∈ S1 . Then,
   
z3 y3
   
 
z1 + y1 
iz1 = z3 and iy1 = y3 . Hence, ~z + ~y = z2 + y2  ∈ S1 since i(z1 + y1 ) = iz1 + iy1 = z3 + y3 . Similarly,
 
z3 + y3
 
234 Section 11.2 Solutions

αz1 
 
α~z = αz2  ∈ S1 since i(αz1 ) = α(iz1 ) = αz3 . Therefore, by the Subspace Test, S1 is a subspace of
 
αz3
 
3
C.
Every ~z ∈ S1 has the form
       
z1   z1  1 0
z2  =  z2  = z1 0 + z2 1
       
z3 iz1 i 0
   

 1 0
   
 
Thus, B =  0 , 1 spans S1 and is clearly linearly independent, so it is a basis for S1 . Conse-


 i 0
     

quently, dim S1 = 2.
         
1 0 1 1 1
(b) Observe that 0 ∈ S2 since 1(0) = 0 and 1 ∈ S2 since 0(1) = 0. However, 0 + 0 = 1 < S2
         
0 0 0 0 0
         
since 1(1) , 0. Consequently, S2 is not closed under addition and hence is not a subspace.
   
z1  y1 
(c) By definition S3 is a subset of C3 and ~0 ∈ S3 since 0 + 0 + 0 = 0. Let ~z = z2  , ~y = y2  ∈ S3 .
   
z3 y3
   
 
z1 + y1 
Then, z1 + z2 + z3 = 0 and y1 + y2 + y3 = 0. Hence, ~z + ~y = z2 + y2  ∈ S1 since (z1 + y1 ) +
 
z3 + y3
 
αz1 
 
(z2 + y2 ) + (z3 + y3 ) = z1 + z2 + z3 + y1 + y2 + y3 = 0 + 0 = 0. Similarly, α~z = αz2  ∈ S1 since
 
αz3
 
αz1 + αz2 + αz3 = α(z1 + z2 + z3 ) = α(0) = 0. Therefore, by the Subspace Test, S3 is a subspace of
C3 .
Every ~z ∈ S1 has the form
       
z1   z1  1  0 
z2  =  z2  = z1 0 + z2  1 
       
z3 −z1 − z2 1 −1
    

 1  0 
   
 
Thus, B =  0 ,  1  spans S3 and is clearly linearly independent, so it is a basis for S3 . Conse-


 1 −1 
     

quently, dim S3 = 2.
11.2.2 (a) Row reducing the matrix A to its reduced row echelon form R gives
   
 1 i  1 0
1 + i −1 + i ∼ 0 1
   
−1 i 0 0
Section 11.2 Solutions 235

The non-zero rows of R form a basis for the rowspace of A. Hence, a basis for Row(A) is
(" # " #)
1 0
,
0 1
The columns from A which correspond to columns in R which have leadings ones form a basis for
the columnspace of A. So, a basis for Col(A) is
   

  1   i  


 , −1 + i

1 + i
  


 
    


 −1  
i 

Solve the homogeneous system A~z = ~0, we find that a basis for Null(A) is the empty set.
To find a basis for the left nullspace, we row reduce AT . We get
" # " #
1 1 + i −1 1 1+i 0

i −1 + i i 0 0 1

Solving AT ~z = ~0, we find that a basis for Null(AT ) is


 


 −1 − i 

1
 

 
 
  0 
   

(b) Row reducing the matrix B to its reduced row echelon form R gives
" # " #
1 1 + i −1 1 1+i 0

i −1 + i i 0 0 1
The non-zero rows of R form a basis for the rowspace of B. Hence, a basis for Row(B) is
   


  1  0 

1 + i , 0
 


 

 0   
1

The columns from B which correspond to columns in R which have leadings ones form a basis for
the columnspace of B. So, a basis for Col(B) is
(" # " #)
1 −1
,
i i
 

 −1 − i 
Solve the homogeneous system B~z = ~0, we find that a basis for Null(B) is 
 
1 .
  
 
 
 0 
 

To find a basis for the left nullspace, we row reduce BT . We get


   
 1 i  1 0
1 + i −1 + i ∼ 0 1
   
−1 i 0 0
Therefore, a basis for Null(BT ) is the empty set.
236 Section 11.2 Solutions

(c) Row reducing the matrix A to its reduced row echelon form R gives
   
1 1 + i 3  1 0 1 
0 2 2 − 2i ∼ 0 1 1 − i
  

i 1−i −i 0 0 0
  

The non-zero rows of R form a basis for the rowspace of A. Hence, a basis for Row(A) is
   

 1 0
   
 
0 , 1



1 0
     

The columns from A which correspond to columns in R which have leadings ones form a basis for
the columnspace of A. So, a basis for Col(A) is
   

 1 1 + i 
  
 
0 ,  2 



 i 1−i 
     

 

  −1  
Solve the homogeneous system A~z = ~0, we find that a basis for Null(A) is 
 
−1 + i .
 
 
  
 1 
 

To find a basis for the left nullspace, we row reduce AT . We get


   
 1 0 i  1 0 i 
1 + i 2 1 − i ∼ 0 1 1 − i
  

3 2 − 2i −i 0 0 0
  

Solving AT ~z = ~0, we find that a basis for Null(AT ) is


 


  −i  

−1 + i
 


  1 
 

11.2.3 If ~z ∈ Cn , then there exists z1 , . . . , zn ∈ C such that


z1     x1  y1 
     
 ..   1 x + iy1  . 
+ i  ...  = ~x + i~y
   
.
 
~z =  .  = .  =
..x + iy   . 
  
  n n
 
zn xn yn
where ~x, ~y ∈ Rn .
   
z1  y1 
11.2.4 (a) Let ~z = z2  and ~y = y2  and α, β ∈ C. Then
   
z3 y3
   

αz1 + βy1  "


 
#
−i(αz1 + βy1 ) + (1 + i)(αz2 + βy2 + (1 + 2i)(αz3 + βy3 )
L(α~z + β~y) = L αz2 + βy2  =
 
(−1 + i)(αz1 + βy1 ) − 2i(αz2 + βy2 ) − 3i(αz3 + βy3 )
αz2 + βy2
 
" # " #
−iz1 + (1 + i)z2 + (1 + 2i)z3 −iy1 + (1 + i)y2 + (1 + 2i)y3
=α +β = αL(~z) + βL(~y)
(−1 + i)z1 − 2iz2 − 3iz3 (−1 + i)y1 − 2iy2 − 3iy3
Section 11.2 Solutions 237

Thus L is linear.
" # " # " #
−i 1+i 1 + 2i
We have L(1, 0, 0) = , L(0, 1, 0) = , and L(0, 0, 1) = . Hence [L] =
−1 + i −2i −3i
" #
−i 1 + i 1 + 2i
.
−1 + i −2i −3i
(b) Row reducing [L] we get
" # " #
−i 1 + i 1 + 2i 1 −1 + i 0

−1 + i −2i −3i 0 0 1
 

 1 − i 
A basis for ker(L) is the general solution of [L]~x = ~0, hence a basis is 
 
 1  .
 

 0 
 

(" # " #)
−i 1 + 2i
The range of L is equal to the columnspace of [L]. Thus, a basis for the range of L is , .
−1 + i −3i
11.2.5 ~ ,~z ∈ C2 and α, β ∈ C. Then
(a) Let w
" #
αz1 + βw1 + (1 + i)(αz2 + βw2 )
L(α~z + β~
w) = L(αz1 + βw1 , αz2 + βw2 ) =
(1 + i)(αz1 + βw1 ) + 2i(αz2 + βw2 )
" # " #
z1 + (1 + i)z2 w1 + (1 + i)w2
=α +β
(1 + i)z1 + 2iz2 (1 + i)w1 + 2iw2
= αL(~z) + βL(~
w)

Hence, L is linear.
" #
z
(b) If ~z = 1 ∈ ker(L), then
z2
" # " #
0 z + (1 + i)z2
= L(~z) = 1
0 (1 + i)z1 + 2iz2
Hence, we need to solve the homogeneous system of equations

z1 + (1 + i)z2 = 0
(1 + i)z1 + 2iz2 = 0

Row reducing the corresponding coefficient matrix gives


" # " #
1 1+i 1 1+i

1 + i 2i 0 0
(" #)
−1 − i
Hence, a basis for ker(L) is .
1
238 Section 11.2 Solutions

Every vector in the range of L has the form


" # " # " #
z1 + (1 + i)z2 1 1+i
= z1 + z2
(1 + i)z1 + 2iz2 1+i 2i
" # " #
1 1
= z1 + (1 + i)z2
1+i 1+i
" #
1

= z1 + (1 + i)z2
1+i
(" #)
1
Therefore, a basis for Range(L) is .
1+i
" #
~ −1 − 1
(c) If ~z ∈ ker(L), then L(~z) = 0 = 0~z. Consequently, we pick ~z1 = . In part (b) we showed
1
" # " #
1 1

that L(~z) = z1 + (1 + i)z2 . Hence, if we take~z2 = , we get
1+i 1+i
" # " #
1 1

L(~z2 ) = 1 + (1 + i)(1 + i) = (1 + 2i)
1+i 1+i

Thus, if we take B = {~z1 ,~z2 } we get

L(~z1 ) = ~0 = 0~z1 + 0~z2


L(~z2 ) = (1 + 2i)~z2 = 0~z1 + (1 + 2i)~z2

Hence, " #
0 0
[L]B =
0 1 + 2i
11.2.6 We have

1 −1 i 1 0 0
1 1
1 + i −i i = 1 + i 1 1 =
=0
i i
1 − i −1 + 2i 1 + 2i 1 − i i i

11.2.7 We have    
 1 1 2 1 0 0   1 0 0 2 + 2i −i −2 + i 
0 1 + i 1 0 1 0  ∼  0 1 0 1 −i i
   
 
i 1 + i 1 + 2i 0 0 1 0 0 1 −1 − i i 1−i
Hence,
 −1  
 1 1 2   2 + 2i −i −2 + i 
 0 1 + i 1  =  1 −i i
   

i 1 + i 1 + 2i −1 − i i 1−i
Section 11.3 Solutions 239

11.2.8 We prove this by induction. If A = [a] is a 1 × 1 matrix, then

det A = det[a] = a = det[a] = det A

Assume the result holds for n − 1 × n − 1 matrices and consider an n × n matrix A. If we expand det A
along the first row, we get by definition of the determinant
n
X
det A = a1iC1i (A)
i=1

where C1i (A) represents the cofactors of A. But, each of these cofactors is the determinant of an n − 1 ×
n − 1 matrix, so we have by our inductive hypothesis that C1i (A) = C1i (A). Hence,
n
X n
X
det A = a1iC1i (A) = a1iC1i (A) = det A
i=1 i=1

11.3 Problem Solutions


11.3.1 (a) We have

2 − λ 1 + i
C(λ) = = λ2 − 3λ = λ(λ − 3)
1 − i 1 − λ

Hence, the eigenvalues are λ1 = 0 and λ2 = 3. For λ1 = 0 we get


" # " #
2 1+i 1 (1 + i)/2
A − 0I = ∼
1−i 1 0 0
(" #)
1+i
Hence, a basis for Eλ1 is . For λ2 = 3 we get
−2
"# " #
−1 1 + i 1 −1 − i
A − 3I = ∼
1 − i −2 0 0
(" #)
1+i
Hence, a basis for Eλ2 is .
1
#" " #
1+i 1+i 0 0
Therefore, A is diagonalized by P = to D = .
−2 1 0 3
(b) We have

3 − λ 5
C(λ) = = λ2 + 16
−5 −3 − λ
240 Section 11.3 Solutions

Solving λ2 + 16 = 0 we find that the eigenvalues are λ1 = 4i and λ2 = −4i. For λ1 = 4i we get
" # " #
3 − 4i 5 5 3 + 4i
A − 4iI = ∼
−5 −3 − 4i 0 0
(" #)
3 + 4i
Hence, a basis for Eλ1 is . For λ2 = −4i we get
−5
" # " #
3 + 4i 5 5 3 − 4i
A + 4iI = ∼
−5 −3 + 4i 0 0
(" #)
3 − 4i
Hence, a basis for Eλ2 is .
−5
" # " #
3 + 4i 3 − 4i 4i 0
Therefore, A is diagonalized by P = to D = .
−5 −5 0 −4i
(c) We have

2 − λ 2 −1 −λ 0 λ
C(λ) = −4 1 − λ 2 = −4 1 − λ 2
2 2 −1 − λ 2
2 −1 − λ

0 0 λ
= −2 1 − λ
2 = −λ(λ2 − 2λ + 5)
1 − λ 2 −1 − λ

Hence, the eigenvalues are λ1 = 0 and the roots of λ2 − 2λ + 5. By the quadratic formula, we get
that λ2 = 1 + 2i and λ3 = 1 − 2i. For λ1 = 0 we get
   
 2 2 −1 1 0 −1/2
A − 0I = −4 1 2  ∼ 0 1 0 
   
2 2 −1 0 0 0
   

  

 1
 
 
Hence, a basis for Eλ1 is  0. For λ2 = 1 + 2i we get

2

 

   
1 − 2i 2 −1  1 0 −1
A − λ2 I =  −4 −2i 2  ∼ 0 1 −i 
   
2 2 −2 − 2i 0 0 0
   

  

 1
 
 
Hence, a basis for Eλ2 is   i  . For λ3 = 1 − 2i we get


1
   

   
1 + 2i 2 −1  1 0 −1
A − λ3 I =  −4 2i 2  ∼ 0 1 i 
   
2 2 −2 + 2i 0 0 0
   
Section 11.3 Solutions 241

  

  1 
 
 
Hence, a basis for Eλ3 is  −i.

  1 

 

   
1 1 1  0 0 0 
Therefore, A is diagonalized by P = 0 i −i to D = 0 1 + 2i 0 .
   
1 1 1 0 0 1 − 2i
   

(d) We have

1 − λ i
C(λ) = = λ2
i −1 − λ

Hence, the only eigenvalue is λ1 = 0 with algebraic multiplicity 2. We get


" # " #
1 i 1 i
A − 0I = ∼
i −1 0 0
(" #)
−i
Hence, a basis for Eλ1 is . Hence, the geometric multiplicity is 1 and so the matrix is not
1
diagonalizable.
(e) We have

2 − λ 1 −1 2 − λ 0 −1
C(λ) = 2 1−λ 0 = 2 1−λ 0
3 −1 2 − λ 3
1 − λ 2 − λ

2 − λ 0 −1
= 2 1−λ 0 = −(λ − 1)(λ2 − 4λ + 5)
1 0 2 − λ

Hence, the eigenvalues are λ1 = 1 and the roots of λ2 − 4λ + 5. By the quadratic formula, we get
that λ2 = 2 + i and λ3 = 2 − i. For λ1 = 1 we get
   
1 1 −1 1 0 0 
A − I = 2 0 0  ∼ 0 1 −1
   
3 −1 1 0 0 0
   

  

 0
 
 
Hence, a basis for Eλ1 is  1. For λ2 = 2 + i we get


1
 

   
−i 1 −1 1 0 −(1 + 2i)/5
A − λ2 I =  2 −1 − i 0  ∼ 0 1 −(3 − i)/5 
   
3 −1 −i 0 0 0
   
242 Section 11.3 Solutions

 

 1 + 2i 

 
Hence, a basis for Eλ2 is  3 + i . For λ3 = 2 − i we get

 
  
  5 
   

   
 i 1 −1 1 0 −(1 − 2i)/5
A − λ3 I = 2 −1 + i 0  ∼ 0 1 −(3 + i)/5 
   
3 −1 i 0 0 0
   

 

 1 − 2i 

 
Hence, a basis for Eλ3 is  3 − i .

 
  
 5 
 
 
 

   
0 1 + 2i 1 − 2i 1 0 0 
Therefore, A is diagonalized by P = 1 3 + i 3 − i  to D = 0 2 + i 0 .
   
1 5 5 0 0 2−i
   

(f) We have

1 + i − λ 1 0 1 + i − λ 1 0
C(λ) = 1 1−λ −i = 0 1 − λ −1 − i + λ
1 0 1 − λ 1 0 1 − λ
= (1 + i − λ)(1 − λ)2 + (−1 − i + λ) = −(λ − (1 + i))[λ2 − 2λ + 1 − 1]
= −(λ − (1 + i))(λ − 2)λ

Hence, the eigenvalues are λ1 = 1 + i, λ2 = 2, and λ3 = 0. For λ1 = 1 + i we get


   
0 1 0  1 0 −i
A − λ1 I = 1 −i −i ∼ 0 1 0 
   
1 0 −i 0 0 0
   

  

  i 
 
 
Hence, a basis for Eλ1 is  0 . For λ2 = 2 we get

1

   

   
−1 + i 1 0  1 0 −1 
A − λ2 I =  1 −1 −i  ∼ 0 1 −1 + i
   
1 0 −1 0 0 0
   

 


  1  

Hence, a basis for Eλ2 is  1 − i . For λ3 = 0 we get
 
 1 

  

   
−1 + i 1 0  1 0 1 
A − λ3 I =  1 −1 −i  ∼ 0 1 −1 − i
   
1 0 −1 0 0 0
   
Section 11.3 Solutions 243

 

  −1  

 
Hence, a basis for Eλ3 is  1 + i .

 
 
 1 
  

   
 i 1 −1  1 + i 0 0
Therefore, A is diagonalized by P = 0 1 − i 1 + i to D =  0 2 0.
   
1 1 1 0 0 0
   

(g) We have

5 − λ −1 + i 2i 1 − λ 0 i − iλ
C(λ) = −2 − 2i 2 − λ 1 − i = −2 − 2i 2 − λ 1 − i
4i −1 − i −1 − λ 4i −1 − i −1 − λ

1 − λ 0 0
= −2 − 2i 2 − λ −1 + i


4i −1 − i 3 − λ
= (1 − λ)[λ2 − 5λ + 4) = −(λ − 1)2 (λ − 4)
Hence, the eigenvalues are λ1 = 1 and λ2 = 4. For λ1 = 1 we get
   
 4 −1 + i 2i  1 (−1 + i)/4 i/2
A − λ1 I = −2 − 2i 1 1 − i ∼ 0 1 0 
   
4i −1 − i −2 0 0 0
   
   

 1 − i −i 

   
Hence, a basis for Eλ1 is  4 , 0 . For λ2 = 4 we get

 

 
 
 
 


 0 
2 
   
   
 1 −1 + i 2i  1 0 −i 
A − λ2 I = −2 − 2i −2 1 − i ∼ 0 1 (1 − i)/2
   
4i −1 − i −5 0 0 0
   
 

  −2i  

 
Hence, a basis for Eλ2 is  −1 + i .

 
  
 2 
 
 
 

   
1 − i −i −2i  1 0 0
Therefore, A is diagonalized by P =  4 0 −1 + i to D = 0 1 0.
   
0 2 2 0 0 4
   

(h) We have

−6 − 3i − λ −2 −3 − 2i i − λ −2 −3 − 2i
C(λ) = 10 2−λ 5 =
0 2−λ 5

8 + 6i 3 4 + 4i − λ −2i + 2λ 3 4 + 4i − λ

i − λ −2 −3 − 2i
= 0 2−λ 5
0 −1 −2 − λ
= (i − λ)(λ2 − 1) = −(λ − i)2 (λ + i)
244 Section 11.3 Solutions

Hence, the eigenvalues are λ1 = i and λ2 = −i. For λ1 = i we get


   
−6 − 4i −2 −3 − 2i 1 0 1/2
A − λ1 I =  10 2−i 5  ∼ 0 1 0 
   
8 + 6i 3 4 + 3i 0 0 0
   

  

 −1
 
 
Hence, a basis for Eλ1 is   0 .


  2 
 

Thus, A is not diagonalizable since gλ1 < aλ1 .


11.3.2 (a) The characteristic polynomial is

cos θ − λ − sin θ
C(λ) = det(Rθ − λI) = = λ2 − 2 cos θ + 1
sin θ cos θ − λ

Hence, by the quadratic formula, we have



2 cos θ ± 4 cos2 θ − 4 p
λ= = cos θ ± − sin2 θ = cos θ ± i sin θ
2

Observe, that if sin θ = 0, then Rθ is diagonal, so we could just take P = I. So, we now assume
that sin θ , 0.
" # " #
−i sin θ − sin θ 1 −i
For λ1 = cos θ + i sin θ, we get Rθ − λ1 I = ∼
sin θ −i sin θ 0 0
" #
i
We get that an eigenvector of λ1 is .
1
" # " #
i sin θ − sin θ 1 i
Similarly, for λ2 = cos θ − i sin θ, we get Rθ − λ2 I = ∼
sin θ i sin θ 0 0
" #
−i
We get that an eigenvector of λ1 is .
1
" # " #
i −i cos θ + i sin θ 0
Thus, P = and D = .
1 1 0 cos θ − i sin θ

" #
1 0
(b) We have R0 = which is already diagonal, so R0 is diagonalized by P = I as we stated in
0 1
(a).
" √ √ # " #
1/ √2 −1/√ 2 i −i
Rπ/4 = . From our work in a) Rπ/4 is diagonalized by P = to D =
1/ 2 1/ 2 1 1
" √ #
(1 + i)/ 2 0 √
. Indeed, we find that
0 (1 − i)/ 2
" #" √ √ #" # " √ #
−1 1 −i 1 1/ 2 −1/ 2 i −i (1 + i)/ 2 0
P Rπ/4 P = √ √ = √
2 i 1 1/ 2 1/ 2 1 1 0 (1 − i)/ 2
Section 11.4 Solutions 245

11.3.3 If A~z = λ~z, then


A~z = A~z = λ~z = λ~z
Hence, ~z is an eigenvector of A with eigenvalue λ.
11.3.4 Since A is real we know that the other eigenvalue of A is 2 − i. Moreover, by the result of Assignment 10
" to 2 − i is# the conjugate
#6, we get that an eigenvector corresponding " of the# eigenvector corresponding
1+i 1−i 2+i 0
to 2 + i. Hence, A is diagonalized by P = to D = . Then P−1 AP = D, so
i −i 0 2−i
" #" # " # " #
−1 1+i 1−i 2+i 0 1 −i −1 + i 1 2
A = PDP = =
i −i 0 2 − i −2i −i 1 + i −1 3

11.3.5 If A is a 3 × 3 real matrix with a non-real eigenvalue λ, then by Theorem 11.3.1, λ is another eigenvalues
of A. Also, by Corollary 11.3.2, we have that A must have a real eigenvalue. Therefore, A has 3 distinct
eigenvalues and hence is diagonalizable.

11.4 Problem Solutions


p p √ √ √
11.4.1 (a) k~u k = ~u · ~u = 1(1) + 2i(−2i) + (1 − i)(1 + i) = 1 + 4 + 2 = 7.
h~u, ~ui =
p p √ √ √
(b) k~
w k = h~ ~i = w
w, w ~ ·w ~ = (1 + i)(1 − i) + (1 − 2i)(1 + 2i) + i(−i) = 2 + 5 + 1 = 8.
(c) h~u,~zi = ~u · ~z = 1(2) + 2i(2i) + (1 − i)(1 + i) = 2 − 4 + 2 = 0
(d) h~z, ~ui = h~u,~zi = 0
(e) h~u, (2 + i)~zi = (2 + i)h~u,~zi = 0
(f) h~z, w ~ = 2(1 − i) + (−2i)(1 + 2i) + (1 − i)(−i) = 2 − 2i − 2i + 4 − i − 1 = 5 − 5i
~ i = ~z · w

(g) h~w,~zi = w ~ · ~z = w
~ · ~z = ~z · w~ = h~z, w
~ i = 5 − 5i
   
* 3   −2 +
   
(h) h~u + ~z, 2i~
w − i~zi =  0  ,  2 + 2i  = 3(−2) + 0(2 − 2i) + (2 − 2i)(−3 + i) = −10 + 8i
2 − 2i −3 − i
   

11.4.2 (a) We have


hA, Bi = 1(−i) + 2i(2) + (−i)(1 + i) + (1 + i)(3) = 4 + 5i

(b) We have
hB, Ai = i(1) + 2(−2i) + (1 − i)(i) + 3(1 − i) = 4 − 5i

(c) We have p p √
kAk = hA, Ai = 1(1) + 2i(−2i) + (−i)(i) + (1 + i)(1 − i) = 8

(d) We have p p
kBk = hB, Bi = i(−i) + 2(2) + (1 − i)(1 + i) + 3(3) = 4
246 Section 11.4 Solutions

11.4.3 (a) Observe that *" √ # " √ #+


i/ √2 h1/ √2 i 1 1 −i i i
, = √ · √ + √ · √ = + =i
1/ 2 −i/ 2 2 2 2 2 2 2
Hence, the columns are not orthogonal, so the matrix is not unitary.
 √ √ 
(1 + i)/2 (1 − i)/ 6 (1 − i)/ 12

3i/ √12 . Then we find that UU ∗ = I, so U is unitary.
 
(b) Let U =  1/2 0√
−i/ 12
 
1/2 2i/ 6
(c) The matrix is a real orthogonal matrix, and hence is unitary.
(d) Observe that  
 0  r r
 0  = 0 + 0 + 2 =
 1
,0
 4 2

(1 + i)/2

Thus, the matrix is not unitary.


h i
11.4.4 Let U = ~u1 · · · ~un . Then, we have
 ∗

 ~u1  h i
U U = .  ~u1 · · · ~un
..~u∗
n
 ∗
 1~u1 · · · ~u∗1~un 
~ u

 . .. .. 
=  .. . . 

~u∗n~u1 · · · ~u∗n~un

Since {~u1 , . . . , ~un } is an orthonormal basis for Cn we get

~u∗i ~ui = ~uTi ~ui = h~ui , ~ui i = 1

and
~u∗i ~u j = ~uTi ~u j = h~ui , ~u j i = 0
whenever i , j. Thus, U ∗ U = I as required.
11.4.5 (a) We have

~ i = (1 + i)(1 + i) + (2 − i)(−2 + 3i) + (−1 + i)(−1) = 2i − 1 + 8i + 1 − i = 9i


h~z, w
w, 2i~zi = 2ih~z, w
h~ ~ i = −2i(−9i) = −18

(b) A vector orthogonal to ~z in Span{~z, w


~ } is
     
 1 − i   1 + i   0 
w,~zi
h~ 9i
~ =w
~v = perp~z w ~− ~z = −2 − 3i +  2 − i  =  1 − i 
     
k~zk2  9 
−1 −1 + i −2 − i
   
Section 11.5 Solutions 247

(c) Note that Span{~z, w ~ } = Span{~z, ~v} where ~v is the vector from part (b) and {~z, ~v} is orthogonal over
C so we have
< ~u,~z > < ~u, ~v >
projS ~u = 2
~z + ~v
k~zk k~vk2
11.4.6 Denote the vectors in the spanning set by A1 , A2 , and A3 respectively.
Let B1 = A1 . Next, we get
" # " # " #
hA2 , B1 i i 0 i 1 i i/2 1/2
A2 − B1 = − =
kB1 k2 0 i 2 0 0 0 i
" #
i 1
So, we let B2 = . Next, we get
0 2i
" # " # " # " #
hA3 , B1 i hA3 , B2 i 0 2 −2i 1 i 4 i 1 i/3 1/3
B3 = A3 − B1 − B2 = − − =
kB1 k2 kB2 k2 0 i 2 0 0 6 0 2i 0 −i/3
So, an orthogonal basis for S is {B1 , B2 , B3 }.
11.4.7 (a) We have
 ∗ T
T
(A∗ )∗ = A = AT = (AT )T = A

(b) We have
(αA)∗ = (αA)T = αAT = αAT = αA∗
(c) We have
(AB)∗ = (AB)T = BT AT = BT AT = B∗ A∗
~ ∈ Cn we have
(d) For any ~z, w

~ = (A~z)T w
~ i = (A~z) · w
hA~z, w ~ = ~zT AT w
~ = ~zT AT w
~ = ~zT A∗ w
~ = ~zT A∗ w ~ = h~z, A∗ w
~ = ~z · A∗ w ~i

11.4.8 ~ ∈ Cn we have
(a) For any ~z, w

~ i = (U~z)T U w
hU~z, U w ~ = ~zT U ∗ U w
~ = ~zT U T U w ~ = ~zT U ∗ U w
~ = ~zT w
~ = h~z, w
~i

(b) Suppose λ is an eigenvalue of U with eigenvector ~v. We get kU~vk2 = hU~v, U~vi = h~v, ~vi = k~vk2 by
part (a). But we also have kU~vk2 = kλ~vk2 = hλ~v, λ~vi = λλ̄h~v, ~vi = |λ|2 k~vk2 , so since k~vk , 0, we
must have |λ| = 1.
" #
i 0
(c) The matrix U = is unitary since U ∗ U = I and the only eigenvalue is i with multiplicity 2.
0 i
11.4.9 If h~u, ~vi = 0, then we have

k~u + ~vk2 = h~u + ~v, ~u + ~vi = h~u, ~u + ~vi + h~v, ~u + ~vi = h~u, ~ui + h~u, ~vi + h~v, ~ui + h~v, ~vi
= k~uk2 + 0 + 0 + k~vk2 = k~uk2 + k~vk2

The converse is not true. One counter example is: Consider V = C with its standard inner product
and let ~u = 1 + i and ~v = 1 − i. Then k~u + ~vk2 = k2k2 = 4 and k~uk2 + k~vk2 = 2 + 2 = 4, but
h~u, ~vi = (1 + i)(1 + i) = 2i , 0.
248 Section 11.5 Solutions

11.5 Problem Solutions


11.5.1 (a) If A is skew-Hermitian, then A∗ = −A. By Schur’s Theorem, there exists a unitary matrix U such
that U ∗ AU = T is upper triangular. Now, observe that

T ∗ = (U ∗ AU)∗ = U ∗ A ∗ U = U ∗ (−A)U = (−1)U ∗ AU = −T

Thus, T is also skew-Hermitian. Since T is upper triangular, we have that T ∗ is lower triangluar.
Consequently, T is both upper and lower triangular and hence diagonal.
So, U ∗ AU = T is diagonal as required.
(b) If A is skew-Hermitian, then A∗ = −A and hence

AA∗ = A[(−1)A] = (−1)AA = A∗ A

Therefore, A is normal and hence unitarily diagonalizable.


11.5.2 Let λ be an eigenvalue of A with eigenvector ~z. Since A is skew-Hermitian we have A∗ = −A and hence

λh~z,~zi = hλ~z,~zi = hA~z,~zi = h~z, A∗~zi = h~z, −A~zi = h~z, −λ~zi = −λh~z,~zi

Since h~z,~zi , 0 as ~z , ~0, we get that λ = −λ so Re(λ) = 21 (λ + λ) = 0.


11.5.3 (a) If A is unitary, then A∗ = A−1 . By Schur’s Theorem, there exists a unitary matrix U such that
U ∗ AU = T is upper triangular. Now, observe that T is a product of unitary matrices and thus, T is
also unitary by Theorem 11.4.8.
t11 t12 · · · t1n 
 
 0 t
22 · · · t2n  h i

Let T =  . . . .  = ~t1 ~t2 · · · ~tn .
 .. .. . . ..  
 
0 · · · 0 tnn
Since T is unitary, the columns of T must form an orthonormal basis for Cn . Thus,

1 = k~t1 k = |t11 |

Also, we must have


0 = h~t1 , ~t2 i = t11 t12
Since t11 , 0, we must have t12 = 0. Then, we get 1 = k~t2 k = |t22 |. Next, we have

0 = h~t1 , ~t3 i = t11 t13


0 = h~t2 , ~t3 i = t22 t23

Therefore, t13 = t23 = 0. Continuing in this way we get that T is diagonal and all of the diagonal
entries satisfy |tii | = 1.
So, U ∗ AU = T is diagonal as required.
Section 11.5 Solutions 249

(b) If A is unitary, then A∗ = A−1 and so

AA∗ = I = A∗ A

Thus, A is normal and hence unitarily diagonalizable.


11.5.4 (a) Since A∗ = A, we have that A is Hermitian and hence it is also normal.
" #
2 −i
(b) We have that A∗ = , so A is not Hermitian, nor skew-Hermitian. We find that AA∗ =
i 1−i
" #
5 −1 − 3i
, A∗ A, so A is not normal either.
−1 + 3i 3
" #
∗ 0 −1 + i
(c) We have that A = = −A, so A is skew-Hermitian and hence it is also normal.
1+i 0
" #
∗ 1+i 2
(d) We have that A = , so A is not Hermitian, nor skew-Hermitian. We find that AA∗ =
−2i 3
" #
6 2 + 4i
, A∗ A, so A is not normal either.
2 − 4i 13
" #
∗ −i 2
(e) We have that A = , so A is not Hermitian, nor skew-Hermitian. We find that AA∗ =
−2i 5
" #
5 2 + 2i
= A∗ A, so A is normal .
2 − 2i 5
(f) Since A∗ = A, we have that A is Hermitian and hence it is also normal.
11.5.5 (a) We have C(λ) = λ2 − 2aλ + a2 + b2 so by the quadratic formula we get eigenvalues λ = a ± bi.
For λ = a + bi we get " # " #
−bi b 1
A − λI = ⇒ ~z1 =
−b −bi i
For λ = a − bi we get " # " #
bi b −1
A − λI = ⇒ ~z2 =
−b bi i
 √1 − √1 
" #  
a + bi 0
Hence we have D = and U =  i 2 2 .
0 a − bi √i 


2 2

(b) We have C(λ) = λ2 − 5λ + 4 = (λ − 4)(λ − 1). Thus, the eigenvalues are λ1 = 4 and λ2 = 1. For
λ1 = 4 we get " # " #
−2 1 + i 1+i
A − λ1 I = ⇒ ~z1 =
1 − i −1 2
For λ2 = 1 we get " # " #
1 1+i −1 − i
A − λI = ⇒ ~z2 =
1−i 2 2
" #  1+i −1−i 
4 0  √ √ 
Hence we have D = and U =  23 6 
.
0 1 √ √1 
3 6
250 Section 11.5 Solutions

(c) We have C(λ) = λ2 − 8λ + 12 = (λ − 2)(λ − 6). Thus, the eigenvalues are λ1 = 2 and λ2 = 6. For
λ1 = 2 we get " √ # " √ #
√ 3 2−i − 2+i
A − λ1 I = ⇒ ~z1 =
2+i 1 3
For λ2 = 6 we get " √ # "√ #
−1 2−i 2−i
A − λI = √ ⇒ ~z2 =
2+i −3 1
 √ √ 
"
2 0
#  − √ 2+i 2−i 
2 

Hence we have D = and U =  112 1 
.
0 6 √
2
12
(d) We have
1 − λ 0 1
C(λ) = 1 1−λ 0 = (1 − λ)3 + 1 = −λ3 + 3λ2 − 3λ + 2
0 1 1 − λ
Since A is a 3 × 3 real matrix, we know that A must have at least one real eigenvalue. We try the
Rational Roots Theorem to see if the real root is rational. The possible rational roots are ±1 and
±2. We find that λ1 = 2 is a root. Then, by the Factor Theorem and polynomial division, we get
−λ3 + 3λ2 − 3λ + 2 = −(λ − 2)(λ2 − λ + 1)
√ √
3 3
Using the quadratic formula, we find that the other eigenvalues are λ2 = 12 + 2 i and λ3 = 21 − 2 i.
   
−1 0 1  1 0 −1
For λ1 we get A − 2I =  1 −1 0  ∼ 0 1 −1
   
0 1 −1 0 0 0
   
 √ 
1/ 3
 √ 
Thus, a corresponding unit eigenvector is z1 = 1/ 3.
 √ 
1/ 3
For λ2 we get
 √   √ 
 1 − 3 i 0√ 1  1 0 −2/(−1 + 3i)
 2 2 √
A − λ2 I =  1 1
− 23 i
  
2 0 √  ∼ 0 1 2/(1 + 3i) 
3  0 0 0
 1
  
0 1 2 − 2 i
 √ 
2/(− 3 + 3i)
 √ 
Thus, a corresponding unit eigenvector is z2 = −2/( 3 + 3i).
 √ 
1/ 3

By Theorem 11.3.1, we have that a unit eigenvector corresponding to λ3 = λ2 is


 √ 
2/(− 3 − 3i)

~z3 = ~z2 = −2/( 3 − 3i)
 
 √ 
1/ 3
h i √ √
Hence, taking U = ~z1 ~z2 ~z3 gives U ∗ AU = diag(2, 12 + 23 i, 21 − 23 i).
Section 11.5 Solutions 251

(e) We have C(λ) = −(λ − 2)(λ2 − λ + 2) = −(λ − 2)2 (λ + 1) so we get eigenvalues λ = 2 and λ = −1.
For λ = 2 we get
     
 −1 0 1 + i 0 1 + i
C − λI =  0 0 0  ⇒ ~z1 = 1 ,~z2 =  0 
     
1 − i 0 −2 0 1
     

For λ = −1 we get
   
 2 0 1 + i 1 + i
C − λI =  0 3 0  ⇒ ~z3 =  0 
   
1−i 0 1 −2
   

0 1+i 1+i


 
√ √ 
 
2 0 0  3 6 

Hence we have D = 0 2 0  and U = 1 0 0 .
  
0 0 −1 0 √13 −2
   
√ 
6
(f) We have

1 − λ i −i 1 + i − λ i −i
C(λ) = −i −1 − λ i = −1 − i − λ −1 − λ i
i −i −λ 0 −i −λ

1 + i − λ 0 −i − λ 1 + i − λ 0 −i − λ
= −1 − i − λ −1 − λ i = −2λ −1 − λ −λ
0 −i −λ 0
−i −λ

1 + i − λ 0 −i − λ
= −2λ −1 + i − λ 0 = i(i + λ)(−2λ) − λ(1 + i − λ)(−1 + i − λ)
0 −i −λ
= −λ3 + 4λ = −λ(λ − 2)(λ + 2)

Thus, the eigenvalues are λ1 = 0, λ2 = 2, and λ3 = −2.


   
 1 i −i 1 0 (−1 − i)/2
For λ1 we get A − 0I = −i −1 i  ∼ 0 1 (−1 − i)/2
   
i −i 0 0 0 0
   
 √ 
(1 + i)/ 8
 √ 
Thus, a corresponding unit eigenvector is z1 = (1 + i)/ 8.
 √ 
1/ 2
   
−1 i −i  1 0 (1 + 3i)/2
For λ2 we get A − 2I =  −i −3 i  ∼ 0 1 (1 − i)/2 
   
i −i −2 0 0 0
   
 
(−1 − 3i)/4
Thus, a corresponding unit eigenvector is z2 =  (−1 + i)/4 .
 
1/2
 
   
 3 i −i 1 0 (1 − i)/2 
For λ3 we get A + 2I = −i 1 i  ∼ 0 1 (1 + 3i)/2
   
i −i 2 0 0 0
   
252 Section 11.6 Solutions
 
 (−1 + i)/4 
Thus, a corresponding unit eigenvector is z3 = (−1 − 3i)/4.
 
1/2
 
h i
Hence, taking U = ~z1 ~z2 ~z3 gives U ∗ AU = diag(0, 2, −2).
11.5.6 We have A∗ A = AA∗ . So

B∗ B = (A∗ A−1 )∗ (A∗ A−1 ) = (A−1 )∗ AA∗ A−1 = (A−1 )∗ A∗ AA−1 = (AA−1 )∗ (AA−1 ) = I

11.5.7 If AB is Hermitian, then (AB) = (AB)∗ = B∗ A∗ = BA, since A and B are Hermitian.

If AB = BA, then (AB)∗ = B∗ A∗ = BA = AB.


11.5.8 (a) For λ = 2 + i we have " # " #
−2 + i −2 + i 1 1
A − λI = ∼
1−i 1−i 0 0
Thus det(A − λI) = 0 as so λ is an eigenvalue of A.
" #
−1
(b) For our work in (a) we get that a unit eigenvector corresponding to λ = 2 + i is ~u1 = √1 . We
2 1
" #
1
extend {~u1 } to an orthonormal basis for by choosing ~u2 = √12 . We can take
1
" #
1 −1 1
U= √
2 1 1
" #
∗ 2 + i 3 − 2i
T = U AU =
0 1+i

11.5.9 (a) We have


AA∗ = A(iA) = iAA = (iA)A = A∗ A
so A is normal.
(b) Assume that ~v is an eigenvector of A corresponding to λ, we get

iλ~v = iA~v = A∗~v = λ~v

Thus, (iλ − λ)~v = ~0, so iλ = λ since ~v , ~0.


(c) By Schur’s Theorem, A is unitarily similar to an upper triangular matrix T whose diagonal entries
are the eigenvalues λ1 , . . . , λn of A. Thus, we have that det T = λ1 · · · λn and tr T = λ1 + · · · + λn .
The result now follows since similar matrices have the same trace and the same determinant.
Section 11.6 Solutions 253

11.6 Problem Solutions


11.6.1 By the Cayley-Hamilton Theorem, we have that

A3 + 2A − 2I = O
A3 + 2A = 2I
!
1 2
A A +I =I
2

Thus, A−1 = 21 A2 + I.
11.6.2 By the Cayley-Hamilton Theorem, we have that

A3 − 147A + 686I = O
A3 = 147A − 686I
   
 441 −1176 −294 686 0 0 
= −1176 −1323 −588 −  0 686 0 
   
−294 −588 882 0 0 686
   
 
 −245 −1176 −294
= −1176 −2009 −588
 
−294 −588 196
 

 
1 0 0
11.6.3 Observe that λ3 − 3λ + 2 = (λ + 2)(λ − 1)2 . Hence, we one choice is to take A = 0 1 0 = I. This
 
0 0 1
 
gives
A3 − 3A + 2A = I 3 − 3I + 2I = I − 3I + 2I = O

NOTE: The characteristic polynomial of our choice of A is (λ − 1)3 = λ3 − 3λ2 + 3λ − 1. Thus, we have
shown that the converse of the Cayley-Hamilton Theorem is not true.

Você também pode gostar